You are on page 1of 324

Compilation of

Digested Cases for


Remedial Law Review II

8 April 2015

Submitted by:
KRISHA MARIE T. BUELA
Wednesday Class | 3:00-7:00 p.m.

Compilation of Digested Cases for Remedial Law Review II


by: K. M. T. BUELA

Table of Contents
PROVISIONAL REMEDIES ................................................................................... 14
A. Preliminary Attachment (Rule 57) ................................................................ 14
ANITA MANGILA vs. COURT OF APPEALS and LORETA GUINA ............ 14
VICENTE B. CHUIDIAN vs. SANDIGANBAYAN .......................................... 15
ALEJANDRO NG WEE vs. MANUEL TANKIANSEE .................................... 18
SOFIA TORRES et al. vs. NICANOR SATSATIN et al. .................................... 19
B. Preliminary Injunction (Rule 58) .................................................................. 20
TERESITA V. IDOLOR vs. HON. COURT OF APPEALS et. al ...................... 20
RIMEO S. GUSTILO vs. HON. RICARDO S. REAL, SR. ................................ 22
MICHAEL J. LAGROSAS vs. BRISTOL-MYERS SQUIBB (PHIL.) et al. ...... 23
NELSON JENOSA et al. vs. REV. FR. JOSE RENE C. DELARIARTE, O.S.A.
et al. ...................................................................................................................... 24
SOLID BUILDERS, INC. and MEDINA FOODS INDUSTRIES, INC. vs.
CHINA BANKING CORPORATION (CBC) ..................................................... 25
SPOUSES SILVESTRE O. PLAZA et al. vs. GUILLERMO LUSTIVA et. al. . 26
C. Receivership (Rule 59) .................................................................................... 28
SPS. CESAR A. LARROBIS, JR. and VIRGINIA S. LARROBIS vs.
PHILIPPINE VETERANS BANK ....................................................................... 28
EVELINA G. CHAVEZ and AIDA CHAVEZ-DELES vs. COURT OF
APPEALS and ATTY. FIDELA Y. VARGAS .................................................... 29
ANA MARIA A. KORUGA vs. TEODORO O. ARCENAS et al. ..................... 30
D. Replevin (Rule 60) ........................................................................................... 31
JOSE S. OROSA and MARTHA P. OROSA vs. HON. COURT OF APPEALS
and FCP CREDIT CORPORATION ................................................................... 31
SMART COMMUNICATIONS, INC. vs. REGINA M. ASTORGA ................. 32
KENNETH HAO vs. ABE C. ANDRES ............................................................. 33
ROGER V. NAVARRO vs. HON. JOSE L. ESCOBIDO and KAREN T. GO .. 34
SPOUSES DEO AGNER and MARICON AGNER vs. BPI FAMILY SAVINGS
BANK, INC. ......................................................................................................... 36
E. Support (Rule 61) ............................................................................................ 37
Page | 2

Compilation of Digested Cases for Remedial Law Review II


by: K. M. T. BUELA
MANUEL DE ASIS vs. COURT OF APPEALS................................................. 37
PEOPLE OF THE PHILIPPINES vs. MANUEL MANAHAN........................... 38
SPOUSES PRUDENCIO and FILOMENA LIM vs. MA. CHERYL S. LIM ..... 38
CHARLES GOTARDO vs. DIVINA BULING................................................... 39

SPECIAL CIVIL ACTIONS .................................................................................... 41


A. Interpleader (Rule 62) .................................................................................... 41
WACK WACK GOLF & COUNTRY CLUB, INC. vs. LEE E. WON and
BIENVENIDO A. TAN ....................................................................................... 41
ETERNAL GARDENS MEMORIAL PARKS CORPORATION vs.
INTERMEDIATE APPELLATE COURT .......................................................... 43
SUBHASH C. PASRICHA and JOSEPHINE A. PASRICHA vs. DON LUIS
DISON REALTY, INC. ....................................................................................... 44
BANK OF COMMERCE (BOC) vs. PLANTERS DEVELOPMENT BANK
(PDB) and BANGKO SENTRAL NG PILIPINAS ............................................. 45
B. Declaratory Relief and Similar Remedies (Rule 63) .................................... 47
EUFEMIA ALMEDA and ROMEL ALMEDA vs. BATHALA MARKETING
INDUSTRIES, INC. ............................................................................................. 47
REPUBLIC OF THE PHILIPPINES vs. CIPRIANO ORBECIDO III ............... 49
CARMEN DANAO MALANA et al. vs. BENIGNO TAPPA et al. ................... 50
FRANCISCO I. CHAVEZ vs. JUDICIAL AND BAR COUNCIL, SEN.
FRANCIS JOSEPH G. ESCUDERO and REP. NIEL C. TUPAS, JR. ............... 51
SPOUSES CLEMENCIO C. SABITSANA, JR. and MA. ROSARIO M.
SABITSANA vs. JUANITO F. MUERTEGUI.................................................... 53
REPUBLIC OF THE PHILIPPINES vs. HERMINIO HARRY ROQUE et. al .. 54
SOUTHERN HEMISPHERE ENGAGEMENT NETWORK, INC vs. ANTITERRORISM COUNCIL et al ............................................................................. 56
C. Review of Judgments and Final Orders of COMELEC and COA (Rule 64)
57
ALLIANCE FOR NATIONALISM AND DEMOCRACY (ANAD) vs.
COMMISSION ON ELECTIONS ....................................................................... 57
D. Certiorari, Prohibition and Mandamus (Rules 65) ..................................... 59
Certiorari
Page | 3

Compilation of Digested Cases for Remedial Law Review II


by: K. M. T. BUELA
OSCAR R. AMPIL vs. THE HON. OFFICE OF THE OMBUDSMAN, et al. ... 59
A.L. ANG NETWORK, INC. vs. EMMA MONDEJAR..................................... 60
MARK JEROME S. MAGLALANG vs. PHILIPPINE AMUSEMENT AND
GAMING CORPORATION (PAGCOR) ............................................................ 62
PEOPLE OF THE PHILIPPINES vs. THE HONORABLE JUANITO C.
CASTANEDA, JR., et al. ..................................................................................... 64
UNIVERSITY OF THE PHILIPPINES BOARD OF REGENTS vs. HON.
ELSIE LIGOT-TELAN ........................................................................................ 65
ROMAN C. TUASON and REMEDIOS V. TUASON vs. REGISTER OF
DEEDS, CALOOCAN City, et al. ....................................................................... 67
Prohibition
ALFEO D. VIVAS vs. THE MONETARY BOARD OF THE BANGKO
SENTRAL NG PILIPINAS AND THE PHILIPPINE DEPOSIT INSURANCE
CORPORATION .................................................................................................. 68
ROSENDO R. CORALES vs. REPUBLIC OF THE PHILIPPINES .................. 70
ROLANDO TAN, ELENA TAN and LAMBERTO TAN vs. THE
HONORABLE COURT OF APPEALS and the PEOPLE OF THE
PHILIPPINES ....................................................................................................... 71
Mandamus
NILO HIPOS vs. HONORABLE RTC JUDGE TEODORO A. BAY ................ 73
EX-C1C JIMMY B. SANCHEZ and EX-C2C SALVADOR A. METEORO vs.
ROBERTO T. LASTIMOSO, in his capacity as DIRECTOR GENERAL OF
THE PHILIPPINE NATIONAL POLICE ........................................................... 74
SOCIAL JUSTICE SOCIETY (SJS) et al. vs. HON. JOSE L. ATIENZA, JR.... 76
DENNIS A.B. FUNA vs. MANILA ECONOMIC AND CULTURAL OFFICE
and the COMMISSION ON AUDIT .................................................................... 77
E. Quo Warranto (Rule 66) ................................................................................ 79
PEDRO MENDOZA vs. RAY ALLAS and GODOFREDO OLORES .............. 79
MA. LUTGARDA P. CALLEJA et al. vs. JOSE PIERRE A. PANDAY et al. ... 80
LUIS K. LOKIN, JR., as the second nominee of CITIZENS BATTLE AGAINST
CORRUPTION (CIBAC) vs. COMMISSION ON ELECTIONS and the HOUSE
OF REPRESENTATIVES ................................................................................... 81
EFREN RACEL ARA TEA vs. COMMISSiON ON ELECTIONS and ESTELA
D. ANTlPOLO...................................................................................................... 83
EMMANUEL A. DE CASTRO vs. EMERSON S. CARLOS ............................ 85
Page | 4

Compilation of Digested Cases for Remedial Law Review II


by: K. M. T. BUELA
F. Expropriation (Rule 67) ................................................................................. 87
CITY OF MANILA vs. OSCAR SERRANO et al. ............................................ 87
NATIONAL POWER CORPORATION vs. COURT OF APPEALS and
ANTONINO POBRE ........................................................................................... 88
REPUBLIC OF THE PHILIPPINES (DPWH) vs. ISMAEL ANDAYA ............ 91
ASIA'S EMERGING DRAGON CORPORATION vs. DEPARTMENT OF
TRANSPORTATION AND COMMUNICATIONS et al. .................................. 92
SPOUSES LETICIA & JOSE ERVIN ABAD et al. vs. FIL-HOMES REALTY
and DEVELOPMENT CORPORATION and MAGDIWANG REALTY
CORPORATION .................................................................................................. 94
NATIONAL POWER CORPORATION, Petitioner, v. YCLA SUGAR
DEVELOPMENT CORPORATION ................................................................... 96
G. Foreclosure of Real Estate Mortgage (Rule 68) ........................................... 97
JOSE T. RAMIREZ vs. THE MANILA BANKING CORPORATION ............. 97
SPOUSES NICASIO C. MARQUEZ AND ANITA J. MARQUEZ vs. SPOUSES
CARLITO ALINDOG AND CARMEN ALINDOG ........................................... 98
RUSTICO A. ARDIENTE and ASUNCION PALOMARDIENTE vs.
PROVINCIAL SHERIFF, REGISTER OF DEEDS OF QUEZON and
PENINSULA DEVELOPMENT BANK ........................................................... 100
LZK HOLDINGS and DEVELOPMENT CORPORATION vs. PLANTERS
DEVELOPMENT BANK .................................................................................. 101
GOLDENWAY MERCHANDISING CORPORATION vs. EQUITABLE PCI
BANK ................................................................................................................. 103
H. Partition (Rule 69)......................................................................................... 105
CELESTINO BALUS vs. SATURNINO BALUS and LEONARDA BALUS
VDA. DE CALUNOD ........................................................................................ 105
EUGENIO FELICIANO et al. vs. PEDRO CANOZA et al. ............................. 106
CARMELA BROBIO MANGAHAS vs. EUFROCINA A. BROBIO .............. 107
I.

Forcible entry and Unlawful Detainer (Rule 70) ....................................... 108


CARMENCITA SUAREZ vs. MR. AND MRS. FELIX E. EMBOY, JR. AND
MARILOU P. EMBOYDELANTAR .............................................................. 108
ATTY. VIRGILIO P. ALCONERA vs. ALFREDO T. PALLANAN ............... 111
TEODORO S. TEODORO vs. DANILO ESPINO et al. ................................... 113
LOURDES B. FERRER and PROSPERIDAD M. ARANDEZ vs. JUDGE
ROMEO A. RABACA ....................................................................................... 115
Page | 5

Compilation of Digested Cases for Remedial Law Review II


by: K. M. T. BUELA
CGR CORPORATION vs. ERNESTO L. TREYES, JR. .................................. 116
J.

Contempt (Rule 71) ....................................................................................... 118


SECURITIES AND EXCHANGE COMMISSION CHAIRMAN PERFECTO R.
YASAY, JR., et al vs. MANUEL D. RECTO et al. ........................................... 118
SALVADOR SISON vs. JUDGE JOSE F. CAOIBES, JR., Presiding Judge, and
TEODORO S. ALVAREZ, Sheriff IV, Regional Trial Court, Las Pias City,
Branch 253 .......................................................................................................... 120
JUDGE DOLORES L. ESPAOL vs. ATTY. BENJAMIN S. FORMOSO and
SPOUSES BENITO SEE and MARLY SEE ..................................................... 121
P/SUPT. HANSEL M. MARANTAN vs. ATTY. JOSE MANUEL DIOKNO and
MONIQUE CU-UNJIENG LA'O ....................................................................... 122
CAPITOL HILLS GOLF & COUNTRY CLUB, INC. and PABLO B. ROMAN,
JR. vs. MANUEL O. SANCHEZ ....................................................................... 124

SPECIAL PROCEEDINGS .................................................................................... 126


A. Settlement of Estate (Rules 73 to 90) ........................................................... 126
TERESITA N. DE LEON et al. vs. HON. COURT OF APPEALS, HON.
PABLO P. INVENTOR and RAMON NICOLAS ............................................ 126
EDGAR SAN LUIS vs. FELICIDAD SAN LUIS ............................................. 128
EDUARDO G. AGTARAP vs. SEBASTIAN AGTARAP, JOSEPH AGTARAP,
TERESA AGTARAP, WALTER DE SANTOS, and ABELARDO DAGORO
............................................................................................................................ 130
EMILIO A.M. SUNTAY III vs. ISABEL COJUANGCO-SUNTAY ............... 132
JOSE C. LEE AND ALMA AGGABAO et al. vs. REGIONAL TRIAL COURT
OF QUEZON CITY BRANCH 85 et al. ............................................................ 134
THE ESTATE OF HILARIO M. RUIZ, EDMOND RUIZ, Executor vs. THE
COURT OF APPEALS ...................................................................................... 136
UNION BANK OF THE PHILIPPINES vs. EDMUND SANTIBAEZ and
FLORENCE SANTIBAEZ ARIOLA ............................................................. 137
AMELIA GARCIA-QUIAZON, JENNETH QUIAZON and MARIA JENNIFER
QUIAZON vs. MA. LOURDES BELEN, for and in behalf of MARIA
LOURDES ELISE QUIAZON ........................................................................... 139
PILAPIL and HEIRS OF DONATA ORTIZ BRIONES vs. HEIRS OF
MAXIMINO R. BRIONES ................................................................................ 141
RODOLFO C. SABIDONG vs. NICOLASITO S. SOLAS............................... 143

Page | 6

Compilation of Digested Cases for Remedial Law Review II


by: K. M. T. BUELA
B. Escheat (Rule 91)........................................................................................... 145
CASTORIO ALVARICO vs. AMELITA L. SOLA .......................................... 145
C. Guardians and Guardianship (Rules 92 to 97)........................................... 147
PILAR Y. GOYENA vs. AMPARO LEDESMA-GUSTILO ............................ 147
The Incompetent, CARMEN CAIZA, represented by her legal guardian,
AMPARO EVANGELISTA vs. COURT OF APPEALS, et al. ........................ 148
NAPOLEON D. NERI, et al. vs. HEIRS OF HADJI YUSOP UY AND
JULPHA* IBRAHIM UY .................................................................................. 150
NILO OROPESA vs. CIRILO OROPESA ........................................................ 153
EDUARDO T. ABAD vs. LEONARDO BIASON and GABRIEL A. MAGNO
............................................................................................................................ 154
D. Trustees (Rule 98) ......................................................................................... 155
ADVENT CAPITAL AND FINANCE CORPORATION vs. NICASIO I.
ALCANTARA and EDITHA I. ALCANTARA ................................................ 155
LAND BANK OF THE PHILIPPINES vs. LAMBERTO C. PEREZ, et al. ..... 156
EDWARD C. ONG vs. THE COURT OF APPEALS AND THE PEOPLE OF
THE PHILIPPINES ............................................................................................ 158
E. Adoption and Custody of Minors (Rules 99 to 100)................................... 160
CANG vs. COURT OF APPEALS and Spouses RONALD V. CLAVANO and
MARIA CLARA CLAVANO ............................................................................ 160
TOMASA VDA. DE JACOB, as Special Administratrix of the Intestate Estate of
Deceased Alfredo E. Jacob vs. COURT OF APPEALS, et al............................ 161
REPUBLIC OF THE PHILIPPINES vs. HON. JOSE R. HERNANDEZ, et al.162
REPUBLIC OF THE PHILIPPINES vs. THE COURT OF APPEALS, JAIME B.
CARANTO, and ZENAIDA P. CARANTO ...................................................... 164
EUGENIO R. REYES, et al. vs. LIBRADA F. MAURICIO (deceased) and
LEONIDA F. MAURICIO ................................................................................. 165
IN THE MATTER OF THE ADOPTION OF STEPHANIE NATHY ASTORGA
GARCIA ............................................................................................................. 166
F. Habeas Corpuz (Rule 102) ........................................................................... 167
ERLINDA K. ILUSORIO vs. ERLINDA I. BILDNER and SYLVIA K.
ILUSORIO, JOHN DOE and JANE DOE ......................................................... 167
ATTY. EDWARD SERAPIO vs. SANDIGANBAYAN, et al. ......................... 168

Page | 7

Compilation of Digested Cases for Remedial Law Review II


by: K. M. T. BUELA
PANFILO LACSON, et al. vs. SECRETARY HERNANDO PEREZ, et al. .... 171
ANISAH IMPAL SANGCA vs. THE CITY PROSECUTOR OF CEBU CITY
............................................................................................................................ 172
ANITA MANGILA vs. JUDGE HERIBERTO M. PANGILINAN, et al ......... 173
G. Change of Name v. Correction/Cancellation of Entries (Rule 103 v. Rule
108)175
MA. LOURDES BARRIENTOS ELEOSIDA vs. LOCAL CIVIL REGISTRAR
OF QUEZON CITY, and CARLOS VILLENA BORBON ............................... 175
REPUBLIC OF THE PHILIPPINES vs. CARLITO I. KHO, et al. ................... 176
IN RE: PETITION FOR CHANGE OF NAME AND/OR
CORRECTION/CANCELLATION OF ENTRY IN CIVIL REGISTRY OF
JULIAN LIN CARULASAN WANG ................................................................ 177
MA. CRISTINA TORRES BRAZA, et al. vs. THE CITY CIVIL REGISTRAR
OF HIMAMAYLAN CITY, NEGROS OCCIDENTAL, et al. ......................... 179
ROMMEL JACINTO DANTES SILVERIO vs. REPUBLIC OF THE
PHILIPPINES ..................................................................................................... 180
REPUBLIC OF THE PHILIPPINES vs. JENNIFER B. CAGANDAHAN ...... 182
REPUBLIC OF THE PHILIPPINES vs. DR. NORMA S. LUGSANAY UY .. 183
MINORU FUJIKI vs. MARIA PAZ GALELA MARINAY, et al. ................... 184
REPUBLIC OF THE PHILIPPINES vs. MERLINDA L. OLAYBAR ............. 186
H. Prerogative Writs .......................................................................................... 187
DANIEL MASANGKAY TAPUZ et al. vs. HONORABLE JUDGE ELMO DEL
ROSARIO, et al. ................................................................................................. 187
ARMANDO Q. CANLAS, et al. vs. NAPICO HOMEOWNERS ASSN., I
XIII, INC., et al. .................................................................................................. 189
P/SUPT. FELIXBERTO CASTILLO et al. vs. DR. AMANDA T. CRUZ,
NIXON T. CRUZ, and FERDINAND T. CRUZ ............................................... 190
GEN. AVELINO I. RAZON vs. MARY JEAN B. TAGITIS............................ 191
IN THE MATTER OF THE PETITION FOR THE WRIT OF AMPARO AND
THE WRIT OF HABEAS DATA IN FAVOR OF MELISSA C. ROXAS,
MELISSA C. ROXAS vs. GLORIA MACAPAGAL-ARROYO et al. ............. 192
EDITA T. BURGOS vs. GEN. HERMOGENES ESPERON et al. ................... 194
MARICRIS D. DOLOT vs. HON. RAMON PAJE (DENR) ............................. 196

Page | 8

Compilation of Digested Cases for Remedial Law Review II


by: K. M. T. BUELA
EVIDENCE .............................................................................................................. 199
A. Preliminary Consideration ........................................................................... 199
ONG CHIA vs. REPUBLIC OF THE PHILIPPINES and THE COURT OF
APPEALS ........................................................................................................... 199
CECILIA ZULUETA vs. COURT OF APPEALS and ALFREDO MARTIN . 200
PEOPLE OF THE PHILIPPINES vs. JOEL YATAR alias "KAWIT ............... 201
NENA LAZALITA* TATING vs. FELICIDAD TATING MARCELLA, et al.
............................................................................................................................ 202
PEOPLE OF THE PHILIPPINES vs. RODRIGO SALAFRANCA Y BELLO 203
SCC CHEMICALS CORPORATION vs. THE HONORABLE COURT OF
APPEALS, et al. ................................................................................................. 205
B. What Need Not Be Proved ........................................................................... 206
LANDBANK OF THE PHILIPPINES vs. SPOUSES VICENTE BANAL and
LEONIDAS ARENAS-BANAL ........................................................................ 206
THE PEOPLE OF THE PHILIPPINES, plaintiff-appellee, vs. JAILON KULAIS
et al. .................................................................................................................... 207
MENANDRO B. LAUREANO vs. COURT OF APPEALS AND SINGAPORE
AIRLINES LIMITED......................................................................................... 208
CASAN MACODE MAQUILING vs. COMMISSION ON ELECTIONS, et al.
............................................................................................................................ 209
PEOPLE OF THE PHILIPPINES vs. KHADDAFY JANJALANI ................... 209
REPUBLIC OF THE PHILIPPINES vs. SANDIGANBAYAN et al. ............... 211
C. Rules of Admissibility ................................................................................... 212
Object/Real Evidence
JUNIE MALILLIN Y. LOPEZ vs. PEOPLE OF THE PHILIPPINES .............. 212
PEOPLE OF THE PHILIPPINES vs. FELIMON PAGADUAN y TAMAYO . 214
NARCISO SALAS vs. ANNABELLE MATUSALEM .................................... 215
PEOPLE OF THE PHILIPPINES vs. RUPER POSING Y ALAYON ............. 217
PEOPLE OF THE PHILIPPINES vs. ASIR GANI y ALIH and NORMINA
GANI y GALOS ................................................................................................. 219
Documentary Evidence
CITIBANK, N.A. MASTERCARD vs. EFREN S. TEODORO ....................... 220
WILGEN LOON et al. vs. POWER MASTER, INC.et al. ................................ 221
Page | 9

Compilation of Digested Cases for Remedial Law Review II


by: K. M. T. BUELA
THERESITA DIMAGUILA et al. vs. JOSE and SONIA A. MONTEIRO ....... 223
RAFAEL S. ORTAEZ vs. THE COURT OF APPEALS, et al. ...................... 224
LAPULAPU FOUNDATION, INC. and ELIAS Q. TAN vs. COURT OF
APPEALS and ALLIED BANKING CORP. ..................................................... 225
MODESTO LEOVERAS vs. CASIMERO VALDEZ ....................................... 227
HEIRS OF LOURDES SAEZ SABANPAN vs. ALBERTO C. COMORPOSA et
al. ........................................................................................................................ 228
ELLERY MARCH G. TORRES vs. PHILIPPINE AMUSEMENT and GAMING
CORPORATION (PAGCOR) ............................................................................ 229
RUSTAN ANG y PASCUA vs. THE HONORABLE COURT OF APPEALS
and IRISH SAGUD ............................................................................................ 231
Testimonial Evidence
LUISA NAVARRO MARCOS* vs. THE HEIRS OF THE LATE DR. ANDRES
NAVARRO, JR .................................................................................................. 232
PEOPLE OF THE PHILIPPINES vs. SALVADOR GOLIMLIM @ "BADONG"
............................................................................................................................ 233
MAXIMO ALVAREZ vs. SUSAN RAMIREZ ................................................. 234
PEOPLE OF THE PHILIPPINES vs. HON. MARIANO C. CASTAEDA, JR.
and BENJAMIN F. MANALOTO ..................................................................... 235
ENRIQUE RAZON vs. INTERMEDIATE APPELLATE COURT and
VICENTE B. CHUIDIAN .................................................................................. 236
LILIBETH SUNGA-CHAN and CECILIA SUNGA vs. LAMBERTO T. CHUA
............................................................................................................................ 238
TERESITA P. BORDALBA vs. COURT OF APPEALS, HEIRS OF NICANOR
JAYME ............................................................................................................... 240
JOSIELENE LARA CHAN vs. JOHNNY T. CHAN ........................................ 241
JUDGE UBALDINO A. LACUROM v. ATTY. ELLIS F. JACOBA and ATTY.
OLIVIA VELASCO-JACOBA .......................................................................... 242
CLARITA J. SAMALA vs. ATTY. LUCIANO D. VALENCIA ...................... 243
COMMISSIONER JOSE T. ALMONTE et al. vs. HONORABLE CONRADO
M. VASQUEZ and CONCERNED CITIZENS ................................................. 244
OSCAR CONSTANTINO et al. vs. HEIRS OF CONSTANTINO ................... 246
CONRADO C. DOLDOL vs. PEOPLE OF THE PHILIPPINES and THE
HONORABLE COURT OF APPEALS............................................................. 247
Page | 10

Compilation of Digested Cases for Remedial Law Review II


by: K. M. T. BUELA

JOSUE R. LADIANA, Petitioner, v. PEOPLE OF THE PHILIPPINES ........... 248


THE PEOPLE OF THE PHILIPPINES vs. FELICIANO ULIT y TAMPOY ... 249
PEOPLE OF THE PHILIPPINES vs. BENJAMIN SAYABOC y SEGUBA et al.
............................................................................................................................ 250
CARLOS L. TANENGGEE vs. PEOPLE OF THE PHILIPPINES .................. 252
PEOPLE OF THE PHILIPPINES vs. RAUL SANTOS y NARCISO .............. 253
PEOPLE OF THE PHILIPPINES vs. ALFREDO NARDO y ROSALES ........ 254
REPUBLIC OF THE PHILIPPINES vs. HEIRS OF FELIPE ALEJAGA SR. . 255
ANNA LERIMA PATULA, vs. PEOPLE OF THE PHILIPPINES .................. 256
PEOPLE OF THE PHILIPPINES vs. CESARIO MONTAEZ ....................... 258
PEOPLE OF THE PHILIPPINES, vs. THEODORE BERNAL et al. ............... 259
CORAZON DEZOLLER TISON and RENE R. DEZOLLER vs. COURT OF
APPEALS and TEODORA DOMINGO ............................................................ 260
FRANCISCO L. JISON vs. COURT OF APPEALS and MONINA JISON ..... 262
PEOPLE OF THE PHILIPPINES v. FRANK LOBRIGAS et al. ...................... 264
PEOPLE OF THE PHILIPPINES vs. GILBERTO VILLARICO et al. ............ 265
PHILIPPINE AIRLINES, INC. vs. JAIME J. RAMOS et al. ............................ 267
RUDY LAO vs. STANDARD INSURANCE CO., INC. .................................. 268
MEYNARDO SABILI, PETITIONER, vs. COMMISSION ON ELECTIONS
AND FLORENCIO LIBREA ............................................................................. 269
Spouses ANTONIO and LORNA QUISUMBING vs. MANILA ELECTRIC
COMPANY ........................................................................................................ 270
PEOPLE OF THE PHILIPPINES vs. LANIE ORTIZ-MIYAKE ..................... 272
HARRY L. GO, et al. vs. THE PEOPLE OF THE PHILIPPINES and
HIGHDONE COMPANY, LTD., et al............................................................... 273
PEOPLE OF THE PHILIPPINES vs. EDWIN IBAEZ Y ALBANTE and
ALFREDO NULLA Y IBAEZ ........................................................................ 274
PEOPLE OF THE PHILIPPINES vs. PO2 ALBERT ABRIOL et al. ............... 275
LAURA and ERIBERTO BAUTISTA vs. HON. COURT OF APPEALS and
FERNANDO MORELOS .................................................................................. 276
BOBBY ABEL AVELINO Y BULAWAN vs. PEOPLE OF THE
PHILIPPINES ..................................................................................................... 277
PEOPLE OF THE PHILIPPINES vs. EMILIANO DURANAN ....................... 278
Page | 11

Compilation of Digested Cases for Remedial Law Review II


by: K. M. T. BUELA

PEOPLE OF THE PHILIPPINES vs. RAFAEL DIOPITA y GUZMAN ......... 279


D. Burden of Proof and Presumptions ............................................................. 280
FAR EAST BANK & TRUST COMPANY (FEBTC) vs. ROBERT MAR
CHANTE ............................................................................................................ 280
IBAAN RURAL BANK INC. vs. THE COURT OF APPEALS and MR. and
MRS. RAMON TARNATE ............................................................................... 281
SPOUSES REYNALDO ALCARAZ and ESMERALDA ALCARAZ PEDRO
M. TANGGA-AN et al. ...................................................................................... 283
HOSPICIO D. ROSAROSO et al. vs. LUCILA LABORTE SORIA et al. ....... 284
HEIRS OF CIPRIANO TRAZONA vs. HEIRS OF DIONISIO CAADA ..... 285
PEOPLE OF THE PHILIPPINES vs. ROBERTO PADRIGONE ..................... 287
METROPOLITAN BANK & TRUST COMPANY vs. COURT OF APPEALS
and G.T.P. DEVELOPMENT CORPORATION ............................................... 288
FILOMENA G. DELOS SANTOS vs. COMMISSION ON AUDIT ................ 289
PEOPLE OF THE PHILIPPINES vs. HADJI SOCOR CADIDIA .................... 290
PEOPLE OF THE PHILIPPINES vs. JESUS EDUALINO............................... 291
E. Presentation of Evidence .............................................................................. 293
PEOPLE OF THE PHILIPPINES vs. LEONARDO FABRE y VICENTE....... 293
PEOPLE OF THE PHILIPPINES vs. JESUS PEREZ y SEBUNGA ................ 294
PEOPLE OF THE PHILIPPINES vs. JAIME CASTILLANO, SR. et al. ......... 295
PEOPLE OF THE PHILIPPINES vs. ANTONIO PLASENCIA y
DESAMPARADO .............................................................................................. 296
ROSELLA D. CANQUE vs. THE COURT OF APPEALS and SOCOR
CONSTRUCTION CORPORATION ................................................................ 297
YASUO IWASAWA vs. FELISA CUSTODIO GANGAN AND THE LOCAL
CIVIL REGISTRAR OF PASAY CITY ............................................................ 298
ASIAN TERMINALS, INC. vs. PHILAM INSURANCE CO., INC. ............... 300
LOMISES ALUDOS, deceased, substituted by FLORA ALUDOS vs. JOHNNY
M. SUERTE........................................................................................................ 301
WESTMONT INVESTMENT CORPORATION vs. AMOS P. FRANCIA, JR. et
al. ........................................................................................................................ 303
F. Weight and Sufficiency of Evidence ............................................................ 304
PEOPLE OF THE PHILIPPINES vs. DELFIN CALISO .................................. 304
PEOPLE OF THE PHILIPPINE vs. FELIMON PATENTES y ZAMORA ...... 306
SUPREME COURT vs. EDDIE V. DELGADO et al. ....................................... 307
Page | 12

Compilation of Digested Cases for Remedial Law Review II


by: K. M. T. BUELA
GOVERNMENT OF HONG KONG SPECIAL ADMINISTRATIVE REGION
vs. HON. FELIXBERTO T. OLALIA, JR. and JUAN ANTONIO MUOZ ... 308
PEOPLE OF THE PHILIPPINES vs. ALFONSO FONTANILLA y OBALDO
............................................................................................................................ 310
PHILIPPINE COMMERCIAL INTERNATIONAL BANK vs. ANTONIO B.
BALMACEDA and ROLANDO N. RAMOS ................................................... 311
DRA. LEILA A. DELA LLANA vs. REBECCA BIONG ................................ 312
ZACARIA A. CANDAO, et al. vs. PEOPLE OF THE PHILIPPINES AND
SANDIGANBAYAN ......................................................................................... 314
OFFICE OF THE OMBUDSMAN vs. ANTONIO T. REYES ......................... 315
XAVIER C. RAMOS vs. BPI FAMILY SAVINGS BANK INC. and/or
ALFONSO L. SALCEDO, JR. ........................................................................... 316
JESSE U. LUCAS vs. JESUS S. LUCAS .......................................................... 318
PHILIPPINE NATIONAL BANK vs. AMELIO TRIA and JOHN DOE ......... 319
RUBEN DEL CASTILLO vs. PEOPLE OF THE PHILIPPINES ..................... 320
PEOPLE OF THE PHILIPPINES vs. ALBERTO ANTICAMARA y CABILLO
et al. .................................................................................................................... 322
PEOPLE OF THE PHILIPPINES, vs. EFREN DEOCAMPO et al. .................. 323

Page | 13

Compilation of Digested Cases for Remedial Law Review II


by: K. M. T. BUELA
PROVISIONAL REMEDIES
A. Preliminary Attachment (Rule 57)
ANITA MANGILA vs. COURT OF APPEALS and LORETA GUINA
G.R. No. 125027, August 12, 2002
FACTS:
As petitioner Mangila failed to pay private respondent shipping charges, private
respondent filed an action for collection of sum of money. Sheriffs return shown
that summons was not served on the petitioner as the latter had transferred her
residence and left the Philippines for Guam. Construing petitioners departure from
the Philippines as done with intent to defraud her creditors, private respondent filed
a Motion for Preliminary Attachment which the Trial court granted. Subsequently,
the writ together with the Notice of Levy with the Order, Affidavit and Bond, was
served on petitioners household.
The trial court granted the Motion to Discharge Attachment upon filing of
petitioners counter-bond. Trial court rendered a decision against the petitioner. On
appeal, the CA affirm the trial courts ruling. Petitioner filed a petition for review
on certiorari with the SC. Petitioners arguments rests on the question of the validity
of the writ of attachment. Because of failure to serve summons on her before or
simultaneously with the writs implementation, petitioner claims that the trial court
had not acquired jurisdiction over her person and thus the service of the writ is void.
Private respondent, nevertheless, claims that the prior or contemporaneous service
of summons contemplated in Section 5 of Rule 57 provides for exceptions. Among
such exceptions are "where the summons could not be served personally or by
substituted service despite diligent efforts or where the defendant is a resident
temporarily absent therefrom x x x.
ISSUE: Whether or not lack of prior or contemporaneous service of summons with
the writ of preliminary attachment to a defendant who resides in the Philippines but
is temporarily absent in action for collection for sum of money renders the
attachment invalid and the issuing court without jurisdiction.
HELD:
The answer is in the affirmative. The preliminary writ of attachment must be served
after or simultaneous with the service of summons on the defendant whether by
personal service, substituted service or by publication as warranted by the
circumstances of the case. The subsequent service of summons does not confer a
retroactive acquisition of jurisdiction over her person because the law does not
allow for retroactivity of a belated service.
Page | 14

Compilation of Digested Cases for Remedial Law Review II


by: K. M. T. BUELA

The rules provide for certain remedies in cases where personal service could not be
effected on a party. Section 14, Rule 14 of the Rules of Court provides that
whenever the defendants "whereabouts are unknown and cannot be ascertained by
diligent inquiry, service may, by leave of court, be effected upon him by publication
in a newspaper of general circulation x x x." Thus, if petitioners whereabouts could
not be ascertained after the sheriff had served the summons at her given address,
then respondent could have immediately asked the court for service of summons by
publication on petitioner.
Private respondent never showed that she effected substituted service on petitioner
after her personal service failed. Likewise, if it were true that private respondent
could not ascertain the whereabouts of petitioner after a diligent inquiry, still she
had some other recourse under the Rules of Civil Procedure. Moreover, as private
respondent also claims that petitioner was abroad at the time of the service of
summons, this made petitioner a resident who is temporarily out of the country.
This is the exact situation contemplated in Section 16, Rule 14 of the Rules of Civil
Procedure, providing for service of summons by publication.
NB: The SCs disquisition on the required prior or contemporaneous service of
summons never touch the private respondents argument as to exceptions to the
compliance of the said requirement.

VICENTE B. CHUIDIAN vs. SANDIGANBAYAN


G.R. No. 139941, January 19, 2001
FACTS:
Petitioner Chuidian was alleged to be a dummy of the Marcoses in several
companies, one of which is the Asian Reliability Company, Incorporated (ARCI)
which is 98% owned by him. PHILGUARANTEE granted ARCI a loan guarantee
of US$25,000,000.00. In June 1985, Philguarantee sued Chuidian before a US
Court for violating of the terms of the loan as Chuidian not only defaulted in
payment, but also misused the funds by investing them in US corporations for his
personal benefit.
During the pendency of the suit, Philguarantee entered into a compromise
agreement with Chuidian whereby the latter shall assign and surrender title to all
his companies in favor of the Philippine government and the former shall absolve
Chuidian from all civil and criminal liability. It was further stipulated that the
Philippine government shall pay Chuidian the amount of US$5,300,000.00. Initial
payment of US$700,000.00 was actually received by Chuidian. While for the

Page | 15

Compilation of Digested Cases for Remedial Law Review II


by: K. M. T. BUELA
remaining balance of US$4,600,000.00 was to be paid through an irrevocable Letter
of Credit (L/C).
On July 30, 1987, the government filed before the Sandiganbayan civil suit against
Chuidian seeking the reconveyance, reversion, accounting and restitution of all
forms of wealth allegedly procured illegally and stashed away by the defendants.
While the case was pending, the Republic of the Philippines filed a motion for
issuance of a writ of attachment over the L/C, on the ground that Chuidian
embezzled or fraudulently misapplied the funds of ARCI acting in a fiduciary
capacity, justifying issuance of the writ under Section 1(b), Rule 57 of the Rules of
Court. Sandiganbayan finding that there was a prima facie case of fraud committed
issued a writ of attachment.
Four (4) years after the issuance of the order of attachment, Chuidian filed a motion
to lift the attachment based on the following grounds: (1) First, he had returned to
the Philippines; hence, the Sandiganbayan's "most potent ground" for the issuance
of the writ of preliminary attachment no longer existed. Since his absence in the
past was the very foundation of the Sandiganbayan's writ of preliminary attachment,
his presence in the country warrants the immediate lifting thereof; and (2) Second,
there was no evidence at all of initial fraud or subsequent concealment except for
the affidavit submitted by the PCGG Chairman citing mere "belief and information"
and "not on knowledge of the facts."
The Sandiganbayan, however, denied Chuidian's motion for reconsideration of the
denial of his motion to lift attachment. Hence, the instant petition for certiorari
contending that the respondent Sandiganbayan committed grave abuse of discretion
amounting to lack or excess of jurisdiction.
ISSUES:
(1) Whether or not the reappearance of a previously absent person against whom
writ of attachment had been issued is a ground for lifting the writ of preliminary
attachment.
(2) Whether or not a writ of attachment may be discharged when the fraud which is
the ground for its issuance is not present.
HELD:
(1)
No. To quash a writ of preliminary attachment, the petitioner has two (2) options,
namely: (1) to file a counterbond in accordance with Rule 57, Section 12; or (2) to
quash the attachment on the ground that it was irregularly or improvidently issued,
as provided for in Section 13 of the same Rule.

Page | 16

Compilation of Digested Cases for Remedial Law Review II


by: K. M. T. BUELA
It would appear that petitioner chose the latter because the grounds he raised assail
the propriety of the issuance of the writ of attachment. By his own admission,
however, he repeatedly acknowledged that his justifications to warrant the lifting of
the attachment are facts or events that came to light or took place after the writ of
attachment had already been implemented.
The rule contemplates that the defect must be in the very issuance of the attachment
writ. For instance, the attachment may be discharged under Section 13 of Rule 57
when it is proven that the allegations of the complaint were deceptively framed, or
when the complaint fails to state a cause of action. Supervening events which may
or may not justify the discharge of the writ are not within the purview of this
particular rule.
(2)
The merits of the action in which a writ of preliminary attachment has been issued
are not triable on a motion for dissolution of the attachment, otherwise an applicant
for the lifting of the writ could force a trial of the merits of the case on a mere
motion.
When the preliminary attachment is issued upon a ground which is at the same time
the applicant's cause of action; e.g., "an action for money or property embezzled or
fraudulently misapplied or converted to his own use by a public officer, or an officer
of a corporation, or an attorney, factor, broker, agent, or clerk, in the course of his
employment as such, or by any other person in a fiduciary capacity, or for a willful
violation of duty," or "an action against a party who has been guilty of fraud in
contracting the debt or incurring the obligation upon which the action is brought,"
the defendant is not allowed to file a motion to dissolve the attachment under
Section 13 of Rule 57 by offering to show the falsity of the factual averments in the
plaintiff's application and affidavits on which the writ was based and consequently
that the writ based thereon had been improperly or irregularly issued the reason
being that the hearing on such a motion for dissolution of the writ would be
tantamount to a trial of the merits of the action. In other words, the merits of the
action would be ventilated at a mere hearing of a motion, instead of at the regular
trial.

Page | 17

Compilation of Digested Cases for Remedial Law Review II


by: K. M. T. BUELA
ALEJANDRO NG WEE vs. MANUEL TANKIANSEE
G.R. No. 171124, February 13, 2008
FACTS:
Wincorp extended a loan to Hotticks Corp. When the latter defaulted, a collection
suit was instituted against it. The suit ended in a settlement with the latters
president Virata assuming the liabilities of Hotticks as surety. Thereafter, Wincorp
extended another loan to Power Merge, owned by Virata in the amount equal to the
money placement/investment of the petitioner Wee.
Wee then filed an action for damages against Virata, Power Merge and Wincorp
officers, including herein respondent. The complaint alleged that Wincorp officers
connived with Power Merge and Virata in transferring petitioner's money
placements with it (Wincorp) without his knowledge and consent to the loan
account of Power Merge through an agreement that virtually freed the Virata of any
liability.
Upon application by the Petitioner, a writ of preliminary attachment was issued with
50M bond by the petitioner. The writ was assailed by the defendants but their
opposition was denied by the trial court. Denial was further affirmed by the CA and
SC. Later on, respondent again filed before the trial court another Motion to
Discharge Attachment. Ruling that the grounds raised were already passed upon by
it in the previous orders affirmed by the CA and SC, trial court denied the motion.
Respondent filed a certiorari petition before the CA. The appellate court rendered a
Decision reversing and setting aside the aforementioned orders of the trial court and
lifting Writ of Preliminary Attachment. Thus petitioner filed a petitioner for review
on certiorari.
Respondent counters, among others, that the general and sweeping allegation of
fraud against respondent in petitioner's affidavit-respondent as an officer and
director of Wincorp allegedly connived with the other defendants to defraud
petitioner-is not sufficient basis for the trial court to order the attachment of
respondent's properties. A writ of attachment can only be granted on concrete and
specific grounds and not on general averments quoting perfunctorily the words of
the Rules. Connivance cannot also be based on mere association but must be
particularly alleged and established as a fact.
ISSUE: Whether or not a writ of attachment may be discharged on the ground that
the fraud alleged against the person to whom attachment was issued is a general
allegation of fraud when the basis of the main case is also fraud.
HELD:

Page | 18

Compilation of Digested Cases for Remedial Law Review II


by: K. M. T. BUELA
The answer is in the affirmative. For a writ of attachment to issue under this rule,
the applicant must sufficiently show the factual circumstances of the alleged fraud
because fraudulent intent cannot be inferred from the debtor's mere non-payment of
the debt or failure to comply with his obligation. The applicant must then be able to
demonstrate that the debtor has intended to defraud the creditor.
In the instant case, petitioner's October 12, 2000 Affidavit is bereft of any factual
statement that respondent committed a fraud. The affidavit narrated only the alleged
fraudulent transaction between Wincorp and Virata and/or Power Merge.
As to the participation of respondent in the said transaction, the affidavit merely
states that respondent, an officer and director of Wincorp, connived with the other
defendants in the civil case to defraud petitioner of his money placements. No other
factual averment or circumstance details how respondent committed a fraud or how
he connived with the other defendants to commit a fraud in the transaction sued
upon. In other words, petitioner has not shown any specific act or deed to support
the allegation that respondent is guilty of fraud.
The affidavit, being the foundation of the writ, must contain such particulars
as to how the fraud imputed to respondent was committed for the court to
decide whether or not to issue the writ. Absent any statement of other factual
circumstances to show that respondent, at the time of contracting the obligation,
had a preconceived plan or intention not to pay, or without any showing of how
respondent committed the alleged fraud, the general averment in the affidavit that
respondent is an officer and director of Wincorp who allegedly connived with the
other defendants to commit a fraud, is insufficient to support the issuance of a writ
of preliminary attachment.

SOFIA TORRES et al. vs. NICANOR SATSATIN et al.


G.R. No. 166759, November 25, 2009
FACTS:
Petitioners authorized Nicanor, through a Special Power of Attorney, to negotiate
for the sale of their c0-owned properties. Nicanor sold the land to Solar Resources,
Inc. (Solar). Petitioners claimed that Solar Resources, Inc. (Solar) has already paid
the entire purchase price of P35,000,000.00 to Nicanor. However, Nicanor only
remitted the total amount of P9,000,000.00, leaving an unremitted balance of
P19,000,000.00. Despite repeated verbal and written demands, Nicanor failed to
remit to them the balance.
Consequently, petitioners filed a Complaint for sum of money and damages, against
Nicanor et al. Petitioners filed an Ex-Parte Motion for the Issuance of a Writ of
Page | 19

Compilation of Digested Cases for Remedial Law Review II


by: K. M. T. BUELA
Attachment which was granted by the trial court. Respondents filed their answer
with a Motion to Discharge Writ of Attachment anchored on the ground that the
bond issued in favor of the petitioners was defective, because the bonding company
failed to obtain the proper clearance that it can transact business with the RTC of
Dasmarias, Cavite. They added that the various clearances which were issued in
favor of the bonding company were applicable only in the courts of the cities of
Pasay, Pasig, Manila, and Makati, but not in the RTC, Imus, Cavite.
ISSUE: Whether or not a writ of attachment may be discharged on the ground that
company who issued the applicants bond is not among those qualified to transact
business with the issuing court.
HELD:
The answer is in the affirmative. In accepting a surety bond, it is necessary that all
the requisites for its approval are met; otherwise, the bond should be rejected.
Every bond should be accompanied by a clearance from the Supreme Court
showing that the company concerned is qualified to transact business which is valid
only for thirty (30) days from the date of its issuance. However, it is apparent that
the Certification issued by the Office of the Court Administrator (OCA) at the time
the bond was issued would clearly show that the bonds offered by Western Guaranty
Corporation may be accepted only in the RTCs of the cities of Makati, Pasay, and
Pasig. Therefore, the surety bond issued by the bonding company should not have
been accepted by the RTC of Dasmarias, Branch 90, since the certification secured
by the bonding company from the OCA at the time of the issuance of the bond
certified that it may only be accepted in the above-mentioned cities. Thus, the trial
court acted with grave abuse of discretion amounting to lack of or in excess of
jurisdiction when it issued the writ of attachment founded on the said bond.

B. Preliminary Injunction (Rule 58)


TERESITA V. IDOLOR vs. HON. COURT OF APPEALS et. al
G.R. No. 141853, February 7, 2001
FACTS:
Petitioner Idolor obtained a loan from Gumersindo De Guzman secured by a Real
Estate Mortgage with right of extrajudicial foreclosure.
Upon default of Idolor, Gumersindo De Guzman filed an extra judicial foreclosure
of the real estate mortgage pursuant to the parties agreement set forth in the real
estate mortgage. On May 23, 1997, the mortgaged property was sold in a public

Page | 20

Compilation of Digested Cases for Remedial Law Review II


by: K. M. T. BUELA
auction to respondent Gumersindo, as the highest bidder and consequently, the
Sheriff's Certificate of Sale was registered.
Petitioner filed a complaint for annulment of Sheriff's Certificate of Sale with prayer
for the issuance of a temporary restraining order (TRO) and a writ of preliminary
injunction against private respondents. A temporary restraining order was issued by
the trial court. On July 28, 1998, the trial court issued a writ of preliminary
injunction enjoining private respondents, the Deputy Sheriffs and the Registry of
Deeds of Quezon City from causing the issuance of a final deed of sale and
consolidation of ownership of the subject property in favor of the De Guzman
spouses.
ISSUE: Whether or not Preliminary Injunction may be issued in favor of a
mortgagor who filed an action to annul the mortgage agreement and public auction
to enjoin the issuance of the final deed of sale and consolidation of ownership.
HELD:
No. Injunction is a preservative remedy aimed at protecting substantive rights and
interests.6 Before an injunction can be issued, it is essential that the following
requisites be present: 1) there must be aright in esse or the existence of a right to be
protected; 2) the act against which the injunction is to be directed is a violation of
such right. Hence the existence of a right violated, is a prerequisite to the granting
of an injunction. Injunction is not designed to protect contingent or future rights.
Failure to establish either the existence of a clear and positive right which should
be judicially protected through the writ of injunction or that the defendant has
committed or has attempted to commit any act which has endangered or tends to
endanger the existence of said right, is a sufficient ground for denying the
injunction.
In the instant case, petitioner has no more proprietary right to speak of over the
foreclosed property to entitle her to the issuance of a writ of injunction. Petitioner
had one year from the registration of the sheriff's sale to redeem the property but
she failed to exercise her right on or before June 23, 1998, thus spouses de Guzman
are now entitled to a conveyance and possession of the foreclosed property. When
petitioner filed her complaint for annulment of sheriff's sale her right of redemption
had already expired on June 23, 1998, i.e. two (2) days before the filing of the
complaint. It is always a ground for denying injunction that the party seeking it has
insufficient title or interest to sustain it, and no claim to the ultimate relief sought in other words, that she shows no equity. The possibility of irreparable damage
without proof of actual existing right is not aground for an injunction.

Page | 21

Compilation of Digested Cases for Remedial Law Review II


by: K. M. T. BUELA
RIMEO S. GUSTILO vs. HON. RICARDO S. REAL, SR.
A.M. No. MTJ-00-1250, February 28, 2001
FACTS:
Complainant was proclaimed duly elected punong barangay. Libo-on, a contender
of the complainant in the afore-mentioned election filed an election protest with
MCTC and sought the recounting of ballots in two precincts, preliminary
prohibitory injunction, and damages. MCTC Judge issued a TRO without sending
a notice of hearing to the complainant.
On May 30, 1997, complainant filed a petition for certiorari before the Regional
Trial Court. RTC lifted the TRO issued by respondent and declared as null and void
the order nullifying complainant's proclamation as duly elected punong barangay.
On June 11, 1997, respondent issued a second TRO "to maintain the status quo
between the contending parties." Complainant argues that by issuing the second
TRO, respondent reversed the order of the RTC.
Thus, Rimeo S. Gustilo charged respondent Judge Ricardo S. Real with gross
misconduct, gross incompetence, gross ignorance of the law, and violation of the
Anti-Graft and Corrupt Practices. In his comment, Respondent admits that the
annulment of complainant's proclamation, was outside the jurisdiction of his court.
But since the COMELEC ignored Libo-on's petition for correction of erroneous
tabulation and Libo-on had no other remedy under the law, he was constrained to
annul complainant's proclamation, which from the very beginning was illegal.
ISSUE: Whether or not injunction may be issued without hearing on the ground
that the act sought to be enjoined is void ab initio.
HELD:
The answer is in the negative. Supreme Court Administrative Circular No. 20-95
provides: The application for a TRO shall be acted upon only after all parties are
heard in a summary hearing conducted within twenty-four (24) hours after the
records are transmitted to the branch selected by raffle. The records shall be
transmitted immediately after raffle.
The foregoing clearly show that whenever an application for a TRO is filed, the
court may act on the application only after all parties have been notified and heard
in a summary hearing. In other words, a summary hearing may not be dispensed
with. In the instant case, respondent admits that he issued the injunctive writ sought
on May 29, 1997 after receiving the applicant's evidence ex parte. His failure to
abide by Administrative Circular No. 20-95 in issuing the first TRO is grave abuse
of authority, misconduct, and conduct prejudicial to the proper administration of
justice.
Page | 22

Compilation of Digested Cases for Remedial Law Review II


by: K. M. T. BUELA

Respondent admits that his court was not vested with the power or jurisdiction to
annul the proclamation, but seeks to justify his action on the ground that the
proclamation was void ab initio. In so doing, respondent wantonly usurped a power
exclusively vested by law in the COMELEC.

MICHAEL J. LAGROSAS vs. BRISTOL-MYERS SQUIBB (PHIL.) et al.


G.R. No. 168637, September 12, 2008
FACTS:
Lagrosas then filed a complaint for illegal dismissal against Bristol-Myers Squibb
(Phil.), Inc./Mead Johnson Phil. Labor Arbiter declared that the dismissal is illegal.
On appeal, NLRC set aside the Decision of Labor Arbiter. Lagrosas moved for
reconsideration and NLRC issued a Resolution reversing its earlier ruling.
Later, Labor Arbiter Hernandez issued a writ of execution. Bristol-Myers moved to
quash the writ of execution contending that it timely filed a petition for certiorari
with the Court of Appeals. The appellate court gave due course to Bristol-Myers
petition and issued a temporary restraining order (TRO) enjoining the enforcement
of the writ of execution and notices of garnishment. Upon the expiration of the
TRO, the appellate court issued a writ of preliminary injunction. Eventually, the
Appellate Court rendered a decision reversing the NLRC Resolution.
Bristol-Myers moved to release the TRO cash bond and injunction cash bond in
view of the Decision dated January 28, 2005. On August 12, 2005, the appellate
court denied the motion as premature since the decision is not yet final and
executory due to Lagrosas appeal to this Court.
ISSUE: Whether or not bond posted by the winning party before the appellate court
for the issuance of TRO and injunction may be released on the ground that the
decision of the said court is not yet final and executory.
HELD:
The answer is in the affirmative. The injunction bond is intended as a security for
damages in case it is finally decided that the injunction ought not to have been
granted. Its principal purpose is to protect the enjoined party against loss or damage
by reason of the injunction, and the bond is usually conditioned accordingly.
In this case, the CA issued the writ of preliminary injunction to enjoin the
implementation of the writ of execution against Bristol-Myers. When the CA
rendered its decision, it was in favor or Bristol-myers. It also ordered the discharge

Page | 23

Compilation of Digested Cases for Remedial Law Review II


by: K. M. T. BUELA
of the TRO cash bond and injunction cash bond. Thus, both conditions of the writ
of preliminary injunction were satisfied.
Notably, the appellate court ruled that Lagrosas had no right to the monetary awards
granted by the labor arbiter and the NLRC, and that the implementation of the writ
of execution and notices of garnishment was properly enjoined. This in effect
amounted to a finding that Lagrosas did not sustain any damage by reason of the
injunction. To reiterate, the injunction bond is intended to protect Lagrosas against
loss or damage by reason of the injunction only. Contrary to Lagrosas claim, it is
not a security for the judgment award by the labor arbiter. Considering the
foregoing, we hold that the appellate court erred in disallowing the discharge and
release of the injunction cash bond.

NELSON JENOSA et al. vs. REV. FR. JOSE RENE C. DELARIARTE,


O.S.A. et al.
G.R. No. 172138, September 8, 2010
FACTS:
Petitioners are and some of the students (and their parents) of the University who
were caught engaging in hazing outside the school premises.
Dialogues and consultations were conducted among the school authorities, the
apprehended students and their parents. During the 28 November 2002 meeting, the
parties agreed that, instead of the possibility of being charged and found guilty of
hazing, the students who participated in the hazing incident as initiators, including
petitioner students, would just transfer to another school, while those who
participated as neophytes would be suspended for one month. The parents of the
apprehended students, including petitioners, affixed their signatures to the minutes
of the meeting to signify their conformity.5 In view of the agreement, the University
did not anymore convene the Committee on Student Discipline (COSD) to
investigate the hazing incident.
On 3 January 2003, petitioners filed a complaint for injunction and damages.
Petitioners assailed the Principals decision to order the immediate transfer of
petitioner students as a violation of their right to due process because the COSD
was not convened.
ISSUE: Whether or not, injunction may be issued to enjoined an act previously
agreed upon between the applicant and the party against whom the injunction is
sought to be issued.
HELD:
Page | 24

Compilation of Digested Cases for Remedial Law Review II


by: K. M. T. BUELA
The answer is no. Since injunction is the strong arm of equity, he who must apply
for it must come with equity or with clean hands. This is so because among the
maxims of equity are (1) he who seeks equity must do equity, and (2) he who comes
into equity must come with clean hands. The latter is a frequently stated maxim
which is also expressed in the principle that he who has done inequity shall not have
equity. It signifies that a litigant may be denied relief by a court of equity on the
ground that his conduct has been inequitable, unfair and dishonest, or fraudulent, or
deceitful as to the controversy in issue.
Here, petitioners, having reneged on their agreement without any justifiable reason,
come to court with unclean hands. This Court may deny a litigant relief if his
conduct has been inequitable, unfair and dishonest as to the controversy in issue.

SOLID BUILDERS, INC. and MEDINA FOODS INDUSTRIES, INC. vs.


CHINA BANKING CORPORATION (CBC)
G.R. No. 179665, April 3, 2013
FACTS:
CBC granted several loans to Solid Builders (SBI). To secure the loans, Medina
Foods (MFII) executed in CBCs favor several surety agreements and contracts of
real estate mortgage over parcels of land.
On October 5, 2000, claiming that the interests, penalties and charges imposed by
CBC were iniquitous and unconscionable and to enjoin CBC from initiating
foreclosure proceedings, SBI and MFII filed a Complaint "To Compel Execution
of Contract and for Performance and Damages, With Prayer for Writ of Preliminary
Injunction and Ex-Parte Temporary Restraining Order." Trial Court issued an order
granting the application for writ of preliminary injunction.
Aggrieved, CBC filed a Petition for Certiorari with the Court of Appeals where it
claimed that the issuance of writ of preliminary injunction were all issued with
grave abuse of discretion amounting to lack of jurisdiction. CA ruled in favor of
CBC and held that the issuance of a writ of preliminary injunction had no basis as
there were no findings of fact or law which would indicate the existence of any of
the requisites for the grant of an injunctive writ. SBI and MFII filed a motion for
reconsideration but it was denied by the Court of Appeals in a Resolution dated
September 18, 2007. Hence, this petition.
ISSUE: Whether or not a preliminary injunction may be issued to enjoin a creditor
from foreclosing the mortgaged property.
HELD:
Page | 25

Compilation of Digested Cases for Remedial Law Review II


by: K. M. T. BUELA
The answer is in the negative. A writ of preliminary injunction is an extraordinary
event which must be granted only in the face of actual and existing substantial
rights. The duty of the court taking cognizance of a prayer for a writ of preliminary
injunction is to determine whether the requisites necessary for the grant of an
injunction are present in the case before it. In this connection, a writ of preliminary
injunction is issued to preserve the status quo ante, upon the applicants showing of
two important requisite conditions, namely: (1) the right to be protected exists prima
facie, and (2) the acts sought to be enjoined are violative of that right. It must be
proven that the violation sought to be prevented would cause an irreparable
injury
An injury is considered irreparable if it is of such constant and frequent
recurrence that no fair or reasonable redress can be had therefor in a court of
law, or where there is no standard by which their amount can be measured
with reasonable accuracy, that is, it is not susceptible of mathematical
computation. The provisional remedy of preliminary injunction may only be
resorted to when there is a pressing necessity to avoid injurious consequences which
cannot be remedied under any standard of compensation.
In the first place, any injury that SBI and MFII may suffer in case of foreclosure
of the mortgaged properties will be purely monetary and compensable by an
appropriate judgment in a proper case against CBC. Moreover, where there is a
valid cause to foreclose on the mortgages, it cannot be correctly claimed that the
irreparable damage sought to be prevented by the application for preliminary
injunction is the loss of the mortgaged properties to auction sale. The alleged
entitlement of SBI and MFII to the "protection of their properties put up as collateral
for the loans" they procured from CBC is not the kind of irreparable injury
contemplated by law. Foreclosure of mortgaged property is not an irreparable
damage that will merit for the debtor-mortgagor the extraordinary provisional
remedy of preliminary injunction.

SPOUSES SILVESTRE O. PLAZA et al. vs. GUILLERMO LUSTIVA et.


al.
G.R. No. 172909, March 5, 2014
FACTS:
Petitioners filed a Complaint for Injunction, Damages, Attorneys Fees with Prayer
for the Issuance of the Writ of Preliminary Injunction and/or Temporary Restraining
Order against the respondents and the City Government of Butuan. They prayed
that the respondents be enjoined from unlawfully and illegally threatening to take
possession of the subject property. According to the petitioners, they acquired the

Page | 26

Compilation of Digested Cases for Remedial Law Review II


by: K. M. T. BUELA
land from Virginia Tuazon in 1997; Tuazon was the sole bidder and winner in a tax
delinquency sale conducted by the City of Butuan on December 27, 1996.
The RTC found that the auction sale was tainted with irregularity as the bidder was
a government employee disqualified in accordance with Section 89 of the Local
Government Code of 1991 and ordered that prayer for a Writ of Preliminary
Injunction be denied.
Through a petition for review on certiorari under Rule 65, the petitioners challenged
the RTCs order before the CA. While the petition for review on certiorari was
pending before the CA, the petitioners filed an action for specific performance
against the City Government of Butuan. According to the petitioners, they acquired
possession and ownership over the auctioned property when they redeemed it from
Tuazon. The City Government of Butuan must therefore issue them a certificate of
sale. In its October 24, 2005 decision, the CA affirmed the RTCs ruling, found the
petitioners guilty of forum shopping, dismissed the case.
The petitioners filed the present petition for review on certiorari with SC to
challenge the CA rulings.
ISSUE: Whether or not petition for review on certiorari assailing the order denying
the issuance of a preliminary injunction may proceed despite the dismissal of the
main case from which the preliminary injunction was originally prayed for.
HELD:
The answer is in the negative. Upon the dismissal of the main case by the RTC on
August 8, 2013, the question of issuance of the writ of preliminary injunction has
become moot and academic. A case becomes moot and academic when there is no
more issue between the parties or object that can be served in deciding the merits
of the case. Upon the dismissal of the main action, the question of the non-issuance
of a writ of preliminary injunction automatically died with it. A writ of preliminary
injunction is a provisional remedy; it is auxiliary, an adjunct of, and subject to the
determination of the main action. It is deemed lifted upon the dismissal of the main
case, any appeal therefrom notwithstanding.

Page | 27

Compilation of Digested Cases for Remedial Law Review II


by: K. M. T. BUELA
C. Receivership (Rule 59)
SPS. CESAR A. LARROBIS, JR. and VIRGINIA S. LARROBIS vs.
PHILIPPINE VETERANS BANK
G.R. No. 135706, October 1, 2004
FACTS:
Petitioner spouses contracted a monetary loan with respondent Philippine Veterans
Bank secured by a Real Estate Mortgage executed on their lot together with the
improvements thereon. On March 23, 1985, the respondent bank went bankrupt and
was placed under receivership/liquidation by the Central Bank. More than fourteen
years from the time the loan became due and demandable, respondent bank filed a
petition for extrajudicial foreclosure of mortgage of petitioners property. On
October 18, 1995, the property was sold in a public auction by Sheriff Arthur
Cabigon with Philippine Veterans Bank as the lone bidder.
On April 26, 1996, petitioners filed a complaint with the RTC, Cebu City, to declare
the extra-judicial foreclosure and the subsequent sale thereof to respondent bank
null and void. RTC rendered its decision dismissing the complaint. It held that the
defendant banks right to foreclose the mortgaged property prescribes in ten (10)
years but such period was interrupted when it was placed under receivership.
ISSUE: Whether or not the period within which the bank was placed under
receivership and liquidation was a fortuitous event which suspended the running of
the ten-year prescriptive period in bringing actions.
HELD:
The answer is in the negative. One characteristic of a fortuitous event, in a legal
sense and consequently in relations to contract, is that its occurrence must be such
as to render it impossible for a party to fulfill his obligation in a normal manner.
Respondents claims that because of a fortuitous event, it was not able to exercise
its right to foreclose the mortgage on petitioners property; and that since it was
banned from pursuing its business and was placed under receivership from April
25, 1985 until August 1992, it could not foreclose the mortgage on petitioners
property within such period since foreclosure is embraced in the phrase "doing
business," are without merit.
While it is true that foreclosure falls within the broad definition of "doing business.
Foreclosure should not be considered included, however, in the acts prohibited
whenever banks are "prohibited from doing business" during receivership and
liquidation proceedings. Section 29 of the Republic Act No. 265, as amended
known as the Central Bank Act, provides that when a bank is forbidden to do
Page | 28

Compilation of Digested Cases for Remedial Law Review II


by: K. M. T. BUELA
business in the Philippines and placed under receivership, the person
designated as receiver shall immediately take charge of the banks assets and
liabilities, as expeditiously as possible, collect and gather all the assets and
administer the same for the benefit of its creditors, and represent the bank
personally or through counsel as he may retain in all actions or proceedings
for or against the institution, exercising all the powers necessary for these
purposes including, but not limited to, bringing and foreclosing mortgages in
the name of the bank. This is consistent with the purpose of receivership
proceedings, i.e., to receive collectibles and preserve the assets of the bank in
substitution of its former management, and prevent the dissipation of its assets to
the detriment of the creditors of the bank.
There was no legal prohibition imposed upon herein respondent to deter its receiver
and liquidator from performing their obligations under the law. However, the bank
may go after the receiver who is liable to it for any culpable or negligent failure to
collect the assets of such bank and to safeguard its assets.

EVELINA G. CHAVEZ and AIDA CHAVEZ-DELES vs. COURT OF


APPEALS and ATTY. FIDELA Y. VARGAS
G.R. No. 174356, January 20, 2010
FACTS:
Respondent Fidela Y. Vargas owned a five-hectare land. Petitioner Evelina had
been staying in a remote portion of the land. Fidela and Evelina agreed to divide the
gross sales of all products from the land between themselves. Since Fidela was busy
with her law practice, Evelina undertook to hold in trust for Fidela her half of the
profits. Fidela claimed that Evelina had failed to remit her share of the profits and,
despite demand to turn over the administration of the property to Fidela, had refused
to do so. Consequently, Fidela filed a complaint against Evelina for recovery of
possession, rent, and damages with prayer for the immediate appointment of a
receiver
Fidela also filed three estafa cases with the RTC of Olongapo City and a complaint
for dispossession with the Department of Agrarian Reform Adjudication Board
(DARAB) against Evelina and Aida. In all these cases, Fidela asked for the
immediate appointment of a receiver for the property.
ISSUE: Whether or not the filing of an administrative, civil and criminal cases, all
of which with prayer for appointment of receiver constitutes forum shopping.
HELD:

Page | 29

Compilation of Digested Cases for Remedial Law Review II


by: K. M. T. BUELA
The answer is in the negative. By forum shopping, a party initiates two or more
actions in separate tribunals, grounded on the same cause, trusting that one or the
other tribunal would favorably dispose of the matter. The elements of forum
shopping are: (1) identity of parties, or at least such parties as would represent the
same interest in both actions; (2) identity of rights asserted and relief prayed for,
the relief being founded on the same facts; and (3) identity of the two preceding
particulars such that any judgment rendered in the other action will, regardless of
which party is successful, amount to res judicata in the action under consideration.
The above cases are similar only in that they involved the same parties and Fidela
sought the placing of the properties under receivership in all of them. But
receivership is not an action. It is but an auxiliary remedy, a mere incident of the
suit to help achieve its purpose. Consequently, it cannot be said that the grant of
receivership in one case will amount to res judicata on the merits of the other cases.
The grant or denial of this provisional remedy will still depend on the need for it in
the particular action.
NB: A petition for receivership under Section 1(b), Rule 59 of the Rules of Civil
Procedure requires that the property or fund subject of the action is in danger of
being lost, removed, or materially injured, necessitating its protection or
preservation. Its object is the prevention of imminent danger to the property. If the
action does not require such protection or preservation, the remedy is not
receivership. Here Fidelas main gripe is that Evelina and Aida deprived her of her
share of the lands produce. Fidela does not claim that the land has been materially
injured, necessitating its protection and preservation. Hence, receiver may not be
appointed.

ANA MARIA A. KORUGA vs. TEODORO O. ARCENAS et al.


G.R. No. 168332, June 19, 2009
FACTS:
Korugas Complaint charged defendants before the RTC-Makati with violation of
Sections 31 to 34 of the Corporation Code, prohibiting self-dealing and conflict of
interest of directors and officers; invoked her right to inspect the corporations
records under Sections 74 and 75 of the Corporation Code; and prayed for
Receivership and Creation of a Management Committee, pursuant to Rule 59 of the
Rules of Civil Procedure, the Securities Regulation Code, the Interim Rules of
Procedure Governing Intra-Corporate Controversies, the General Banking Law of
2000, and the New Central Bank Act. She accused the directors and officers of
Banco Filipino of engaging in unsafe, unsound, and fraudulent banking practices,
more particularly, acts that violate the prohibition on self-dealing.

Page | 30

Compilation of Digested Cases for Remedial Law Review II


by: K. M. T. BUELA
ISSUE: Whether or not RTC has jurisdiction over cases which seeks to place a
bank under receivership.
HELD:
RTC has no jurisdiction to hear and decide a suit that seeks to place Banco Filipino
under receivership. Under Section 29 of the New Central Bank Act, the appointment
of a receiver under this section shall be vested exclusively with the Monetary Board.
On the strength of these provisions, it is the Monetary Board that exercises
exclusive jurisdiction over proceedings for receivership of banks. Crystal clear in
Section 30 is the provision that says the "appointment of a receiver under this
section shall be vested exclusively with the Monetary Board." The term
"exclusively" connotes that only the Monetary Board can resolve the issue of
whether a bank is to be placed under receivership and, upon an affirmative finding,
it also has authority to appoint a receiver. This is further affirmed by the fact that
the law allows the Monetary Board to take action "summarily and without need for
prior hearing."
And, as a clincher, the law explicitly provides that "actions of the Monetary Board
taken under this section or under Section 29 of this Act shall be final and executory,
and may not be restrained or set aside by the court except on a petition for certiorari
on the ground that the action taken was in excess of jurisdiction or with such grave
abuse of discretion as to amount to lack or excess of jurisdiction."

D. Replevin (Rule 60)


JOSE S. OROSA and MARTHA P. OROSA vs. HON. COURT OF
APPEALS and FCP CREDIT CORPORATION
G.R. No. 111080, April 5, 2000
FACTS:
Petitioners purchased a vehicle from Fiesta Motors payable in installment.
Petitioners executed by promissory note and a chattel mortgage over the vehicle to
secure future payment thereon. Fiesta Motors assigned the PN and mortgage to
FCP.
Petitioner failed to pay 4 monthly installment. FCP Credit Corporation filed a
complaint for replevin and damages against petitioner to recover possession of the
vehicle subject of the chattel mortgage. Writ of Replevin was issued. However,
RTC eventually dismissed private respondent's complaint and ordered private
respondent's surety, Stronghold Insurance Co., Inc. to jointly and severally with
private respondent return to petitioner the 1983 Ford Laser 1.5 Sedan or its and to
Page | 31

Compilation of Digested Cases for Remedial Law Review II


by: K. M. T. BUELA
pay damages to the extent of the value of the bond in the replevin (i.e., double the
value of the sedan plus the return of the car)
The surety company assailed the decision by way of petition for certiorari, but the
same was dismissed by CAs First Division. SC affirmed CAs First Devisioins
decision. Meanwhile, in private respondent's appeal, the CAs Eighth Division
affirmed the ruling of the trial court and ordered respondent to: (1) return the car
subject of the replevin issued; and (2) to pay petitioner the amount equivalent to
the value of the fourteen (14) monthly installments paid by them.
Petitioners assailed the ruling of the CA- 8th division as it amounts to annulling the
decision of the CA-1st division.
ISSUE: Whether or not the dismissal of the main case wherein replevin was availed
of results to the return of the property taken by the sheriff and the payment of the
full amount of the bond posted by the applicant.
HELD:
The answer is in the negative. Court of Appeals that the trial court erred when it
ordered private respondent to return the subject car or its equivalent considering
that petitioner had not yet fully paid the purchase price. Verily, to sustain the trial
court's decision would amount to unjust enrichment. The Court of Appeals was
correct when it instead ordered private respondent to return, not the car itself, but
only the amount equivalent to the fourteen installments actually paid with interest.
To uphold the assertion of the petitioner would result to the return of the car to them
plus payment of the bond equivalent to double the value of the car. It is as if
petitioners initial asset, i.e., the cars value was multiplied to three fold.

SMART COMMUNICATIONS, INC. vs. REGINA M. ASTORGA


G.R. No. 148132, January 28, 2008
FACTS:
Astorga was employed by respondent Smart Communications as District Sales
Manager and enjoyed various benefits including a car plan in the amount of
P455,000.00. In February 1998, SMART launched an organizational realignment
to achieve more efficient operations. The realignment resulted to Astorgas
termination on ground of redundancy.
Astorga to file a Complaint for illegal dismissal. Meanwhile, SMART sent a letter
to Astorga demanding that she pay the current market value of the Honda Civic
Sedan which was given to her under the companys car plan program, or to
Page | 32

Compilation of Digested Cases for Remedial Law Review II


by: K. M. T. BUELA
surrender the same to the company for proper disposition. Astorga, however, failed
and refused to do either, thus prompting SMART to file a suit for replevin
Astorga moved to dismiss on the ground that the regular courts have no jurisdiction
over the complaint because the subject thereof pertains to a benefit arising from an
employment contract; hence, jurisdiction over the same is vested in the labor
tribunal and not in regular courts.
Astorgas motion to dismiss the replevin case was denied. Her subsequent MR was
denied as well. Astorga elevated the denial of her motion via certiorari to the CA.
Granting the petition and, consequently, dismissing the replevin case, the CA held
that the case is intertwined with Astorgas complaint for illegal dismissal
On the other hand, the illegal dismissal case was decided by NLRC in favor of
SMART
ISSUE: Whether or not regular courts has jurisdiction to grant an application for
replevin over a vehicle issued to an employee pursuant to an employment car plan
benefit.
HELD:
The answer is in the affirmative. Contrary to the CAs ratiocination, the RTC
rightfully assumed jurisdiction over the suit and acted well within its discretion in
denying Astorgas motion to dismiss. SMARTs demand for payment of the market
value of the car or, in the alternative, the surrender of the car, is not a labor, but a
civil, dispute. It involves the relationship of debtor and creditor rather than
employee-employer relations. As such, the dispute falls within the jurisdiction of
the regular courts.

KENNETH HAO vs. ABE C. ANDRES


A.M. No. P-07-2384, June 18, 2008
FACTS:
Complainant Hao is one of the defendants in a civil case for replevin while
respondent is the sheriff who implement the writ of replevin. Complainant Hao filed
the instant administrative complaint against respondent and alleged the following:
(1) Andres gave undue advantage to the replevin applicant in the implementation of
the order and that Andres seized the nine motor vehicles in an oppressive manner;
(2) the day after the seizure, the seized vehicles were kept in a compound owned by
the applicant; (3) allowed applicants lawyer Atty. Macadangdang to get a
keymaster to duplicate the vehicles keys in order to take one motor vehicle; and
(4) eight of the nine seized motor vehicles were reported missing.

Page | 33

Compilation of Digested Cases for Remedial Law Review II


by: K. M. T. BUELA
Investigating judge found Andres guilty of serious negligence in the custody of the
nine motor vehicles.
ISSUE: Whether or not a property seized pursuant to a writ of replevin may be kept
in a place owned by the applicant for purposes of safekeeping the following day
immediately after the implementation of the writ.
HELD:
The answer is in the negative. Section 6 of Rule 60, of the Rules of Court provides
that:
If within five (5) days after the taking of the property by the
sheriff, the adverse party does not object to the sufficiency of
the bond, or of the surety or sureties thereon; or if the
adverse party so objects and the court affirms its approval
of the applicants bond or approves a new bond, or if the
adverse party requires the return of the property but his
bond is objected to and found insufficient and he does not
forthwith file an approved bond, the property shall be
delivered to the applicant. If for any reason the property is
not delivered to the applicant, the sheriff must return it to the
adverse party.
In accordance with the said rules, Andres should have waited no less than five days
in order to give the complainant an opportunity to object to the sufficiency of the
bond or of the surety or sureties thereon, or require the return of the seized motor
vehicles by filing a counter-bond. This, he failed to do.
It matters not that Silver was in possession of the seized vehicles merely for
safekeeping as stated in the depository receipts. The rule is clear that the property
seized should not be immediately delivered to the plaintiff, and the sheriff must
retain custody of the seized property for at least five days. Hence, the act of Andres
in delivering the seized vehicles immediately after seizure to Silver for whatever
purpose, without observing the five-day requirement finds no legal justification.

ROGER V. NAVARRO vs. HON. JOSE L. ESCOBIDO and KAREN T. GO


G.R. No. 153788, November 27, 2009
FACTS:
A lease agreement with option to purchase entered into by and between Respondent
and petitioner whereby the latter delivered unto the former six (6) post-dated
checks. As the said checks were dishonored upon presentment, Respondent Karen
Page | 34

Compilation of Digested Cases for Remedial Law Review II


by: K. M. T. BUELA
T. Go filed two complaints for replevin and/or sum of money with damages against
Navarro. In these complaints, Karen Go prayed that the RTC issue writs of replevin
for the seizure of two (2) motor vehicles in Navarros possession.
Petitioner maintains that the complaints were premature because no prior demand
was made on him to comply with the provisions of the lease agreements before the
complaints for replevin were filed.
ISSUE: Whether or not prior demand is required for the issuance of writ of replevin.
HELD:
Demand is not required prior to filing of replevin action. For a writ of replevin to
issue, all that the applicant must do is to file an affidavit and bond. Pursuant to
Section 2, Rule 60 of the Rules, which states:
The applicant must show by his own affidavit or that of some other person who
personally knows the facts:
(a) That the applicant is the owner of the property claimed, particularly describing
it, or is entitled to the possession thereof;
(b) That the property is wrongfully detained by the adverse party, alleging the cause
of detention thereof according to the best of his knowledge, information, and
belief;
(c) That the property has not been distrained or taken for a tax assessment or a fine
pursuant to law, or seized under a writ of execution or preliminary attachment,
or otherwise placed under custodia legis, or if so seized, that it is exempt from
such seizure or custody; and
(d) The actual market value of the property.
The applicant must also give a bond, executed to the adverse party in double the
value of the property as stated in the affidavit aforementioned, for the return of the
property to the adverse party if such return be adjudged, and for the payment to the
adverse party of such sum as he may recover from the applicant in the action.
Nothing in these provisions requires the applicant to make a prior demand on the
possessor of the property before he can file an action for a writ of replevin. Thus,
prior demand is not a condition precedent to an action for a writ of replevin.

Page | 35

Compilation of Digested Cases for Remedial Law Review II


by: K. M. T. BUELA
SPOUSES DEO AGNER and MARICON AGNER vs. BPI FAMILY
SAVINGS BANK, INC.
G.R. No. 182963, June 3, 2013
FACTS:
Petitioners spouses Agner executed a Promissory Note with Chattel Mortgage in
favor of Citimotors, Inc. Thereafter the right and interest of Citimotors, Inc. in the
Promissory Note with Chattel Mortgage was assigned the same to respondent BPI.
For failure to pay four successive installments from May 15, 2002 to August 15,
2002 despite repeated demands, respondent filed an action for Replevin and
Damages. A writ of replevin was issued. Despite this, the subject vehicle was not
seized.
After trial on the merits, RTC ruled for the respondent and ordered petitioners to
jointly and severally pay the amount of Php576,664.04 plus interest at the rate of
72% per annum from August 20, 2002 until fully paid, and the costs of suit. CA
affirmed RTCs decision.
Petitioner elevated the case before the SC and argued that respondents remedy of
resorting to both actions of replevin and collection of sum of money is contrary to
the provision of Article 1484 of the Civil Code
ISSUE: Whether or not the trial court erred in issuing a writ of replevin and
ordering the party against whom writ was issued to pay the applicant the value of
the obligation considering that the writ was never implemented.
HELD:
The answer is in the negative. The vehicle subject matter of this case was never
recovered and delivered to respondent despite the issuance of a writ of replevin. As
there was no seizure that transpired, it cannot be said that petitioners were deprived
of the use and enjoyment of the mortgaged vehicle or that respondent pursued,
commenced or concluded its actual foreclosure. The trial court, therefore, rightfully
granted the alternative prayer for sum of money, which is equivalent to the remedy
of "exacting fulfillment of the obligation." Certainly, there is no double recovery or
unjust enrichment to speak of.

Page | 36

Compilation of Digested Cases for Remedial Law Review II


by: K. M. T. BUELA
E. Support (Rule 61)
MANUEL DE ASIS vs. COURT OF APPEALS
G.R. No. 127578, February 15, 1999
FACTS:
Vircel D. Andres, in her capacity as the legal guardian of the minor, Glen Camil
Andres de Asis, brought Civil Case Q -88-935 for maintenance and support against
Manuel de Asis alleging that Manuel de Asis (the petitioner here) is the father of
subject minor Glen Camil Andres de Asis, and the former refused and/or failed to
provide for the maintenance of the latter, despite repeated demands.
Subsequently, Andres filed a manifestation withdrawing the petition on the
agreement that herein petitioner would no longer pursue his counterclaim. By virtue
of the said manifestation, both the plaintiff and the defendant agreed to move for
the dismissal of the case. Acting thereupon, RTC terminated the case.
On September 7, 1995, another Complaint for maintenance and support was brought
against Manuel A. de Asis, this time in the name of Glen Camil Andres de Asis,
represented by her legal guardian/mother, Vircel D. Andres. Petitioner moved to
dismiss the Complaint on the ground of res judicata, alleging that instant case is
barred by the prior judgment which dismissed with prejudice Civil Case Q -88-935.
Trial court denied the motion to dismiss, as well as Petitioner's motion for
reconsideration. CA likewise denied petitioners petitioner for certiorari assailing
RTCs decision.
ISSUE: Whether or not a second action for support is barred by the previous action
for support which was dismissed upon agreement of the parties.
HELD:
The answer is in the negative. Future support cannot be the subject of a compromise
according to Art. 2035 of the Civil Code.
The right to support being founded upon the need of the recipient to maintain his
existence, he is not entitled to renounce or transfer the right for this would mean
sanctioning the voluntary giving up of life itself. The right to life cannot be
renounce; hence, support which is the means to attain the former, cannot be
renounced.
The manifestation sent in by respondent's mother in the first case, amounted to
renunciation as it severed the vinculum that gives the minor, Glen Camil, the right
to claim support from his putative parent, the petitioner. As the dismissal of the first
Page | 37

Compilation of Digested Cases for Remedial Law Review II


by: K. M. T. BUELA
case is anchored on an invalid compromise, it cannot have the effect of barring the
second action for support.

PEOPLE OF THE PHILIPPINES vs. MANUEL MANAHAN


G.R. No. 128157, September 29, 1999
FACTS:
Maning was found guilty of rape and sentenced to death by the court a quo. He was
also ordered to indemnify the victim P50,000.00 as moral damages, pay the costs,
and acknowledge and support the offspring of his indiscretion.
ISSUE: Whether or not support may be granted in a criminal action for rape.
HELD:
The answer is the affirmative. Article 345 of The Revised Penal Code provides that
persons guilty of rape shall also be sentenced to "acknowledge the offspring, unless
the law should prevent him from doing so," and "in every case to support the
offspring."
NB: compulsory acknowledgment of the child Melanie Tibigar is not proper there
being a legal impediment in doing so as it appears that the accused is a married man.
As pronounced by this Court in People v. Guerrero, the rule is that if the rapist is a
married man, he cannot be compelled to recognize the offspring of the crime, should
there be any, as his child, whether legitimate or illegitimate." Consequently, that
portion of the judgment under review is accordingly deleted. In any case, we sustain
that part ordering the accused to support the child as it is in accordance with law.

SPOUSES PRUDENCIO and FILOMENA LIM vs. MA. CHERYL S. LIM


G.R. No. 163209, October 30, 2009
FACTS:
Respondent Cheryl S. Lim married Edward Lim. They have three children. The
family resided at the house of petitioners, Edwards parents in Forbes Park, Makati
City. On 14 October 1990, Cheryl abandoned the Forbes Park residence, bringing
the children with her (then all minors), after a violent confrontation with Edward.
Cheryl, for herself and her children, sued petitioners, Edward, Chua Giak and
Mariano (petitioners) in the Regional Trial Court of Makati City, Branch 140 (trial
court) for support.

Page | 38

Compilation of Digested Cases for Remedial Law Review II


by: K. M. T. BUELA
Relying on provisions found in Title IX of the Civil Code, as amended, on Parental
Authority, petitioners theorize that their liability is activated only upon default of
parental authority, conceivably either by its termination or suspension during the
childrens minority. Because at the time respondents sued for support, Cheryl and
Edward exercised parental authority over their children, petitioners submit that the
obligation to support the latters offspring ends with them.
ISSUE: Whether or not grandparents may be sued for support of their grandchildren
and daughter-in-law.
HELD:
Petitioners Liable to Provide Support but only to their Grandchildren. By statutory
and jurisprudential mandate, the liability of ascendants to provide legal support to
their descendants is beyond cavil.
Although the obligation to provide support arising from parental authority ends
upon the emancipation of the child, the same obligation arising from spousal and
general familial ties ideally lasts during the obligee's lifetime.. Also, while parental
authority under Title IX (and the correlative parental rights) pertains to parents,
passing to ascendants only upon its termination or suspension, the obligation to
provide legal support passes on to ascendants not only upon default of the parents
but also for the latters inability to provide sufficient support.

CHARLES GOTARDO vs. DIVINA BULING


G.R. No. 165166, August 15, 2012
FACTS:
Divina Buling filed a complaint for compulsory recognition and support pendente
lite, claiming that the petitioner is the father of her child Gliffze. During the
pendency of the case, the RTC, on the respondents motion, granted a P2,000.00
monthly child support, retroactive from March 1995.
RTC dismissed the complaint for insufficiency of evidence proving Gliffzes
filiation. CA consequently set aside the RTC decision and ordered the petitioner to
recognize his minor son Gliffze. It also reinstated the RTC order granting a P
2,000.00 monthly child support.
ISSUE: Whether or not the amount in the order granting support is fix and may not
be changed.
HELD:

Page | 39

Compilation of Digested Cases for Remedial Law Review II


by: K. M. T. BUELA
The answer is in the negative. Support consists of everything indispensable for
sustenance, dwelling, clothing, medical attendance, education and transportation, in
keeping with the financial capacity of the family. Thus, the amount of support is
variable and, for this reason, no final judgment on the amount of support is
made as the amount shall be in proportion to the resources or means of the
giver and the necessities of the recipient. It may be reduced or increased
proportionately according to the reduction or increase of the necessities of the
recipient and the resources or means of the person obliged to support.

Page | 40

Compilation of Digested Cases for Remedial Law Review II


by: K. M. T. BUELA
SPECIAL CIVIL ACTIONS
A. Interpleader (Rule 62)
WACK WACK GOLF & COUNTRY CLUB, INC. vs. LEE E. WON and
BIENVENIDO A. TAN
G.R. No. L-23851, March 26, 1976
FACTS:
Wack Wack Golf & Country Club filed a complaint for interpleader alleging two
causes of action. For the first cause of action, it alleges both Lee E. Won and
Bienvenido Tan claims to be the owner of membership fee certificate 201. Wons
claim is by virtue of the decision rendered in civil case 26044 in CFI Manila while
Tans claim is by virtue of membership fee certificate 201-serial no. 1199 issued to
him by "Swan, Culbertson and Fritz," the original owner and holder of membership
fee certificate 201. For its second cause of action, it alleged that the membership
fee certificate issued to Won by CFI of Manila is null and void because issued in
violation of its by-laws, which require the surrender and cancellation of the
outstanding membership fee certificate 201 before issuance may be made to the
transferee of a new certificate duly signed by its president and secretary.
The Corporation prayed that (a) an order be issued requiring Lee and Tan to
interplead and litigate their conflicting claims; and (b) judgment be rendered, after
hearing, declaring who of the two is the lawful owner of membership fee certificate
201, and ordering the surrender and cancellation of membership fee certificate 201serial no. 1478 issued in the name of Lee.
The trial court dismissed the complaint with the following findings: (1) that the
allegations in its amended and supplemental complaint do not constitute a valid
ground for an action of interpleader, and in holding that "the principal motive for
the present action is to reopen the Manila Case and collaterally attack the decision
of the said Court"; and (2) that the decision in civil case 26044 of the CFI of Manila
constitutes res judicata and bars its present action.
ISSUE: Whether or not an interpleader suit may prosper when one of the claimants
is a winning litigant in a previous suit involving the subject matter of the said
interpleader.
HELD:
The answer is in the negative for three reasons, namely: (1) action of interpleader
was filed inexcusably late, for which reason it is barred by laches or unreasonable
delay; (2) present application for interpleader would in effect be a collateral attack
upon the final judgment in the said civil case; and (3) interpleader suit would
Page | 41

Compilation of Digested Cases for Remedial Law Review II


by: K. M. T. BUELA
compel the winning litigant to establish his rights anew, and thereby increase
instead of diminish litigations, which is one of the purposes of an interpleader suit.
Action of interpleader was filed inexcusably late.
A stakeholder should use reasonable diligence to hale the contending claimants to
court. He need not await actual institution of independent suits against him before
filing a bill of interpleader. He should file an action of interpleader within a
reasonable time after a dispute has arisen without waiting to be sued by either of
the contending claimants. Otherwise, he may be barred by laches or undue
delay. But where he acts with reasonable diligence in view of the environmental
circumstances, the remedy is not barred.
In this case, the Corporation has not shown any justifiable reason why it did not file
an application for interpleader in civil case 26044 to compel the appellees herein to
litigate between themselves their conflicting claims of ownership. It was only after
adverse final judgment was rendered against it that the remedy of interpleader was
invoked by it. By then it was too late, because to be entitled to this remedy the
applicant must be able to show that he has not been made independently liable to
any of the claimants.
Application for interpleader would in effect be a collateral attack upon the final
judgment in the said civil case.
A successful litigant cannot later be impleaded by his defeated adversary in an
interpleader suit and compelled to prove his claim anew against other adverse
claimants, as that would in effect be a collateral attack upon the judgment.
Instant interpleader suit is contrary to the purposes of an interpleader suit which
is to diminish litigation.
To now permit the Corporation to bring Lee to court after the latter's successful
establishment of his rights in civil case 26044 to the membership fee certificate 201,
is to increase instead of to diminish the number of suits, which is one of the purposes
of an action of interpleader, with the possibility that the latter would lose the
benefits of the favorable judgment. This cannot be done because having elected to
take its chances of success in said civil case 26044, with full knowledge of all the
fact, the Corporation must submit to the consequences of defeat.

Page | 42

Compilation of Digested Cases for Remedial Law Review II


by: K. M. T. BUELA
ETERNAL GARDENS MEMORIAL PARKS CORPORATION vs.
INTERMEDIATE APPELLATE COURT
G.R. No. 73794, September 19, 1988
FACTS:
Petitioner Eternal Gardens Memorial Parks Corporation and private respondent
North Philippine Union Mission Corporation of the Seventh Day Adventists
(MISSION for short) executed a Land Development Agreement whereby the former
undertook to introduce and construct at its own expense and responsibility
necessary improvements on the property owned by private respondent into a
memorial park to be subdivided into and sold as memorial plot lots, at a stipulated
area and price per lot. Out of the proceeds from the sale, private respondent is
entitled to receive 40% of the net gross collection from the project to be remitted
monthly by petitioner to private respondent through a designated depositary trustee
bank.
All went well until Maysilo Estate asserted its claim of ownership over the parcel
of land in question. Confronted with such conflicting claims, petitioner as plaintiff
filed a complaint for interpleader against private respondent MISSION and Maysilo
Estate. Private respondent presented a motion for the placing on judicial deposit the
amounts due and unpaid from petitioner. Acting on such motion, the trial court
denied the motion for judicial deposit.
Private respondent assailed the decision of the trial court denying the motion for
judicial deposit before the Intermediate Appellate Court (IAC). IAC issued an order
setting aside the assailed order of the trial court and directed Eternal Gardens to to
deposit whatever amounts are due from it under the Land Development Agreement
of October 6, 1976 with a reputable bank to be designated by the respondent court
to be the depository trustee of the said amounts to be paid to whoever shall be found
entitled thereto. Petitioner opposed the IAC decision before the Supreme Court
ISSUE: Whether or not the stakeholder or the person who filed a complaint for
interpleader may refuse the order for judicial deposit of the subject matter of the
interpleader.
HELD:
The answer is in the negative. Under the circumstances, there appears to be no
plausible reason for petitioner's objections to the deposit of the amounts in litigation
after having asked for the assistance of the lower court by filing a complaint for
interpleader where the deposit of aforesaid amounts is not only required by the
nature of the action but is a contractual obligation of the petitioner under the Land
Development Program.

Page | 43

Compilation of Digested Cases for Remedial Law Review II


by: K. M. T. BUELA
The essence of an interpleader, aside from the disavowal of interest in the property
in litigation on the part of the petitioner, is the deposit of the property or funds in
controversy with the court. It is a rule founded on justice and equity: "that the
plaintiff may not continue to benefit from the property or funds in litigation
during the pendency of the suit at the expense of whoever will ultimately be
decided as entitled thereto."

SUBHASH C. PASRICHA and JOSEPHINE A. PASRICHA vs. DON LUIS


DISON REALTY, INC.
G.R. No. 136409, March 14, 2008
FACTS:
Respondent Don Luis Dison Realty, Inc. and petitioners executed two Contracts of
Lease whereby the former, as lessor, agreed to lease to the latter several units of the
San Luis Building. Petitioners, in turn, agreed to pay monthly rentals.
While the contracts were in effect, petitioners dealt with Francis Pacheco (Pacheco),
then General Manager of private respondent. Thereafter, Pacheco was replaced by
Roswinda Bautista. Petitioners religiously paid the monthly rentals until May 1992.
After that, however, despite repeated demands, petitioners continuously refused to
pay the stipulated rent. Consequently, respondent filed a complaint for ejectment
was filed by private respondent through its representative, Ms. Bautista, before the
Metropolitan Trial Court.
Petitioners admitted their failure to pay the stipulated rent for the leased premises
starting July until November 1992, but claimed that such refusal was justified
because of the internal squabble in respondent company as to the person authorized
to receive payment.
ISSUE: Whether or not obligor may refuse payment of an obligation on the ground
that he does not know to whom payment must be made.
HELD:
The answer is in the negative. Non-payment of rentals because ostensibly they did
not know to whom payment should be made did not justify their failure to pay,
because if such were the case, they were not without any remedy. They should have
availed of the provisions of the Civil Code of the Philippines on the consignation
of payment and of the Rules of Court on interpleader.
Article 1256 of the Civil Code provides: If the creditor to whom tender of payment
has been made refuses without just cause to accept it, the debtor shall be released
from responsibility by the consignation of the thing or sum due. Consignation shall
Page | 44

Compilation of Digested Cases for Remedial Law Review II


by: K. M. T. BUELA
be made by depositing the things due at the disposal of a judicial authority, before
whom the tender of payment shall be proved in a proper case, and the announcement
of the consignation in other cases.
Moreover, Section 1, Rule 62 of the Rules of Court provides: Whenever conflicting
claims upon the same subject matter are or may be made against a person who
claims no interest whatever in the subject matter, or an interest which in whole or
in part is not disputed by the claimants, he may bring an action against the
conflicting claimants to compel them to interplead and litigate their several claims
among themselves.
Otherwise stated, an action for interpleader is proper when the lessee does not know
to whom payment of rentals should be made due to conflicting claims on the
property (or on the right to collect).60 The remedy is afforded not to protect a person
against double liability but to protect him against double vexation in respect of one
liability.
Notably, instead of availing of the above remedies, petitioners opted to refrain from
making payments.

BANK OF COMMERCE (BOC) vs. PLANTERS DEVELOPMENT BANK


(PDB) and BANGKO SENTRAL NG PILIPINAS
G.R. Nos. 154470-71, September 24, 2012
FACTS:
RCBC owned two sets of Central Bank Bills (CB Bills): (1) 7 CB Bills worth
70Million; and (2) 2 CB Bills worth 20Million. The first set was sold to BOC which
the latter in turn sold to PDB. PDB, in turn, sold to the BOC Treasury Bills worth
P 70 million, with maturity date of June 29, 1994. The second set of CB Bills was
sold by RCBC to PDB and subsequently acquired by BOC. All in all, the BOC
acquired the first and Second sets of CB bills.
On June 30, 1994, upon learning of the transfers involving the CB bills, PDB
requested the BSP to record its claim in the BSPs books, explaining that its nonpossession of the CB bills is "on account of imperfect negotiations thereof and/or
subsequent setoff or transfer."
BSP denied the request, invoking Section 8 of CB Circular No. 28 (Regulations
Governing Open Market Operations, Stabilization of the Securities Market, Issue,
Servicing and Redemption of the Public Debt) which requires the presentation of
the bond before a registered bond may be transferred on the books of the BSP.

Page | 45

Compilation of Digested Cases for Remedial Law Review II


by: K. M. T. BUELA
In light of these BSP responses and the impending maturity of the CB bills, the PDB
filed with the RTC two separate petitions for Mandamus, Prohibition and Injunction
with prayer for Preliminary Injunction and Temporary Restraining Order.
The BOC filed its Answer, praying for the dismissal of the petition. It argued that
the PDB has no cause of action against it since the PDB is no longer the owner of
the CB bills. On the other hand, the BSP countered that the PDB cannot invoke
Section 10 (d) 4 of CB Circular No. 28 because this section applies only to an
"owner" and a "person presenting the bond," of which the PDB is neither.
Alternatively, the BSP asked that an interpleader suit be allowed between and
among the claimants to the subject CB bills on the position that while it is able and
willing to pay the subject CB bills face value, it is duty bound to ensure that
payment is made to the rightful owner.
PDB agrees that the various claimants should now interplead and substantiate their
respective claims on the subject CB bills. However, the total face value of the
subject CB bills should be deposited in escrow with a private bank to be disposed
of only upon order of the RTC.
Accordingly, the BOC and the PDB entered into two separate Escrow Agreements.
Accordingly, the BSP released the maturity proceeds of the CB bills by crediting
the Demand Deposit Account of the PDB and of the BOC with 50% each of the
maturity proceeds of the amount in escrow.
RTC granted the BSPs motion to interplead. In October 2000, the BOC filed its
Amended Consolidated Answer with Compulsory Counterclaim, reiterating its
earlier arguments asserting ownership over the subject CB bills. In May 2001, the
PDB filed an Omnibus Motion, questioning the RTCs jurisdiction over the BOCs
"additional counterclaims."
ISSUE: Whether or not a motion for interpleader may be made an alternative
defense in an answer.
HELD:
The answer is in the affirmative. The remedy of interpleader, as a special civil
action, is primarily governed by the specific provisions in Rule 62 of the Rules of
Court and secondarily by the provisions applicable to ordinary civil actions.136
Indeed, Rule 62 does not expressly authorize the filing of a complaint-ininterpleader as part of, although separate and independent from, the answer.
Similarly, Section 5, Rule 6, in relation to Section 1, Rule 9 of the Rules of Court
does not include a complaint-in-interpleader as a claim, a form of defense, or as an
objection that a defendant may be allowed to put up in his answer or in a motion to
Page | 46

Compilation of Digested Cases for Remedial Law Review II


by: K. M. T. BUELA
dismiss. This does not mean, however, that the BSPs "counter-complaint/crossclaim for interpleader" runs counter to general procedures.
What is quite unique in this case is that the BSP did not initiate the interpleader suit
through an original complaint but through its Answer. This circumstance becomes
understandable if it is considered that insofar as the BSP is concerned, the PDB
does not possess any right to have its claim recorded in the BSPs books;
consequently, the PDB cannot properly be considered even as a potential claimant
to the proceeds of the CB bills upon maturity. Thus, the interpleader was only an
alternative position, made only in the BSPs Answer.
Apart from a pleading, the rules allow a party to seek an affirmative relief from the
court through the procedural device of a motion. While captioned "Answer with
counter complaint/cross-claim for interpleader," the RTC understood this as in the
nature of a motion, seeking relief which essentially consists in an order for the
conflicting claimants to litigate with each other so that "payment is made to the
rightful or legitimate owner" of the subject CB bills.

B. Declaratory Relief and Similar Remedies (Rule 63)


EUFEMIA ALMEDA and ROMEL
MARKETING INDUSTRIES, INC.
G.R. No. 150806, January 28, 2008

ALMEDA

vs.

BATHALA

FACTS:
Bathala Marketing Industries, Inc., as lessee, renewed its Contract of Lease with
Ponciano L. Almeda with the following terms: (1) leased property is 7,348.25
square meters of the Almeda Compound in Pasong Tamo; (2) for a monthly rental
of P1,107,348.69; (4) term of four (4) years from May 1, 1997
During the effectivity of the contract, Ponciano died. Thereafter, respondent dealt
with petitioners, Poncianos heirs.
On two separate dates, petitioners advised respondent that the formers monthly
rentals shall be inclusive of VAT and that monthly rental shall be increased by 73%.
Petitioners asserted that the changes in the monthly rental is pursuant to condition
no. 6 and 7 of the contract of lease.
Respondent refused to pay the VAT and adjusted rentals as demanded by petitioners
but continued to pay the stipulated amount set forth in their contract. On February
18, 1998, respondent instituted an action for declaratory relief for purposes of

Page | 47

Compilation of Digested Cases for Remedial Law Review II


by: K. M. T. BUELA
determining the correct interpretation of condition Nos. 6 and 7 of the lease contract
to prevent damage and prejudice.
On March 10, 1998, petitioners in turn filed an action for ejectment, rescission and
damages against respondent for failure of the latter to vacate the premises after the
demand made by the former.
Petitioners claim that the instant petition is not proper because a separate action for
rescission, ejectment and damages had been commenced before another court; thus,
the construction of the subject contractual provisions should be ventilated in the
same forum.
ISSUE: Whether or not a petition for declaratory relief may be heard despite the
existence of a pending action for rescission involving the same parties and contract
terms.
HELD:
The answer is in the affirmative. It is true that in Panganiban v. Pilipinas Shell
Petroleum Corporation we held that the petition for declaratory relief should be
dismissed in view of the pendency of a separate action for unlawful detainer.
However, we cannot apply the same ruling to the instant case. In Panganiban, the
unlawful detainer case had already been resolved by the trial court before the
dismissal of the declaratory relief case; and it was petitioner in that case who
insisted that the action for declaratory relief be preferred over the action for
unlawful detainer. Conversely, in the case at bench, the trial court had not yet
resolved the rescission/ejectment case during the pendency of the declaratory
relief petition. In fact, the trial court, where the rescission case was on appeal,
itself initiated the suspension of the proceedings pending the resolution of the
action for declaratory relief.
We are not unmindful of the doctrine enunciated in Teodoro, Jr. v. Mirasol where
the declaratory relief action was dismissed because the issue therein could be
threshed out in the unlawful detainer suit. Yet, again, in that case, there was already
a breach of contract at the time of the filing of the declaratory relief petition. This
dissimilar factual milieu proscribes the Court from applying Teodoro to the instant
case.
Given all these attendant circumstances, the Court is disposed to entertain the
instant declaratory relief action instead of dismissing it, notwithstanding the
pendency of the ejectment/rescission case before the trial court. The resolution of
the present petition would write finis to the parties' dispute, as it would settle once
and for all the question of the proper interpretation of the two contractual
stipulations subject of this controversy.

Page | 48

Compilation of Digested Cases for Remedial Law Review II


by: K. M. T. BUELA
REPUBLIC OF THE PHILIPPINES vs. CIPRIANO ORBECIDO III
G.R. No. 154380, October 5, 2005
FACTS:
On May 24, 1981, Cipriano Orbecido III married Lady Myros M. Villanueva. In
1986, Ciprianos wife left for the United States bringing along their son Kristoffer.
A few years later, Cipriano discovered that his wife had been naturalized as an
American citizen. Sometime in 2000, Cipriano learned from his son that his wife
had obtained a divorce decree and then married a certain Innocent Stanley.
Cipriano thereafter filed with the trial court a petition for authority to remarry
invoking Paragraph 2 of Article 26 of the Family Code. Finding merit in the
petition, the court granted the same. The Republic, herein petitioner, through the
Office of the Solicitor General (OSG), sought reconsideration but it was denied.
In this petition, the OSG raises a pure question of law: The OSG contends that
Paragraph 2 of Article 26 of the Family Code is not applicable to the instant case
because it only applies to a valid mixed marriage; that is, a marriage celebrated
between a Filipino citizen and an alien.
ISSUE: Whether or not a party may file a petition, not denominated as petition for
declaratory relief, invoking applicability of a legal provision to his case.
HELD:
Yes. We note that the petition for authority to remarry filed before the trial court
actually constituted a petition for declaratory relief. In this connection, Section 1,
Rule 63 of the Rules of Court provides: Any person interested under a deed, will,
contract or other written instrument, or whose rights are affected by a statute,
executive order or regulation, ordinance, or other governmental regulation may,
before breach or violation thereof, bring an action in the appropriate Regional
Trial Court to determine any question of construction or validity arising, and for a
declaration of his rights or duties, thereunder. xxxx
The requisites of a petition for declaratory relief are: (1) there must be a
justiciable controversy; (2) the controversy must be between persons whose
interests are adverse; (3) that the party seeking the relief has a legal interest in
the controversy; and (4) that the issue is ripe for judicial determination.
This case concerns the applicability of Paragraph 2 of Article 26 to a marriage
between two Filipino citizens where one later acquired alien citizenship, obtained a
divorce decree, and remarried while in the U.S.A. The interests of the parties are
also adverse, as petitioner representing the State asserts its duty to protect the
institution of marriage while respondent, a private citizen, insists on a declaration
Page | 49

Compilation of Digested Cases for Remedial Law Review II


by: K. M. T. BUELA
of his capacity to remarry. Respondent, praying for relief, has legal interest in the
controversy. The issue raised is also ripe for judicial determination inasmuch as
when respondent remarries, litigation ensues and puts into question the validity of
his second marriage.

CARMEN DANAO MALANA et al. vs. BENIGNO TAPPA et al.


G.R. No. 181303, September 17, 2009
FACTS:
Petitioners alleged that they are the owners of a parcel of land which they inherited
from Anastacio Danao. During the lifetime of Anastacio, he had allowed Consuelo
Pauig to build on and occupy a portion of the subject property with the agreement
that the latter would vacate the said land at any time that Anastacio and his heirs
might need it. Averring that they already needed it, petitioners demanded that
respondents vacate the same. Respondents, however, refused to heed petitioners
demand.
As Respondents presented dubious documents supporting their claim of ownership
before the proceedings in Lupong Tagapamayapa, petitioners filed before the RTC
their Complaint for Reivindicacion, Quieting of Title, and Damages against
respondents before RTC - Tuguegarao City. RTC dismissed petitioners Complaint
on the ground of lack of jurisdiction being that the suit is a real action involving a
real property with assessed value less than P20,000.00 and hereby dismisses the
same without prejudice.
Petitioners filed a Motion for Reconsideration and argued that their principal cause
of action was for quieting of title; the accion reivindicacion was included merely to
enable them to seek complete relief from respondents. Petitioners Complaint
should not have been dismissed, since Section 1, Rule 63 of the Rules of Court13
states that an action to quiet title falls under the jurisdiction of the RTC.
ISSUE: Whether or not an action for Quieting of Title falls under the jurisdiction
of RTC pursuant to the second paragraph of Section 1 of Rule 63.
HELD:
The answer is in the negative. The second paragraph of Section 1, Rule 63 of the
Rules of Court specifically refers to (1) an action for the reformation of an
instrument, recognized under Articles 1359 to 1369 of the Civil Code; (2) an action
to quiet title, authorized by Articles 476 to 481 of the Civil Code; and (3) an action
to consolidate ownership required by Article 1607 of the Civil Code in a sale with
a right to repurchase. These three remedies are considered similar to declaratory

Page | 50

Compilation of Digested Cases for Remedial Law Review II


by: K. M. T. BUELA
relief because they also result in the adjudication of the legal rights of the litigants,
often without the need of execution to carry the judgment into effect.
To determine which court has jurisdiction over the actions identified in the second
paragraph of Section 1, Rule 63 of the Rules of Court, said provision must be read
together with those of the Judiciary Reorganization Act of 1980, as amended.
It is important to note that Section 1, Rule 63 of the Rules of Court does not
categorically require that an action to quiet title be filed before the RTC. It
repeatedly uses the word "may" that an action for quieting of title "may be brought
under [the] Rule" on petitions for declaratory relief, and a person desiring to file a
petition for declaratory relief "may x x x bring an action in the appropriate Regional
Trial Court." The use of the word "may" in a statute denotes that the provision is
merely permissive and indicates a mere possibility, an opportunity or an option.
In contrast, the mandatory provision of the Judiciary Reorganization Act of 1980,
as amended, uses the word "shall" and explicitly requires the MTC to exercise
exclusive original jurisdiction over all civil actions which involve title to or
possession of real property where the assessed value does not exceed P20,000.00

FRANCISCO I. CHAVEZ vs. JUDICIAL AND BAR COUNCIL, SEN.


FRANCIS JOSEPH G. ESCUDERO and REP. NIEL C. TUPAS, JR.
G.R. No. 202242, April 16, 2013
FACTS:
In his initiatory pleading, petitioner asked the Court to determine 1] whether the
first paragraph of Section 8, Article VIII of the 1987 Constitution allows more than
one (1) member of Congress to sit in the JBC; and 2] if the practice of having two
(2) representatives from each House of Congress with one (1) vote each is
sanctioned by the Constitution.
On July 17, 2012, the Court ruled that the Judicial and Bar Council is hereby
enjoined to reconstitute itself so that only one (1) member of Congress will sit as a
representative in its proceedings, in accordance with Section 8(1), Article VIII of
the 1987 Constitution.
On July 31, 2012, respondents filed a motion for reconsideration. Through the
subject motion, respondents pray that the Court reconsider its decision and dismiss
the petition on the following grounds: 1] that allowing only one representative from
Congress in the JBC would lead to absurdity considering its bicameral nature; 2]
that the failure of the Framers to make the proper adjustment when there was a shift
from unilateralism to bicameralism was a plain oversight; 3] that two
Page | 51

Compilation of Digested Cases for Remedial Law Review II


by: K. M. T. BUELA
representatives from Congress would not subvert the intention of the Framers to
insulate the JBC from political partisanship; and 4] that the rationale of the Court
in declaring a seven-member composition would provide a solution should there be
a stalemate is not exactly correct.
ISSUE: Whether or not both the Senate and the House of Representatives is entitled
to only one seat in the Judicial and Bar Council.
HELD:
The language used in the Constitution must be taken to have been deliberately
chosen for a definite purpose. Every word employed in the Constitution must be
interpreted to exude its deliberate intent which must be maintained inviolate against
disobedience and defiance. What the Constitution clearly says, according to its text,
compels acceptance and bars modification even by the branch tasked to interpret it.
For this reason, the Court cannot accede to the argument of plain oversight in order
to justify constitutional construction. As stated in the July 17, 2012 Decision, in
opting to use the singular letter "a" to describe "representative of Congress," the
Filipino people through the Framers intended that Congress be entitled to only one
(1) seat in the JBC. Had the intention been otherwise, the Constitution could have,
in no uncertain terms, so provided, as can be read in its other provisions.
A reading of the 1987 Constitution would reveal that several provisions were indeed
adjusted as to be in tune with the shift to bicameralism. One example is Section 4,
Article VII, which provides that a tie in the presidential election shall be broken "by
a majority of all the Members of both Houses of the Congress, voting separately."
Another is Section 8 thereof which requires the nominee to replace the VicePresident to be confirmed "by a majority of all the Members of both Houses of the
Congress, voting separately." Similarly, under Section 18, the proclamation of
martial law or the suspension of the privilege of the writ of habeas corpus may be
revoked or continued by the Congress, voting separately, by a vote of at least a
majority of all its Members." In all these provisions, the bicameral nature of
Congress was recognized and, clearly, the corresponding adjustments were made as
to how a matter would be handled and voted upon by its two Houses.
Thus, to say that the Framers simply failed to adjust Section 8, Article VIII, by sheer
inadvertence, to their decision to shift to a bicameral form of the legislature, is not
persuasive enough. Respondents cannot just lean on plain oversight to justify a
conclusion favorable to them. It is very clear that the Framers were not keen on
adjusting the provision on congressional representation in the JBC because it was
not in the exercise of its primary function to legislate. JBC was created to support
the executive power to appoint, and Congress, as one whole body, was merely
assigned a contributory non-legislative function.
Page | 52

Compilation of Digested Cases for Remedial Law Review II


by: K. M. T. BUELA

The underlying reason for such a limited participation can easily be discerned.
Congress has two (2) Houses. The need to recognize the existence and the role of
each House is essential considering that the Constitution employs precise language
in laying down the functions which particular House plays, regardless of whether
the two Houses consummate an official act by voting jointly or separately. Whether
in the exercise of its legislative or its non-legislative functions such as inter alia, the
power of appropriation, the declaration of an existence of a state of war, canvassing
of electoral returns for the President and Vice-President, and impeachment, the
dichotomy of each House must be acknowledged and recognized considering the
interplay between these two Houses. In all these instances, each House is
constitutionally granted with powers and functions peculiar to its nature and with
keen consideration to 1) its relationship with the other chamber; and 2) in
consonance with the principle of checks and balances, as to the other branches of
government.

SPOUSES CLEMENCIO C. SABITSANA, JR. and MA. ROSARIO M.


SABITSANA vs. JUANITO F. MUERTEGUI
G.R. No. 181359, August 5, 2013
FACTS:
Garcia executed an unnotarized Deed of Sale in favor of respondent Juanito over a
parcel of unregistered land. Immediately thereafter, Juanitos father Domingo Sr.
and brother Domingo Jr. took actual possession of the lot and planted thereon
coconut and ipil-ipil trees. On October 17, 1991, Garcia sold the lot to the Muertegui
family lawyer, petitioner Atty. Clemencio C. Sabitsana, Jr. (Atty. Sabitsana),
through a notarized deed of absolute sale.8 The sale was registered with the Register
of Deeds on February 6, 1992.9 TD No. 1996 was cancelled and a new one, TD No.
5327, was issued in Atty. Sabitsanas name.
When Domingo Sr. passed away, his heirs applied for registration and coverage of
the lot under the Public Land Act or Commonwealth Act No. 141. Atty. Sabitsana
opposed the application, claiming that he was the true owner of the lot.
On April 11, 2000, Juanito filed an action for quieting of title and preliminary
injunction before Regional Trial Court (RTC) of Naval, Biliran, against herein
petitioners Atty. Sabitsana and his wife, Rosario, claiming that they bought the lot
in bad faith and are exercising acts of possession and ownership over the same,
which acts thus constitute a cloud over his title.
In their Answer with Counterclaim, petitioners asserted mainly that the sale to
Juanito is null and void absent the marital consent of Garcias wife, Soledad Corto
Page | 53

Compilation of Digested Cases for Remedial Law Review II


by: K. M. T. BUELA
(Soledad); that they acquired the property in good faith and for value; and that the
Complaint is barred by prescription and laches. They likewise insisted that the
Regional Trial Court (RTC) of Naval, Biliran did not have jurisdiction over the
case, which involved title to or interest in a parcel of land the assessed value of
which is merely P1,230.00.
ISSUE: Whether or not RTC has jurisdiction over an action for quieting of title
where the subject property is valued below 20,000.00 outside Metro Manila.
HELD:
On the question of jurisdiction, it is clear under the Rules that an action for quieting
of title may be instituted in the RTCs, regardless of the assessed value of the real
property in dispute. Under Rule 63 of the Rules of Court, an action to quiet title to
real property or remove clouds therefrom may be brought in the appropriate RTC.

REPUBLIC OF THE PHILIPPINES vs. HERMINIO HARRY ROQUE et.


al
G.R. No. 204603, September 24, 2013
FACTS:
On July 17, 2007, private respondents filed a Petition for declaratory relief before
the RTC, assailing the constitutionality of the following sections of RA 9372: (a)
Section 3, for being void for vagueness; (b) Section 7, for violating the right to
privacy of communication and due process and the privileged nature of priestpenitent relationships; (c)Section 18, for violating due process, the prohibition
against ex post facto laws or bills of attainder, the Universal Declaration of Human
Rights, and the International Covenant on Civil and Political Rights, as well as for
contradicting Article 125 of the Revised Penal Code, as amended; (d) Section 26,
for violating the right to travel; and (e) Section 27, for violating the prohibition
against unreasonable searches and seizures.
On February 27, 2012, petitioners filed the subject motion to dismiss, contending
that private respondents failed to satisfy the requisites for declaratory relief.
Likewise, they averred that the constitutionality of RA 9372 had already been
upheld by the Court in the Southern Hemisphere cases.
On April 23, 2012, the RTC issued an Order24 which denied the subject motion to
dismiss, finding that the Court did not pass upon the constitutionality of RA 9372
and that private respondents petition for declaratory relief was properly filed.

Page | 54

Compilation of Digested Cases for Remedial Law Review II


by: K. M. T. BUELA
ISSUE: Whether or not a declaratory relief, the subject matter of which is a new
legislation, may be given due course despite the fact that no controversy had arisen
from the said legislation.
HELD:
Case law states that the following are the requisites for an action for declaratory
relief:
First , the subject matter of the controversy must be a deed, will, contract or other
written instrument, statute, executive order or regulation, or ordinance; second , the
terms of said documents and the validity thereof are doubtful and require judicial
construction; third , there must have been no breach of the documents in question;
fourth , there must be an actual justiciable controversy or the "ripening seeds" of
one between persons whose interests are adverse; fifth , the issue must be ripe for
judicial determination; and sixth, adequate relief is not available through other
means or other forms of action or proceeding.
While the first, second, and third requirements appear to exist in this case, the
fourth, fifth, and sixth requirements, however, remain wanting.
As to the fourth requisite, there is serious doubt that an actual justiciable
controversy or the "ripening seeds" of one exists in this case.
Pertinently, a justiciable controversy refers to an existing case or controversy that
is appropriate or ripe for judicial determination, not one that is conjectural or merely
anticipatory. Corollary thereto, by "ripening seeds" it is meant, not that sufficient
accrued facts may be dispensed with, but that a dispute may be tried at its inception
before it has accumulated the asperity, distemper, animosity, passion, and violence
of a full blown battle that looms ahead. The concept describes a state of facts
indicating imminent and inevitable litigation provided that the issue is not settled
and stabilized by tranquilizing declaration.
A perusal of private respondents petition for declaratory relief would show that
they have failed to demonstrate how they are left to sustain or are in immediate
danger to sustain some direct injury as a result of the enforcement of the assailed
provisions of RA 9372. Not far removed from the factual milieu in the Southern
Hemisphere cases, private respondents only assert general interests as citizens,
and taxpayers and infractions which the government could prospectively
commit if the enforcement of the said law would remain untrammeled. As their
petition would disclose, private respondents fear of prosecution was solely based
on remarks of certain government officials which were addressed to the general
public. They, however, failed to show how these remarks tended towards any
prosecutorial or governmental action geared towards the implementation of RA
Page | 55

Compilation of Digested Cases for Remedial Law Review II


by: K. M. T. BUELA
9372 against them. In other words, there was no particular, real or imminent threat
to any of them.
The possibility of abuse in the implementation of RA 9372 does not avail to take
the present petitions out of the realm of the surreal and merely imagined. Such
possibility is not peculiar to RA 9372 since the exercise of any power granted by
law may be abused. Allegations of abuse must be anchored on real events before
courts may step in to settle actual controversies involving rights which are legally
demandable and enforceable.
As to the fifth requisite for an action for declaratory relief, neither can it be inferred
that the controversy at hand is ripe for adjudication since the possibility of abuse,
based on the above-discussed allegations in private respondents petition, remain
highly-speculative and merely theorized. It is well-settled that a question is ripe for
adjudication when the act being challenged has had a direct adverse effect on the
individual challenging it This private respondents failed to demonstrate in the case
at bar.
Finally, as regards the sixth requisite, the Court finds it irrelevant to proceed with a
discussion on the availability of adequate reliefs since no impending threat or injury
to the private respondents exists in the first place.

SOUTHERN HEMISPHERE ENGAGEMENT NETWORK, INC vs.


ANTI-TERRORISM COUNCIL et al
G.R. No. 178552, October 5, 2010
FACTS:
Before the Court are six petitions challenging the constitutionality of Republic Act
No. 9372 (RA 9372), "An Act to Secure the State and Protect our People from
Terrorism," otherwise known as the Human Security Act of 2007, signed into law
on March 6, 2007.
ISSUE: Whether or not constitutionality of a law may be assailed via petition for
certiorari.
HELD:
Preliminarily, certiorari does not lie against respondents who do not exercise
judicial or quasi-judicial functions. Section 1, Rule 65 of the Rules of Court is clear:
Section 1. Petition for certiorari.When any tribunal, board
or officer exercising judicial or quasi-judicial functionshas
acted without or in excess of its or his jurisdiction, or with
Page | 56

Compilation of Digested Cases for Remedial Law Review II


by: K. M. T. BUELA
grave abuse of discretion amounting to lack or excess of
jurisdiction, and there is no appeal, nor any plain, speedy,
and adequate remedy in the ordinary course of law, a person
aggrieved thereby may file a verified petition in the proper
court, alleging the facts with certainty and praying that
judgment be rendered annulling or modifying the
proceedings of such tribunal, board or officer, and granting
such incidental reliefs as law and justice may require.
Parenthetically, petitioners do not even allege with any modicum of particularity
how respondents acted without or in excess of their respective jurisdictions, or with
grave abuse of discretion amounting to lack or excess of jurisdiction.
The impropriety of certiorari as a remedy aside, the petitions fail just the same.
Without any justiciable controversy, the petitions have become pleas for declaratory
relief, over which the Court has no original jurisdiction. Then again, declaratory
actions characterized by "double contingency," where both the activity the
petitioners intend to undertake and the anticipated reaction to it of a public official
are merely theorized, lie beyond judicial review for lack of ripeness.
The possibility of abuse in the implementation of RA 9372 does not avail to take
the present petitions out of the realm of the surreal and merely imagined. Such
possibility is not peculiar to RA 9372 since the exercise of any power granted by
law may be abused. Allegations of abuse must be anchored on real events before
courts may step in to settle actual controversies involving rights which are legally
demandable and enforceable.

C. Review of Judgments and Final Orders of COMELEC and COA (Rule 64)
ALLIANCE FOR NATIONALISM AND DEMOCRACY (ANAD) vs.
COMMISSION ON ELECTIONS
G.R. No. 206987, September 10, 2013
FACTS:
On 7 November 2012, the COMELEC En Banc promulgated a Resolution canceling
petitioners Certificate of Registration and/or Accreditation.
In the assailed Resolution dated 11 May 2013, the COMELEC affirmed the
cancellation of petitioners Certificate of Registration and/or Accreditation and
disqualified it from participating in the 2013 Elections. The COMELEC held that
while ANAD can be classified as a sectoral party lacking in well-defined political
constituencies, its disqualification still subsists for violation of election laws and
Page | 57

Compilation of Digested Cases for Remedial Law Review II


by: K. M. T. BUELA
regulations, particularly for its failure to submit at least five nominees, and for its
failure to submit its Statement of Contributions and Expenditures for the 2007
Elections.
Hence, the present petition raising the issues of whether or not the COMELEC
gravely abused its discretion in promulgating the assailed Resolution without the
benefit of a summary evidentiary hearing mandated by the due process clause, and
whether or not the COMELEC erred in finding that petitioner submitted only three
nominees and that it failed to submit its Statement of Contributions and
Expenditures in the 2007 Elections.
ISSUE: Whether or not a resolution of COMELEC may be assailed via petition
for certiorari on the ground of its failure to set the case for a second evidentiary
hearing.
HELD:
The only question that may be raised in a petition for certiorari under Section 2,
Rule 64 of the Rules of Court is whether or not the COMELEC acted with grave
abuse of discretion amounting to lack or excess of jurisdiction. For a petition for
certiorari to prosper, there must be a clear showing of caprice and arbitrariness in
the exercise of discretion.
"Grave abuse of discretion," under Rule 65, has a specific meaning. It is the
arbitrary or despotic exercise of power due to passion, prejudice or personal
hostility; or the whimsical, arbitrary, or capricious exercise of power that amounts
to an evasion or a refusal to perform a positive duty enjoined by law or to act at all
in contemplation of law. For an act to be struck down as having been done with
grave abuse of discretion, the abuse of discretion must be patent and gross.
ANAD claims that the COMELEC gravely abused its discretion when it
promulgated the assailed Resolution without giving ANAD the benefit of a
summary evidentiary hearing, thus violating its right to due process. It is to be noted,
however, that ANAD was already afforded a summary hearing on23 August 2013,
during which Mr. Domingo M. Balang, ANADs president, authenticated
documents and answered questions from the members of the COMELEC pertinent
to ANADs qualifications.

Page | 58

Compilation of Digested Cases for Remedial Law Review II


by: K. M. T. BUELA
D. Certiorari, Prohibition and Mandamus (Rules 65)
1. Certiorari (Section 1)
OSCAR R. AMPIL vs. THE HON. OFFICE OF THE OMBUDSMAN,
et al.
G.R. No. 192685, July 31, 2013
FACTS:
ASB Realty Corporation (ASB) and Malayan Insurance Company (MICO)
entered into a Joint Project Development Agreement (JPDA) for the
construction of "The Malayan Tower." wherein the latter shall provide the real
property while former would construct and shoulder the cost of construction and
development of the condominium building. Due to financial difficulties, ASB
was unable to perform its obligations to MICO. Thus, MICO and ASB executed
a Memorandum of Agreement (MOA), allowing MICO to assume the entire
responsibility for the development and completion of The Malayan Tower. The
MOA specifies the entitlement of both ASB and MICO to net saleable areas of
The Malayan Tower representing their investments.
On 11 March 2005, two sets of Condominium Certificates of Title (CCTs) were
issued by Espenesin for 38 units and the allotted parking spaces in Malayan
Tower. The first was in the name of MICO and the second in the name of ASB.
The second set of CCTs was issued upon the instruction of Serrano an officer
of MICO. Ampil, unsecured creditor of the ASB charged Espenesin with
violation of Sections 3(a) and (e) of Republic Act No. 3019 before the Office of
the Ombudsman.
Ombudsman dismissed Ampils complaint on the ground of lack of probable
cause for the alleged commission of falsification. Thereafter, Ampil filed a
petition for review under Rule 43 of the Rules of Court before the appellate
court. And as already stated, the appellate court affirmed the Ombudsmans
resolution.
ISSUE: Whether or not Ombudsmans discretionary power to determine the
existence of probable cause may be assailed via petition for certiorari under
Rule 65 of the Rules of Court
HELD:
The Supreme Court have consistently hewed to the policy of non-interference
with the Ombudsmans exercise of its constitutionally mandated powers. The
Ombudsmans finding to proceed or desist in the prosecution of a criminal case
can only be assailed through certiorari proceedings before this Court on the
Page | 59

Compilation of Digested Cases for Remedial Law Review II


by: K. M. T. BUELA
ground that such determination is tainted with grave abuse of discretion which
contemplates an abuse so grave and so patent equivalent to lack or excess of
jurisdiction.
However, on several occasions, the court have interfered with the
Ombudsmans discretion in determining probable cause: (a) To afford
protection to the constitutional rights of the accused; (b) When necessary for the
orderly administration of justice or to avoid oppression or multiplicity of
actions; (c) When there is a prejudicial question which is sub judice; (d) When
the acts of the officer are without or in excess of authority; (e) Where the
prosecution is under an invalid law, ordinance or regulation; (f) When double
jeopardy is clearly apparent; (g) Where the court has no jurisdiction over the
offense; (h) Where it is a case of persecution rather than prosecution; (i) Where
the charges are manifestly false and motivated by the lust for vengeance.
The fourth circumstance is present in this case. Despite the admission by
Espenesin that he had altered the CCTs and the Ombudsmans findings thereon,
the Ombudsman abruptly dismissed Ampils complaint-affidavit. A finding of
probable cause needs only to rest on evidence showing that more likely than not
a crime has been committed and there is enough reason to believe that it was
committed by the accused. It need not be based on clear and convincing
evidence of guilt, neither on evidence establishing absolute certainty of guilt.
As Registrar of Deeds, Espenesin was duty bound to inquire and ascertain the
reason for Serranos new instruction on those specific set of CCTs and not just
heed Serranos bidding. He heads the Office of Register of Deeds which is
constituted by law as "a public repository of records of instruments affecting
registered or unregistered lands x x x in the province or city wherein such office
is situated." He should not have so easily taken Serranos word that the
amendment Serrano sought was to correct simple and innocuous error.
Espenesin could have then easily asked, as he is obliged to, for a contract or an
authenticated writing to ascertain which units and parking slots were really
allotted for ASB and MICO. His actions would then be based on what is
documented and not merely by a lame claim of bona fides mistake.

A.L. ANG NETWORK, INC. vs. EMMA MONDEJAR


G.R. No. 200804, January 22, 2014
FACTS:
Petitioner filed a complaint for sum of money under the Rule of Procedure for
Small Claims Cases before the MTCC, seeking to collect from respondent the

Page | 60

Compilation of Digested Cases for Remedial Law Review II


by: K. M. T. BUELA
amount of P23,111.71 which represented her unpaid water bills for the period
June 1, 2002 to September 30, 2005.
On June 10, 2011, the MTCC rendered a Decision in favor of respondent.
Aggrieved, petitioner filed a petition for certiorari before the RTC, ascribing
grave abuse of discretion on the part of the MTCC in finding that petitioner
failed to establish with certainty respondents obligation, and in not ordering the
latter to pay the full amount sought to be collected.
On November 23, 2011, the RTC issued a Decision dismissing the petition for
certiorari, finding that the said petition was only filed to circumvent the nonappealable nature of small claims cases as provided under Section 2322 of the
Rule of Procedure on Small Claims Cases.
ISSUE: Whether or not a decision on a complaint falling under small claims
cases may be questioned via petition for certiorari under Rule 65 when the Rule
of Procedure for Small Claims Cases states that decisions rendered in such case
is final and unappealable.
HELD:
The answer is in the affirmative. Considering the final nature of a small claims
case decision under the above-stated rule, the remedy of appeal is not allowed,
and the prevailing party may, thus, immediately move for its execution.
Nevertheless, the proscription on appeals in small claims cases, similar to
other proceedings where appeal is not an available remedy, does not
preclude the aggrieved party from filing a petition for certiorari under
Rule 65 of the Rules of Court. This general rule has been enunciated in the
case of Okada v. Security Pacific Assurance Corporation, wherein it was held
that:
In a long line of cases, the Court has consistently ruled that "the extraordinary
writ of certiorari is always available where there is no appeal or any other plain,
speedy and adequate remedy in the ordinary course of law."
Truly, an essential requisite for the availability of the extraordinary remedies
under the Rules is an absence of an appeal nor any "plain, speedy and adequate
remedy" in the ordinary course of law, one which has been so defined as a
"remedy which (would) equally (be) beneficial, speedy and sufficient not
merely a remedy which at some time in the future will bring about a revival of
the judgment x x x complained of in the certiorari proceeding, but a remedy
which will promptly relieve the petitioner from the injurious effects of that
judgment and the acts of the inferior court or tribunal" concerned.

Page | 61

Compilation of Digested Cases for Remedial Law Review II


by: K. M. T. BUELA

MARK JEROME S. MAGLALANG vs. PHILIPPINE AMUSEMENT


AND GAMING CORPORATION (PAGCOR)
G.R. No. 190566, December 11, 2013
FACTS:
Petitioner was a teller at the Casino Filipino which was operated by respondent
PAGCOR. Due to an altercation with a customer, Petitioner was charged with
Discourtesy towards a casino customer. He was later on found him guilty of
Discourtesy towards a casino customer and imposed on him a 30-day
suspension for this first offense. Aggrieved, petitioner filed a Motion for
Reconsideration seeking a reversal of the boards decision and further prayed in
the alternative that if he is indeed found guilty as charged, the penalty be only a
reprimand as it is the appropriate penalty. His Motion for Reconsideration was
denied.
Hence, petitioner filed a petition for certiorari under Rule 65 of the 1997 Rules
of Civil Procedure, as amended, before the CA. Moreover, petitioner ascribed
grave abuse of discretion amounting to lack or excess of jurisdiction to the acts
of PAGCOR in adjudging him guilty of the charge, in failing to observe the
proper procedure in the rendition of its decision and in imposing the harsh
penalty of a 30-day suspension. Justifying his recourse to the CA, petitioner
explained that he did not appeal to the Civil Service Commission (CSC) because
the penalty imposed on him was only a 30-day suspension which is not within
the CSCs appellate jurisdiction. He also claimed that discourtesy in the
performance of official duties is classified as a light offense which is punishable
only by reprimand.
In its assailed Resolution dated September 30, 2009, the CA outrightly
dismissed the petition for certiorari for being premature as petitioner failed to
exhaust administrative remedies before seeking recourse from the CA. Invoking
Section 2(1), Article IX-B of the 1987 Constitution, the CA held that the CSC
has jurisdiction over issues involving the employer-employee relationship in all
branches, subdivisions, instrumentalities and agencies of the Government,
including government-owned or controlled corporations with original charters
such as PAGCOR.
ISSUE: Whether or not a petition for certiorari filed directly before the CA
assailing a decision for which the law provides no remedy of appeal may be
dismissed on the ground of failure to exhaust administrative remedies.
HELD:

Page | 62

Compilation of Digested Cases for Remedial Law Review II


by: K. M. T. BUELA
The answer is in the negative. Under the doctrine of exhaustion of
administrative remedies, before a party is allowed to seek the intervention of the
court, he or she should have availed himself or herself of all the means of
administrative processes afforded him or her. Hence, if resort to a remedy within
the administrative machinery can still be made by giving the administrative
officer concerned every opportunity to decide on a matter that comes within his
or her jurisdiction, then such remedy should be exhausted first before the court's
judicial power can be sought. The premature invocation of the intervention of
the court is fatal to ones cause of action.
However, the doctrine of exhaustion of administrative remedies is not absolute
as it admits of the following exceptions: (1) when there is a violation of due
process; (2) when the issue involved is purely a legal question; (3) when the
administrative action is patently illegal amounting to lack or excess of
jurisdiction; (4) when there is estoppel on the part of the administrative agency
concerned; (5) when there is irreparable injury; (6) when the respondent is a
department secretary whose acts as an alter ego of the President bears the
implied and assumed approval of the latter; (7) when to require exhaustion of
administrative remedies would be unreasonable; (8) when it would amount to a
nullification of a claim; (9) when the subject matter is a private land in land case
proceedings; (10) when the rule does not provide a plain, speedy and adequate
remedy, and (11) when there are circumstances indicating the urgency of
judicial intervention, and unreasonable delay would greatly prejudice the
complainant; (12) where no administrative review is provided by law; (13)
where the rule of qualified political agency applies and (14) where the issue of
non-exhaustion of administrative remedies has been rendered moot.29
The case before us falls squarely under exception number 12 since the law per
se provides no administrative review for administrative cases whereby an
employee like petitioner is covered by Civil Service law, rules and regulations
and penalized with a suspension for not more than 30 days.
Decisions of administrative or quasi-administrative agencies which are declared
by law final and unappealable are subject to judicial review if they fail the test
of arbitrariness, or upon proof of gross abuse of discretion, fraud or error of law.
In sum, there being no appeal or any plain, speedy, and adequate remedy in the
ordinary course of law in view of petitioner's allegation that P AGCOR has acted
without or in excess of jurisdiction, or with grave abuse of discretion amounting
to lack or excess of jurisdiction, the CA's outright dismissal of the petition for
certiorari on the basis of non-exhaustion of administrative remedies is bereft of
any legal standing and should therefore be set aside.

Page | 63

Compilation of Digested Cases for Remedial Law Review II


by: K. M. T. BUELA
PEOPLE OF THE PHILIPPINES vs. THE HONORABLE JUANITO C.
CASTANEDA, JR., et al.
G.R. No. 208290, December 11, 2013
FACTS:
Private respondents were charged before the CTA for misdeclaration of goods.
Subsequent to the filing by prosecution of its Formal Offer of Evidence, Private
Respondent filed their Omnibus Motion to File Demurrer to Evidence which
was granted by the CTA.
Despite opposition, the CTA dismissed the case against Garcia and Vestidas Jr.
in its March 26, 2013 Resolution, for failure of the prosecution to establish their
guilt beyond reasonable doubt. According to the CTA, "no proof whatsoever
was presented by the prosecution showing that the certified true copies of the
public documents offered in evidence against both accused were in fact issued
by the legal custodians.
The prosecution filed its motion for reconsideration, but it was denied by the
CTA, stressing, among others, that to grant it would place the accused in double
jeopardy. Hence, this petition for certiorari, ascribing grave abuse of discretion
on the part of the CTA
ISSUE: Whether or not a final decision in a criminal complaint may be assailed
by way of petition for certiorari.
HELD:
The answer is in the affirmative. While a judgment of acquittal in a criminal
case may be assailed in a petition for certiorari under Rule 65 of the Rules of
Court, it must be shown that there was grave abuse of discretion amounting to
lack or excess of jurisdiction or a denial of due process. In this case, a perusal
of the challenged resolutions of the CTA does not disclose any indication of
grave abuse of discretion on its part or denial of due process. The records are
replete with indicators that the petitioner actively participated during the trial
and, in fact, presented its offer of evidence and opposed the demurrer.
Grave abuse of discretion is defined as capricious or whimsical exercise of
judgment as is equivalent to lack of jurisdiction. The abuse of discretion must
be patent and gross as to amount to an evasion of a positive duty or a virtual
refusal to perform a duty enjoined by law, or to act at all in contemplation of
law, as where the power is exercised in an arbitrary and despotic manner by
reason of passion and hostility. Here, the subject resolutions of the CTA have
been issued in accordance with the rules on evidence and existing jurisprudence.

Page | 64

Compilation of Digested Cases for Remedial Law Review II


by: K. M. T. BUELA

UNIVERSITY OF THE PHILIPPINES BOARD OF REGENTS vs.


HON. ELSIE LIGOT-TELAN
G.R. No. 110280, October 12, 1993
FACTS:
Ramon P. Nadal, a student enrolled in the UP College of Law, applied for
Socialized Tuition Fee and Assistance Program (STFAP).
U.P. charged Nadal before the Student Disciplinary Tribunal (SDT). SDT
rendered a decision finding him guilty of "wilfully and deliberately withholding
information in his STFAP application about the income of his mother, who is
living abroad, in support of the studies of his brothers Antonio and Federico,
which is tantamount to acts of dishonesty in relation to his studies.
As such, the SDT imposed upon Nadal the penalty of expulsion from the
University and required him to reimburse all STFAP benefits he had received
but if he does not voluntarily make reimbursement. The Executive Committee
affirmed the decision of the SDT. Nadal appealed to the Board of Regents
(BOR). BOR affirmed the decision of the SDT. Nadal forthwith filed a motion
for reconsideration of the BOR decision.
In the morning of March 29, 1993, the BOR found Nadal guilty and imposed
upon him the penalties of suspension for one (1) year effective March 29, 1993,
non-issuance of any certificate of good moral character during the suspension
and/or as long as Nadal has not reimbursed the STFAP benefits he had received
with 12% interest per annum from march 30, 1993 and non-issuance of his
transcript of records until he has settled his financial obligations with the
university.
Nadal filed with the Regional Trial Court of Quezon City a petition for
mandamus with preliminary injunction and prayer for a temporary restraining
order against President Abueva, the BOR, Oscar M. Alfonso, Cesar A.
Buenaventura, Armand V. Fabella and Olivia C. Caoili.
Petitioners filed the instant petition for certiorari and prohibition with prayer for
the issuance of an injunction and alleged that RTC judge gravely abused her
discretion in issuing a writ of preliminary injunction thereby preventing the
BOR from implementing the suspension penalty it had imposed on Nadal.
Private respondent opposed the petition and argued that Dr. Caoili, not having
been authorized by the Board of Regents as a collegial body to file the instant
petition, and Dr. Abueva, who verified the petition, not being the "Board of
Page | 65

Compilation of Digested Cases for Remedial Law Review II


by: K. M. T. BUELA
Regents" nor "the University of the Philippines," they are not real parties in
interest who should file the same.
ISSUE: Whether or not a respondent to a petition for certiorari may assail the
legal standing of the petitioner when the former had specifically named the latter
in a petition for mandamus for which the injunction was issued and is assailed
in the certiorari petition.
HELD:
The answer is in the negative. A real party in interest is one "who stands to be
benefited or injured by the judgment or the party entitled to the avails of the
suit. 'Interest' within the meaning of the rule means material interest, an interest
in issue and to be affected by the decree, as distinguished from mere interest in
the question involved, or a mere incidental interest."
Undoubtedly, the U.P. Board of Regents has an interest to protect inasmuch as
what is in issue here is its power to impose disciplinary action against a student
who violated the Rules and Regulations on Student Conduct and Discipline by
withholding information in connection with his application for STFAP benefits,
which information, if disclosed, would have sufficed to disqualify him from
receiving the financial assistance he sought. Such dishonesty, if left unpunished,
would have the effect of subverting a commendable program into which the
University officials had devoted much time and expended precious resources,
from the conceptualization to the implementation stage, to rationalize the
socialized scheme of tuition fee payments in order that more students may
benefit from the public funds allocated to the State University.
Having specifically named Drs. Abueva and Caoili as respondents in the
petition for mandamus that he filed below, Nadal is now estopped from
questioning their personality to file the instant petition. Moreover, under Sec.
7 of the U.P. Charter (Act 1870) and Sec. 11 of the University Code "all process"
against the BOR shall be served on "the president or secretary thereof'." It is in
accordance with these legal provisions that Dr. Caoili is named as a petitioner.
Necessarily, Dr. Abueva, the University President and member of the BOR, has
to verify the petition. It is not mandatory, however, that each and every member
of the BOR be named petitioners. As the Court has time and again held, an
action may be entertained, notwithstanding the failure to include an
indispensable party where it appears that the naming of the party would be but
a formality. Mandamus is never issued in doubtful cases, a showing of a clear
and certain right on the part of the petitioner being required. It is of no avail
against an official or government agency whose duty requires the exercise of
discretion or judgment.

Page | 66

Compilation of Digested Cases for Remedial Law Review II


by: K. M. T. BUELA

ROMAN C. TUASON and REMEDIOS V. TUASON vs. REGISTER


OF DEEDS, CALOOCAN City, et al.
G.R. No. 70484, January 29, 1988
FACTS:
Petitioner Spouses Tuasons bought from Carmel Farms, Inc. a piece of land in
Caloocan City. The said land had been earlier purchased by Carmel from the
Government by installment with the following clause: in the event of default
by a purchaser to pay any installment of purchase money and interest thereon,
the Chief of the Bureau of Public Lands had the duty at once to protect the
Government from loss by bringing suit to obtain judicial authority to enforce
the Government's lien.
On September 14, 1973, Marcos issued Presidential Decree No. 293 which
invalidated inter alia the title of the Tuasons' vendor, Carmel. Said Presidential
Decree No. 293 made the finding that Carmel had failed to complete payment
of the price. The Tuason Spouses thereupon filed with this Court a petition for
certiorari assailing the Marcos decree as an arbitrary measure which deprived
them of their property in favor of a selected group.
Marcos' Solicitor General sought to sustain the decree. In his comment on the
petition, he questioned the propriety of the remedy of certiorari resorted to by
the petitioners, it not appearing that the public respondents were being sued as
judicial or quasi-judicial officers who had acted without or in excess of their
jurisdiction, or with grave abuse of discretion.
ISSUE: Whether or not a presidential proclamation annulling a registered title
over a real property may be assailed via petition for certiorari
HELD:
It is true that the extraodinary writ of certiorari may properly issue to nullify
only judicial or quasi-judicial acts, unlike the writ of prohibition which may be
directed against acts either judicial or ministerial. Section 1, Rule 65 of the
Rules of Court deals with the writ of certiorari in relation to "any tribunal, board
or officer exercising judicial functions, while Section 2 of the same Rule treats
of the writ of prohibition in relation to "proceedings of any tribunal, corporation,
board, or person ... exercising functions judicial or ministerial." But the petition
will be shown upon analysis to be in reality directed against an unlawful
exercise of judicial power.
The decree reveals that Mr. Marcos exercised an obviously judicial function.
He made a determination of facts, and applied the law to those facts, declaring
Page | 67

Compilation of Digested Cases for Remedial Law Review II


by: K. M. T. BUELA
what the legal rights of the parties were in the premises. These acts essentially
constitute a judicial function, or an exercise of jurisdiction which is the
power and authority to hear or try and decide or determine a cause. He adjudged
it to be an established fact that neither the original purchasers nor their
subsequent transferees have made full payment of all installments of the
purchase money and interest on the lots claimed by Carmel Farms, Inc.,
including those on which the dwellings of the members of ... (the) Association
(of homeowners) stand." And applying the law to that situation, he made the
adjudication that "title to said land has remained with the Government, and the
land now occupied by the members of said association has never ceased to form
part of the property of the Republic of the Philippines," and that 'any and all acts
affecting said land and purporting to segregate it from the said property of the
Republic ... (were) null and void ab initio as against the law and public policy.
These acts may thus be properly struck down by the writ of certiorari, because
done by an officer in the performance of what in essence is a judicial function,
if it be shown that the acts were done without or in excess of jurisdiction, or
with grave abuse of discretion. Since Mr. Marcos was never vested with judicial
power, such power, as everyone knows, being vested in the Supreme Court and
such inferior courts as may be established by law the judicial acts done by
him were in the circumstances indisputably perpetrated without jurisdiction.
The acts were completely alien to his office as chief executive, and utterly
beyond the permissible scope of the legislative power that he had assumed as
head of the martial law regime.

2. Prohibition (Section 2)
ALFEO D. VIVAS vs. THE MONETARY BOARD OF THE BANGKO
SENTRAL NG PILIPINAS AND THE PHILIPPINE DEPOSIT
INSURANCE CORPORATION
G.R. No. 191424, August 7, 2013
FACTS:
On March 4, 2010, the MB issued Resolution No. 27623 placing Eurocredit
Bank under receivership. Assailing MB Resolution No. 276, Vivas (member of
Eurocredit Bank new management team) filed this petition for prohibition
before this Court, ascribing grave abuse of discretion to the MB for prohibiting
Eurocredit Bank from continuing its banking business and for placing it under
receivership.
ISSUE: Whether or not a petition for prohibition is the proper remedy to assail
an order of the BSP to place a bank under receivership.
Page | 68

Compilation of Digested Cases for Remedial Law Review II


by: K. M. T. BUELA

HELD:
The answer is in the negative. Vivas Availed of the Wrong Remedy.
To begin with, Vivas availed of the wrong remedy. The MB issued Resolution
No. 276, dated March 4, 2010, in the exercise of its power under R.A. No. 7653.
Under Section 30 thereof, any act of the MB placing a bank under
conservatorship, receivership or liquidation may not be restrained or set aside
except on a petition for certiorari. Pertinent portions of Section 30, R.A. 7653
read: x x x x The actions of the Monetary Board taken under this section or
under Section 29 of this Act shall be final and executory, and may not be
restrained or set aside by the court except on petition for certiorari on the
ground that the action taken was in excess of jurisdiction or with such grave
abuse of discretion as to amount to lack or excess of jurisdiction. The petition
for certiorari may only be filed by the stockholders of record representing the
majority of the capital stock within ten (10) days from receipt by the board of
directors of the institution of the order directing receivership, liquidation or
conservatorship.
Prohibition is already unavailing. Granting that a petition for prohibition is
allowed, it is already an ineffective remedy under the circumstances obtaining.
Prohibition or a "writ of prohibition" is that process by which a superior court
prevents inferior courts, tribunals, officers, or persons from usurping or
exercising a jurisdiction with which they have not been vested by law, and
confines them to the exercise of those powers legally conferred. Its office is to
restrain subordinate courts, tribunals or persons from exercising jurisdiction
over matters not within its cognizance or exceeding its jurisdiction in matters of
which it has cognizance.
Indeed, prohibition is a preventive remedy seeking that a judgment be rendered
which would direct the defendant to desist from continuing with the commission
of an act perceived to be illegal. As a rule, the proper function of a writ of
prohibition is to prevent the doing of an act which is about to be done. It is
not intended to provide a remedy for acts already accomplished.
Though couched in imprecise terms, this petition for prohibition apparently
seeks to prevent the acts of closing of ECBI and placing it under receivership.
Resolution No. 276, however, had already been issued by the MB and the
closure of ECBI and its placement under receivership by the PDIC were already
accomplished. Apparently, the remedy of prohibition is no longer appropriate.
Settled is the rule that prohibition does not lie to restrain an act that is
already a fait accompli.

Page | 69

Compilation of Digested Cases for Remedial Law Review II


by: K. M. T. BUELA

ROSENDO R. CORALES vs. REPUBLIC OF THE PHILIPPINES


G.R. No. 186613, August 27, 2013
FACTS:
Petitioner Corales appointed Dr. Angeles as Municipal Administrator during his
Second and Third term as mayor of Nagcarlan, Laguna. However, the
Sangguniang Bayan disapproved petitioner Dr. Angeles appointment on the
ground of nepotism, unfitness and unsatisfactory performance. Even so,
petitioner Dr. Angeles continued to discharge the functions and duties of a
Municipal Administrator for which he received an annual salary.
Andal, Provincial State Auditor of Laguna, issued an Audit Observation
Memorandum (AOM) addressed to petitioner Corales who was asked to
comment/reply. The AOM states that: As Angeles appointment was not
confirmed by Sangguniang Bayan, he is considered merely a de facto officer.
As De Facto officer, he may be entitlted to emoluments of the office for actual
services rendered but instead of the municipality of Nagcarlan, it should be
Petitioner Corales who should shoulder the petitioner Dr. Angeles salary
according to the Local Government Code.
Instead of submitting his comment/reply thereon, petitioners filed a Petition for
Prohibition and Mandamus against Andal to recall its AOM and to eventually
desist from collecting reimbursement from petitioner Corales for the salaries
paid to and received by petitioner Dr. Angeles for the latters services as
Municipal Administrator.
OSG, on Andals behalf, filed a Motion to Dismiss. The trial court denied the
said Motion to Dismiss on the ground that Andal was merely a nominal party.
Respondent Republic filed a Petition for Certiorari with the CA ascribing grave
abuse of discretion amounting to lack or excess of jurisdiction on the part of the
trial court in rendering the Orders dated 17 May 2007 and 5 September 2007,
as it unjustly denied respondents right to actively prosecute the case through a
mere declaration that it was a nominal party despite a clear showing that the
Petition for Prohibition referred to the respondent as a real party in interest.
Court of Appeals granted respondents Petition for Certiorari dismissed
petitioners Petition for Prohibition.
ISSUE: Whether or not a petition for prohibition may be availed of to assail an
Audit Observation Memorandum of the Provincial State Auditor stating that
according to law the petitioner is liable to pay and requiring the petitioner to
submit a comment thereon.
Page | 70

Compilation of Digested Cases for Remedial Law Review II


by: K. M. T. BUELA

HELD:
The answer is in the negative. Settled is the rule that for the courts to exercise
the power of judicial review, the following must be extant: (1) there must be an
actual case calling for the exercise of judicial power; (2) the question must be
ripe for adjudication; and (3) the person challenging must have the "standing."
An actual case or controversy involves a conflict of legal rights, an assertion of
opposite legal claims, susceptible of judicial resolution as distinguished from a
mere hypothetical or abstract difference or dispute. There must be a contrariety
of legal rights that can be interpreted and enforced on the basis of existing law
and jurisprudence. Closely related thereto is that the question must be ripe for
adjudication. A question is considered ripe for adjudication when the act being
challenged has had a direct adverse effect on the individual challenging it.
A mere issuance stating a possible liability and requiring a person to comment
thereon cannot be considered an actual controversy over which the power of
judicial review may be exercise by way of petition for prohibition.
In this case, petitioners action for prohibition was premature. The audit
investigative process was still in its initial phase. There was yet no Notice of
Disallowance issued. And, even granting that the AOM issued to petitioner
Corales is already equivalent to an order, decision or resolution of the Auditor
or that such AOM is already tantamount to a directive for petitioner Corales to
reimburse the salaries paid to petitioner Dr. Angeles, still, the action for
prohibition is premature since there are still many administrative remedies
available to petitioners to contest the said AOM.
Prohibition, being a preventive remedy to seek a judgment ordering the
defendant to desist from continuing with the commission of an act perceived to
be illegal, may only be resorted to when there is "no appeal or any other plain,
speedy, and adequate remedy in the ordinary course of law."

ROLANDO TAN, ELENA TAN and LAMBERTO TAN vs. THE


HONORABLE COURT OF APPEALS and the PEOPLE OF THE
PHILIPPINES
G.R. No. 164966, June 8, 2007
FACTS:
James L. King (King) charged Petitioners with violation of Batas Pambansa
Bilang 22 (B.P. 22) and Estafa. In a Joint Resolution dated November 8, 2002,

Page | 71

Compilation of Digested Cases for Remedial Law Review II


by: K. M. T. BUELA
public respondent Montero found probable cause. Accordingly an information
was filed.
On December 17, 2002, petitioners filed a Petition for Prohibition and
Injunction with Preliminary Injunction and Prayer for Temporary Restraining
Order before the Court of Appeals. They sought to restrain the trial court from
proceeding with the subject criminal cases against them and prayed that the
same be dismissed.
On November 24, 2003, the Court of Appeals issued the assailed Decision
dismissing the petition for lack of merit. After their motion for reconsideration
was denied, petitioners interposed the instant petition for review on certiorari to
review the Decision of the Court of Appeals which dismissed their petition for
prohibition. Therefore, the principal issue is whether resort to the extraordinary
remedy of prohibition was proper.
ISSUE: Whether or not an extraordinary remedy of writ of prohibition may be
availed of to assail the validity of a prosecutors resolution finding probable
cause over a criminal complaint.
HELD:
The answer is in the negative. Basic is the rule that the writ of prohibition is an
extraordinary remedy to prevent the unlawful and oppressive exercise of legal
authority and to provide for a fair and orderly administration of justice.
It is available only when there is no appeal or any plain, speedy and
adequate remedy in the ordinary course of law, and when the proceedings
are done without or in excess of jurisdiction or with grave abuse of
discretion. The petitioner must allege in his petition and establish facts to
show that any other existing remedy is not speedy or adequate.
In fine, the arguments raised in their petition for prohibition ineluctably shows
that petitioners are principally questioning the factual and legal bases of the
finding of probable cause against them. This is but a veiled attempt to litigate
issues which should have been timely appealed to the Secretary of Justice via a
petition for review. However, petitioners, through their own fault, failed to avail
themselves of this remedy. Countless times we have ruled that the extraordinary
remedy of certiorari or prohibition is not a substitute for a lost appeal.

Page | 72

Compilation of Digested Cases for Remedial Law Review II


by: K. M. T. BUELA
3. Mandamus (Section 3)
NILO HIPOS vs. HONORABLE RTC JUDGE TEODORO A. BAY
G.R. Nos. 174813-15, March 17, 2009
FACTS:
Two Information for the crime of rape and one Information for the crime of acts
of lasciviousness were filed against petitioners. Private complainants AAA and
BBB filed a Motion for Reinvestigation asking Judge Bay to order the City
Prosecutor of Quezon City to study if the proper Information had been filed
against petitioners and their co-accused. Judge Bay granted the Motion and
ordered a reinvestigation of the cases.
On the other hand, petitioners filed their Joint Memorandum to Dismiss the
Cases before the City Prosecutor on the ground of lack of probable cause.
On 10 August 2004, the Office of the City Prosecutor affirmed the Information
filed against petitioners. Thereafter, 2nd Assistant City Prosecutor Lamberto C.
de Vera, treating the Joint Memorandum to Dismiss as an appeal of the 10
August 2004 Resolution, reversed the Resolution dated 10 August 2004. On the
same date, the City Prosecutor filed a Motion to Withdraw Information before
Judge Bay.
On 2 October 2006, Judge Bay denied the Motion to Withdraw Information in
an Order of even date. Without moving for a reconsideration of the above
assailed Order, petitioners filed the present Petition for Mandamus
ISSUE: Whether or not the Supreme Court may compel the trial court to dismiss
the case through a writ of mandamus by virtue of the resolution of the office of
the city prosecutor finding no probable cause against the accused and
subsequently filing a motion to withdraw information.
HELD:
The answer is in the negative. Mandamus is an extraordinary writ commanding
a tribunal, corporation, board, officer or person, immediately or at some other
specified time, to do the act required to be done, when the respondent unlawfully
neglects the performance of an act which the law specifically enjoins as a duty
resulting from an office, trust, or station; or when the respondent excludes
another from the use and enjoyment of a right or office to which the latter is
entitled, and there is no other plain, speedy and adequate remedy in the ordinary
course of law.

Page | 73

Compilation of Digested Cases for Remedial Law Review II


by: K. M. T. BUELA
As an extraordinary writ, the remedy of mandamus lies only to compel an
officer to perform a ministerial duty, not a discretionary one; mandamus will
not issue to control the exercise of discretion by a public officer where the law
imposes upon him the duty to exercise his judgment in reference to any manner
in which he is required to act, because it is his judgment that is to be exercised
and not that of the court.
In the case at bar, the act which petitioners pray that we compel the trial court
to do is to grant the Office of the City Prosecutors Motion for Withdrawal of
Information against petitioners. In effect, petitioners seek to curb Judge Bays
exercise of judicial discretion.
There is indeed an exception to the rule that matters involving judgment and
discretion are beyond the reach of a writ of mandamus, for such writ may be
issued to compel action in those matters, when refused. However, mandamus is
never available to direct the exercise of judgment or discretion in a particular
way or the retraction or reversal of an action already taken in the exercise of
either. In other words, while a judge refusing to act on a Motion to Withdraw
Information can be compelled by mandamus to act on the same, he cannot be
compelled to act in a certain way, i.e., to grant or deny such Motion. In the case
at bar, Judge Bay did not refuse to act on the Motion to Withdraw Information;
he had already acted on it by denying the same.
Accordingly, mandamus is not available anymore. If petitioners believed that
Judge Bay committed grave abuse of discretion in the issuance of such Order
denying the Motion to Withdraw Information, the proper remedy of petitioners
should have been to file a Petition for Certiorari against the assailed Order of
Judge Bay.

EX-C1C JIMMY B. SANCHEZ and EX-C2C SALVADOR A.


METEORO vs. ROBERTO T. LASTIMOSO, in his capacity as
DIRECTOR GENERAL OF THE PHILIPPINE NATIONAL POLICE
G.R. No. 161735, September 25, 2007
FACTS:
Petitioner were former constable in the Philippine Constabulary (PC) that were
discharged from the service due to administrative cases filed against them. On
appeal, they were both cleared of all charges and thereafter applied for
reinstatement but their applications were not acted upon even up to the
integration of the PC into the PNP.

Page | 74

Compilation of Digested Cases for Remedial Law Review II


by: K. M. T. BUELA
On the account of the integration, NAPOLCOM issued two resolutions, to wit:
(1) Resolution No. 98-037 - considering as absorbed into the police force,
among others, those who had been discharged by virtue of pending
administrative or criminal cases but who were later acquitted or had their cases
dismissed, and who subsequently filed petitions for reinstatement that were not
acted upon by the PNP; and (2) Resolution No. 98-105 - affirming and
confirming the absorption into the PNP 126 ex-PC constables including therein
the petitioners.
As no absorption order had yet been issued by the Chief of the PNP, the
constables in the list requested the assistance of the Secretary of the Department
of Interior and Local Government (DILG). DILG Secretary sent a memorandum
to the Chief of the PNP.
As petition for absorption is yet to be acted upon, petitioners filed a petition for
mandamus with RTC against the PNP Chief. During the pendency of the said
petition, NAPOLCOM issued Resolution No. 99-061 on April 19, 1999
recalling the earlier Resolution No. 98-105.
ISSUE: Whether or not a public official who has the power to appoint/reinstate
may be compelled by mandamus to appoint or reinstate petitioners previously
discharged from service.
HELD:
No. In order that a writ of mandamus may aptly issue, it is essential that, on the
one hand, petitioner has a clear legal right to the claim that is sought and
that, on the other hand, respondent has an imperative duty to perform that
which is demanded of him. Mandamus will not issue to enforce a right, or to
compel compliance with a duty, which is questionable or over which a
substantial doubt exists. The principal function of the writ of mandamus is to
command and to expedite, not to inquire and to adjudicate. Thus, it is neither
the office nor the aim of the writ to secure a legal right but to implement that
which is already established. Unless the right to relief sought is unclouded,
mandamus will not issue.
Viewed in light of the said guideposts, the PNP Chiefs issuance of the orders
for the absorption of herein petitioners in the police force is not compellable by
a writ of mandamus precisely because the same does not involve a performance
of a ministerial duty. Let it be noted that petitioners were discharged from the
PC service, subsequently cleared of the charges against them, applied for
reinstatement but their applications were not acted upon until the integration of
the PC into the PNP in 1990 when R.A. No. 697523 was enacted. Thus, we no
longer speak of the reinstatement of the petitioners to the service because the
Page | 75

Compilation of Digested Cases for Remedial Law Review II


by: K. M. T. BUELA
Philippine Constabulary no longer exists, but of their employment in the PNP
which is, as we held in Gloria v. De Guzman, technically an issuance of a new
appointment. The power to appoint is essentially discretionary to be
performed by the officer in which it is vested according to his best lights,
the only condition being that the appointee should possess the qualifications
required by law.
Consequently, it cannot be the subject of an application for a writ of mandamus.
Furthermore, the petitioners do not have a clear legal right over the issuance
of the absorption orders. They cannot claim the right to be issued an
appointment based on the NAPOLCOM issuances, specifically Resolution Nos.
98-037 and 98-105. Suffice it to state that R.A. No. 6975 clearly provides that
the power to appoint PNP personnel with the rank of "Police Officer I" to
"Senior Police Officer IV" to which petitioners may be appointed27 is vested in
the PNP regional director or in the Chief of the PNP as the case may be, and not
in the NAPOLCOM.

SOCIAL JUSTICE SOCIETY (SJS) et al. vs. HON. JOSE L. ATIENZA,


JR.
G.R. No. 156052, March 7, 2007
FACTS:
Sangguniang Panlungsod of Manila enacted Ordinance No. 8027 which became
effective on December 28, 2001. Ordinance No. 8027 reclassified the area
described therein from industrial to commercial and directed the owners and
operators of businesses disallowed under Section 1 to cease and desist from
operating their businesses within six months from the date of effectivity of the
ordinance. Among the businesses situated in the area are the so-called
"Pandacan Terminals" of the oil companies Caltex (Philippines), Inc., Petron
Corporation and Pilipinas Shell Petroleum Corporation.
However, implementation of the ordinance was repeatedly postpone. Thus,
petitioners filed this original action for mandamus on December 4, 2002 praying
that Mayor Atienza be compelled to enforce Ordinance No. 8027 and order the
immediate removal of the terminals of the oil companies. Petitioners contend
that respondent has the mandatory legal duty, under Section 455 (b) (2) of the
Local Government Code (RA 7160), to enforce Ordinance No. 8027 and order
the removal of the Pandacan Terminals of the oil companies. Instead, he has
allowed them to stay.
ISSUE: Whether or not mandamus may be issued in order to compel the
enforcement of an ordinance.
Page | 76

Compilation of Digested Cases for Remedial Law Review II


by: K. M. T. BUELA

HELD:
The answer is in the affirmative. Mandamus is an extraordinary writ that is
employed to compel the performance, when refused, of a ministerial duty that
is already imposed on the respondent and there is no other plain, speedy and
adequate remedy in the ordinary course of law. The petitioner should have a
well-defined, clear and certain legal right to the performance of the act and it
must be the clear and imperative duty of respondent to do the act required to be
done.
When a mandamus proceeding concerns a public right and its object is to
compel a public duty, the people who are interested in the execution of the laws
are regarded as the real parties in interest and they need not show any specific
interest. Besides, as residents of Manila, petitioners have a direct interest in the
enforcement of the citys ordinances. Respondent never questioned the right of
petitioners to institute this proceeding.
On the other hand, the Local Government Code imposes upon respondent the
duty, as city mayor, to "enforce all laws and ordinances relative to the
governance of the city." One of these is Ordinance No. 8027. As the chief
executive of the city, he has the duty to enforce Ordinance No. 8027 as long as
it has not been repealed by the Sanggunian or annulled by the courts.21 He has
no other choice. It is his ministerial duty to do so.
These officers cannot refuse to perform their duty on the ground of an alleged
invalidity of the statute imposing the duty. The reason for this is obvious. It
might seriously hinder the transaction of public business if these officers were
to be permitted in all cases to question the constitutionality of statutes and
ordinances imposing duties upon them and which have not judicially been
declared unconstitutional. Officers of the government from the highest to the
lowest are creatures of the law and are bound to obey it.

DENNIS A.B. FUNA vs. MANILA ECONOMIC AND CULTURAL


OFFICE and the COMMISSION ON AUDIT
G.R. No. 193462, February 4, 2014
FACTS:
Petitioner sent a letter to the COA requesting for a "copy of the latest financial
and audit report" of the MECO invoking, for that purpose, his "constitutional
right to information on matters of public concern." In reply to the said letter,
Assistant Commissioner Naranjo issued a memorandum referring the
petitioners request to COA Assistant Commissioner Emma M. Espina for
Page | 77

Compilation of Digested Cases for Remedial Law Review II


by: K. M. T. BUELA
"further disposition." In this memorandum, however, Assistant Commissioner
Naranjo revealed that the MECO was "not among the agencies audited by any
of the three Clusters of the Corporate Government Sector."
Taking the 25 August 2010 memorandum as an admission that the COA had
never audited and examined the accounts of the MECO, the petitioner filed the
instant petition for mandamus on 8 September 2010. Petitioner filed the suit in
his capacities as "taxpayer, concerned citizen, a member of the Philippine Bar
and law book author." He impleaded both the COA and the MECO.
Petitioner posits that by failing to audit the accounts of the MECO, the COA is
neglecting its duty under Section 2(1), Article IX-D of the Constitution to audit
the accounts of an otherwise bona fide GOCC or government instrumentality.
It is the adamant claim of the petitioner that the MECO is a GOCC without an
original charter or, at least, a government instrumentality, the funds of which
partake the nature of public funds.
COA argues that the instant petition already became moot when COA
Chairperson Maria Gracia M. Pulido-Tan (Pulido-Tan) issued Office Order No.
2011-69850 on 6 October 2011.51 The COA notes that under Office Order No.
2011-698, Chairperson Pulido-Tan already directed a team of auditors to
proceed to Taiwan, specifically for the purpose of auditing the accounts of,
among other government agencies based therein, the MECO.
ISSUE: Whether or not a petition for mandamus may be denied on the ground
of mootness despite allegation of violation of the constitution and involving
paramount public interest.
HELD:
No. The "moot and academic" principle is not a magical formula that can
automatically dissuade the courts in resolving a case. Courts will decide cases,
otherwise moot and academic, if: first, there is a grave violation of the
Constitution; second, the exceptional character of the situation and the
paramount public interest is involved; third, when constitutional issue raised
requires formulation of controlling principles to guide the bench, the bar, and
the public; and fourth, the case is capable of repetition yet evading review.
An allegation as serious as a violation of a constitutional or legal duty, coupled
with the pressing public interest in the resolution of all related issues, prompts
this Court to pursue a definitive ruling thereon, if not for the proper guidance of
the government or agency concerned, then for the formulation of controlling
principles for the education of the bench, bar and the public in general. For this
purpose, the Court invokes its symbolic function.
Page | 78

Compilation of Digested Cases for Remedial Law Review II


by: K. M. T. BUELA

Assuming that the allegations of neglect on the part of the COA were true,
Office Order No. 2011-698 does not offer the strongest certainty that they would
not be replicated in the future. In the first place, Office Order No. 2011-698 did
not state any legal justification as to why, after decades of not auditing the
accounts of the MECO, the COA suddenly decided to do so. Neither does it
state any determination regarding the true status of the MECO. The
justifications provided by the COA, in fact, only appears in the memorandum it
submitted to this Court for purposes of this case.

E. Quo Warranto (Rule 66)


PEDRO MENDOZA vs. RAY ALLAS and GODOFREDO OLORES
G.R. No. 131977, February 4, 1999
FACTS:
Mendoza filed a petition for quo warranto against Allas as the latter was appointed
(by President Ramos) to the formers position while the former was temporarily
designated in Cagayan. The trial court ruled in favor of petitioner and ordered the
ouster of respondent Allas from the position of Director III, and at the same time
directed the reinstatement of petitioner to the same position with payment of full
back salaries and other benefits appurtenant thereto.
Allas appealed to the CA. While the case was pending before said court, respondent
Allas was promoted to the position of Deputy Commissioner of Customs for
Assessment and Operations. The CA dismissed the appeal upon the motion of the
Petitioner. The order of dismissal became final and entry of judgment was made on
March 19, 1996.
On May 9, 1996, petitioner filed with the court a quo a Motion for Execution of its
decision. On July 24, 1996, the court denied the motion on the ground that the
contested position vacated by respondent Allas was now being occupied by Olores
who was not a party to the quo warranto petition. He alleges that he should have
been reinstated despite respondent Olores' appointment because the subject position
was never vacant to begin with. Petitioner's removal was illegal and he was deemed
never to have vacated his office when respondent Allas was appointed to the same.
Respondent Allas' appointment was null and void and this nullity allegedly extends
to respondent Olores, his successor-in-interest.
ISSUE: Whether or not a petition for quo warranto that was granted may be
executed against the new person occupying the position vacated by respondent in
the said petition.
Page | 79

Compilation of Digested Cases for Remedial Law Review II


by: K. M. T. BUELA

HELD:
No. A petition for quo warranto is a proceeding to determine the right of a person
to the use or exercise of a franchise or office and to oust the holder from its
enjoyment, if his claim is not well-founded, or if he has forfeited his right to enjoy
the privilege.
A judgment in quo warranto does not bind the respondent's successor in office,
even though such successor may trace his title to the same source. This follows from
the nature of the writ of quo warranto itself. It is never directed to an officer as
such, but always against the person to determine whether he is constitutionally
and legally authorized to perform any act in, or exercise any function of the office
to which he lays claim. In the case at bar, the petition for quo warranto was filed
by petitioner solely against respondent Allas. What was threshed out before the trial
court was the qualification and right of petitioner to the contested position as against
respondent Ray Allas, not against Godofredo Olores.

MA. LUTGARDA P. CALLEJA et al. vs. JOSE PIERRE A. PANDAY et al.


G.R. No. 168696, February 28, 2006
FACTS:
On May 16, 2005, respondents filed a petition with the Regional Trial Court of San
Jose, Camarines Sur for quo warranto with Damages and Prayer for Mandatory and
Prohibitory Injunction, Damages and Issuance of Temporary Restraining Order
against herein petitioners. Respondents alleged that from 1985 up to the filing of
the petition with the trial court, they had been members of the board of directors
and officers of St. John Hospital, Incorporated, but sometime in May 2005,
petitioners, who are also among the incorporators and stockholders of said
corporation, forcibly and with the aid of armed men usurped the powers which
supposedly belonged to Respondents.
ISSUE: Whether or not a petition for quo warranto is the proper remedy to assail
the persons who usurped the powers of a board member of a private corporation.
HELD:
No. Rule 66 of the 1997 Rules of Civil Procedure does not apply to quo warranto
cases against persons who usurp an office in a private corporation.
As declared by Justice Jose Y. Feria in Unilongo v. Court of Appeals, Section 1,
Rule 66 of the 1997 Rules of Civil Procedure is "limited to actions of quo warranto
against persons who usurp a public office, position or franchise; public officers who

Page | 80

Compilation of Digested Cases for Remedial Law Review II


by: K. M. T. BUELA
forfeit their office; and associations which act as corporations without being legally
incorporated
Clearly, the present Rule 66 only applies to actions of quo warranto against persons
who usurp a public office, position or franchise; public officers who forfeit their
office; and associations which act as corporations without being legally
incorporated despite the passage of R.A. No. 8799. It is, therefore, The Interim
Rules of Procedure Governing Intra-Corporate Controversies Under R.A. No. 8799
(hereinafter the Interim Rules) which applies to the petition for quo warranto filed
by respondents before the trial court since what is being questioned is the authority
of herein petitioners to assume the office and act as the board of directors and
officers of St. John Hospital, Incorporated.

LUIS K. LOKIN, JR., as the second nominee of CITIZENS BATTLE


AGAINST CORRUPTION (CIBAC) vs. COMMISSION ON ELECTIONS
and the HOUSE OF REPRESENTATIVES
G.R. Nos. 179431-32, June 22, 2010
FACTS:
CIBAC, a party-list submitted a list of five nominees from which its representatives
would be chosen should CIBAC obtain the required number of qualifying votes in
May 2007 Election. The second nominee in the list is herein petitioner. However,
the president of CIBAC filed a certificate of nomination, substitution and
amendment of the list of nominees whereby it withdrew the nominations of Second,
Fourth and Fifth nominees.
After the election, as CIBAC was declared to be entitled to two seats, it was
requested to the Secretary General of the House of Representative to have the
petitioner sworn in as the second nominee of the party list. The said request was not
granted in view of the pending motion for substitution of nominees. COMELEC
approves the withdrawal and substitution of the nomination. As a result, the
COMELEC en banc proclaimed Cruz-Gonzales (third nominee) as the official
second nominee of CIBAC.
Hence, Lokin seeks through mandamus to compel respondent COMELEC to
proclaim him as the official second nominee of CIBAC and assails Section 13 of
Resolution No. 7804 promulgated on January 12, 2007.
The COMELEC posits that once the proclamation of the winning party-list
organization has been done and its nominee has assumed office, any question
relating to the election, returns and qualifications of the candidates to the House of
Representatives falls under the jurisdiction of the HRET pursuant to Section 17,
Page | 81

Compilation of Digested Cases for Remedial Law Review II


by: K. M. T. BUELA
Article VI of the 1987 Constitution. Thus, Lokin should raise the question he poses
herein either in an election protest or in a special civil action for quo warranto in
the HRET, not in a special civil action for certiorari in this Court.
ISSUE: Whether or not special civil action for quo warranto is the proper remedy
of an alleged winning party-list nominee seeking to oust proclaimed nominee of the
same party list.
HELD:
No. A special civil action for quo warranto refers to questions of disloyalty to the
State, or of ineligibility of the winning candidate. The objective of the action is to
unseat the ineligible person from the office, but not to install the petitioner in his
place. Any voter may initiate the action, which is, strictly speaking, not a contest
where the parties strive for supremacy because the petitioner will not be seated even
if the respondent may be unseated.
The controversy involving Lokin is neither an election protest nor an action for quo
warranto, for it concerns a very peculiar situation in which Lokin is seeking to be
seated as the second nominee of CIBAC. Although an election protest may properly
be available to one party-list organization seeking to unseat another party-list
organization to determine which between the defeated and the winning party-list
organizations actually obtained the majority of the legal votes, Lokins case is not
one in which a nominee of a particular party-list organization thereby wants to
unseat another nominee of the same party-list organization. Neither does an action
for quo warranto lie, considering that the case does not involve the ineligibility and
disloyalty of Cruz-Gonzales to the Republic of the Philippines, or some other cause
of disqualification for her.
Lokin has correctly brought this special civil action for certiorari against the
COMELEC to seek the review of the September 14, 2007 resolution of the
COMELEC in accordance with Section 7 of Article IX-A of the 1987 Constitution,
notwithstanding the oath and assumption of office by Cruz-Gonzales. The
constitutional mandate is now implemented by Rule 64 of the 1997 Rules of Civil
Procedure, which provides for the review of the judgments, final orders or
resolutions of the COMELEC and the Commission on Audit. As Rule 64 states, the
mode of review is by a petition for certiorari in accordance with Rule 65 to be filed
in the Supreme Court within a limited period of 30 days. Undoubtedly, the Court
has original and exclusive jurisdiction over Lokins petitions for certiorari and for
mandamus against the COMELEC.

Page | 82

Compilation of Digested Cases for Remedial Law Review II


by: K. M. T. BUELA
EFREN RACEL ARA TEA vs. COMMISSiON ON ELECTIONS and
ESTELA D. ANTlPOLO
G.R. No. 195229, October 9, 2012
FACTS:
Lonzanida and Antipolo were candidates for 2010 Mayoralty candidates. Lonzanida
was disqualify pursuant to Rodolfos petition under Section 78 of the Omnibus
Election Code. The ground for the disqualification is to the fact that Lonzanida was
elected, and had served, as mayor of San Antonio, Zambales for four (4)
consecutive terms immediately prior to the term for the May 2010 elections.
Lonzanida filed an MR. Pending the resolution of the MR, 2010 Election resulted
to Lonzanida and Aratea garnered the highest number of votes and were
respectively proclaimed Mayor and Vice-Mayor.
Pursuant to a DILG opinion that in light of Lonzanidas pending MR, the office of
the mayor is considered vacant, Aratea took his oath of office as Acting Mayor.
On 11 August 2010, the COMELEC En Banc resolved to deny Lonzanidas MR,
thereby upholding his disqualification. On 25 August 2010, Antipolo, contender in
2010 mayoralty election who garnered the second highest votes, filed a Motion for
Leave to Intervene and to Admit Attached Petition-in-Intervention. She claimed her
right to be proclaimed as Mayor.
In his Comment filed, Aratea opposed the Antipolos motion. However,
COMELEC allowed the intervention and ruled that Lonzanidas disqualification
and ineligibility to hold public office is established both in fact and in law on
election day itself. COMELEC further ruled that it should be Antipolo and not
Aratea that should be the mayor.
ISSUE: Whether or not the second placer should in election should be declared as
mayor in the event that the winning candidate was disqualify.
HELD:
In eligibility requirements and disqualification issues of election candidates, there
are three remedies available, namely: (1) cancellation of a CoC; (2) disqualification
from candidacy or from holding office; and (3) quo warranto. These three (3)
remedies may be differentiated from each other into three ways, i.e., as to grounds,
period of filing and effects.
As to the grounds
In cancellation of a CoC, the ground is essentially lack of eligibility under the
pertinent constitutional and statutory provisions on qualifications or eligibility for
public office; the governing provisions are Sections 78 and 69 of the OEC. In a
Page | 83

Compilation of Digested Cases for Remedial Law Review II


by: K. M. T. BUELA
disqualification case, as mentioned above, the grounds are traits, conditions,
characteristics or acts of disqualification, individually applicable to a candidate.
The grounds for disqualification are different from, and have nothing to do with, a
candidates CoC. Section 78 of the Omnibus Election Code states that a certificate
of candidacy may be denied or cancelled when there is false material representation
of the contents of the certificate of candidacy. Material representation may relate to
the qualifications required of the public office he/she is running for.
In a quo warranto petition, the grounds to oust an elected official from his office
are ineligibility and disloyalty to the Republic of the Philippines. This is provided
under Section 253 of the OEC and governed by the Rules of Court as to procedures.
While quo warranto and cancellation share the same ineligibility grounds, they
differ as to the time these grounds are cited. Under section 78, the qualifications for
elective office are misrepresented in the certificate of candidacy and the
proceedings must be initiated before the elections, whereas a petition for quo
warranto under section 253 may be brought on the basis of two grounds - (1)
ineligibility or (2) disloyalty to the Republic of the Philippines, and must be initiated
within ten days after the proclamation of the election results. Under section 253, a
candidate is ineligible if he is disqualified to be elected to office, and he is
disqualified if he lacks any of the qualifications for elective office.
As to the period for filing:
The period to file a petition to deny due course to or cancel a CoC depends on the
provision of law invoked. If the petition is filed under Section 78 of the OEC, the
petition must be filed within twenty-five (25) days from the filing of the CoC.
However, if the petition is brought under Section 69 of the same law, the petition
must be filed within five (5) days from the last day of filing the CoC.
On the other hand, the period to file a disqualification case is at any time before the
proclamation of a winning candidate, as provided in COMELEC Resolution No.
8696.
As to the effects of a successful suit:
A candidate whose CoC was denied due course or cancelled is not considered a
candidate at all. A "candidate" whose CoC has been cancelled or denied due course
cannot be substituted for lack of a CoC, to all intents and purposes. Similarly, a
successful quo warranto suit results in the ouster of an already elected official from
office; substitution, for obvious reasons, can no longer apply.
On the other hand, a candidate who was simply disqualified is merely prohibited
from continuing as a candidate or from assuming or continuing to assume the
functions of the office; substitution can thus take place under the terms of Section
Page | 84

Compilation of Digested Cases for Remedial Law Review II


by: K. M. T. BUELA
77 of the OEC.32 However, a three-term candidate with a valid and subsisting CoC
cannot be substituted if the basis of the substitution is his disqualification on account
of his three-term limitation. Disqualification that is based on a breach of the threeterm limit rule cannot be invoked as this disqualification can only take place after
election where the three-term official emerged as winner. As in a quo warranto, any
substitution is too late at this point.
As to the effects of a successful suit on the right of the second placer in the
elections:
In any of these three remedies, the doctrine of rejection of the second placer applies.
As an exceptional situation, however, the candidate with the second highest number
of votes (second placer) may be validly proclaimed as the winner in the elections
should the winning candidate be disqualified by final judgment before the elections.
In a CoC cancellation proceeding, the law is silent on the legal effect of a judgment
cancelling the CoC and does not also provide any temporal distinction. Given,
however, the formal initiatory role a CoC plays and the standing it gives to a
political aspirant, the cancellation of the CoC based on a finding of its invalidity
effectively results in a vote for an inexistent "candidate" or for one who is deemed
not to be in the ballot. Although legally a misnomer, the "second placer" should be
proclaimed the winner as the candidate with the highest number of votes for the
contested position. This same consequence should result if the cancellation case
becomes final after elections, as the cancellation signifies non-candidacy from the
very start, i.e., from before the elections.
In this case, the SC ruled that Lonzanida's certificate of candidacy was
cancelled because he was ineligible or not qualified to run for Mayor. Whether
his certificate of candidacy is cancelled before or after the elections is
immaterial because the cancellation on such ground means he was never a
candidate from the very beginning, his certificate of candidacy being void ab
initio. There was only one qualified candidate for Mayor in the May 2010
elections - Anti polo, who therefore received the highest number of votes.
NB: note that the petition filed against Lonzanida is not for the cancellation of his
CoC but for disqualification for false misrepresenatation.

EMMANUEL A. DE CASTRO vs. EMERSON S. CARLOS


G.R. No. 194994, April 16, 2013
FACTS:
In 2009, Petitioner was appointed by PGMA as assistant general manager for
operations (AGMO) of the MMDA. In 2010, Exec. Sec. Ochoa issued OPPage | 85

Compilation of Digested Cases for Remedial Law Review II


by: K. M. T. BUELA
Memorandum Circular No. 2 stating that: All non-Career Executive Service
Officials (non-CESO) occupying Career Executive Service (CES) positions in all
agencies of the executive branch shall remain in office and continue to perform their
duties and discharge their responsibility until October 31, 2010 or until their
resignations have been accepted and/or until their respective replacements have
been appointed or designated, whichever comes first, unless they are reappointed in
the meantime. Pursuant to the said Circular petitioner was terminated.
When Petitioner sought a clarification from the Career Executive Service Board
(CESB) as to the proper classification of the position of AGMO, CESB Executive
Director stated that the position of AGMO had not yet been classified and petitioner
was not covered by OP Memorandum Circular Nos. 1 and 2.
Demanding payment of his salary and reinstatement in the monthly payroll,
petitioner sent a letter to MMD Administration Service. For his failure to obtain an
action or a response from MMDA, he elevated his demand to the Office of the
President. Meanwhile a new AGMO for MMDA was appointed. Hence, Petitioner
filed instant Petition for the issuance of a writ of quo warranto seeking to oust
respondent from the position of AGMO.
In his comment, respondent posits that the AGMO position belongs to the CES;
thus, in order to have security of tenure, petitioner, must be a Career Executive
Service official (CESO). Respondent maintains that the function of an AGM is
executive and managerial in nature.
ISSUE: Whether or not petition for quo warranto may be availed of to question an
alleged illegal termination wherein the petitioners right over the position sought
to be reinstated is disputed.
HELD:
No. In a quo warranto proceeding, the person suing must show that he has a clear
right to the office allegedly held unlawfully by another. Absent a showing of that
right, the lack of qualification or eligibility of the supposed usurper is immaterial.
A petition for quo warranto is a proceeding to determine the right of a person to use
or exercise a franchise or an office and to oust the holder from the enjoyment,
thereof, if the claim is not well-founded, or if his right to enjoy the privilege has
been forfeited." Where the action is filed by a private person, in his own name, he
must prove that he is entitled to the controverted position, otherwise, respondent
has a right to the undisturbed possession of the office.
In this case, AGMO position was held to be within the coverage of the CES. An
AGMO performs functions that are managerial in character; exercises management
Page | 86

Compilation of Digested Cases for Remedial Law Review II


by: K. M. T. BUELA
over people, resource, and/or policy; and assumes functions like planning,
organizing, directing, coordinating, controlling, and overseeing the activities of
MMDA. The position requires the application of managerial or supervisory skills
necessary to carry out duties and responsibilities involving functional guidance,
leadership, and supervision. For the foregoing reasons, the position of AGMO is
within the coverage of the CES.
While, the law permits, on many occasions, the appointment of non-CES eligibles
to CES positions in the government in the absence of appropriate eligibles and when
there is necessity in the interest of public service to fill vacancies in the government.
But in all such cases, the appointment is at best merely temporary as it is said to
be conditioned on the subsequent obtention of the required CES eligibility.
Petitioner undisputedly lacked CES eligibility. Thus, he did not hold the position of
AGMO in a permanent capacity or acquire security of tenure in that position.
Otherwise stated, his appointment was temporary and "co-terminus with the
appointing authority."

F. Expropriation (Rule 67)


CITY OF MANILA vs. OSCAR SERRANO et al.
G.R. No. 142304, June 20, 2001
FACTS:
City Council of Manila enacted Ordinance No. 7833, authorizing the expropriation
of certain properties in Manilas First District in Tondo. Upon motion by petitioner,
the trial court issued an order, directing petitioner to deposit the amount of
P1,825,241.00 equivalent to the assessed value of the properties. After petitioner
had made the deposit, the trial court issued another order, directing the issuance of
a writ of possession in favor of petitioner.
Respondents filed a petition for certiorari with the Court of Appeals held that in
accordance with the ruling in Filstream International Inc. v. Court of Appeals, the
other modes of acquisition of lands enumerated in 9-10 of the law must first be
tried by the city government before it can resort to expropriation. As petitioner
failed to show that it had done so, the Court of Appeals gave judgment for
respondents and enjoined petitioner from expropriating Lot 1-C.
ISSUE: Whether or not compliance an LGU in an expropriation proceeding to the
legal requirement that the government must try other modes of acquisition before
resorting to expropriation is condition sine qua non before the court may issue a
writ of possession.
Page | 87

Compilation of Digested Cases for Remedial Law Review II


by: K. M. T. BUELA

HELD:
The answer is in the negative. Upon the filing of the complaint or at any time
thereafter and after due notice to the defendant, the plaintiff shall have the right to
take or enter upon the possession of the real property involved if he deposits with
the authorized government depositary an amount equivalent to the assessed value
of the property for purposes of taxation to be held by such bank subject to the orders
of the court. Such deposit shall be in money, unless in lieu thereof the court
authorizes the deposit of a certificate of deposit of a government bank of the
Republic of the Philippines payable on demand to the authorized government
depositary.
After such deposit is made the court shall order the sheriff or other proper officer to
forthwith place the plaintiff in possession of the property involved and promptly
submit a report thereof to the court with service of copies to the parties. Thus, a
writ of execution may be issued by a court upon the filing by the government
of a complaint for expropriation sufficient in form and substance and upon
deposit made by the government of the amount equivalent to the assessed value
of the property subject to expropriation. Upon compliance with these
requirements, the issuance of the writ of possession becomes ministerial. In this
case, these requirements were satisfied and, therefore, it became the ministerial duty
of the trial court to issue the writ of possession.
The Court of Appeals, however, ruled that petitioner failed to comply with the
requirements laid down in 9-10 of R.A. No. 7279 and reiterated in the Filstream
ruling. This is error. The ruling in Filstream was necessitated because an order of
condemnation had already been issued by the trial court in that case. Thus, the
judgment in that case had already become final. In this case, the trial court has not
gone beyond the issuance of a writ of possession. Hearing is still to be held to
determine whether or not petitioner indeed complied with the requirements
provided in R.A. No. 7279. It is, therefore, premature at this stage of the
proceedings to find that petitioner resorted to expropriation without first trying the
other modes of acquisition enumerated in 10 of the law.

NATIONAL POWER CORPORATION vs. COURT OF APPEALS and


ANTONINO POBRE
G.R. No. 106804, August 12, 2004
FACTS:
NPC is authorized by law to acquire property and exercise the right of eminent
domain. Private respondent Pobre is the owner of a 68,969 square-meter land
located in Tiwi, Albay. In 1963, Pobre began developing the Property as a resortPage | 88

Compilation of Digested Cases for Remedial Law Review II


by: K. M. T. BUELA
subdivision. On 18 February 1972 when Pobre leased to NPC for one year eleven
lots from the approved subdivision plan. Subsequently, NPC filed two expropriation
case over a portion of the said property, i.e., first for the 8,311.60 square-meter
portion of the Property and second for the 5,554 square-meter lot (1 September
1979).
On 2 January 1985, NPC filed a motion to dismiss the second expropriation case on
the ground that NPC had found an alternative site. The trial court granted NPCs
motion to dismiss but the trial court allowed Pobre to adduce evidence on his claim
for damages. Thereafter, trial court issued a judgment ordering the plaintiff to pay
the defendant P3,448,450.00 PESOS which is the fair market value of the
subdivision of defendant with an area of sixty eight thousand nine hundred sixty
nine (68,969) square meters, plus legal rate of interest per annum from September
6, 1979 and damages.
The Court of Appeals affirmed the decision of the trial court. NPC insists that at the
time that it moved for the dismissal of its complaint, Pobre had yet to serve an
answer or a motion for summary judgment on NPC. Thus, NPC as plaintiff had the
right to move for the automatic dismissal of its complaint. NPC relies on Section 1,
Rule 17 of the 1964 Rules of Court, the Rules then in effect. NPC argues that the
dismissal of the complaint should have carried with it the dismissal of the entire
case including Pobres counterclaim.
ISSUE: Whether or not the rule on dismissal of action as a matter of right is
applicable to a special civil action of expropriation.
HELD:
No. Section 1, Rule 17 of the 1964 Rules of Court provided the exception to the
general rule that the dismissal of the complaint is addressed to the sound discretion
of the court. For as long as all of the elements of Section 1, Rule 17 were present
the dismissal of the complaint rested exclusively on the plaintiffs will. The
defending party and even the courts were powerless to prevent the dismissal. The
courts could only accept and record the dismissal.
A plain reading of Section 1, Rule 17 of the 1964 Rules of Court makes it obvious
that this rule was not intended to supplement Rule 67 of the same Rules. Section 1,
Rule 17 of the 1964 Rules of Court, provided that:
SECTION 1. Dismissal by the plaintiff. An action may be
dismissed by the plaintiff without order of court by filing a
notice of dismissal at any time before service of the answer
or of a motion for summary judgment. Unless otherwise
stated in the notice, the dismissal is without prejudice, except
Page | 89

Compilation of Digested Cases for Remedial Law Review II


by: K. M. T. BUELA
that a notice operates as an adjudication upon the merits
when filed by a plaintiff who has once dismissed in a
competent court an action based on or including the same
claim. A class suit shall not be dismissed or compromised
without approval of the court.
While Section 1, Rule 17 spoke of the service of answer or summary judgment,
the Rules then did not require the filing of an answer or summary judgment in
eminent domain cases. In lieu of an answer, Section 3 of Rule 67 required the
defendant to file a single motion to dismiss where he should present all of his
objections and defenses to the taking of his property for the purpose specified in the
complaint. In short, in expropriation cases under Section 3 of Rule 67, the motion
to dismiss took the place of the answer.
The records show that Pobre had already filed and served on NPC his motion to
dismiss/answer even before NPC filed its own motion to dismiss. NPC filed its
notice of dismissal of the complaint on 2 January 1985.
In expropriation cases, there is no such thing as the plaintiffs matter of right to
dismiss the complaint precisely because the landowner may have already suffered
damages at the start of the taking. The plaintiffs right in expropriation cases to
dismiss the complaint has always been subject to court approval and to certain
conditions. The exceptional right that Section 1, Rule 17 of the 1964 Rules of Court
conferred on the plaintiff must be understood to have applied only to other civil
actions. The 1997 Rules of Civil Procedure abrogated this exceptional right.
Thus, NPCs theory that the dismissal of its complaint carried with it the dismissal
of Pobres claim for damages is baseless. There is nothing in Rule 67 of the 1964
Rules of Court that provided for the dismissal of the defendants claim for damages,
upon the dismissal of the expropriation case. Case law holds that in the event of
dismissal of the expropriation case, the claim for damages may be made either in a
separate or in the same action, for all damages occasioned by the institution of the
expropriation case. The dismissal of the complaint can be made under certain
conditions, such as the reservation of the defendants right to recover damages
either in the same or in another action. The trial court in this case reserved Pobres
right to prove his claim in the same case, a reservation that has become final due to
NPCs own fault.

Page | 90

Compilation of Digested Cases for Remedial Law Review II


by: K. M. T. BUELA
REPUBLIC OF THE PHILIPPINES (DPWH) vs. ISMAEL ANDAYA
G.R. No. 160656, June 15, 2007
FACTS:
Republic of the Philippines (Republic) negotiated with Andaya to enforce the 60meter easement of right-of-way. The easement was for concrete levees and
floodwalls for Phase 1, Stage 1 of the Lower Agusan Development Project. The
parties, however, failed to reach an agreement.
On December 13, 1995, the Republic instituted an action to enforce the easement
of right-of-way or eminent domain. The trial court issued a writ of possession on
April 26, 1996. It also constituted a Board of Commissioners (Board) to determine
the just compensation. Eventually, the trial court issued an Order of Expropriation
upon payment of just compensation.
On December 10, 1998, the Board reported that the project would affect a total of
10,380 square meters of Andayas properties, 4,443 square meters of which will be
for the 60-meter easement. The Board also reported that the easement would
diminish the value of the remaining 5,937 square meters. As a result, it
recommended the payment of consequential damages amounting to P2,820,430 for
the remaining area.
ISSUE: Whether or not the Republic liable for just compensation if in enforcing
the legal easement of right-of-way on a property, the remaining area would be
rendered unusable and uninhabitable. (Indirect expropriation)
HELD:
The answer is in the affirmative. Taking, in the exercise of the power of eminent
domain, occurs not only when the government actually deprives or dispossesses the
property owner of his property or of its ordinary use, but also when there is a
practical destruction or material impairment of the value of his property. Using this
standard, there was undoubtedly a taking of the remaining area of Andayas
property. True, no burden was imposed thereon and Andaya still retained title and
possession of the property. But, as correctly observed by the Board and affirmed
by the courts a quo, the nature and the effect of the floodwalls would deprive
Andaya of the normal use of the remaining areas. It would prevent ingress and
egress to the property and turn it into a catch basin for the floodwaters coming from
the Agusan River.
For this reason, in our view, Andaya is entitled to payment of just compensation,
which must be neither more nor less than the monetary equivalent of the land. One
of the basic principles enshrined in our Constitution is that no person shall be
deprived of his private property without due process of law; and in expropriation
Page | 91

Compilation of Digested Cases for Remedial Law Review II


by: K. M. T. BUELA
cases, an essential element of due process is that there must be just compensation
whenever private property is taken for public use.

ASIA'S EMERGING DRAGON CORPORATION vs. DEPARTMENT OF


TRANSPORTATION AND COMMUNICATIONS et al.
G.R. No. 169914, April 18, 2008

Page | 92

Compilation of Digested Cases for Remedial Law Review II


by: K. M. T. BUELA

Page | 93

Compilation of Digested Cases for Remedial Law Review II


by: K. M. T. BUELA
SPOUSES LETICIA & JOSE ERVIN ABAD et al. vs. FIL-HOMES
REALTY and DEVELOPMENT CORPORATION and MAGDIWANG
REALTY CORPORATION
G.R. No. 189239, November 24, 2010
FACTS:
Respondents are co-owners of two lots situated in Sucat, Paraaque City. They filed
a complaint for unlawful detainer on May 7, 2003 against above-named petitioners
before MeTC. During the pendency of the case, the City of Paraaque filed
expropriation proceedings covering the lots before the Regional Trial Court with
the intention of establishing a socialized housing project therein for distribution to
the occupants including petitioners. A writ of possession was consequently issued
and a Certificate of Turn-over given to the City.
On March 3, 2008, MeTC, rendered judgment in the unlawful detainer case against
petitioners. On appeal, the Regional Trial Court reversed the MeTC decision and
dismissed respondents complaint. The RTC went on to rule that the issuance of a
writ of possession in favor of the City bars the continuation of the unlawful detainer
proceedings, and since the judgment had already been rendered in the expropriation
proceedings which effectively turned over the lots to the City, the MeTC has no
jurisdiction to "disregard the . . . final judgment and writ of possession" due to nonpayment of just compensation.
Respondent filed a petition for review before the CA. CA, citing Republic v.
Gingoyon, held that the issuance of a writ of possession in the expropriation
proceedings does not signify the completion of the expropriation proceedings.
Hence, the MeTC ruling was reinstated.
ISSUE: Whether or not an unlawful detainer case involving a property, which is
later on subjected to expropriation proceedings, is deemed suspended upon the
institution of expropriation proceedings.
HELD:
As a general rule, ejectment proceedings, due to its summary nature, are not
suspended or their resolution held in abeyance despite the pendency of a civil action
regarding ownership.
Section 1 of Commonwealth Act No. 53810 enlightens, however:
Section 1. When the Government seeks to acquire, through purchase or
expropriation proceedings, lands belonging to any estate or chaplaincy
(cappellania), any action for ejectment against the tenants occupying said lands
shall be automatically suspended, for such time as may be required by the
Page | 94

Compilation of Digested Cases for Remedial Law Review II


by: K. M. T. BUELA
expropriation proceedings or the necessary negotiations for the purchase of the
lands, in which latter case, the period of suspension shall not exceed one year.
To avail himself of the benefits of the suspension, the tenants shall pay to the
landowner the current rents as they become due or deposit the same with the court
where the action for ejectment has been instituted. (emphasis and underscoring
supplied)
Petitioners did not comply with any of the acts mentioned in the law to avail of the
benefits of the suspension. They nevertheless posit that since the lots are the subject
of expropriation proceedings, respondents can no longer assert a better right of
possession; and that the City Ordinance authorizing the initiation of expropriation
proceedings designated them as beneficiaries of the lots, hence, they are entitled to
continue staying there.
Expropriation of lands consists of two stages:
The first is concerned with the determination of the authority of the plaintiff to
exercise the power of eminent domain and the propriety of its exercise in the context
of the facts involved in the suit. The second phase of the eminent domain action is
concerned with the determination by the court of "the just compensation for the
property sought to be taken." It is only upon the completion of these two stages that
expropriation is said to have been completed. The process is not complete until
payment of just compensation. Accordingly, the issuance of the writ of possession
in this case does not write finis to the expropriation proceedings. To effectuate the
transfer of ownership, it is necessary for the NPC to pay the property owners the
final just compensation.
In the present case, the mere issuance of a writ of possession in the expropriation
proceedings did not transfer ownership of the lots in favor of the City. Such issuance
was only the first stage in expropriation. There is even no evidence that judicial
deposit had been made in favor of respondents prior to the Citys possession of the
lots, contrary to Section of the LGC.
Respecting petitioners claim that they have been named beneficiaries of the lots,
the city ordinance authorizing the initiation of expropriation proceedings does not
state so. Petitioners cannot thus claim any right over the lots on the basis of the
ordinance.

Page | 95

Compilation of Digested Cases for Remedial Law Review II


by: K. M. T. BUELA
NATIONAL POWER CORPORATION, Petitioner, v. YCLA SUGAR
DEVELOPMENT CORPORATION
G.R. No. 193936, December 11, 2013
FACTS:
YCLA is the registered owner of three parcels of land situated in Puerto Galera. In
order to complete its 69 KV CalapanMamburao Island Grid Project in Puerto
Galera, Oriental Mindoro, NPC had to construct transmission lines that would
traverse several private properties, including the said parcels of land owned by
YCLA. Accordingly, on December 2, 1997, NPC filed a Complaint for
expropriation with the RTC against YCLA and several other individuals.
On May 2, 2001, the Board of Commissioners submitted its Report, which fixed the
amount of just compensation of the subject properties at P500.00 per sqm. YCLA
objected to the amount recommended by the Board of Commissioners. On
September 15, 2003, after conducting an ocular inspection requested by YCLA, the
Board of Commissioners submitted its second Report, which fixed the just
compensation of the subject properties at P1,000.00 per sqm.
RTC rendered a Decision and adopted the report and recommendation of the Board
of Commissioners. NPC appealed the RTC Decision to the CA, alleging that the
amount of P1,000.00 per sqm recommended by the Board of Commissioners as the
reasonable amount of just compensation, which was adopted by the RTC, is too
excessive considering that the subject properties were barren and undeveloped
agricultural lands at the time it instituted the action for expropriation.
ISSUE: Whether or not the Board of Commissioners tasked to determine the just
compensation may revised its previous determination according to the fair market
value of the subject property determine through an ocular inspection conducted five
years after the taking.
HELD:
No. It is settled that the amount of just compensation is to be ascertained as of the
time of the taking, which usually coincides with the commencement of the
expropriation proceedings. Where the institution of the action precedes entry into
the property, the amount of just compensation is to be ascertained as of the time of
the filing of the complaint.
In this case, in arriving at the amount of just compensation, both the RTC and the
CA relied heavily on the Board of Commissioners Report dated September 15,
2003, which, in turn, was arrived at after conducting an ocular inspection of the
subject properties on August 27, 2003. However, the Board of Commissioners
recommendation as to the amount of just compensation was based on the prevailing
Page | 96

Compilation of Digested Cases for Remedial Law Review II


by: K. M. T. BUELA
market value of the subject properties in 2003. What escaped the attention of the
lower courts is that the prevailing market value of the subject properties in 2003
cannot be used to determine the amount of just compensation considering that the
Complaint for expropriation was filed by NPC on December 2, 1997.

G. Foreclosure of Real Estate Mortgage (Rule 68)


JOSE T. RAMIREZ vs. THE MANILA BANKING CORPORATION
G.R. No. 198800, December 11, 2013
FACTS:
Jose T. Ramirez mortgaged two parcels of land in favor of respondent The Manila
Banking Corporation to secure his P265,000 loan. The real estate mortgage provides
that all correspondence relative to the mortgage including notifications of
extrajudicial actions shall be sent to petitioner Ramirez at his given address.
Respondent filed a request for extrajudicial foreclosure of real estate mortgage9
before Atty. Hipolito Saez on the ground that Ramirez failed to pay his loan despite
demands. During the auction sale on September 8, 1994, respondent was the only
bidder for the mortgaged properties.10 Thereafter, a certificate of sale11 was issued
in its favor as the highest bidder.
Ramirez sued respondent for annulment of sale and prayed that the certificate of
sale be annulled on the ground, among others, that paragraph N of the real estate
mortgage was violated for he was not notified of the foreclosure and auction sale.
The trial court ruled that the extrajudicial foreclosure proceedings were null and
void and the certificate of sale is invalid. The CA reversed the trial courts decision
ISSUE: Whether or not the breach in the stipulation requiring personal notice to
the mortgagor annuls a concluded extrajudicial foreclosure sale.
HELD:
Yes. A contract is the law between the parties and absent any showing that its
provisions are wholly or in part contrary to law, morals, good customs, public order,
or public policy, it shall be enforced to the letter by the courts. When respondent
failed to send the notice of extrajudicial foreclosure sale to Ramirez, it committed
a contractual breach of said paragraph N sufficient to render the extrajudicial
foreclosure sale on September 8, 1994 null and void.
Section 3, Act No. 3135 reads: The Act only requires (1) the posting of notices of
sale in three public places, and (2) the publication of the same in a newspaper of
general circulation. Personal notice to the mortgagor is not necessary.
Page | 97

Compilation of Digested Cases for Remedial Law Review II


by: K. M. T. BUELA
Nevertheless, the parties to the mortgage contract are not precluded from exacting
additional requirements.
In Carlos Lim, et al. v. Development Bank of the Philippines, we held that unless
the parties stipulate, personal notice to the mortgagor in extrajudicial foreclosure
proceedings is not necessary because Section 3 of Act No. 3135 only requires the
posting of the notice of sale in three public places and the publication of that notice
in a newspaper of general circulation. In this case, the parties stipulated in paragraph
N of the real estate mortgage that all correspondence relative to the mortgage
including notifications of extrajudicial actions shall be sent to mortgagor Ramirez
at his given address. Respondent had no choice but to comply with this contractual
provision it has entered into with Ramirez. The contract is the law between them.
Hence, we cannot agree with the bank that paragraph N of the real estate mortgage
does not impose an additional obligation upon it to provide personal notice of the
extrajudicial foreclosure sale to the mortgagor Ramirez.

SPOUSES NICASIO C. MARQUEZ AND ANITA J. MARQUEZ vs.


SPOUSES CARLITO ALINDOG AND CARMEN ALINDOG
G.R. No. 184045, January 22, 2014
FACTS:
Petitioner Anita Marquez extended a loan to Benjamin Gutierrez secured by a Real
Estate Mortgage over a parcel of land located in Tagaytay. The mortgage was duly
annotated on the dorsal portion of TCT No. T-13443, which Sps. Marquez had
verified as clean prior to the mortgage.
Since Gutierrez defaulted in the payment of his loan obligation, Anita sought the
extra-judicial foreclosure of the subject property. At the public auction sale, Anita
emerged as the highest bidder. Upon Gutierrezs failure to redeem the same
property within the prescribed period, title was consolidated under the name of Sps.
Marquez, which, however, bore an annotation of adverse claim dated March 2, 2000
in the names of respondents-spouses Alindog. Said annotation was copied from an
earlier annotation on TCT No. T-13443 made only after the subject propertys
mortgage to Sps. Marquez.
Subsequently, Sps. Alindog filed a civil case for annulment of real estate mortgage
and certificate of sale against Sps. Marquez. Meanwhile, Anita filed an ex-parte
petition for the issuance of a writ of possession before the RTC. RTC granted the
same. Claiming that they would suffer irreparable injury, Sps. Alindog sought the
issuance of a temporary restraining order (TRO) and/or writ of preliminary
injunction. After further proceedings on the injunction case, the RTC issued a writ

Page | 98

Compilation of Digested Cases for Remedial Law Review II


by: K. M. T. BUELA
of preliminary injunction enjoining Sps. Marquez from taking possession of the
subject property until after the controversy has been fully resolved on the merits.
Aggrieved, Sps. Marquez moved for reconsideration, essentially pointing out that,
as the confirmed and registered owners of the subject property, they are entitled to
its possession as a matter of right. They argued that pursuant to Sections 728 and
829 of Act No. 3135, as amended by Act No. 4118, the RTC was legally bound to
place them in possession of the subject property pending resolution of the
annulment case. Further, it is their position that the purpose for the issuance of the
injunctive writ i.e., to restrain the implementation of the writ of possession had
already been rendered moot and academic by its actual enforcement in the interim.
ISSUE: Whether or not the trial court may issue a writ of preliminary injunction to
enjoin the possession of the winning bidder in a foreclosure sale after the lapse of
the redemption period.
HELD:
The answer is in the negative. It is an established rule that the purchaser in an
extra-judicial foreclosure sale is entitled to the possession of the property and
can demand that he be placed in possession of the same either during (with
bond) or after the expiration (without bond) of the redemption period therefor.
A writ of possession duly applied for by said purchaser should issue as a matter
of course, and thus, merely constitutes a ministerial duty on the part of the
court.
The ministerial issuance of a writ of possession in favor of the purchaser in an extrajudicial foreclosure sale, however, admits of an exception. Section 33, Rule 39 of
the Rules of Court (Rules) pertinently provides that the possession of the mortgaged
property may be awarded to a purchaser in an extra-judicial foreclosure unless a
third party is actually holding the property by adverse title or right.
The phrase a third party who is actually holding the property adversely to the
judgment obligor contemplates a situation in which a third party holds the property
by adverse title or right, such as that of a co-owner, tenant or usufructuary. The coowner, agricultural tenant, and usufructuary possess the property in their own right,
and they are not merely the successor or transferee of the right of possession of
another co-owner or the owner of the property. Notably, the property should not
only be possessed by a third party, but also held by the third party adversely to the
judgment obligor." The third person must therefore claim a right superior to that of
the original mortgagor.
In this case, it is clear that the issuance of a writ of possession in favor of Sps.
Marquez, who had already consolidated their title over the extra-judicially
Page | 99

Compilation of Digested Cases for Remedial Law Review II


by: K. M. T. BUELA
foreclosed property, is merely ministerial in nature. The general rule as herein stated
and not the exception found under Section 33, Rule 39 of the Rules should apply
since Sps. Alindog hinged their claim over the subject property on their purported
purchase of the same from its previous owner, i.e., Sps. Gutierrez (with Gutierrez
being the original mortgagor). Accordingly, it cannot be seriously doubted that Sps.
Alindog are only the latters (Sps. Gutierrez) successors-in-interest who do not have
a right superior to them.

RUSTICO A. ARDIENTE and ASUNCION PALOMARDIENTE vs.


PROVINCIAL SHERIFF, REGISTER OF DEEDS OF QUEZON and
PENINSULA DEVELOPMENT BANK
G.R. No. 148448, August 17, 2004
FACTS:
Spouses Ardiente obtained a loan from the Peninsula Development Bank secured
by Real Estate Mortgage over a parcel of land. Out of the proceeds of the loan, the
Ardientes purchased a mini bus. As Ardientes failed to pay the loan, the bank extrajudicially foreclosed the mortgage and the parcels of land covered thereby were sold
at public auction to the bank which was the highest bidder.
Two days before the period to redeem the foreclosed mortgage expired, Spouses
Ardiente filed before the RTC a complaint for Annulment of Auction Sale with
Preliminary Injunction and Damages Alleging that the requisite of notifying the
mortgagors of the intended extra-judicial foreclosure sale was not duly complied
with. The bank, on the other hand, filed its Answer with Counterclaim and alleged
that petitioners were duly notified of the extra-judicial foreclosure and public
auction sale. There was sufficient notice and publication served to all concerned of
said public auction sale of the properties offered as collaterals
By Decision, the trial court, noting the absence of documentary evidence showing
strict compliance with the statutory requirements on publication of notice of extrajudicial foreclosure of mortgage, declared the extra-judicial foreclosure and the sale
of the mortgaged properties null and void. However, the Court of Appeals reversed
the decision of the trial court after finding the argument of the defendant-appellants
bank et al. that the lack of required notice and publication of the extra-judicial
foreclosure of mortgage was not averred in the complaint, hence, cannot be the basis
of an adverse judgment.
ISSUE: Whether or not the failure of mortgagee to comply with the publication
requirements may be raised on appeal.
HELD:

Page | 100

Compilation of Digested Cases for Remedial Law Review II


by: K. M. T. BUELA
No. The office, purpose or function of the complaint is to inform the defendant
clearly and definitely of the claims made against him so that he may be prepared to
meet the issues at the trial. Otherwise stated, if the wrong or omission of the
defendant is not alleged in the complaint, then the defendant would be precluded
from presenting evidence to refute the imputation of such wrong or present
justification for the alleged omission. In this case, even perfunctory reading of the
Petition and the Amended Petition, readily reveals the absence of any averment
relating to the required posting and publication of the notice of foreclosure sale.
Understandably then, the defendant-appellant Bank saw no need to present the
Sheriffs Certification of Posting and the newspaper where the notice was published
as well as the publishers affidavit. Clearly, the presumption that the Provincial
Sheriff of Quezon has discharged his official duty in a regular manner and that the
defendant-appellant Bank complied with the requirements under the law will
suffice.
Despite the banks repeated claim that the statutory requirements governing extrajudicial foreclosure had been complied with, the plea of banks lack of publication
of notice of foreclosure was not raised by petitioners either in the Amended
Complaint or in the Reply and Answer to Counterclaim. It was not also raised
during the trial as the entire transcripts of the stenographic notes of the proceedings
before the trial court show. Nor even in their memorandum filed before the trial
court, petitioners having merely assailed the lack of "personal" notification to them
of any "intended" extrajudicial foreclosure and the "grossly and greatly inadequate"
purchase price of the lands.
Indeed, as correctly held by the respondent Court, the issue of lack of publication
of the notice of foreclosure of the mortgage was raised only on appeal. Petitioner
does not represent that he directly attacked in his complaint the validity of the
foreclosure because of such lack of notice.

LZK HOLDINGS and DEVELOPMENT CORPORATION vs. PLANTERS


DEVELOPMENT BANK
G.R. No. 187973, January 20, 2014
FACTS:
LZK Holdings obtained a loan from Planters Bank secured the same with a Real
Estate Mortgage over its lot located in La Union. Due to LZK Holdings' failure to
pay its loan, the mortgaged property was extrajudicially foreclosed with Planters
Bank emerged as the highest bidder during the auction sale and its certificate of sale
was registered.

Page | 101

Compilation of Digested Cases for Remedial Law Review II


by: K. M. T. BUELA
LZK Holdings filed a complaint for annulment of extra judicial foreclosure,
mortgage contract, promissory note and damages with prayer for issuance of
TRO/Writ of Preliminary injunction before RTC-Makati. On the other hand,
Planters Bank filed an ex-parte motion for the issuance of a writ of possession with
the RTC-San Fernando.
In the annulment case, RTC-Makati issued a writ of preliminary injunction On April
3, 2000. Consequently, the ex-parte proceedings for the issuance of a writ of
possession was suspended by the RTC-San Fernando in view of the injunction
issued by the RTC-Makati. RTC-Makati declared as null and void the consolidated
title of Planters Bank in an Order dated June 2, 2000. Such ruling was affirmed by
the CA sustained by the SC in a Resolution dated September 13, 2004.
Planters Bank also appealed the Order of the RTC-San Fernando which held in
abeyance the resolution of its ex parte motion for the issuance of a writ of
possession. CA granted the appeal and SC affirmed such decision, to wit: Until the
foreclosure sale of the property in question is annulled by a court of competent
jurisdiction, petitioner is bereft of valid title and of the right to prevent the issuance
of a writ of possession to Planters Bank. Until then, it is the trial court's ministerial
function to grant the possessory writ to Planters Bank.
Armed with the above ruling, Planters Bank filed before the RTC-San Fernando a
motion to set ex-parte hearing for the issuance of a writ of possession. LZK
Holdings opposed the motion. RTC-San Fernando denied the opposition and set the
hearing on April 14, 2008. On April 8, 2008, the RTC-San Fernando declaring the
scheduled hearing moot and academic and granting Planter Bank's ex-parte motion
for the issuance of a writ of possession
ISSUE:
Whether or not hearing in a petition for issuance of writ of possession may be
dispensed with when the SC had already decreed the propriety of the issuance of
such writ.
HELD:
The answer is in the affirmative. Under the principle of conclusiveness of judgment,
the right of Planter's Bank to a writ of possession as adjudged in G.R. No. 167998
is binding and conclusive on the parties.
The doctrine of res judicata by conclusiveness of judgment postulates that "when a
right or fact has been judicially tried and determined by a court of competent
jurisdiction, or when an opportunity for such trial has been given, the judgment of
the court, as long as it remains unreversed, should be conclusive upon the parties
and those in privity with them."
Page | 102

Compilation of Digested Cases for Remedial Law Review II


by: K. M. T. BUELA

All the elements of the doctrine are present in this case. The final judgment in G.R.
No. 167998 was rendered by the Court pursuant to its jurisdiction over the review
of decisions and rulings of the CA. It was a judgment on the merits of Planters
Banks's right to apply for and be issued a writ of possession. Lastly, the parties in
G.R. No. 167998 are the same parties involved in the present case.
The proceeding in a petition for a writ of possession is ex parte and summary in
nature. It is a judicial proceeding brought for the benefit of one party only and
without notice by the court to any person adverse of interest. It is a proceeding
wherein relief is granted without giving the person against whom the relief is sought
an opportunity to be heard.
By its very nature, an ex parte petition for issuance of a writ of possession is a nonlitigious proceeding. It is a judicial proceeding for the enforcement of one's right of
possession as purchaser in a foreclosure sale. It is not an ordinary suit filed in court,
by which one party sues another for the enforcement of a wrong or protection of a
right, or the prevention or redress of a wrong. (Citations omitted)
Given the ex-parte nature of the proceedings for a writ of possession, the R TC did
not err in cancelling the previously scheduled hearing and in granting Planters
Bank's motion without affording notice to LZK Holdings or allowing it to
participate.
NB: Was the initial decision of the trial court, which was affirmed by CA and SC,
declaring the consolidated title of Planters Bank null and void abrogated by the
subsequent decision of the SC upholding the issuance of the writ of possession?

GOLDENWAY MERCHANDISING CORPORATION vs. EQUITABLE


PCI BANK
G.R. No. 195540, March 13, 2013
FACTS:
Petitioner executed a Real Estate Mortgage in favor of Equitable PCI Bank over its
real properties to secure a P2,000,000.00 loan. As petitioner failed to settle its loan
obligation, respondent extra-judicially foreclosed the mortgage on December 13,
2000. During the public auction, respondent emerged as the highest bidder and
accordingly, a Certificate of Sale was issued. On February 16, 2001, the Certificate
of Sale was registered.
In a letter dated March 8, 2001, petitioners counsel offered to redeem the
foreclosed properties
Page | 103

Compilation of Digested Cases for Remedial Law Review II


by: K. M. T. BUELA
However, petitioner was told that such redemption is no longer possible because the
certificate of sale had already been registered.
On December 7, 2001, petitioner filed a complaint for specific performance and
damages against the respondent, asserting that it is the one-year period of
redemption under Act No. 3135 which should apply and not the shorter redemption
period provided in R. A. No. 8791 (General Banking Law). Petitioner argued that
applying Section 47 of R.A. 8791 to the real estate mortgage executed in 1985
would result in the impairment of obligation of contracts and violation of the
equal protection clause under the Constitution.
On January 8, 2007, the trial court rendered its decision dismissing the complaint
as well as the counterclaim. Petitioner appealed to the CA which affirmed the trial
courts decision.
ISSUE: Whether or not Section 47 of R.A. No. 8791 (The General Banking Law
of 2000) which took effect on June 13, 2000, providing for a shorter redemption
period in foreclosure sale where the debtor is a juridical entity, be validly applied
in this case when the real estate mortgage contract was executed in 1985 and the
mortgage foreclosed when R.A. No. 8791 was already in effect.
HELD:
The answer is in the affirmative. The right of redemption being statutory, it must be
exercised in the manner prescribed by the statute, and within the prescribed time
limit, to make it effective. Furthermore, as with other individual rights to contract
and to property, it has to give way to police power exercised for public welfare.
Under section 47 of R.A. No. 8791 otherwise known as "The General Banking Law
of 2000, notwithstanding Act 3135, juridical persons whose property is being
sold pursuant to an extrajudicial foreclosure, shall have the right to redeem
the property until, but not after, the registration of the certificate of
foreclosure sale with the applicable Register of Deeds which in no case shall be
more than three (3) months after foreclosure, whichever is earlier. Owners of
property that has been sold in a foreclosure sale prior to the effectivity of this Act
shall retain their redemption rights until their expiration.
Section 47 did not divest juridical persons of the right to redeem their foreclosed
properties but only modified the time for the exercise of such right by reducing the
one-year period originally provided in Act No. 3135. The new redemption period
commences from the date of foreclosure sale, and expires upon registration of the
certificate of sale or three months after foreclosure, whichever is earlier. There is
likewise no retroactive application of the new redemption period because Section

Page | 104

Compilation of Digested Cases for Remedial Law Review II


by: K. M. T. BUELA
47 exempts from its operation those properties foreclosed prior to its effectivity and
whose owners shall retain their redemption rights under Act No. 3135.

H. Partition (Rule 69)


CELESTINO BALUS vs. SATURNINO BALUS and LEONARDA BALUS
VDA. DE CALUNOD
G.R. No. 168970, January 15, 2010
FACTS:
During his lifetime, Rufo obtained a loan from Rural Bank of Maigo secured with
a mortgaged on herein disputed land. Rufo failed to pay his loan. As a result, the
mortgaged property was foreclosed and was subsequently sold to the Bank. A new
title was issued in the Bank's name before Rufo died on July 6, 1984.
On October 10, 1989, herein petitioner and respondents (heis of Rufo) executed an
Extrajudicial Settlement of Estate adjudicating to each of them a specific one-third
portion of the subject property. Three years after the execution of the Extrajudicial
Settlement, herein respondents bought the subject property from the Bank.
Meanwhile, petitioner continued possession of the subject lot.
On June 27, 1995, respondents filed a Complaint for Recovery of Possession and
Damages against petitioner, contending that they had already informed petitioner
of the fact that they were the new owners of the disputed property, but the petitioner
still refused to surrender possession of the same to them.
Petitioner posits that the subject Extrajudicial Settlement is, in and by itself, a
contract between him and respondents, because it contains a provision whereby the
parties agreed to continue their co-ownership of the subject property by
"redeeming" or "repurchasing" the same from the Bank. This agreement, petitioner
contends, is the law between the parties and, as such, binds the respondents. As a
result, petitioner asserts that respondents' act of buying the disputed property from
the Bank without notifying him inures to his benefit as to give him the right to claim
his rightful portion of the property, comprising 1/3 thereof, by reimbursing
respondents the equivalent 1/3 of the sum they paid to the Bank.
ISSUE: Whether or not an extra-judicial settlement of estate involving a foreclosed
property where title is already issued in favor of the mortgagor can be a basis for
assertion of co-ownership.
HELD:

Page | 105

Compilation of Digested Cases for Remedial Law Review II


by: K. M. T. BUELA
The answer is in the negative. In the first place, there is no co-ownership to talk
about and no property to partition, as the disputed lot never formed part of the estate
of their deceased father.
The deceased father lost ownership of the subject property during his lifetime, it
only follows that at the time of his death, the disputed parcel of land no longer
formed part of his estate to which his heirs may lay claim. Stated differently,
petitioner and respondents never inherited the subject lot from their father.
Petitioner's contention that he and his siblings intended to continue their supposed
co-ownership of the subject property contradicts the provisions of the subject
Extrajudicial Settlement where they clearly manifested their intention of having the
subject property divided or partitioned by assigning to each of the petitioner and
respondents a specific 1/3 portion of the same. Partition calls for the segregation
and conveyance of a determinate portion of the property owned in common. It seeks
a severance of the individual interests of each co-owner, vesting in each of them a
sole estate in a specific property and giving each one a right to enjoy his estate
without supervision or interference from the other. In other words, the purpose of
partition is to put an end to co-ownership, an objective which negates petitioner's
claims in the present case.

EUGENIO FELICIANO et al. vs. PEDRO CANOZA et al.


G.R. No. 161746, September 1, 2010
FACTS:
When Antonio Feliciano passed away on May 20, 1930, he left behind his only
property, a parcel of land. Leona, Maria, Pedro and Salina, all surnamed Feliciano,
declared themselves to be the only surviving heirs of Antonio Feliciano and
executed an extrajudicial settlement of Antonio Felicianos estate and appropriated
among themselves the said parcel of land, to the exclusion of the heirs of Esteban
Feliciano and Doroteo Feliciano, deceased children of Antonio Feliciano.
On even date, Leona, Maria, Pedro and Salina executed a deed of absolute sale over
the property in favor of the late Jacinto Feliciano (Pedros portion), Felisa Feliciano
(Salinas portion) and Pedro Canoza (Leona and Marias portions).
On October 18, 1993, surviving heirs of the late Esteban Feliciano and Doroteo
Feliciano, filed a complaint against Salina Feliciano, Felisa Feliciano, Pedro
Canoza and the heirs of the late Jacinto Feliciano, namely Delia, Rosauro, Elsa,
Nardo and Ponciano, all surnamed Feliciano, for the Declaration of Nullity of
Documents and Title, Recovery of Real Property and Damages. They alleged that

Page | 106

Compilation of Digested Cases for Remedial Law Review II


by: K. M. T. BUELA
the settlement of the estate and sale were done without their participation and
consent as heirs of Esteban and Doroteo.
On August 3, 1998, the trial court rendered a Decision in favor of the plaintiffs.
Trial courts decision was reversed on appeal. The CA ruled that prescription had
set in. It reasoned that when petitioners filed the instant complaint for the annulment
of the extrajudicial settlement of Antonio Felicianos estate, more than four (4)
years had elapsed from the issuance of the free patents. As regards the portion
claimed by the late Jacinto Feliciano, sixteen (16) years had elapsed from the time
the free patent was issued to him before petitioners filed the complaint, while in the
case of Canoza, fourteen (14) years had elapsed from the issuance of the free patent
in Canozas favor. Hence, according to the CA, the action for the annulment of the
documents had prescribed.
ISSUE: Whether or not the validity of an extra-judicial partition of an estate
wherein some of the compulsory heirs where excluded may be assailed after more
than four years had lapsed from the issuance of new titles of the subsequent
transferees thereof.
HELD:
The answer is in the negative. Evidently, the applicable prescriptive period to
institute the action to annul the deed of extrajudicial settlement was four (4) years
counted from the discovery of fraud as held in the case of Gerona v. De Guzman.
However, the records show that petitioners complaint was filed only on October
18, 1993, or almost sixteen (16) years after Jacinto Feliciano was issued Free Patent
No. (IV-4) 012293 onNovember 28, 1977, and almost fourteen (14) years from the
time Pedro Canoza was issued OCT No. P-364 on November 28, 1979. As
petitioners are deemed to have obtained constructive notice of the fraud upon the
registration of the Free Patent, they clearly failed to institute the present civil action
within the allowable period. The same result obtains even if their complaint is
treated as one (1) essentially for reconveyance as more than ten (10) years have
passed since petitioners cause of action accrued. The CA committed no error in
dismissing their complaint.

CARMELA BROBIO MANGAHAS vs. EUFROCINA A. BROBIO


G.R. No. 183852, October 20, 2010
FACTS:
Pacifico died intestate and was survived by his wife, respondent Eufrocina A.
Brobio, and four legitimate and three illegitimate children. Petitioner Carmela
Brobio Mangahas is one of the illegitimate children. The heirs of the deceased
executed a Deed of Extrajudicial Settlement of Estate. Respondent agreed to pay
Page | 107

Compilation of Digested Cases for Remedial Law Review II


by: K. M. T. BUELA
the petitioner for the latters share in the estate. Upon demand, Respondent failed
to pay but subsequently issued a Promissory Note in favor of the petitioner.
When the promissory note fell due, respondent failed and refused to pay despite
demand. Thus, petitioner filed a Complaint for Specific Performance with Damages
against respondent. Trial court rendered a decision in favor of petitioner.
CA reversed the trial court decision and held that the waiver of petitioners share in
the three properties, as expressed in the deed of extrajudicial settlement, may not be
considered as the consideration of the promissory note, considering that petitioner
signed the Deed way back in 2002 and she had already received the consideration
of P150,000.00 for signing the same. The CA went on to hold that if petitioner
disagreed with the amount she received, then she should have filed an action for
partition.
ISSUE: Whether or not an action for partition is the proper remedy to contest the
terms of the extra-judicial settlement.
HELD:
The remedy suggested by the CA is not the proper one under the circumstances. An
action for partition implies that the property is still owned in common. Considering
that the heirs had already executed a deed of extrajudicial settlement and waived
their shares in favor of respondent, the properties are no longer under a state of coownership; there is nothing more to be partitioned, as ownership had already been
merged in one person.

I. Forcible entry and Unlawful Detainer (Rule 70)


CARMENCITA SUAREZ vs. MR. AND MRS. FELIX E. EMBOY, JR.
AND MARILOU P. EMBOYDELANTAR
G.R. No. 187944, March 12, 2014
FACTS:
Spouses Padilla owned Lot No. 1907A which he partitioned among their heirs
among which are Claudia and Vicente who are the predecessors of herein
Respondent and Petitioner respectively. In 2004, respondents (heirs of Claudia)
were asked by the Heirs of Vicente (predecessor of petitioner), to vacate the subject
lot and to transfer to Lot No. 1907A5, a landlocked portion (of the same inherited
property) sans a right of way. They refused to comply insisting that Claudias
inheritance pertained to Lot No. 1907A2. Thereafter, respondents received from
Petitioner Carmencita a demand letter requiring them to vacate the subject lot and
informing them that Carmencita had already purchased the subject lot from the
Page | 108

Compilation of Digested Cases for Remedial Law Review II


by: K. M. T. BUELA
Heirs of Vicente. Respondents ignore the demand and filed a complaint for
nullification of the partition and for the issuance of new TCTs covering the heirs
respective portions of Lot No. 1907A.
Carmencita filed before the MTCC and against the respondents a complaint for
unlawful detainer. She alleged that she bought the subject lot from Heirs of Vicente,
the registered owners thereof and the persons who allowed the respondents to
occupy the same by mere tolerance. Respondent opposed the said action on the
following grounds: (1) complaint for unlawful detainer was fundamentally
inadequate lack of specific averment as to when and how possession by tolerance
of the respondents began; (2) the pendency of another action (partition) anchored
on the issue of ownership justifies the suspension of an ejectment suit involving the
same real property.
The MTCC upheld Carmencitas claims and ordered the respondents to vacate the
subject lot and remove at their expense all the improvements they had built thereon.
RTC affirmed in its entirety the MTCC ruling. CA rendered a decision reversing
the disquisitions of the courts a quo and dismissing Carmencitas complaint for
unlawful detainer.
ISSUES:
(1) Whether or not a complaint for unlawful detainer which merely alleges prior
legal possession by tolerance without specific averment as to when and how
possession by tolerance of the defendants began is sufficient to vest the court
with jurisdiction.
(2) Whether or not an action for unlawful detainer may be suspended by an action
for partition filed by the defendant putting in issue the ownership of the property
subject of the ejectment proceedings.
HELD:
(1) No. In a complaint for unlawful detainer, the following key jurisdictional facts
must be alleged and sufficiently established:
1) initially, possession of property by the defendant was by contract with or by
tolerance of the plaintiff;
2) eventually, such possession became illegal upon notice by plaintiff to
defendant of the termination of the latters right of possession;
3) thereafter, the defendant remained in possession of the property and
deprived the plaintiff of the enjoyment thereof; and
4) within one year from the last demand on defendant to vacate the property,
the plaintiff instituted the complaint for ejectment.40
In the case at bar, the first requisite mentioned above is markedly absent.
Carmencita failed to clearly allege and prove how and when the respondents entered
Page | 109

Compilation of Digested Cases for Remedial Law Review II


by: K. M. T. BUELA
the subject lot and constructed a house upon it. Carmencita was likewise
conspicuously silent about the details on who specifically permitted the respondents
to occupy the lot, and how and when such tolerance came about. Instead,
Carmencita cavalierly formulated a legal conclusion, sans factual substantiation,
that (a) the respondents initial occupation of the subject lot was lawful by virtue of
tolerance by the registered owners, and (b) the respondents became deforciants
unlawfully withholding the subject lots possession after Carmencita, as purchaser
and new registered owner, had demanded for the former to vacate the property. It
is worth noting that the absence of the first requisite assumes even more importance
in the light of the respondents claim that for decades, they have been occupying
the subject lot as owners thereof.
Again, this Court stresses that to give the court jurisdiction to effect the
ejectment of an occupant or deforciant on the land, it is necessary that the
complaint must sufficiently show such a statement of facts as to bring the party
clearly within the class of cases for which the statutes provide a remedy,
without resort to parol testimony, as these proceedings are summary in nature.
In short, the jurisdictional facts must appear on the face of the complaint. When
the complaint fails to aver facts constitutive of forcible entry or unlawful detainer,
as where it does not state how entry was effected or how and when dispossession
started, the remedy should either be an accion publiciana or accion reivindicatoria.
(2) The answer is in the affirmative.
As a general rule, therefore, a pending civil action involving ownership of the
same property does not justify the suspension of ejectment proceedings. The
underlying reasons for the above ruling were that the actions in the Regional Trial
Court did not involve physical or de facto possession, and, on not a few occasions,
that the case in the Regional Trial Court was merely a ploy to delay disposition of
the ejectment proceeding, or that the issues presented in the former could quite as
easily be set up as defenses in the ejectment action and there resolved.
Only in rare instances is suspension allowed to await the outcome of the pending
civil action. One such exception is Vda. de Legaspi v. Avendao, wherein the
Court declared:
x x x. Where the action, therefore, is one of illegal detainer, as distinguished from
one of forcible entry, and the right of the plaintiff to recover the premises is
seriously placed in issue in a proper judicial proceeding, it is more equitable and
just and less productive of confusion and disturbance of physical possession, with
all its concomitant inconvenience and expenses, for the Court in which the issue of
legal possession, whether involving ownership or not, is brought to restrain, should
a petition for preliminary injunction be filed with it, the effects of any order or
Page | 110

Compilation of Digested Cases for Remedial Law Review II


by: K. M. T. BUELA
decision in the unlawful detainer case in order to await the final judgment in the
more substantive case involving legal possession or ownership. It is only where
there has been forcible entry that as a matter of public policy the right to
physical possession should be immediately set at rest in favor of the prior
possession regardless of the fact that the other party might ultimately be found
to have superior claim to the premises involved, thereby to discourage any
attempt to recover possession thru force, strategy or stealth and without
resorting to the courts.
Like Vda. de Legaspi, the respondents suit is one of unlawful detainer and not of
forcible entry. And most certainly, the ejectment of petitioners would mean a
demolition of their house, a matter that is likely to create the confusion,
disturbance, inconveniences and expenses mentioned in the said exceptional case.
Necessarily, the affirmance of the MCTC Decision would cause the respondent to
go through the whole gamut of enforcing it by physically removing the petitioners
from the premises they claim to have been occupying since 1937. (Respondent is
claiming ownership only of the land, not of the house.) Needlessly, the litigants as
well as the courts will be wasting much time and effort by proceeding at a stage
wherein the outcome is at best temporary, but the result of enforcement is
permanent, unjust and probably irreparable.

ATTY. VIRGILIO P. ALCONERA vs. ALFREDO T. PALLANAN


A.M. No. P-12-3069, January 20, 2014
FACTS:
Rafols was a defendant in an unlawful detainer case of which Alconera is his
counsel. MTCC ruled against Rafols. Rafols through Alconera appealed with RTC.
While the appeal is pending, RTC issued a writ of possession upon the motion of
the complainant in the ejectment suit. Alconera filed a motion for reconsideration
assailing the writ of possession issued.
On March 17, 2011, herein Respondent Sheriff enforced the writ of execution
against Rafols. During the enforcement of the writ, Alconera and the Sheriff had a
verbal disagreement over the phone as the former is asserting that the writ should
not be enforced pending the resolution of his motion for reconsideration.
On April 6, 2011, Alconera went to RTC Br. 36 with his daughter to confront
respondent sheriff. The face-off escalated into a heated argument was recorded on
video.

Page | 111

Compilation of Digested Cases for Remedial Law Review II


by: K. M. T. BUELA
Alconera filed a Complaint-Affidavit against the respondent sheriff for grave
misconduct before this Court on April 6, 2011. Respondent filed his comment
where he averred that the duty of a court sheriff in enforcing a writ of execution is
ministerial, and without a TRO enjoining it, a sheriff is duty bound to implement it.
ISSUE: Whether or not a motion seeking the reconsideration of the court in issuing
a writ of possession in an unlawful detainer suit would stop the enforcement of the
writ.
HELD:
The answer is in the negative. In ejectment cases, the rulings of the courts are
immediately executory and can only be stayed via compliance with Section 19, Rule
70 of the Rules of Court. Under said Sec. 19, Rule 70, a judgment on a forcible
entry and detainer action is made immediately executory to avoid further injustice
to a lawful possessor. The defendant in such a case may have such judgment stayed
only by (a) perfecting an appeal; (b) filing a supersedeas bond; and (c) making
a periodic deposit of the rental or reasonable compensation for the use and
occupancy of the property during the pendency of the appeal. The failure of the
defendant to comply with any of these conditions is a ground for the outright
execution of the judgment, the duty of the court in this respect being ministerial and
imperative. Hence, if the defendant-appellant has perfected the appeal but failed to
file a supersedeas bond, the immediate execution of the judgment would
automatically follow. Conversely, the filing of a supersedeas bond will not stay the
execution of the judgment if the appeal is not perfected. Necessarily then, the
supersedeas bond should be filed within the period for the perfection of the appeal.
In the case at bar, complainant lost his clients case and appealed to the RTC. His
client has also been periodically depositing rental with the court for the use of the
property pending appeal. However, as ruled by the RTC, the bond filed did not meet
the legal requirements because first and foremost, the bond posted was a property
bond, not cash nor surety. Furthermore, Rafols did not own the property he posted
as bond and besides, it was also not issued in favour of the plaintiff in the ejectment
case. Because of the non-compliance with the requirements under the above-quoted
rule, the execution of the judgment was not effectively stayed. The only exceptions
to non-compliance are the existence of fraud, accident, mistake or excusable
negligence which prevented the defendant from posting the supersedeas bond
or making the monthly deposit, or the occurrence of supervening events which
brought about a material change in the situation of the parties and which
would make the execution inequitable. But whether or not these obtain in the case
at bar is an issue best left to the court that issued the writ of execution.

Page | 112

Compilation of Digested Cases for Remedial Law Review II


by: K. M. T. BUELA
TEODORO S. TEODORO vs. DANILO ESPINO et al.
G.R. No. 189248, February 5, 2014
FACTS:
The subject of the instant controversy is a lot included in a property registered in
the name of Genaro Theodoro. Genaro had five children: Santiago; Maria, from
whom respondents descended and trace their claim of ownership and right of
possession; Petra, Mariano, Petitioner Teodoro Teodoros father; and Ana.
Of all Genaros children, only Petra occupied the subject lot, living at the ancestral
house. After Petras death, her purported will, a holographic will, was probated
which Decision has become final and executory. In the will, Petra, asserting
ownership, devised the subject property to Teodoro Teodoro.
Teodoro Teodoro effected the demolition of the ancestral house, intending to use
the subject property for other purposes. Soon thereafter, respondents, who resided
at portions of Lot No. 2476 that surround the subject property on which the ancestral
house previously stood, erected a fence on the surrounding portion, barricaded its
frontage, and put up a sign thereat, effectively dispossessing Teodoro Teodoro of
the property bequeathed to him by Petra. After Teodoro Teodoros demand for
respondents to vacate the subject property went unheeded, he filed the complaint
for forcible entry against respondents. In their Answer, respondents asserted their
own ownership and possession of the subject property.
Given both parties respective claims of ownership over the subject property via
succession from their ascendants, who are all compulsory heirs of Genaro in whose
name the subject property is still registered, the MTC ruled that respondents cannot
be disturbed in their possession of the subject property "until and unless the question
of ownership over the same is finally resolved before the appropriate court.
In contrast, the RTC ruled in favor of Theodoro. The appellate court, refusing to
touch and rule on the issue of ownership, declared that there is no sufficient
evidence of Teodoro Teodoros prior actual physical possession over the subject
property. Thus, the appellate court dismissed Teodoro Teodoros complaint for lack
of merit.
ISSUE: Whether or not exclusive ownership of a property subject of an action for
forcible entry should be proven before rightful possessor may be ajudged.
HELD:
The answer is in the negative. Exclusive ownership is not among those needed to
be proved to be entitled to possession in an action for forcible entry.

Page | 113

Compilation of Digested Cases for Remedial Law Review II


by: K. M. T. BUELA
The ground rules in forcible entry cases:
(1) One employs force, intimidation, threat, strategy or stealth to deprive another of
physical possession of real property.
(2) Plaintiff must allege and prove prior physical possession of the property in
litigation until deprived thereof by the defendant. This requirement implies that
the possession of the disputed land by the latter was unlawful from the
beginning.
(3) The sole question for resolution hinges on the physical or material possession
(possession de facto) of the property. Neither a claim of juridical possession
(possession de jure) nor an averment of ownership by the defendant can, at the
outset, preclude the court from taking cognizance of the case.
(4) Ejectment cases proceed independently of any claim of ownership, and the
plaintiff needs merely to prove prior possession de facto and undue deprivation
thereof.
In this case, both parties assert prior and exclusive physical possession in the
concept of owner acquired through succession from the same decedent, their aunt
and grand aunt, respectively, Petra. In turn, Petra inherited the property from her
father Genaro, in whose name the subject property is still registered.
Teodoro has not proven exclusive ownership, the MTC was right. But exclusive
ownership of Lot No. 2476 or a portion thereof is not in this case required of
Teodoro Teodoro for him to be entitled to possession. Co-ownership, the finding of
both the MTC at first instance and by the RTC on appeal, is sufficient.
as found by the trial courts, the whole of Lot No. 2476 including the portion now
litigated is, owing to the fact that it has remained registered in the name of Genaro
who is the common ancestor of both parties herein, co-owned property. All, or both
Teodoro Teodoro and respondents are entitled to exercise the right of possession as
co-owners.
Neither party can exclude the other from possession. Although the property remains
unpartitioned, the respondents in fact possess specific areas. Teodoro Teodoro can
likewise point to a specific area, which is that which was possessed by Petra.
Teodoro Teodoro cannot be dispossessed of such area, not only by virtue of Petra's
bequeathal in his favor but also because of his own right of possession that comes
from his co-ownership of the property. As the RTC concluded, petitioners, as heirs
substituting Teodoro Teodoro in this suit, should be restored in the lawful
possession of the disputed area.

Page | 114

Compilation of Digested Cases for Remedial Law Review II


by: K. M. T. BUELA
LOURDES B. FERRER and PROSPERIDAD M. ARANDEZ vs. JUDGE
ROMEO A. RABACA
A.M. No. MTJ-05-1580, October 6, 2010
FACTS:
This administrative case charges Hon. Romeo A. Rabaca, then the Presiding Judge
of Branch 25 of the Metropolitan Trial Court of Manila (MeTC), with ignorance of
the law, disregard of the law, dereliction of duty, knowingly rendering an unjust
interlocutory order, and violation of the Code of Conduct for Government Officials.
The complainants were the President and the Executive Director of the plaintiff in
an ejectment suit. In the said suit, respondent Judge rendered a decision in favor of
the plaintiff. Consequently, plaintiffs counsel filed a motion for immediate
execution, praying that a writ of execution be issued "for the immediate execution
of the aforesaid Judgment. Respondent Judge denied the motion for immediate
execution on the ground that notice of appeal had been timely filed hence
jurisdiction over the case is now elevated to the Regional Trial Court.
ISSUE: Whether or not a motion for issuance of writ of execution in an ejectment
suit may no longer be issued when notice of appeal had already been filed.
HELD:
No. It is basic rule in ejectment cases that the execution of judgment in favor
of the plaintiff is a matter of right and mandatory. This has been the consistent
ruling of the Court in a number of cases involving the same issue posed before
the respondent judge. Respondent Judge is expected to know this and his
justification of erroneous application of the law, although mitigating, could not
exculpate him from liability.
Indeed, respondent Judge should have granted the plaintiffs motion for immediate
execution considering that the defendant did not file the sufficient supersedeas bond
despite having appealed. Granting the plaintiffs motion for immediate execution
became his ministerial duty upon the defendants failure to file the sufficient
supersedeas bond. Section 19, Rule 70, of the Rules of Court clearly imposes such
duty, viz:
Section 19. Immediate execution of judgment; how to stay same. If judgment is
rendered against the defendant, execution shall issue immediately upon motion,
unless an appeal has been perfected and the defendant to stay execution files a
sufficient supersedeas bond, approved by the Municipal Trial Court and executed
in favor of the plaintiff to pay the rents, damages, and costs accruing down to the
time of the judgment appealed from, and unless, during the pendency of the appeal,
he deposits with the appellate court the amount of rent due from time to time under
Page | 115

Compilation of Digested Cases for Remedial Law Review II


by: K. M. T. BUELA
the contract, if any, as determined by the judgment of the Municipal Trial Court.
In the absence of a contract, he shall deposit with the Regional Trial Court the
reasonable value of the use and occupation of the premises for the preceding
month or period at the rate determined by the judgment of the lower court on or
before the tenth day of each succeeding month or period. The supersedeas bond
shall be transmitted by the Municipal Trial Court, with the other papers, to the
clerk of the Regional Trial Court to which the action is appealed.

CGR CORPORATION vs. ERNESTO L. TREYES, JR.


G.R. No. 170916, April 27, 2007
FACTS:
Petitioners claimed to have occupied 37.3033 hectares of public land even before
the notarized separate Fishpond Lease Agreement in their respective favor were
approved in October 2000 by the Secretary of Agriculture for a period of twentyfive (25) years or until December 31, 2024.
On November 18, 2000, Ernesto L. Treyes, Jr. (respondent) allegedly forcibly and
unlawfully entered the leased properties and once inside barricaded the entrance to
the fishponds, set up a barbed wire fence along the road going to petitioners
fishponds, and harvested several tons of milkfish, fry and fingerlings owned by
petitioners.
On November 22, 2000, petitioners promptly filed complaints for Forcible Entry
against Ernesto M. Treyes, Sr. and respondent. In a separate move, petitioners filed
in March 2004 with the Bacolod RTC a complaint for damages against respondent.
Respondent filed a Motion to Dismiss petitioners complaint for damages on three
grounds litis pendentia, res judicata and forum shopping. Bacolod RTC dismissed
petitioners complaint on the ground of prematurity, it holding that a complaint for
damages may only be maintained "after a final determination on the forcible entry
cases has been made."
ISSUE: Whether, during the pendency of their separate complaints for forcible
entry, petitioners can independently institute and maintain an action for damages
which they claim arose from incidents occurring after the dispossession by
respondent of the premises.
HELD:
The answer is in the affirmative. The recoverable damages in forcible entry and
detainer cases thus refer to "rents" or "the reasonable compensation for the use and

Page | 116

Compilation of Digested Cases for Remedial Law Review II


by: K. M. T. BUELA
occupation of the premises" or "fair rental value of the property" and attorneys fees
and costs.
Settled is the rule that in ejectment cases, the only damage that can be recovered is
the fair rental value or the reasonable compensation for the use and occupation of
the property. Considering that the only issue raised in ejectment is that of rightful
possession, damages which could be recovered are those which the plaintiff could
have sustained as a mere possessor, or those caused by the loss of the use and
occupation of the property, and not the damages which he may have suffered but
which have no direct relation to his loss of material possession. Other damages
must thus be claimed in an ordinary action.
As reflected in the earlier-quoted allegations in the complaint for damages of herein
petitioners, their claim for damages have no direct relation to their loss of
possession of the premises. It had to do with respondents alleged harvesting and
carting away several tons of milkfish and other marine products in their fishponds,
ransacking and destroying of a chapel built by petitioner CGR Corporation, and
stealing religious icons and even decapitating the heads of some of them, after the
act of dispossession had occurred.
Surely, one of the elements of litis pendentia - that the identity between the pending
actions, with respect to the parties, rights asserted and reliefs prayed for, is such that
any judgment rendered on one action will, regardless of which is successful, amount
to res judicata in the action under consideration - is not present, hence, it may not
be invoked to dismiss petitioners complaint for damages.
Res judicata may not apply because the court in a forcible entry case has no
jurisdiction over claims for damagesother than the use and occupation of the
premises and attorneys fees. Neither may forum-shopping justify a dismissal of the
complaint for damages, the elements of litis pendentia not being present, or where
a final judgment in the forcible entry case will not amount to res judicata in the
former.
Petitioners filing of an independent action for damages other than those sustained
as a result of their dispossession or those caused by the loss of their use and
occupation of their properties could not thus be considered as splitting of a cause of
action.

Page | 117

Compilation of Digested Cases for Remedial Law Review II


by: K. M. T. BUELA
J. Contempt (Rule 71)
SECURITIES AND EXCHANGE COMMISSION CHAIRMAN
PERFECTO R. YASAY, JR., et al vs. MANUEL D. RECTO et al.
G.R. No. 129521, September 7, 1999
FACTS:
SEC Chairman Yasay, upon request of certain stockholders of Interport Resources
Corporation, issued a temporary restraining order (TRO) enjoining the Interport
Resources Corporation from holding the July 9, 1996 scheduled annual meeting of
the stockholders. Notwithstanding the SEC's TRO, the stockholders proceeded with
the meeting on July 9, 1996, presided over by respondent Manalaysay.
On July 10, 1996, the SEC declared the stockholders meeting of Interport Resources
Corporation held on July 9, 1996, null and void and directed respondents to appear
before the SEC on July 15, 1996, at 3:00 p.m., to show cause why no disciplinary
action should be taken against them or why they should not be cited in contempt.
At the hearing on July 15, 1996, respondent Manalaysay questioned the validity of
the TRO as well as the contempt proceedings in light of the TRO issued by the
Court of Appeals restraining the SEC from enforcing its TRO.
After the hearing, the SEC issued an order stating Manalaysay and Ricalde are
GUILTY OF CONTEMPT. After due proceedings, on April 14, 1997, the Court of
Appeals promulgated its decision reversing and setting aside the SEC order
declaring respondents guilty of contempt. On May 2, 1997, petitioners filed a
motion for reconsideration of the decision. However, on June 11, 1997, the Court
of Appeals denied the motion. Petitioner appealed to the SC.
On October 7, 1997, respondents filed their comment. In the main, respondents
submit that contempt is criminal in character and their exoneration from a charge
of contempt amounts to an acquittal from which an appeal would not lie.
ISSUE: Whether or not the contempt instituted motu proprio and holding of guilty
thereof by a quasi-judicial agency when reversed on appeal before the CA may
again be appealed to the SC.
HELD:
We agree with respondents that the charge of contempt partakes of the nature of a
criminal offense. The exoneration of the contemner from the charge amounts to an
acquittal from which an appeal would not lie.
A distinction is made between a civil and criminal contempt. Civil contempt is the
failure to do something ordered by a court to be done for the benefit of a party. A
Page | 118

Compilation of Digested Cases for Remedial Law Review II


by: K. M. T. BUELA
criminal contempt is any conduct directed against the authority or dignity of the
court. Petitioners argue that the contempt committed by respondents was civil in
nature, as the temporary restraining order the SEC issued was for the benefit of a
party to a case. The contention is untenable.
"Civil contempt proceedings are generally held to be remedial and civil in their
nature; that is, they are proceedings for the enforcement of some duty, and
essentially a remedy for coercing a person to do the thing required." "In general,
civil contempt proceedings should be instituted by an aggrieved party, or his
successor, or someone who has a pecuniary interest in the right to be protected." If
the contempt is initiated by the court or tribunal exercising the power to punish a
given contempt, it is criminal in nature, and the proceedings are to be conducted in
accordance with the principles and rules applicable to criminal cases. The State is
the real prosecutor.
"The real character of the proceedings in contempt cases is to be determined by the
relief sought or by the dominant purpose. The proceedings are to be regarded as
criminal when the purpose is primarily punishment, and civil when the purpose is
primarily compensatory or remedial."
"But whether the first or the second, contempt is still a criminal proceeding in
which acquittal, for instance, is a bar to a second prosecution. The distinction
is for the purpose only of determining the character of punishment to be
administered."
In this case, the contempt is not civil in nature, but criminal, imposed to vindicate
the dignity and power of the Commission; hence, as in criminal proceedings, an
appeal would not lie from the order of dismissal of, or an exoneration from, a charge
of contempt."
At any rate, the SEC order directing respondents to show cause why they should
not be cited in contempt was highly improper. The Court of Appeals issued on July
8, 1996, a temporary restraining order against the order of the SEC of June 28, 1996
directing the Interport Resources Corporation to desist from holding the
stockholders' meeting on July 9, 1996. Contrary to the view of petitioners, the effect
of the temporary restraining order of the Court of Appeals directing the SEC to
desist from enforcing its own TRO was to allow such meeting to proceed as
scheduled. More, the Court of Appeals in its final decision nullified the SEC's order.
Hence, there was no willful disobedience to a lawful order of the SEC. Respondents
were not guilty of contempt.

Page | 119

Compilation of Digested Cases for Remedial Law Review II


by: K. M. T. BUELA
SALVADOR SISON vs. JUDGE JOSE F. CAOIBES, JR., Presiding Judge,
and TEODORO S. ALVAREZ, Sheriff IV, Regional Trial Court, Las Pias
City, Branch 253
A.M. No. RTJ-03-1771, May 27, 2004
FACTS:
On September 29, 1999, Sison, an MMDA officer apprehended Caoibes III for
violating a no right turn sign while the latter was driving along EDSA. Caoibes III
introduce himself to be the son of RTC Judge Caoibes Jr. and plead that he had an
official errand for his father. Notwithstanding such introduction and plea, Sison
confiscated the drivers license of Caoibies III, even bragging in the process that he
did the same to somebody who introduced himself as a lawyer the day before.
On September 15, 1999, Judge Caoibes Jr. issued an order requiring Sison to appear
before him to explain a traffic incident involving his son and to show cause why he
should not be cited as in contempt of court. Because of the complainants failure to
appear as directed, Judge Caoibes Jr., issued another Order dated September 22,
1999 for the complainants arrest and commitment, and for the latter to personally
appear for hearing before his sala on September 29, 1999. On the scheduled hearing,
the complainant appeared and executed an affidavit admitting to the court that he
made a mistake and that it was all a misunderstanding. The respondent judge,
thereafter, lifted the September 22, 1999 Order.
Sison filed an administrative case against Judge Caoibes, Jr., and alleged the
following: (1) that former was charged for indirect contempt for apprehending the
latters son for traffic violation; (2) that former was arrested and was only released
upon admitting his mistakes in relation to the apprehension under duress; and (3)
that the acts of respondents in arresting him without any warrant of arrest before a
charge of indirect contempt is heard constitute the gravest ABUSE OF
AUTHORITY ever committed by the respondents.
ISSUE: Whether or not the act of an MMDA officer in apprehending a person, who
while not an official employee of the judiciary was allegedly performing official
function for his father who is a judge, is contumacious act under Section 3, Rule 71.
HELD:
The answer is in the negative. The act complained against must be any of those
specified in Sec. 3, Rule 71, 1997 Rules of Civil Procedure; otherwise, there is
no contempt of court, which requires that the person obstructed should be
performing a duty connected with judicial functions.
The respondent Judge was not justified to so consider the act and remarks of Sison
as thereby displaying arrogance towards and deliberate disregard of the usual
respect, courtesy and accommodation due to a court of law and its representative.
Page | 120

Compilation of Digested Cases for Remedial Law Review II


by: K. M. T. BUELA
The refusal of Sison and the supposed remarks should not cause resentment on the
part of the respondent Judge (whom Sison most likely did not yet know at the time)
because he knew, as a public official himself, that Sison was only doing his duty of
enforcing evenly the particular traffic regulation against swerving into a one-way
street from the wrong direction, regardless of the office or position of the violators
father.
The foregoing renders clear that the respondent Judge had no legitimate basis by
which to consider Sisons apprehension of his son as indirect contempt.

JUDGE DOLORES L. ESPAOL vs. ATTY. BENJAMIN S. FORMOSO


and SPOUSES BENITO SEE and MARLY SEE
G.R. No. 150949, June 21, 2007
FACTS:
On April 15, 1994, Sharcons Builders Philippines, Inc. (Sharcons) bought from
Evanswinda Morales a piece of land purported to be registered under the latters
name.
Thus, Title in Evanswindas name was cancelled and in lieu thereof, TCT No. T511462 was issued in the name of Sharcons. However, when the latters workers
tried to fence and take possession of the lot, they were prevented by the caretaker
of spouses Joseph and Enriqueta Mapua. The caretaker claimed that spouses Mapua
are the owners of the land. Sharcons verified the status of the title and found that
TCT No. T-107163 was indeed registered in the names of spouses Mapua as early
as July 13, 1979.
On January 25, 2000, Sharcons filed a complaint for quieting of title and impleaded
as defendants spouses Mapua, Evanswinda Morales, and the Register of Deeds of
Trece Martires City. In the course of the proceedings, Judge Dolores L. Espaol
came across Civil Case No. 623-92 wherein it was adjudged that Sharcons' TCT
and other supporting documents are falsified and that respondents are responsible
therefor. Hence, Judge Espaol issued an Order declaring respondents guilty of
direct contempt of court and ordered their confinement for ten (10) days in the
municipal jail of Dasmarias, Cavite.
By virtue of a warrant of arrest issued by petitioner, respondents were confined in
municipal jail. That same day, respondents filed a motion for bail and a motion to
lift the order of arrest. But they were denied outright by petitioner.
Respondents then filed with the Court of Appeals a petition for a writ of habeas
corpus which was granted.
Page | 121

Compilation of Digested Cases for Remedial Law Review II


by: K. M. T. BUELA

ISSUE: Whether or not the act of presenting falsified documents before a court
constitutes direct contempt.
HELD:
The answer is in the negative. The use of falsified and forged documents is a
contumacious act. However, it constitutes indirect contempt not direct contempt.
Pursuant to Section 3, Rule 71, such act is an improper conduct which degrades the
administration of justice. In Santos v. Court of First Instance of Cebu, Branch VI,
we ruled that the imputed use of a falsified document, more so where the falsity of
the document is not apparent on its face, merely constitutes indirect contempt, and
as such is subject to such defenses as the accused may raise in the proper
proceedings. Thus, following Section 3, Rule 71, a contemner may be punished only
after a charge in writing has been filed, and an opportunity has been given to the
accused to be heard by himself and counsel. Moreover, settled is the rule that a
contempt proceeding is not a civil action, but a separate proceeding of a criminal
nature in which the court exercises limited jurisdiction. Thus, the modes of
procedure and the rules of evidence in contempt proceedings are assimilated as far
as practicable to those adapted to criminal prosecutions. Perforce, petitioner judge
erred in declaring summarily that respondents are guilty of direct contempt and
ordering their incarceration. She should have conducted a hearing with notice to
respondents.

P/SUPT. HANSEL M. MARANTAN vs. ATTY. JOSE MANUEL DIOKNO


and MONIQUE CU-UNJIENG LA'O
G.R. No. 205956, February 12, 2014
FACTS:
Marantan, et al were charged with homicide. The said case involve an incident
which transpired on November 7, 2005, where Anton Cu-Unjieng (son of
respondent LaO), Francis Xavier Manzano, and Brian Anthony Dulay, were shot
and killed by police officers in front of the AIC Gold Tower at Ortigas Center,
which incident was captured by a television crew from UNTV 37 (Ortigas incident).
In G.R. No. 199462, LaO, together with the other petitioners, prayed that the
corresponding informations for homicide be withdrawn; and that charges for
murder be filed. In the meantime, on January 6, 2013, a shooting incident occurred
in Atimonan, Quezon, where Marantan was the ground commander in a policemilitary team, which resulted in the death of thirteen (13) men (Atimonan incident).
Marantan alleges that, riding on the unpopularity of the Atimonan incident, LaO
and her counsel, Atty. Diokno, and one Ernesto Manzano, organized and conducted
Page | 122

Compilation of Digested Cases for Remedial Law Review II


by: K. M. T. BUELA
a televised/radio broadcasted press conference. During the press conference, they
maliciously made intemperate and unreasonable comments on the conduct of the
Court in handling G.R. No. 199462, as well as contumacious comments on the
merits of the criminal cases before the RTC, branding Marantan and his co-accused
guilty of murder in the Ortigas incident.
Marantan submits that the respondents violated the sub judice rule, making them
liable for indirect contempt under Section 3(d) of Rule 71 of the Rules of Court, for
their contemptuous statements and improper conduct tending directly or indirectly
to impede, obstruct or degrade the administration of justice. He argues that their
pronouncements and malicious comments delved not only on the supposed inaction
of the Court in resolving the petitions filed, but also on the merits of the criminal
cases before the RTC and prematurely concluded that he and his co-accused are
guilty of murder.
The respondents, in their Comment, argue that there was no violation of the sub
judice rule as their statements were legitimate expressions of their desires, hopes
and opinions which were taken out of context and did not actually impede, obstruct
or degrade the administration of justice in a concrete way.
ISSUE: Whether or not comments made in national television regarding an ongoing case violates sub judice rule and consequently a contemptuous act.
HELD:
The answer is in the negative. The sub judice rule restricts comments and
disclosures pertaining to the judicial proceedings in order to avoid prejudging the
issue, influencing the court, or obstructing the administration of justice. A violation
of this rule may render one liable for indirect contempt under Sec. 3(d), Rule 71 of
the Rules of Court, which reads:
Section 3. Indirect contempt to be punished after charge and hearing. x x x a
person guilty of any of the following acts may be punished for indirect contempt:
xxx
(d) Any improper conduct tending, directly or indirectly, to impede, obstruct, or
degrade the administration of justice.
For a comment to be considered as contempt of court "it must really appear" that
such does impede, interfere with and embarrass the administration of justice.
The power of contempt is inherent in all courts in order to allow them to conduct
their business unhampered by publications and comments which tend to impair the
impartiality of their decisions or otherwise obstruct the administration of justice. As
important as the maintenance of freedom of speech, is the maintenance of the
Page | 123

Compilation of Digested Cases for Remedial Law Review II


by: K. M. T. BUELA
independence of the Judiciary. The "clear and present danger" rule may serve as an
aid in determining the proper constitutional boundary between these two rights.
The "clear and present danger" rule means that the evil consequence of the comment
must be "extremely serious and the degree of imminence extremely high" before an
utterance can be punished. There must exist a clear and present danger that the
utterance will harm the administration of justice. Freedom of speech should not
be impaired through the exercise of the power of contempt of court unless there is
no doubt that the utterances in question make a serious and imminent threat to the
administration of justice. It must constitute an imminent, not merely a likely, threat.
In this case, the comments seem to be what the respondents claim to be an
expression of their opinion that their loved ones were murdered by Marantan. This
is merely a reiteration of their position in G.R. No. 199462, which precisely calls
the Court to upgrade the charges from homicide to murder. The mere restatement
of their argument in their petition cannot actually, or does not even tend to,
influence the Court.
As to the conduct of the Court, a review of the respondents' comments reveals that
they were simply stating that it had not yet resolved their petition. There was no
complaint, express or implied, that an inordinate amount of time had passed since
the petition was filed without any action from the Court. There appears no attack or
insult on the dignity of the Court either.
"A public utterance or publication is not to be denied the constitutional protection
of freedom of speech and press merely because it concerns a judicial proceeding
still pending in the courts, upon the theory that in such a case, it must necessarily
tend to obstruct the orderly and fair administration of justice." By no stretch of the
imagination could the respondents' comments pose a serious and imminent threat
to the administration of justice. No criminal intent to impede, obstruct, or degrade
the administration of justice can be inferred from the comments of the respondents.

CAPITOL HILLS GOLF & COUNTRY CLUB, INC. and PABLO B.


ROMAN, JR. vs. MANUEL O. SANCHEZ
G.R. No. 182738, February 24, 2014
FACTS:
Respondent, a stockholder of petitioner Capitol Hills Golf & Country Club, Inc.
filed a petition for the nullification of the annual meeting of stockholders of May
21, 2002 and the special meeting of stockholders of April 23, 2002. On August 12,
2002, respondent filed a Motion for Production and Inspection of Documents,
which the court granted in an Order dated September 10, 2002.
Page | 124

Compilation of Digested Cases for Remedial Law Review II


by: K. M. T. BUELA

On December 9, 2002, then Presiding Judge Bruselas issued an Order for the
immediate implementation of the September 10, 2002 Order. On several occasions,
petitioners failed to comply with the directive to produce documents for inspection.
On September 3, 2007, the trial court issued an order directing the defendants to
strictly comply with this September 10, 2002 order. Failure of the defendants to
comply with all the requirements of the order dated September 10, 2002 will
result in this court citing all the defendants in contempt of court. This Court
shall order defendants solidarily to pay a fine of P10,000.00 for every day of
delay to comply with the order of September 10, 2002 until the defendants shall
have fully and completely complied with the said order.
Further sanctions shall be meted upon defendants should the Court find that
defendants have been in bad faith in complying with the order of September 10,
2002 despite the order of this Court.
Petitioners questioned the aforesaid Resolution via Petition for Certiorari.
Petitioners claim that the threatened citation for contempt is not in line with the
policy that there should be willfulness or that the contumacious act be done
deliberately in disregard of the authority of the court. CA ruled that there is no
indication that the RTC committed grave abuse of discretion amounting to lack or
excess of jurisdiction.
ISSUE: Whether or not an order of the court stating that failure to strictly comply
with it will result to the persons being directed thereby to be cited for contempt
constitutes a final order in an indirect contempt proceeding. Hence it violates due
process.
HELD:
The answer is in the negative. The September 3, 2007 Resolution could be treated
as a mere reiteration of the September 10, 2002 Order. It is not yet a "judgment or
final order of a court in a case of indirect contempt" as contemplated under the
Rules. The penalty mentioned therein only serves as a reminder to caution
petitioners of the consequence of possible non-observance of the long-overdue
order to produce and make available for inspection and photocopying of the
requested records/documents. In case of another failure or refusal to comply with
the directive, the court or respondent could formally initiate the indirect contempt
proceedings pursuant to the mandatory requirements of the Rules and existing
jurisprudence.

Page | 125

Compilation of Digested Cases for Remedial Law Review II


by: K. M. T. BUELA
SPECIAL PROCEEDINGS
A. Settlement of Estate (Rules 73 to 90)
TERESITA N. DE LEON et al. vs. HON. COURT OF APPEALS, HON.
PABLO P. INVENTOR and RAMON NICOLAS
G.R. No. 128781, August 6, 2002
FACTS:
Petitioners and Respondents are heirs of Rafael and Salud Nicolas. Petioner Teresita
N. de Leon was appointed administratrix of the estate of Rafael C. Nicolas.
Respondent Ramon G. Nicolas, an oppositorapplicant in the intestate proceedings,
filed a "Motion for Collation," claiming that deceased Rafael Nicolas, during his
lifetime, had given some real properties to his children by gratuitous title and that
administratrix-petitioner Teresita failed to include the same in the inventory of the
estate of the decedent.
On November 11, 1994, the RTC issued an Order for collation of the properties
mentioned in Ramons order of collation.
Teresita N. de Leon filed a Motion for Reconsideration alleging that the properties
subject of the Order "were already titled in their names years ago" and that titles
may not be collaterally attacked in a motion for collation. On February 23, 1995,
the RTC issued an Order denying said motion, ruling that it is within the jurisdiction
of the court to determine whether titled properties should be collated, citing Section
2, Rule 90 of the Rules of Court which provides that the final order of the court
concerning questions as to advancements made shall be binding on the person
raising the question and on the heir.
Petitioners filed with the Court of Appeals a petition for certiorari, prohibition and
mandamus with prayer for a temporary restraining order and writ of preliminary
injunction. Petitioners claimed that respondent judge acted with grave abuse of
discretion when it ordered the collation of the properties already titled under
petitioners name.
Court of Appeals found the petition devoid of merit, ruling that the Order dated
November 11, 1994 directing the inclusion of the properties therein enumerated in
the estate of the deceased Rafael Nicolas had already become final for failure of
petitioners to appeal from the order of collation.
ISSUE: Whether or not an order of collation is a final order and failure to appeal
therefrom makes it unassailable.

Page | 126

Compilation of Digested Cases for Remedial Law Review II


by: K. M. T. BUELA
HELD:
The answer is in the negative. A probate court or one in charge of proceedings
whether testate or intestate cannot adjudicate or determine title to properties claimed
to be a part of the estate and which are claimed to belong to outside parties. All that
the said court could do as regards said properties is to determine whether they
should or should not be included in the inventory or list of properties to be
administered by the administrator. If there is no dispute, well and good, but if there
is, then the parties, the administrator, and the opposing parties have to resort to an
ordinary action for a final determination of the conflicting claims of title because
the probate court cannot do so
Private respondents reliance on Section 2, Rule 90 of the Rules of Court, to wit:
"SEC. 2. Questions as to advancement to be determined. Questions as to
advancement made, or alleged to have been made, by the deceased to any
heir may be heard and determined by the court having jurisdiction of the
estate proceedings; and the final order of the court thereon shall be binding
on the person raising the question and on the heir."
in support of his claim that the assailed Order is a final order and therefore
appealable and that due to petitioners failure to appeal in due time, they are now
bound by said Order, is not feasible.
What seems to be a conflict between the above-quoted Rule and the aforediscussed
jurisprudence that the Order in question is an interlocutory and not a final order is
more apparent than real. This is because the questioned Order was erroneously
referred to as an order of collation both by the RTC and the appellate court. For all
intents and purposes, said Order is a mere order including the subject properties in
the inventory of the estate of the decedent.
The order of exclusion (or inclusion) is not a final order; that it is interlocutory in
the sense that it did not settle once and for all the title to the subject lots; that the
prevailing rule is that for the purpose of determining whether a certain property
should or should not be included in the inventory, the probate court may pass upon
the title thereto but such determination is not conclusive and is subject to the final
decision in a separate action regarding ownership which may be instituted by the
parties.

Page | 127

Compilation of Digested Cases for Remedial Law Review II


by: K. M. T. BUELA
EDGAR SAN LUIS vs. FELICIDAD SAN LUIS
G.R. No. 133743, February 6, 2007
FACTS:
Felicisimo T. San Luis, during his lifetime, contracted three marriages, to wit: (1)
first with Virginia Sulit out of which were born six children, one of which is herein
petitioner; (2) on 1968 he was married to Merry Lee, with whom he had a son,
Tobias; but said marriage was divorced by virtue of a decree granting said divorce
by Family court of Hawaii; (3) On 1984, he married Felicidad San Luis with whom
he had had no children but lived with her for 18 years from the time of their marriage
up to his death on December 18, 1992.
Claiming to be the widow of the decedent, Respondent sought the settlement of
Felicisimos estate and filed a petition for letters of administration before RTC
Makati. Petitioner filed a motion to dismiss on the ground that respondent has no
legal personality to file the petition because she was only a mistress of Felicisimo
since the latter, at the time of his death, was still legally married to Merry Lee.
Petitioner further asserted that paragraph 2, Article 26 of the Family Code cannot
be given retroactive effect to validate respondents bigamous marriage with
Felicisimo because this would impair vested rights in derogation of Article 256 of
the Family Code.
On September 12, 1995, the trial court finding merit on petitioners arguments
dismissed the petition for letters of administration. On appeal, Court of Appeals
reversed the orders of the trial court. The appellante court ruled: (1) that under
Section 1, Rule 73 of the Rules of Court, the term "place of residence" of the
decedent, for purposes of fixing the venue of the settlement of his estate, refers to
the personal, actual or physical habitation, or actual residence or place of abode of
a person as distinguished from legal residence or domicile. Thus, the petition for
letters of administration was properly filed in Makati City; and (2) that Felicisimo
had legal capacity to marry respondent by virtue of paragraph 2, Article 26 of the
Family Code and the rulings in Van Dorn v. Romillo, Jr. 30 and Pilapil v. IbaySomera. On July 2, 1998, Petitioners appealed to this Court via the instant petition
for review on certiorari.
ISSUE: Whether or not a surviving spouse whose marriage to the decedent is
assailed on the ground of prior marriage may be considered an interested party in
the settlement of the decedents estate.
HELD:
The answer is in the affirmative. Even assuming that Felicisimo was not
capacitated to marry respondent in 1974, nevertheless, we find that the latter
has the legal personality to file the subject petition for letters of administration,
Page | 128

Compilation of Digested Cases for Remedial Law Review II


by: K. M. T. BUELA
as she may be considered the co-owner of Felicisimo as regards the properties
that were acquired through their joint efforts during their cohabitation.
Section 6, Rule 78 of the Rules of Court states that letters of administration may be
granted to the surviving spouse of the decedent. However, Section 2, Rule 79
thereof also provides in part:
SEC. 2. Contents of petition for letters of administration. A petition for
letters of administration must be filed by an interested person and must
show, as far as known to the petitioner: x x x.
An "interested person" has been defined as one who would be benefited
by the estate, such as an heir, or one who has a claim against the estate,
such as a creditor. The interest must be material and direct, and not
merely indirect or contingent.
In the instant case, respondent would qualify as an interested person who has a
direct interest in the estate of Felicisimo by virtue of their cohabitation, the
existence of which was not denied by petitioners. If she proves the validity of the
divorce and Felicisimos capacity to remarry, but fails to prove that her marriage
with him was validly performed under the laws of the U.S.A., then she may be
considered as a co-owner under Article 144 of the Civil Code. This provision
governs the property relations between parties who live together as husband and
wife without the benefit of marriage, or their marriage is void from the beginning.
It provides that the property acquired by either or both of them through their work
or industry or their wages and salaries shall be governed by the rules on coownership. In a co-ownership, it is not necessary that the property be acquired
through their joint labor, efforts and industry. Any property acquired during the
union is prima facie presumed to have been obtained through their joint efforts.
Hence, the portions belonging to the co-owners shall be presumed equal, unless the
contrary is proven.
Meanwhile, if respondent fails to prove the validity of both the divorce and the
marriage, the applicable provision would be Article 148 of the Family Code which
has filled the hiatus in Article 144 of the Civil Code by expressly regulating the
property relations of couples living together as husband and wife but are
incapacitated to marry. In Saguid v. Court of Appeals, we held that even if the
cohabitation or the acquisition of property occurred before the Family Code took
effect, Article 148 governs.

Page | 129

Compilation of Digested Cases for Remedial Law Review II


by: K. M. T. BUELA
EDUARDO G. AGTARAP vs. SEBASTIAN AGTARAP, JOSEPH
AGTARAP, TERESA AGTARAP, WALTER DE SANTOS, and
ABELARDO DAGORO
G.R. No. 177099, June 8, 2011
FACTS:
Joaquin died intestate on November 21, 1964 without any known debts or
obligations. During his lifetime, Joaquin contracted two marriages, first with Lucia
(died on April 24, 1924) with whom he had three children, Jesus (died prior to 1994
without issue), Milagros (died on 1996 with a will subject to a separate probate
proceedings) and Jose (survived by three children, namely, Gloria, Joseph, and
Teresa herein respondent); and second with Caridad with whom he also had three
childrenEduardo (herein petitioner), Sebastian, and Mercedes. At the time of his
death, Joaquin left two parcels of land with improvements in Pasay City. On
September 15, 1994, petitioner Eduardo filed a verified petition for the judicial
settlement of the estate of his deceased father Joaquin Agtarap.
Joseph, Gloria, and Teresa filed their answer/opposition. They alleged that the two
subject lots belong to the conjugal partnership of Joaquin with Lucia, and that, upon
Lucias death in April 1924, they became the pro indiviso owners of the subject
properties. On October 23, 2000, RTC issued an Order of Partition and declared
that bulk of the estate property were acquired during the existence of the second
marriage as shown by TCT No. (38254) and TCT No. (38255) which showed on its
face that decedent was married to Caridad Garcia
Eduardo, Sebastian, and oppositors Joseph and Teresa filed their respective motions
for reconsideration. On August 27, 2001, the RTC issued a resolution declaring that
the real estate properties belonged to the conjugal partnership of Joaquin and Lucia.
On appeal, CA affirmed the decision of the RTC and included in its resolution is
the settlement not only of the estate of Joaquin, but also the estates of Lucia, Jesus,
Jose, Mercedes, Gloria, and Milagros.
Aggrieved, Sebastian and Eduardo filed their respective motions for
reconsideration. They proferred the following contentions: (1) that the estate must
have already been settled in light of the payment of the estate and inheritance tax
by Milagros, Joseph, and Teresa, resulting to the issuance of TCT No. 8925 in
Milagros name and of TCT No. 8026 in the names of Milagros and Jose; (2) CA
erroneously settled, together with the settlement of the estate of Joaquin, the estates
of Lucia, Jesus, Jose, Mercedes, Gloria, and Milagros, in contravention of the
principle of settling only one estate in one proceeding. He particularly questions the
distribution of the estate of Milagros in the intestate proceedings despite the fact
that a proceeding was conducted in another court for the probate of the will of
Milagros, bequeathing all to Eduardo whatever share that she would receive from
Joaquins estate. He states that this violated the rule on precedence of testate over
Page | 130

Compilation of Digested Cases for Remedial Law Review II


by: K. M. T. BUELA
intestate proceedings; and (3) that RTC, acting as an intestate court with limited
jurisdiction, was not vested with the power and authority to determine questions of
ownership, which properly belongs to another court with general jurisdiction.
ISSUES:
(1) Whether or not estate is considered settled upon payment of inheritance tax.
(2) Whether or not the share of the heir who died pending the settlement of the
estate of predecessor decedent may be settled in the said pending settlement
case of the decedent.
(3) Whether or not the intestate court has the authority to determine questions of
ownership over properties covered by the estate.
HELD:
(1)
No. Sebastians claim that Joaquins estate could have already been settled in 1965
after the payment of the inheritance tax cannot be upheld. Payment of the
inheritance tax, per se, does not settle the estate of a deceased person. As provided
in Section 1, Rule 90 of the Rules of Courtan estate is settled and distributed
among the heirs only after the payment of the debts of the estate, funeral charges,
expenses of administration, allowance to the widow, and inheritance tax. The
records of these cases do not show that these were complied with in 1965.
(2)
The answer is in the affirmative provided the estate of the heirs who died during the
pendency of the settlement their predecessor is not subject of a separate
proceedings.
Pursuant to Section 1, Rule 90 of the Rules of Court, as cited above, the RTC was
specifically granted jurisdiction to determine who are the lawful heirs of Joaquin,
as well as their respective shares after the payment of the obligations of the estate,
as enumerated in the said provision. The inclusion of Lucia, Jesus, Jose, Mercedes,
and Gloria in the distribution of the shares was merely a necessary consequence of
the settlement of Joaquins estate, they being his legal heirs.
However, we agree with Eduardos position that the CA erred in distributing
Joaquins estate pertinent to the share allotted in favor of Milagros. Eduardo was
able to show that a separate proceeding was instituted for the probate of the will
allegedly executed by Milagros before the RTC, Branch 108, Pasay City.34 While
there has been no showing that the alleged will of Milagros, bequeathing all of her
share from Joaquins estate in favor of Eduardo, has already been probated and
approved, prudence dictates that this Court refrain from distributing Milagros share
in Joaquins estate.

Page | 131

Compilation of Digested Cases for Remedial Law Review II


by: K. M. T. BUELA
(3)
As to the issue on the ownership of the subject real properties, we hold that the
RTC, as an intestate court, had jurisdiction to resolve the same.
The general rule is that the jurisdiction of the trial court, either as a probate or an
intestate court, relates only to matters having to do with the probate of the will
and/or settlement of the estate of deceased persons, but does not extend to the
determination of questions of ownership that arise during the proceedings.
However, this general rule is subject to exceptions as justified by expediency and
convenience.
First, the probate court may provisionally pass upon in an intestate or a testate
proceeding the question of inclusion in, or exclusion from, the inventory of a piece
of property without prejudice to the final determination of ownership in a separate
action. Second, if the interested parties are all heirs to the estate, or the question is
one of collation or advancement, or the parties consent to the assumption of
jurisdiction by the probate court and the rights of third parties are not impaired, then
the probate court is competent to resolve issues on ownership. Verily, its
jurisdiction extends to matters incidental or collateral to the settlement and
distribution of the estate, such as the determination of the status of each heir and
whether the property in the inventory is conjugal or exclusive property of the
deceased spouse.
We hold that the general rule does not apply to the instant case considering that the
parties are all heirs of Joaquin and that no rights of third parties will be impaired by
the resolution of the ownership issue. More importantly, the determination of
whether the subject properties are conjugal is but collateral to the probate courts
jurisdiction to settle the estate of Joaquin.

EMILIO A.M. SUNTAY III vs. ISABEL COJUANGCO-SUNTAY


G.R. No. 183053, October 10, 2012
FACTS:
The decedent Cristina Aguinaldo-Suntay (Cristina) died intestate on 4 June 1990.
Cristina was survived by her spouse, Dr. Federico Suntay (Federico) and five
grandchildren: three legitimate grandchildren, including herein respondent, Isabel;
and two illegitimate grandchildren, including petitioner Emilio III, all by Federicos
and Cristinas only child, Emilio A. Suntay (Emilio I), who predeceased his parents.
The illegitimate grandchildren, Emilio III and Nenita, were both reared from
infancy by the spouses Federico and Cristina. Their legitimate grandchildren, Isabel
Page | 132

Compilation of Digested Cases for Remedial Law Review II


by: K. M. T. BUELA
and her siblings, Margarita and Emilio II, lived with their mother Isabel Cojuangco,
following the separation of Isabels parents, Emilio I and Isabel Cojuangco. Isabels
parents, along with her paternal grandparents, were involved in domestic relations
cases, including a case for parricide filed by Isabel Cojuangco against Emilio I.
Emilio I was eventually acquitted.
Three years after Cristinas death, Federico adopted his illegitimate grandchildren,
Emilio III and Nenita. On 26 October 1995, respondent Isabel, filed a petition for
the issuance of letters of administration over Cristinas estate. Federico, opposed
the petition.
On 13 November 2000, Federico died. Almost a year thereafter or on 9 November
2001, the trial court rendered a decision appointing Emilio III as administrator of
decedent Cristinas intestate estate. On appeal, the Court of Appeals reversed and
set aside the decision of the RTC and appointed Petitioner Isabel Cojuangco-Suntay
as administratrix of the intestate estate of Cristina Aguinaldo Suntay. On appeal by
certiorari, Supreme Court reversed and set aside the ruling of the appellate court
included Emilio III as co-administrator of Cristinas estate, giving weight to his
interest in Federicos estate. Petitioner filed a motion for reconsideration and argued
that the explicit provisions of Section 6, Rule 78 of the Rules of Court on the order
of preference for the issuance of letters of administration cannot be ignored and that
Article 992 of the Civil Code must be followed.
ISSUE: Whether or not a legitimate child has preference over the illegitimate child
with respect to the appointment as administrator of the estate of their grandparent.
HELD:
The answer is in the affirmative. The general rule in the appointment of
administrator of the estate of a decedent is laid down in Section 6, Rule 78 of the
Rules of Court. The rule lists an order of preference, in the appointment of an
administrator. This order of preference, which categorically seeks out the surviving
spouse, the next of kin and the creditors in the appointment of an administrator, has
been reinforced in jurisprudence.
The paramount consideration in the appointment of an administrator over the
estate of a decedent is the prospective administrators interest in the estate. This
is the same consideration which Section 6, Rule 78 takes into account in
establishing the order of preference in the appointment of administrator for the
estate. The rationale behind the rule is that those who will reap the benefit of a
wise, speedy and economical administration of the estate, or, in the alternative,
suffer the consequences of waste, improvidence or mismanagement, have the
highest interest and most influential motive to administer the estate correctly.
In all, given that the rule speaks of an order of preference, the person to be
Page | 133

Compilation of Digested Cases for Remedial Law Review II


by: K. M. T. BUELA
appointed administrator of a decedents estate must demonstrate not only an
interest in the estate, but an interest therein greater than any other candidate.
The court further drew a distinction between the heirs categorized as next of kin,
the nearest of kin in the category being preferred. The "next of kin" has been defined
as those persons who are entitled under the statute of distribution to the decedents
property (citations omitted). It is generally said that "the nearest of kin, whose
interest in the estate is more preponderant, is preferred in the choice of
administrator. Among members of a class the strongest ground for preference is
the amount or preponderance of interest. As between next of kin, the nearest of kin
is to be preferred.
It would be a grave abuse of discretion for the probate court to imperiously set aside
and insouciantly ignore that directive without any valid and sufficient reason
therefor.
Mere demonstration of interest in the estate to be settled does not ipso facto entitle
an interested person to co-administration thereof. Neither does squabbling among
the heirs nor adverse interests necessitate the discounting of the order of preference
set forth in Section 6, Rule 78. Indeed, in the appointment of administrator of the
estate of a deceased person, the principal consideration reckoned with is the interest
in said estate of the one to be appointed as administrator. Given Isabels
unassailable interest in the estate as one of the decedents legitimate grandchildren
and undoubted nearest "next of kin.

JOSE C. LEE AND ALMA AGGABAO et al. vs. REGIONAL TRIAL


COURT OF QUEZON CITY BRANCH 85 et al.
G.R. No. 146006, February 23, 2004
FACTS:
Dr. Juvencio P. Ortaez died on July 21, 1980 and was survived by his wife (Juliana
Salgado Ortaez), three legitimate children (Rafael, Jose and Antonio Ortaez) and
five illegitimate children by Ligaya Novicio (herein private respondent).
During the pendency of the intestate proceeding, Juliana Ortaez and her two
children, Special Administrators Rafael and Jose Ortaez, entered into a
memorandum of agreement (MOA) for the extrajudicial settlement of the estate of
Dr. Juvencio Ortaez, partitioning the estate (including the Philinterlife shares of
stock) among themselves.
Pursuant to the MOA, Juliana S. Ortaez and Jose Ortaez, claiming to be owners,
sold the Philinterlife shares to Filipino Loan Assistance Group (FLAG). Having
Page | 134

Compilation of Digested Cases for Remedial Law Review II


by: K. M. T. BUELA
failed to repurchase the shares, petitioner FLAG, after one year, consolidated in its
name the ownership of the Philinterlife shares of stock
On July 12, 1995, private respondent filed a motion for appointment of special
administrator of Philinterlife shares of stock. The motion was granted by the
intestate court. Special Administratrix Enderes filed an urgent motion: (1) to declare
void ab initio the memorandum of agreement dated March 4, 1982; and (2) to
declare the partial nullity of the extrajudicial settlement of the decedents estate.
On August 29, 1997, the intestate court granted the motions of Special
Administratrix Enderes for the annulment of the MOA. Jose Ortaez assailed the
order of the intestate court by filing a petition for certiorari in the Court of Appeals.
The appellate court denied his petition. He then elevated the CA decision to the SC
via petition for review. The SC dismissed the petition of Special Administrator Jose
Ortaez and such decision became final and was subsequently recorded in the book
of entries of judgments.
On July 6, 2000, the intestate court granted the motion for execution filed by private
respondent. Petitioners Lee and Aggabao, officers of FLAG filed before the CA a
petition for certiorari.
ISSUE: Whether or not sale of property included in the inventory of the estate by
some of the heirs made during the pendency of the intestate proceeding without
intestate courts approval may be declared null and void.
HELD:
The answer is in the affirmative. The rule is clear that (1) any disposition of estate
property by an administrator or prospective heir pending final adjudication
requires court approval and (2) any unauthorized disposition of estate
property can be annulled by the probate court, there being no need for a
separate action to annul the unauthorized disposition.
An heir can sell his right, interest, or participation in the property under
administration under Art. 533 of the Civil Code which provides that possession of
hereditary property is deemed transmitted to the heir without interruption from the
moment of death of the decedent. However, an heir can only alienate such portion
of the estate that may be allotted to him in the division of the estate by the probate
or intestate court after final adjudication, that is, after all debtors shall have been
paid or the devisees or legatees shall have been given their shares. This means that
an heir may only sell his ideal or undivided share in the estate, not any specific
property therein.

Page | 135

Compilation of Digested Cases for Remedial Law Review II


by: K. M. T. BUELA
In the present case, Juliana Ortaez and Jose Ortaez sold specific properties of the
estate (1,014 and 1,011 shares of stock in Philinterlife) in favor of petitioner FLAG.
This they could not lawfully do pending the final adjudication of the estate by the
intestate court because of the undue prejudice it would cause the other claimants to
the estate, as what happened in the present case.

THE ESTATE OF HILARIO M. RUIZ, EDMOND RUIZ, Executor vs. THE


COURT OF APPEALS
G.R. No. 118671, January 29, 1996
FACTS:
Hilario M. Ruiz executed a holographic will naming as his heirs his only son,
Edmond Ruiz, his adopted daughter, Montes and his three granddaughters (private
respondents) with Edmond as the executor. Four years after the testator's death,
private respondent Montes filed a petition for the probate and approval of Hilario
Ruiz's will and for the issuance of letters testamentary to Edmond Ruiz.
During the pendency of the probate proceedings, the court ordered Edmond to
deposit with the Branch Clerk the rental deposit and payments representing the oneyear lease of the Valle Verde property. In compliance, on January 25, 1993,
Edmond turned over the amount net of expenses for repair and maintenance on the
estate.
On July 28, 1993, petitioner Testate Estate of Hilario Ruiz, with Edmond Ruiz as
executor, filed a motion praying for the release of the rent payments deposited with
the Branch Clerk. Respondent Montes opposed the motion and concurrently filed a
Motion and prayed for the release of the said rent payments to the testators
grandchildren and for the distribution of the testator's properties, specifically the
Valle Verde property and the Blue Ridge apartments, in accordance with the
provisions of the holographic will.
The probate court denied petitioner's motion but granted respondent Montes'
motion. The probate court, on December 22, 1993, ordered the release of the funds
to Edmond but only "such amount as may be necessary to cover the expenses of
administration and allowances for support" of the testator's three granddaughters
subject to collation and deductible from their share in the inheritance. The court,
however, held in abeyance the release of the titles to respondent Montes and the
three granddaughters until the lapse of six months from the date of first publication
of the notice to creditors.
Petitioner assailed this order before the Court of Appeals but the CA dismissed the
petition and sustained the probate court's order. Hence, Petitioner filed a petition
Page | 136

Compilation of Digested Cases for Remedial Law Review II


by: K. M. T. BUELA
for review on certiorari before the SC and argued that affirming the Probate Courts
order would have the following effect: (1) disallow the executor/administrator of
the estate of the late Hilario M. Ruiz to take possession of all the real and personal
properties of the estate; (2) grant support, during the pendency of the settlement of
an estate, to certain persons not entitled thereto; and (3) prematurely partition and
distribute the estate pursuant to the provisions of the holographic will even before
its intrinsic validity has been determined, and despite the existence of unpaid debts
and obligations of the estate.
ISSUE:
The issue for resolution is whether the probate court, after admitting the will to
probate but before payment of the estate's debts and obligations, has the authority
to grant an allowance from the funds of the estate for the support of the testator's
grandchildren.
HELD:
It is settled that allowances for support under Section 3 of Rule 83 should not be
limited to the "minor or incapacitated" children of the deceased. Article 188 of the
Civil Code of the Philippines, the substantive law in force at the time of the testator's
death, provides that during the liquidation of the conjugal partnership, the
deceased's legitimate spouse and children, regardless of their age, civil status or
gainful employment, are entitled to provisional support from the funds of the
estate.14 The law is rooted on the fact that the right and duty to support, especially
the right to education, subsist even beyond the age of majority.
Be that as it may, grandchildren are not entitled to provisional support from the
funds of the decedent's estate. The law clearly limits the allowance to "widow and
children" and does not extend it to the deceased's grandchildren, regardless of their
minority or incapacity.16 It was error, therefore, for the appellate court to sustain
the probate court's order granting an allowance to the grandchildren of the testator
pending settlement of his estate.

UNION BANK OF THE PHILIPPINES vs. EDMUND SANTIBAEZ and


FLORENCE SANTIBAEZ ARIOLA
G.R. No. 149926, February 23, 2005
FACTS:
During his lifetime, Efraim M. Santibaez obtained two (2) set of loans used to by
three (3) tractors which remained unpaid upon his death. Upon his death, his heirs
commenced a testate settlement proceeding pursuant to his holographic will. During
the pendency of the testate proceedings, the surviving heirs, Edmund and his sister
Florence executed a Joint Agreement wherein they agreed to divide between
Page | 137

Compilation of Digested Cases for Remedial Law Review II


by: K. M. T. BUELA
themselves and take possession of the three (3) tractors. Each of them was to assume
the indebtedness of their late father to FCCC, corresponding to the tractor
respectively taken by them.
FCCC assigned all its assets and liabilities to Petitioner Union Savings and
Mortgage Bank. Pursuant to the assigned assets and liabilities, Union sent demand
letters to Edmund, but the latter failed to heed the same and refused to pay. Thus,
the petitioner filed a Complaint for sum of money against the heirs of Efraim
Santibaez, Edmund and Florence.
On December 7, 1988, respondent Florence filed her Answer and alleged that
considering that the joint agreement signed by her and her brother Edmund was not
approved by the probate court, it was null and void; hence, she was not liable to the
petitioner under the joint agreement.
The trial court found that the claim of the petitioner should have been filed with the
probate court before which the testate estate of the late Efraim Santibaez was
pending, as the sum of money being claimed was an obligation incurred by the said
decedent. The trial court ruled in favor of Florence and Edmund.
ISSUES:
(1) Whether or not the heirs may, during the pendency of the probate proceedings,
enter into an agreement distributing among themselves properties of the testator
and by virtue of such agreement assumed the indebtedness corresponding to
such properties.
(2) Whether or not a creditor of the testator may, during the pendency of the probate
proceedings, file a separate complaint for collection against the heirs based on
an agreement executed by the heirs whereby they distribute the properties of the
estate and assume the debts corresponding to such property.
HELD:
(1)
The answer is in the negative. The act of the heirs in the instant case of dividing the
3 tractors amounts to partition. Every act intended to put an end to indivision among
co-heirs and legatees or devisees is deemed to be a partition, although it should
purport to be a sale, an exchange, a compromise, or any other transaction. Thus, in
executing any joint agreement which appears to be in the nature of an extra-judicial
partition, as in the case at bar, court approval is imperative, and the heirs cannot just
divest the court of its jurisdiction over that part of the estate.
It must be stressed that the probate proceeding had already acquired jurisdiction
over all the properties of the deceased, including the three (3) tractors. To dispose

Page | 138

Compilation of Digested Cases for Remedial Law Review II


by: K. M. T. BUELA
of them in any way without the probate courts approval is tantamount to divesting
it with jurisdiction which the Court cannot allow.
(2)
The loan was contracted by the decedent. The petitioner, purportedly a creditor of
the late Efraim Santibaez, should have thus filed its money claim with the probate
court in accordance with Section 5, Rule 86 of the Revised Rules of Court.
The filing of a money claim against the decedents estate in the probate court is
mandatory. This requirement is for the purpose of protecting the estate of the
deceased by informing the executor or administrator of the claims against it, thus
enabling him to examine each claim and to determine whether it is a proper one
which should be allowed. The plain and obvious design of the rule is the speedy
settlement of the affairs of the deceased and the early delivery of the property to the
distributees, legatees, or heirs. `The law strictly requires the prompt presentation
and disposition of the claims against the decedent's estate in order to settle the
affairs of the estate as soon as possible, pay off its debts and distribute the residue.
The question that now comes to fore is whether the heirs assumption of the
indebtedness of the decedent is binding. We rule in the negative. Perusing the joint
agreement, it provides that the heirs as parties thereto "have agreed to divide
between themselves and take possession and use the above-described chattel and
each of them to assume the indebtedness corresponding to the chattel taken as herein
after stated which is in favor of First Countryside Credit Corp." The assumption of
liability was conditioned upon the happening of an event, that is, that each heir shall
take possession and use of their respective share under the agreement. It was made
dependent on the validity of the partition, and that they were to assume the
indebtedness corresponding to the chattel that they were each to receive.

AMELIA GARCIA-QUIAZON, JENNETH QUIAZON and MARIA


JENNIFER QUIAZON vs. MA. LOURDES BELEN, for and in behalf of
MARIA LOURDES ELISE QUIAZON
G.R. No. 189121, July 31, 2013
FACTS:
Eliseo died intestate on 12 December 1992. On 12 September 1994, Elise filed a
Petition for Letters of Administration and claimed that she is the natural child of
Eliseo having been conceived and born at the time when her parents were both
capacitated to marry each other. Insisting on the legal capacity of Eliseo and
Lourdes to marry, Elise impugned the validity of Eliseos marriage to Amelia by
claiming that it was bigamous for having been contracted during the subsistence of
the latters marriage with one Filipito Sandico (Filipito). To prove her filiation to

Page | 139

Compilation of Digested Cases for Remedial Law Review II


by: K. M. T. BUELA
the decedent, Elise, among others, attached to the Petition for Letters of
Administration her Certificate of Live Birth4 signed by Eliseo as her father.
Claiming that the venue of the petition was improperly laid, Amelia, wife of the
decedent, together with her children, Jenneth and Jennifer, opposed the issuance of
the letters of administration by filing an Opposition/Motion to Dismiss. The
petitioners asserted that as shown by his Death Certificate, Eliseo was a resident of
Capas, Tarlac and not of Las Pias City, at the time of his death. Pursuant to Section
1, Rule 73 of the Revised Rules of Court, the petition for settlement of decedents
estate should have been filed in Capas, Tarlac and not in Las Pias City. In addition
to their claim of improper venue, the petitioners averred that there are no factual
and legal bases for Elise to be appointed administratix of Eliseos estate.
The lower court ruled that the venue of the petition was properly laid in Las Pias
City and directed the issuance of Letters of Administration to Elise upon posting
the necessary bond. On appeal, the decision of the trial court was affirmed in toto
ISSUES:
(1) Whether or not the residence of the decedent as indicated in the death certificate
should be taken into account for purposes of determining the venue for the
probate of the will.
(2) Whether or not the natural child of the decedent may be appointed as an
administrator.
HELD:
(1)
Under Section 1, Rule 73 of the Rules of Court, the petition for letters of
administration of the estate of a decedent should be filed in the RTC of the province
where the decedent resides at the time of his death
The word resides" should be viewed or understood in its popular sense, meaning,
the personal, actual or physical habitation of a person, actual residence or place of
abode. It signifies physical presence in a place and actual stay thereat. Venue for
ordinary civil actions and that for special proceedings have one and the same
meaning. As thus defined, "residence," in the context of venue provisions, means
nothing more than a persons actual residence or place of abode, provided he resides
therein with continuity and consistency.
While the recitals in death certificates can be considered proofs of a decedents
residence at the time of his death, the contents thereof, however, is not binding on
the courts. Both the RTC and the Court of Appeals found that Eliseo had been living
with Lourdes, deporting themselves as husband and wife, from 1972 up to the time
of his death in 1995. This finding is consistent with the fact that in 1985, Eliseo
Page | 140

Compilation of Digested Cases for Remedial Law Review II


by: K. M. T. BUELA
filed an action for judicial partition of properties against Amelia before the RTC of
Quezon City, Branch 106, on the ground that their marriage is void for being
bigamous.20 That Eliseo went to the extent of taking his marital feud with Amelia
before the courts of law renders untenable petitioners position that Eliseo spent the
final days of his life in Tarlac with Amelia and her children. It disproves rather than
supports petitioners submission that the lower courts findings arose from an
erroneous appreciation of the evidence on record. Factual findings of the trial court,
when affirmed by the appellate court, must be held to be conclusive and binding
upon this Court.
(2)
Section 2 of Rule 79 provides that a petition for Letters of Administration must be
filed by an interested person
An "interested party," in estate proceedings, is one who would be benefited in the
estate, such as an heir, or one who has a claim against the estate, such as a creditor.
Also, in estate proceedings, the phrase "next of kin" refers to those whose
relationship with the decedent is such that they are entitled to share in the estate as
distributees.
In the instant case, Elise, as a compulsory heir who stands to be benefited by the
distribution of Eliseos estate, is deemed to be an interested party. With the
overwhelming evidence on record produced by Elise to prove her filiation to Eliseo,
the petitioners pounding on her lack of interest in the administration of the
decedents estate, is just a desperate attempt to sway this Court to reverse the
findings of the Court of Appeals. Certainly, the right of Elise to be appointed
administratix of the estate of Eliseo is on good grounds. It is founded on her right
as a compulsory heir, who, under the law, is entitled to her legitimate after the debts
of the estate are satisfied.

PILAPIL and HEIRS OF DONATA ORTIZ BRIONES vs. HEIRS OF


MAXIMINO R. BRIONES
G.R. No. 150175, February 5, 2007
FACTS:
Maximino was married to Donata but their union did not produce any children.
When Maximino died on 1 May 1952, Donata instituted intestate proceedings to
settle her husbands estate. CFI issued Letters of Administration appointing Donata
as the administratrix of Maximinos estate. Subsequently, CFI issue an Order, dated
2 October 1952, awarding ownership of the aforementioned real properties to
Donata.

Page | 141

Compilation of Digested Cases for Remedial Law Review II


by: K. M. T. BUELA
Donata died on 1 November 1977. Erlinda, one of Donatas nieces, instituted with
the RTC a petition for the administration of the intestate estate of Donata. Erlinda
and her husband, Gregorio, were appointed by the RTC as administrators of
Donatas intestate estate.
On 3 March 1987, the heirs of Maximino filed a Complaint with the RTC against
the heirs of Donata for the partition, annulment, and recovery of possession of real
property. They alleged that Donata, as administratrix of the estate of Maximino,
through fraud and misrepresentation, in breach of trust, and without the knowledge
of the other heirs, succeeded in registering in her name the real properties belonging
to the intestate estate of Maximino.
After trial in due course, the RTC rendered its Decision, dated 8 April 1986, in favor
of the heirs of Maximino. The heirs of Donata appealed the RTC Decision before
the Court of Appeals but the latter court affirmed the decision. Unsatisfied the
Decision of the Court of Appeals, the heirs of Donata elevated the case before the
SC. SC reversed the decisions of CA and RTC and dismissed the Complaint for
partition, annulment, and recovery of possession of real property filed by the heirs
of Maximino in Civil Case No. CEB-5794.
ISSUE: Whether or not a judgment awarding ownership of the properties included
in the decedents estate to his surviving wife may be assailed on the ground of fraud
after more than 30 years had lapse from the promulgation of the said judgment.
HELD:
The answer is in the negative. The heirs of Maximino failed to prove by clear and
convincing evidence that Donata managed, through fraud, to have the real
properties, belonging to the intestate estate of Maximino, registered in her name. In
the absence of fraud, no implied trust was established between Donata and the heirs
of Maximino under Article 1456 of the New Civil Code. Donata was able to register
the real properties in her name, not through fraud or mistake, but pursuant to an
Order, dated 2 October 1952, issued by the CFI in Special Proceedings No. 928-R.
The CFI Order, presumed to be fairly and regularly issued, declared Donata as the
sole, absolute, and exclusive heir of Maximino; hence, making Donata the singular
owner of the entire estate of Maximino, including the real properties, and not merely
a co-owner with the other heirs of her deceased husband.
The CFI Order, dated 2 October 1952, issued in Special Proceedings No. 928-R,
effectively settled the intestate estate of Maximino by declaring Donata as the sole,
absolute, and exclusive heir of her deceased husband. The issuance by the CFI of
the said Order, as well as its conduct of the entire Special Proceedings No. 928-R,
enjoy the presumption of validity pursuant to the Section 3(m) and (n) of Rule 131
of the Revised Rules of Court, reproduced below
Page | 142

Compilation of Digested Cases for Remedial Law Review II


by: K. M. T. BUELA

(m) That official duty has been regularly performed;


(n) That a court, or judge acting as such, whether in the
Philippines or elsewhere, was acting in the lawful exercise
of jurisdiction.
By reason of the foregoing provisions, this Court must presume, in the absence of
any clear and convincing proof to the contrary, that the CFI in Special Proceedings
No. 928-R had jurisdiction of the subject matter and the parties, and to have
rendered a judgment valid in every respect; and it could not give credence to the
following statements made by the Court of Appeals in its Decision.
While it is true that since the CFI was not informed that Maximino still had
surviving siblings and so the court was not able to order that these siblings be given
personal notices of the intestate proceedings, it should be borne in mind that the
settlement of estate, whether testate or intestate, is a proceeding in rem, and
that the publication in the newspapers of the filing of the application and of
the date set for the hearing of the same, in the manner prescribed by law, is a
notice to the whole world of the existence of the proceedings and of the hearing
on the date and time indicated in the publication. The publication requirement
of the notice in newspapers is precisely for the purpose of informing all interested
parties in the estate of the deceased of the existence of the settlement proceedings,
most especially those who were not named as heirs or creditors in the petition,
regardless of whether such omission was voluntarily or involuntarily made.

RODOLFO C. SABIDONG vs. NICOLASITO S. SOLAS


A.M. No. P-01-1448, June 25, 2013
FACTS:
The subject of this controversy is Lot No. 11 which is part of the Hodges Estate.
Hodges Estate is the subject of a pending intestate proceedings.
Herein complainant is the son of Trinidad Sabidong, one of the longtime occupants
of Lot 11. Lot 11 was the subject of an ejectment suit filed by the Hodges Estate,
docketed at the MTCC Iloilo City, Branch 4. On May 31, 1983, a decision was
rendered in said case ordering the occupants to immediately vacate the portion of
Lot 11 leased to her and to pay the plaintiff rentals due, attorneys fees, expenses
and costs. At the time, respondent was the Clerk of Court III of MTCC, Branch 3,
Iloilo City.
On January 8, 1986, respondent submitted an Offer to Purchase Lot 11 with an area
of 234 square meters for the amount of P35,100. Under the Order dated November
Page | 143

Compilation of Digested Cases for Remedial Law Review II


by: K. M. T. BUELA
18, 1986 issued by the probate court, respondents Offer to Purchase Lot 11 was
approved.
On January 21, 1987, the probate court issued another Order granting respondents
motion for issuance of a writ of possession in his favor. The writ of possession over
Lot 11 was eventually issued on June 27, 1989. On November 21, 1994, a Deed of
Sale With Mortgage covering Lot 11 was executed. Lot 11 was thereby conveyed
to respondent on installment for the total purchase price of P50,000. Thus, a new
certificate of title in the name of respondent was issued.
On June 14, 1999, this Court received the sworn letter-complaint asserting that as
court employee respondent cannot buy property in litigation (consequently he is not
a buyer in good faith), commit deception, dishonesty, oppression and grave abuse
of authority.
ISSUE: Whether or not an estate proceeding may be considered a pending litigation
in relation to the property included is such estate which disallows a court official
from purchasing the same on the ground of Art. 1491 of the Civil Code.
HELD:
The answer is in the affirmative. Article 1491 (5) of the Civil Code provides that he
following persons cannot acquire by purchase, even at a public or judicial auction,
either in person or through the mediation of another: Justices, judges, prosecuting
attorneys, clerks of superior and inferior courts, and other officers and employees
connected with the administration of justice, the property and rights in litigation or
levied upon an execution before the court within whose jurisdiction or territory they
exercise their respective functions; this prohibition includes the act of acquiring by
assignment and shall apply to lawyers, with respect to the property and rights which
may be the object of any litigation in which they may take part by virtue of their
profession.
For the prohibition to apply, the sale or assignment of the property must take place
during the pendency of the litigation involving the property. Where the property is
acquired after the termination of the case, no violation of paragraph 5, Article 1491
of the Civil Code attaches.
In the case at bar, when respondent purchased Lot 11-A on November 21, 1994, the
Decision in Civil Case No. 14706 which was promulgated on May 31, 1983 had
long become final. Be that as it may, it cannot be said that the property is no longer
"in litigation" at that time considering that it was part of the Hodges Estate then
under settlement proceedings (Sp. Proc. No. 1672).

Page | 144

Compilation of Digested Cases for Remedial Law Review II


by: K. M. T. BUELA
A thing is said to be in litigation not only if there is some contest or litigation over
it in court, but also from the moment that it becomes subject to the judicial action
of the judge. A property forming part of the estate under judicial settlement
continues to be subject of litigation until the probate court issues an order declaring
the estate proceedings closed and terminated. The rule is that as long as the order
for the distribution of the estate has not been complied with, the probate proceedings
cannot be deemed closed and terminated. The probate court loses jurisdiction of an
estate under administration only after the payment of all the debts and the remaining
estate delivered to the heirs entitled to receive the same. Since there is no evidence
to show that Sp. Proc. No. 1672 in the RTC of Iloilo, Branch 27, had already been
closed and terminated at the time of the execution of the Deed of Sale With
Mortgage dated November 21, 1994, Lot 11 is still deemed to be "in litigation"
subject to the operation of Article 1491 (5) of the Civil Code.
This notwithstanding, it was held that the sale of Lot 11 in favor of respondent did
not violate the rule on disqualification to purchase property because Sp. Proc. No.
1672 was then pending before another court (RTC) and not MTCC where he was
Clerk of Court.

B. Escheat (Rule 91)


CASTORIO ALVARICO vs. AMELITA L. SOLA
G.R. No. 138953, June 6, 2002
FACTS:
Fermina A. Lopez, a widown, was an awardee of Lots Nos. 4, 5, 3-B, 3-C and 6-B,
Sgs-3451 and being the winning bidder at the auction sale of these parcels by the
Bureau of Lands. On May 28, 1983, Fermina executed a Deed of Self-Adjudication
and Transfer of Rights over Lot 5 in favor of Amelita, who agreed to assume all the
obligations, duties, and conditions imposed upon Fermina under MSA Application
No. V-81066. The document of transfer was filed with the Bureau of Lands.
Bureau of Lands issued an order approving the transfer of rights and granting the
amendment of the application from Fermina to Amelita. Consequently, an OCT was
issued in the name of Amelita and her husband.
On June 24, 1993, herein petitioner filed Civil Case No. CEB-1419110 for
reconveyance against Amelita. He claimed that on January 4, 1984, Fermina
donated the land to him and immediately thereafter, he took possession of the same.
He averred that the donation to him had the effect of withdrawing the earlier transfer
to Amelita.

Page | 145

Compilation of Digested Cases for Remedial Law Review II


by: K. M. T. BUELA
For her part, Amelita maintained that the donation to petitioner is void because
Fermina was no longer the owner of the property when it was allegedly donated to
petitioner, the property having been transferred earlier to her. She added that the
donation was void because of lack of approval from the Bureau of Lands, and that
she had validly acquired the land as Fermina's rightful heir. She also denied that she
is a trustee of the land for petitioner. After trial, the RTC rendered a decision in
favor of petitioner. On appeal, RTC decision was reversed.
ISSUE: Whether or not a person imputing bad faith on the transfer of land patents
may assailed the validity of an OCT subsequently issued to the transferee of the
land patents.
HELD:
The answer is in the negative. this allegation of bad faith on the part of Amelita Sola
in acquiring the title is devoid of evidentiary support. For one, the execution of
public documents, as in the case of Affidavits of Adjudication, is entitled to the
presumption of regularity, hence convincing evidence is required to assail and
controvert them. Second, it is undisputed that OCT No. 3439 was issued in 1989 in
the name of Amelita. It requires more than petitioner's bare allegation to defeat the
Original Certificate of Title which on its face enjoys the legal presumption of
regularity of issuance. A Torrens title, once registered, serves as notice to the whole
world. All persons must take notice and no one can plead ignorance of its
registration.
Even assuming that respondent Amelita Sola acquired title to the disputed property
in bad faith, only the State can institute reversion proceedings under Sec. 101 of the
Public Land Act, to wit: All actions for reversion to the Government of lands of the
public domain or improvements thereon shall be instituted by the Solicitor General
or the officer acting in his stead, in the proper courts, in the name of the Republic
of the Philippines.
In other words, a private individual may not bring an action for reversion or any
action which would have the effect of canceling a free patent and the corresponding
certificate of title issued on the basis thereof, such that the land covered thereby will
again form part of the public domain. Only the Solicitor General or the officer acting
in his stead may do so. Since Amelita Sola's title originated from a grant by the
government, its cancellation is a matter between the grantor and the grantee.30
Clearly then, petitioner has no standing at all to question the validity of Amelita's
title. It follows that he cannot "recover" the property because, to begin with, he has
not shown that he is the rightful owner thereof.

Page | 146

Compilation of Digested Cases for Remedial Law Review II


by: K. M. T. BUELA
C. Guardians and Guardianship (Rules 92 to 97)
PILAR Y. GOYENA vs. AMPARO LEDESMA-GUSTILO
G.R. No. 147148, January 13, 2003
FACTS:
On July 8, 1996, respondent (Amparo) filed a "PETITION FOR LETTERS OF
GUARDIANSHIP" over the person and properties of her sister Julieta, as the latter
is not in a position to care for herself, and that she needs the assistance of a guardian
to manage her interests in on-going corporate and agricultural enterprises. Petitioner
(Goyena) opposed the petition for letters of guardianship on the following grounds:
(1) that Julieta Ledesma is competent and sane and there is absolutely no need to
appoint a guardian to take charge of her person/property; (2) Respondent is not fit
to be appointed as the guardian of Julieta Ledesma since their interests are
antagonistic; and (3) because Petitioner thinks that the Respondent dislikes her.
The trial court found Julieta "incompetent and incapable of taking care of herself
and her property" and appointed respondent as guardian of her person and
properties. On appeal of petitioner, the Court of Appeals affirmed the trial court's
decision. CA held that there are really no antagonistic interests to speak of between
petitioner [Amparo] and Julieta, they being co-owners of certain properties. There
is also no showing that petitioner's business decisions in the past had resulted in the
prejudice of Julieta.
Petitioner's Motion for Reconsideration of the Court of Appeals decision having
been denied, she filed the present petition
ISSUE: Whether or not a grant for letters of guardianship may be assailed on the
ground of antagonistic interest and mere presumed dislike by the appointed
guardian of the oppositor.
HELD:
While, it was held in the case of Garchitorena v. Sotelo that a person with
antagonistic interest with that of the ward may not be appointed as guardian, no
such antagonistic interest exist in the present case as between the appointed
guardian, i.e., the Respondent and the ward, Julieta.
In the selection of a guardian, a large discretion must be allowed the judge who
deals directly with the parties. As a rule, when it appears that the judge has exercised
care and diligence in selecting the guardian, and has given due consideration to the
reasons for and against his action which are urged by the interested parties, his
action should not be disturbed unless it is made very clear that he has fallen into
grievous error.
Page | 147

Compilation of Digested Cases for Remedial Law Review II


by: K. M. T. BUELA

In the case at bar, petitioner has not shown that the lower courts committed any
error.
Finally, it should be noted that Petitioner opposed the petition for the appointment
of respondent as guardian before the trial court because, among other reasons, she
felt she was disliked by respondent, a ground which does not render respondent
unsuitable for appointment as guardian. Furthermore, Petitioner concealed the
deteriorating state of mind of Julieta before the trial court, which is reflective of a
lack of good faith.

The Incompetent, CARMEN CAIZA, represented by her legal guardian,


AMPARO EVANGELISTA vs. COURT OF APPEALS, et al.
G.R. No. 110427, February 24, 1997
FACTS:
Amparo A. Evangelista, judicially appointed legal guardian of the person and estate
of Carmen Caiza, representing the latter in the said capacity filed an unlawful
detainer suit against the Respondent Estradas. The Complaint alleges that: (1)
plaintiff Caiza was the absolute owner of the property in question; (2) that out of
kindness, she had allowed the Estrada Spouses, their children et al. to temporarily
reside in her house, rent-free; (3) Caiza already had urgent need of the house on
account of her advanced age and failing health, "so funds could be raised to meet
her expenses for support, maintenance and medical treatment;" (4) through her
guardian, Caiza had asked the Estradas verbally and in writing to vacate the house;
and (5) the complaint was filed within one (1) year from the date of first letter of
demand dated February 3, 1990.
In their Answer with Counterclaim, the defendants declared that Caiza executed a
holographic will on September 4, 1988 by which she "bequeathed" to the Estradas
the house and lot in question.
Judgment was rendered by the MetroTC in Caiza's favor. But on appeal, the
decision was reversed by the RTC. RTC held that the "action by which the issue of
defendants' possession should be resolved is accion publiciana. Later on, the
Appellate Court affirmed the RTC's judgment in toto. Hence, Caiza, through her
guardian, elevated the case before the SC.
While the case was pending before the SC, Carmen Caiza died and her heirs
the aforementioned guardian, Amparo Evangelista, and Ramon C. Nevado, her
niece and nephew, respectively were by this Court's leave, substituted for her.

Page | 148

Compilation of Digested Cases for Remedial Law Review II


by: K. M. T. BUELA
ISSUE:
(1) Whether or not a holographic will purportedly executed by the registered owner
of the property in favor of the defendants is a valid defense against an action for
unlawful detainer.
(2) Whether or not a legal guardian had authority to bring an action for unlawful
detainer in representation of his/her ward.
(3) Whether or not a guardian may continue to represent the ward after the latter's
death.
HELD:
(1)
The answer is in the negative. A will is essentially ambulatory; at any time prior to
the testator's death, it may be changed or revoked; and until admitted to probate, it
has no effect whatever and no right can be claimed thereunder, the law being quite
explicit: "No will shall pass either real or personal property unless it is proved and
allowed in accordance with the Rules of Court" (ART. 838, NCC).
Thus, at the time of the institution of the action of desahucio, the Estradas had no
legal right to the property, whether as possessors by tolerance or sufferance, or as
owners. They could not claim the right of possession by sufferance; that had been
legally ended. They could not assert any right of possession flowing from their
ownership of the house; their status as owners is dependent on the probate of the
holographic will by which the property had allegedly been bequeathed to them
an event which still has to take place; in other words, prior to the probate of the will,
any assertion of possession by them would be premature and inefficacious.
Metropolitan Trial Courts, Municipal Trial Courts, and Municipal Circuit Trial
Courts nevertheless have the undoubted competence to resolve "the issue of
ownership . . only to determine the issue of possession.
(2)
The answer is in the affirmative. In bringing the action of desahucio, Evangelista
was merely discharging the duty to attend to "the comfortable and suitable
maintenance of the ward" explicitly imposed on her by Section 4, Rule 96 of the
Rules of Court, viz.: A guardian must manage the estate of his ward frugally and
without waste, and apply the income and profits thereof, so far as maybe necessary,
to the comfortable and suitable maintenance of the ward and his family, if there be
any; and if such income and profits be insufficient for that purpose, the guardian
may sell or encumber the real estate, upon being authorized by order to do so, and
apply to such of the proceeds as may be necessary to such maintenance.
(3)

Page | 149

Compilation of Digested Cases for Remedial Law Review II


by: K. M. T. BUELA
While it is indeed well-established rule that the relationship of guardian and ward
is necessarily terminated by the death of either the guardian or the ward, 39 the rule
affords no advantage to the Estradas. Amparo Evangelista, as niece of Carmen
Caiza, is one of the latter's only two (2) surviving heirs, the other being Caiza's
nephew, Ramon C. Nevado. On their motion and by Resolution of this Court of
June 20, 1994, they were in fact substituted as parties in the appeal at bar in place
of the deceased, in accordance with Section 17, Rule 3 of the Rules of Court, viz.:
xxx The heirs of the deceased may be allowed to be substituted for the deceased,
without requiring the appointment of an executor or administrator and the court may
appoint guardian ad litem for the minor heirs. Xxxxx
To be sure, an ejectment case survives the death of a party. Caiza's demise did not
extinguish the desahucio suit instituted by her through her guardian. That action,
not being a purely personal one, survived her death; her heirs have taken her place
and now represent her interests in the appeal at bar.

NAPOLEON D. NERI, et al. vs. HEIRS OF HADJI YUSOP UY AND


JULPHA* IBRAHIM UY
G.R. No. 194366, October 10, 2012
FACTS:
Anunciacion had seven children, two (2) from her first marriage with Gonzalo Illut
(Gonzalo), namely: Eutropia and Victoria, and five (5) from her second marriage
with Enrique Neri (Enrique), namely: Napoleon, Alicia, Visminda, Douglas and
Rosa. Throughout the marriage of spouses Enrique and Anunciacion, they acquired
several homestead properties.
When Anunciacion died intestate, Enrique, in his personal capacity and as natural
guardian of his minor children Rosa and Douglas, together with Napoleon, Alicia,
and Visminda executed an Extra-Judicial Settlement of the Estate with Absolute
Deed of Sale on July 7, 1979, adjudicating among themselves the said homestead
properties, and thereafter, conveying them to the late spouses Hadji Yusop Uy and
Julpha Ibrahim Uy (spouses Uy)for a consideration of P 80,000.00.
17 years after, the children of Enrique filed a complaint for annulment of sale of the
said homestead properties against spouses Uy arguing that (1) the sale was made
within the prohibited period; and (2) children of Anunciacion from her first
marriage were excluded and deprived of their legitimes.
RTC ruled that the sale is still void because Eutropia and Victoria were deprived of
their hereditary rights and that Enrique had no judicial authority to sell the shares
of his minor children, Rosa and Douglas.
Page | 150

Compilation of Digested Cases for Remedial Law Review II


by: K. M. T. BUELA

On appeal, the CA reversed the ruling of the RTC. CA found it unconscionable to


permit the annulment of the sale considering spouses Uys possession thereof for
17 years. Furthermore, CA held that the extrajudicial settlement and sale valid as
far as the heirs of Anunciacion on the following grounds: (1) as to the children of
the first marriage, while they are not bound by the extrajudicial settlement, they
belatedly filed their claim or more than 2 years from knowledge of their exclusion
as heirs in 1994 when their stepfather died; (2) as to the children of the second
marriage who as a minor at the time of the sale, because they were deemed to have
ratified the sale when they failed to question it upon reaching the age of majority.
It also found laches to have set in because of their inaction for a long period of time.
ISSUE:
(1) Whether or not compulsory heirs who have not notice of the fact that the some
of the heirs executed an extrajudicial settlement and sale are bound thereof.
(2) Whether or not surviving spouse of the decedent, as natural guardian of the
minor heirs may dispose the share of such minor heirs in the properties
inherited.
(3) Whether or not the heirs who have no notice of the execution of the extrajudicial
settlement and sale or minor heirs who was represented by their surviving parent
in the said settlement may still question extrajudicial settlement and sale after
17 years.
HELD:
(1)
No. Section 1, Rule 74 of the Rules of Court provides: Extrajudicial settlement by
agreement between heirs. x x x The fact of the extrajudicial settlement or
administration shall be published in a newspaper of general circulation in the
manner provided in the next succeeding section; but no extrajudicial settlement
shall be binding upon any person who has not participated therein or had no notice
thereof.
In the execution of the Extra-Judicial Settlement of the Estate with Absolute Deed
of Sale in favor of spouses Uy, all the heirs of Anunciacion should have
participated. Considering that Eutropia and Victoria were admittedly excluded and
that then minors Rosa and Douglas were not properly represented therein, the
settlement was not valid and binding upon them and consequently, a total nullity.
(2)
No. Articles 320 and 326 of the Civil Code, the laws in force at the time of the
execution of the settlement and sale, provide:

Page | 151

Compilation of Digested Cases for Remedial Law Review II


by: K. M. T. BUELA
ART. 320. The father, or in his absence the mother, is the
legal administrator of the property pertaining to the child
under parental authority. If the property is worth more than
two thousand pesos, the father or mother shall give a bond
subject to the approval of the Court of First Instance.
ART. 326. When the property of the child is worth more than
two thousand pesos, the father or mother shall be considered
a guardian of the childs property, subject to the duties and
obligations of guardians under the Rules of Court.
Corollarily, Section 7, Rule 93 of the Rules of Court also provides: When the
property of the child under parental authority is worth two thousand pesos or less,
the father or the mother, without the necessity of court appointment, shall be his
legal guardian. When the property of the child is worth more than two thousand
pesos, the father or the mother shall be considered guardian of the childs property,
with the duties and obligations of guardians under these Rules, and shall file the
petition required by Section 2 hereof. For good reasons, the court may, however,
appoint another suitable persons.
Administration includes all acts for the preservation of the property and the receipt
of fruits according to the natural purpose of the thing. Any act of disposition or
alienation, or any reduction in the substance of the patrimony of child, exceeds the
limits of administration. Thus, a father or mother, as the natural guardian of the
minor under parental authority, does not have the power to dispose or encumber the
property of the latter. Such power is granted by law only to a judicial guardian of
the wards property and even then only with courts prior approval secured in
accordance with the proceedings set forth by the Rules of Court.
Consequently, the disputed sale entered into by Enrique in behalf of his minor
children without the proper judicial authority, unless ratified by them upon reaching
the age of majority, is unenforceable in accordance with Articles 1317 and 1403(1)
of the Civil Code.
(3)
Yes. Contrary to the ruling of the CA, the prescriptive period of 2 years provided in
Section 1 Rule 74 of the Rules of Court reckoned from the execution of the
extrajudicial settlement finds no application to petitioners Eutropia, Victoria and
Douglas, who were deprived of their lawful participation in the subject estate.
Besides, an "action or defense for the declaration of the inexistence of a contract
does not prescribe" in accordance with Article 1410 of the Civil Code.

Page | 152

Compilation of Digested Cases for Remedial Law Review II


by: K. M. T. BUELA
NILO OROPESA vs. CIRILO OROPESA1
G.R. No. 184528, April 25, 2012
FACTS:
Nilo Oropesa (petitioner) filed a petition to be appointed as guardians over the
property of his father, Cirilo Oropesa (Respondent). He alleged among others that:
(1) the respondent has been afflicted with several maladies and has been sickly for
over ten (10) years after suffering from stroke on 2003; (2) that his judgment and
memory were impaired; (3) that due to his age and medical condition, he cannot,
without outside aid, manage his property wisely, and has become an easy prey for
deceit and exploitation by people around him, particularly Ms. Ma. Luisa Agamata,
his girlfriend.
Respondent opposed the petition for guardianship. After the presentation of
evidence of the petitioner, Respondent filed his "Omnibus Motion (1) to Declare
the petitioner to have waived the presentation of his Offer of Exhibits and the
presentation of his Evidence Closed since they were not formally offered; (2) To
Expunge the Documents of the Petitioner from the Record; and (3) To Grant leave
to the Oppositor to File Demurrer to Evidence.
The trial court granted respondents demurrer to evidence. The Trial Court noted
the absence of any testimony of a medical expert which states that Gen. Cirilo O.
Oropesa does not have the mental, emotional, and physical capacity to manage his
own affairs. On the contrary, Oppositors evidence includes a Neuropsychological
Screening Report which states that Gen. Oropesa, (1) performs on the average range
in most of the domains that were tested; (2) is capable of mental calculations; and
(3) can provide solutions to problem situations. The Report concludes that Gen.
Oropesa possesses intact cognitive functioning, except for mildly impaired abilities
in memory, reasoning and orientation. It is the observation of the Court that
oppositor is still sharp, alert and able.
Petitioner moved for reconsideration but this was denied by the trial court.
Accordingly, petitioners Motion for Reconsideration is denied for lack of merit.
Petitioner elevated the case to the Court of Appeals but his appeal was dismissed.
ISSUE: Whether or not testimony of a medical expert is required to prove the
incompetency of the purported ward in a petition for guardianship.
HELD:
A finding that a person is incompetent should be anchored on clear, positive and
definite evidence. In an analogous guardianship case wherein the soundness of mind
of the proposed ward was at issue, it was held that "where the sanity of a person is
1

LEONARDO-DE CASTRO, J.
Page | 153

Compilation of Digested Cases for Remedial Law Review II


by: K. M. T. BUELA
at issue, expert opinion is not necessary and that the observations of the trial judge
coupled with evidence establishing the persons state of mental sanity will
suffice."
The trial court in its order denying the petitioners motion for reconsideration stated
its own observation of respondents physical and mental state, i.e., that oppositor
is still sharp, alert and able.

EDUARDO T. ABAD vs. LEONARDO BIASON and GABRIEL A.


MAGNO
G.R. No. 191993, December 5, 2012
FACTS:
Abad filed a petition for guardianship over the person and properties of Maura B.
Abad. When the petition was called for hearing, nobody entered an opposition and
Abad was allowed to present evidence ex parte. On June 14, 2007, Leonardo Biason
(Biason) filed a Motion for Leave to File Opposition to the Petition. Biason alleged
that he is also a nephew of Maura and opposed the appointment of Abad as Mauras
guardian as he cannot possibly perform his duties as such since he resides in Quezon
City while Maura maintains her abode in Pangasinan. Biason prayed that he be
appointed as Mauras guardian since he was previously granted by the latter with a
power of attorney to manage her properties.
RTC denied Abads petition and appointing Biason as Mauras guardian. Abad filed
a motion for reconsideration of the foregoing decision but the RTC denied the same.
On appeal, CA affirmed the decision of the RTC.
Thus, Abad filed a Petition for Review on Certiorari with the SC. Abad contends
that: (1) CA erred in affirming the RTCs decision despite the fact that it did not
hold any hearing to determine whether Biason possessed all the qualifications for a
guardian as provided by law; (2) he was not given the opportunity to submit
evidence to controvert Biasons appointment; and (3) the fact that he does not reside
in the place where the ward is residing is not a ground for his disqualification as
guardian.
Pending the resolution of the instant petition, Maura filed a Manifestation and
Motion, informing this Court that Biason died. She averred that Biasons death
rendered moot and academic the issues raised in the petition. She thus prayed that
the petition be dismissed and the guardianship be terminated.
ISSUE: Whether or not the death of the appointed guardian terminate the
guardianship and renders all issues assailing his appointment moot.
Page | 154

Compilation of Digested Cases for Remedial Law Review II


by: K. M. T. BUELA

HELD:
Yes. With Biasons demise, it has become impractical and futile to proceed with
resolving the merits of the petition. It is a well-established rule that the relationship
of guardian and ward is necessarily terminated by the death of either the
guardian or the ward. The supervening event of death rendered it pointless to
delve into the propriety of Biasons appointment since the juridical tie between him
and Maura has already been dissolved. The petition, regardless of its disposition,
will not afford Abad, or anyone else for that matter, any substantial relief.

D. Trustees (Rule 98)


ADVENT CAPITAL AND FINANCE CORPORATION vs. NICASIO I.
ALCANTARA and EDITHA I. ALCANTARA
G.R. No. 183050, January 25, 2012
FACTS:
Advent Capital and the Alcantaras entered into a trust agreement wherein the former
shall manage the latters several trust accounts and in lieu thereof, the latter shall
pay trust fees which could automatically be deducted from the Portfolio at the
end of each calendar quarter. However, Advent Capital failed to automatically
deduct the trust fees due to them at the time when they were still in possession of
the trust account. The trust account is now in the possession of Belson.
Subsequently, Advent Capital filed a petition for rehabilitation. Atty. Danilo L.
Concepcion was appointed as its rehabilitation receiver. Concepcion found that
respondents Alcantara owed Advent Capital P27,398,026.59, representing trust
fees. Concepcion filed a motion before the rehabilitation court to direct Belson to
release the cash dividends that Belson held under the Alcantaras Trust Account 95013.
Alcantaras opposed the motion and claimed that the money in the trust account
belonged to them under their Trust Agreement with Advent Capital. The latter, they
said, could not claim any right or interest in the dividends generated by their
investments since Advent Capital merely held these in trust for the Alcantaras, the
trustors-beneficiaries. For this reason, Atty. Concepcion had no right to compel the
delivery of the dividends to him as receiver.
The rehabilitation court granted Atty. Concepcions motion. Complying with the
rehabilitation courts order and Atty. Concepcions demand letter, Belson turned
over the subject dividends to him. Meanwhile, the Alcantaras filed a special civil
action of certiorari before the CA, seeking to annul the rehabilitation courts order.
Page | 155

Compilation of Digested Cases for Remedial Law Review II


by: K. M. T. BUELA
CA rendered a decision, granting the petition and directing Atty. Concepcion to
account for the dividends and deliver them to the Alcantaras.
The CA pointed out that the cash dividends in Belsons care cannot be summarily
applied to the payment of such charges. To enforce its lien, Advent Capital has to
file a collection suit. The rehabilitation court cannot simply enforce the latters
claim by ordering Belson to deliver the money to it.
ISSUE: Whether or not trust fees stipulated to be automatically deducted from the
trust account per trust agreement may be ordered by the rehabilitation court to be
delivered to the trustee when the trust account is already in the possession of a third
person.
HELD:
The answer is in the negative. The real owner of the trust property is the trustorbeneficiary. In this case, the trustors-beneficiaries are the Alcantaras. Thus, Advent
Capital could not dispose of the Alcantaras portfolio on its own. The income and
principal of the portfolio could only be withdrawn upon the Alcantaras written
instruction or order to Advent Capital.
Ultimately, the issue is what court has jurisdiction to hear and adjudicate the
conflicting claims of the parties over the dividends that Belson held in trust for their
owners. Certainly, not the rehabilitation court which has not been given the power
to resolve ownership disputes between Advent Capital and third parties. Neither
Belson nor the Alcantaras are its debtors or creditors with interest in the
rehabilitation.
Advent Capital must file a separate action for collection to recover the trust fees
that it allegedly earned and, with the trial courts authorization if warranted, put the
money in escrow for payment to whoever it rightly belongs. Having failed to collect
the trust fees at the end of each calendar quarter as stated in the contract, all it had
against the Alcantaras was a claim for payment which is a proper subject for an
ordinary action for collection. It cannot enforce its money claim by simply filing a
motion in the rehabilitation case for delivery of money belonging to the Alcantaras
but in the possession of a third party.

LAND BANK OF THE PHILIPPINES vs. LAMBERTO C. PEREZ, et al.


G.R. No. 166884, June 13, 2012
FACTS:
LBP extended a credit accommodation to ACDC through the execution of a Credit
Line Agreement. ACDC used the Letters of Credit/Trust Receipts Facility of the
Page | 156

Compilation of Digested Cases for Remedial Law Review II


by: K. M. T. BUELA
Agreement to buy construction materials. The respondents, as officers and
representatives of ACDC, executed trust receipts in connection with the
construction materials. The trust receipts matured, but ACDC failed to return to
LBP the proceeds of the construction projects or the construction materials subject
of the trust receipts.
When ACDC failed to comply with the demands of LBP, the latter filed the
affidavit-complaint for estafa before the City Prosecutors Office.
ACDC argued that its clients for the construction projects have not yet paid them;
thus, ACDC had yet to receive the proceeds of the materials that were the subject
of the trust receipts and were allegedly used for these constructions. As there were
no proceeds received from these clients, no misappropriation thereof could have
taken place.
ISSUE: Whether or not a criminal complaint for estafa is the proper remedy for an
alleged violation of the Trust Receipt Law by the Trustee.
HELD:
The answer is in the affirmative. However, the transaction between the parties to
this case is not a trust receipt. Violations of Trust Receipts Law are criminally
punishable, but no criminal complaint for violation of Article 315, paragraph 1(b)
of the Revised Penal Code, in relation with P.D. 115, should prosper against a
borrower who was not part of a genuine trust receipt transaction.
It is fundamental in a trust receipt transaction that the person who advanced
payment for the merchandise becomes the absolute owner of said merchandise and
continues as owner until he or she is paid in full, or if the goods had already been
sold, the proceeds should be turned over to him or to her. In all trust receipt
transactions, the trustee has the obligation to either the return of the proceeds of the
sale or the return or recovery of the goods, whether raw or processed.
When both parties enter into an agreement knowing that the return of the goods
subject of the trust receipt is not possible even without any fault on the part of the
trustee, it is not a trust receipt transaction penalized under Section 13 of P.D. 115;
the only obligation actually agreed upon by the parties would be the return of the
proceeds of the sale transaction. This transaction becomes a mere loan, where the
borrower is obligated to pay the bank the amount spent for the purchase of the
goods.
The fact that LBP had knowingly authorized the delivery of construction materials
to a construction site ACDCs projects, repudiates the idea that LBP intended to be
the owner of those construction materials. As a government financial institution,
Page | 157

Compilation of Digested Cases for Remedial Law Review II


by: K. M. T. BUELA
LBP should have been aware that the materials were to be used for the construction
of an immovable property, as well as a property of the public domain. As an
immovable property, the ownership of whatever was constructed with those
materials would presumably belong to the owner of the land, under Article 445 of
the Civil Code
It should be noted line of work that the borrowers were engaged in was construction.
In the case of materials used in the manufacture of finished products, these finished
products if not the raw materials or their components similarly remain in the
possession of the trustee until they are sold. But the goods and the materials that are
used for a construction project are often placed under the control and custody of the
clients employing the contractor, who can only be compelled to return the materials
if they fail to pay the contractor and often only after the requisite legal proceedings.
The contractors difficulty and uncertainty in claiming these materials (or the
buildings and structures which they become part of), as soon as the bank
demands them, disqualify them from being covered by trust receipt
agreements.
Since these transactions are not trust receipts, an action for estafa should not be
brought against the respondents, who are liable only for a loan. As the law stands
today, there can be no violation of the right against imprisonment for non-payment
of a debt.
Furthermore, the proceedings regarding the criminal aspect of this case, should be
dismissed as it does not appear from the records that the complaint was filed with
the participation or consent of the Office of the Solicitor General (OSG).

EDWARD C. ONG vs. THE COURT OF APPEALS AND THE PEOPLE


OF THE PHILIPPINES
G.R. No. 119858, April 29, 2003
FACTS:
Petitioner representing ARMAGRI executed two sets of Trust Receipts in favor of
Solidbank. When the trust receipts became due and demandable, ARMAGRI failed
to pay or deliver the goods to the Bank despite several demand letters.
Consequently, Petitioner was charged with two counts of estafa for violation of the
Trust Receipts Law.
The trial court convicted him of the said charged. On appeal with CA affirmed the
decision of the trial court. Petitioner elevated the case before the SC. Petitioner
contends that he is neither an officer nor a director of AMAGRI and that he merely
acted as an agent of ARMAGRI. Petitioner asserts that nowhere in the trust receipts
Page | 158

Compilation of Digested Cases for Remedial Law Review II


by: K. M. T. BUELA
did he assume personal responsibility for the undertakings of ARMAGRI which
was the entrustee.
ISSUE: Whether or not a person who signs as an agent of the corporation a trust
receipt agreement may be held liable for estafa in the event that the said agreement
had been breach.
HELD:
The answer is in the affirmative. petitioner comes within the purview of Section 13
of the Trust Receipts Law which provides: If the violation is committed by a
corporation, partnership, association or other juridical entities, the penalty
provided for in this Decree shall be imposed upon the directors, officers, employees
or other officials or persons therein responsible for the offense, without prejudice
to the civil liabilities arising from the offense.
In the instant case, the Bank was the entruster while ARMAGRI was the entrustee.
Being the entrustee, ARMAGRI was the one responsible to account for the goods
or its proceeds in case of sale. However, the criminal liability for violation of the
Trust Receipts Law falls on the human agent responsible for the violation.
Petitioner, who admits being the agent of ARMAGRI, is the person responsible for
the offense for two reasons. First, petitioner is the signatory to the trust receipts, the
loan applications and the letters of credit. Second, despite being the signatory to the
trust receipts and the other documents, petitioner did not explain or show why he is
not responsible for the failure to turn over the proceeds of the sale or account for
the goods covered by the trust receipts.
The Trust Receipts Law expressly makes the corporation's officers or employees or
other persons therein responsible for the offense liable to suffer the penalty of
imprisonment. In the instant case, petitioner signed the two trust receipts on behalf
of ARMAGRI 24 as the latter could only act through its agents.
True, petitioner acted on behalf of ARMAGRI. However, it is a well-settled rule
that the law of agency governing civil cases has no application in criminal cases.
When a person participates in the commission of a crime, he cannot escape
punishment on the ground that he simply acted as an agent of another party. In the
instant case, the Bank accepted the trust receipts signed by petitioner based on
petitioner's representations. It is the fact of being the signatory to the two trust
receipts, and thus a direct participant to the crime, which makes petitioner a person
responsible for the offense.

Page | 159

Compilation of Digested Cases for Remedial Law Review II


by: K. M. T. BUELA
E. Adoption and Custody of Minors (Rules 99 to 100)
CANG vs. COURT OF APPEALS and Spouses RONALD V. CLAVANO
and MARIA CLARA CLAVANO
G.R. No. 105308, September 25, 1998
FACTS:
Petitioner Herbert Cang and Anna Marie Clavano were legally separated with the
custody of their three (3) children remained with the latter. After the separation,
Petitioner then left for the United States where he divorced Anna Marie.
On September 25, 1987, private respondents Spouses Clavano, respectively the
brother and sister-in-law of Anna Marie, filed for the adoption of the three minor
Cang children. The petition bears the signature of then 14-year-old Keith signifying
consent to his adoption. Anna Marie likewise filed an affidavit of consent alleging
that her husband had "evaded his legal obligation to support" his children; that her
brothers and sisters including Ronald V. Clavano, had been helping her in taking
care of the children; that because she would be going to the United States to attend
to a family business, "leaving the children would be a problem and would naturally
hamper (her) job-seeking venture abroad;" and that her husband had "long forfeited
his parental rights
Upon learning of the petitioner for adoption, petitioner immediately returned to the
Philippines and filed an opposition thereto. The trial court issued a decree of
adoption. The said decree was affirmed by the CA. Petitioner filed a motion for
reconsideration but was denied. Hence, Petitioner filed a petition for review on
certiorari before the SC alleging that the petition for adoption was fatally defective
as it did not have his written consent as a natural father as required by Article 31
(2) of Presidential Decree No. 603, the Child and Youth Welfare Code, and Article
188 (2) of the Family Code.
ISSUE: Whether or not minor children may be legally adopted without the written
consent of a natural parent on the ground that the latter has abandoned them.
HELD:
While the law requires that the written consent of the natural parent is indispensable
for the validity of the decree of adoption. Nevertheless, the requirement of written
consent can be dispensed with if the parent has abandoned the child or that
such parent is "insane or hopelessly intemperate." The court may acquire
jurisdiction over the case even, without the written consent of the parents or one of
the parents provided that the petition for adoption alleges facts sufficient to warrant
exemption from compliance therewith. This is in consonance with the liberality
with which this Court treats the procedural aspect of adoption.

Page | 160

Compilation of Digested Cases for Remedial Law Review II


by: K. M. T. BUELA
In the instant case, records disclose that petitioner's conduct did not manifest a
settled purpose to forego all parental duties and relinquish all parental claims over
his children as to, constitute abandonment. Physical estrangement alone, without
financial and moral desertion, is not tantamount to abandonment. 24 While
admittedly, petitioner was physically absent as he was then in the United States, he
was not remiss in his natural and legal obligations of love, care and support for his
children. He maintained regular communication with his wife and children through
letters and telephone. He used to send packages by mail and catered to their whims.

TOMASA VDA. DE JACOB, as Special Administratrix of the Intestate


Estate of Deceased Alfredo E. Jacob vs. COURT OF APPEALS, et al.
G.R. No. 135216, August 19, 1999
FACTS:
Petitioner filed a petition for settlement of the estate of the deceased Alfredo.
During the pendency of the said proceedings, Respondent sought to intervene
therein claiming his share of the deceaseds estate as Alfredo's adopted son and as
his sole surviving heir.
Respndent presented the Order in Special Proceedings No. 192 issued by then
Presiding Judge Moya granting the petition for adoption filed by deceased Alfredo
which declared therein Pedro Pilapil as the legally adopted son of Alfredo.
As the presiding judge was already 79 years old and was suffering from "glaucoma",
his deposition was taken at his residence. During the taking of the deposition, when
asked to identify his signature on the alleged adoption decree, he said: I do not
remember having issued such an order and the signature reading Jose; I cant make
out clearly what comes after the name; Jose Moya is not my signature.
The trial court then consulted two (2) handwriting experts to test the authenticity
and genuineness of Judge Moya's signature. Confronted with two (2) conflicting
reports, the trial court sustained the findings of Atty. Pagui declaring the signature
of Judge Moya in the challenged Order as genuine and authentic.
ISSUE: Whether or not the fact of adoption may be proved merely by a decree of
adoption wherein the signature of the issuing judge is in question.
HELD:
The burden of proof in establishing adoption is upon the person claiming such
relationship. This Respondent Pilapil failed to do. Moreover, the evidence presented
by petitioner shows that the alleged adoption is a sham.

Page | 161

Compilation of Digested Cases for Remedial Law Review II


by: K. M. T. BUELA
The alleged Order was purportedly made in open court. In his Deposition, however,
Judge Moya declared that he did not dictate decisions in adoption cases. The only
decisions he made in open court were criminal cases, in which the accused pleaded
guilty. Moreover, Judge Moya insisted that the branch where he was assigned was
always indicated in his decisions and orders; yet the questioned Order did not
contain this information. Furthermore, Pilapils conduct gave no indication that he
recognized his own alleged adoption, as shown by the documents that he signed and
other acts that he performed thereafter.
In the same vein, no proof was presented that Dr. Jacob had treated him as an
adopted child. Likewise, both the Bureau of Records Management in Manila and
the Office of the Local Civil Registrar of Tigaon, Camarines Sur, issued
Certifications that there was no record that Pedro Pilapil had been adopted by Dr.
Jacob. Taken together, these circumstances inexorably negate the alleged adoption
of respondent.

REPUBLIC OF THE PHILIPPINES vs. HON. JOSE R. HERNANDEZ, et


al.
G.R. No. 117209, February 9, 1996
FACTS:
Private respondent spouses filed a petition to adopt the minor Kevin Earl Bartolome
Moran. In the very same petition, private respondents prayed for the change of the
first name or said minor adoptee to Aaron Joseph, the same being the name with
which he was baptized in keeping with religious tradition and by which he has been
called by his adoptive family, relatives and friends since he arrived at private
respondents' residence.
Petitioner opposed the inclusion of the relief for change of name in the same petition
for adoption. Petitioner further contends that what the law allows is the change of
the surname of the adoptee, as a matter of right, to conform with that of the adopter
and as a natural consequence of the adoption thus granted. If what is sought is the
change of the registered given or proper name, and since this would involve a
substantial change of one's legal name, a petition for change of name under Rule
103 should accordingly be instituted, with the substantive and adjective requisites
therefor being conformably satisfied.
Private respondents, on the contrary, admittedly filed the petition for adoption with
a prayer for change of name predicated upon Section 5, Rule 2 which allows
permissive joinder of causes of action in order to avoid multiplicity of suits and in
line with the policy of discouraging protracted and vexatious litigations.

Page | 162

Compilation of Digested Cases for Remedial Law Review II


by: K. M. T. BUELA
ISSUE: Whether or not a petition for change of name of the adopted may be filed
jointly with the petition for adoption.
HELD:
The answer is in the negative. If a change in one's name is desired, this can only be
done by filing and strictly complying with the substantive and procedural
requirements for a special proceeding for change of name under Rule 103 of the
Rules of Court, wherein the sufficiency of the reasons or grounds therefor can be
threshed out and accordingly determined.
A petition for change of name being a proceeding in rem. It is an independent and
discrete special proceeding, in and by itself, governed by its own set of rules. A
fortiori, it cannot be granted by means of any other proceeding. To consider it as a
mere incident or an offshoot of another special proceeding would be to denigrate its
role and significance as the appropriate remedy available under our remedial law
system.
Neither can the allowance of the subject petition, by any stretch of imagination and
liberality, be justified under the rule allowing permissive joinder of causes of action.
While the rule allows a plaintiff to join as many separate claims as he may have,
there should nevertheless be some unity in the problem presented and a common
question of law and fact involved, subject always to the restriction thereon regarding
jurisdiction, venue and joinder of parties. Unlimited joinder is not authorized.
Turning now to the present petition, while it is true that there is no express
prohibition against the joinder of a petition for adoption and for change of name,
there is no relation between these two petitions, nor are they of the same nature or
character, much less do they present any common question of fact or law, which
conjointly would warrant their joinder. In short, these petitions do not rightly
meet the underlying test of conceptual unity demanded to sanction their
joinder under our Rules.
The policy of avoiding multiplicity of suits which underscores the rule on
permissive joinder of causes of action is addressed to suits that are intimately related
and also present interwoven and dependent issues which can be most expeditiously
and comprehensively settled by having just one judicial proceeding, but not to suits
or actions whose subject matters or corresponding reliefs are unrelated or diverse
such that they are best taken up individually.

Page | 163

Compilation of Digested Cases for Remedial Law Review II


by: K. M. T. BUELA
REPUBLIC OF THE PHILIPPINES vs. THE COURT OF APPEALS,
JAIME B. CARANTO, and ZENAIDA P. CARANTO
G.R. No. 103695, March 15, 1996
FACTS:
private respondents' petition for the adoption of Midael C. Mazon with prayer for
the correction of the minor's first name "Midael" to "Michael." The Solicitor
General opposed the petition insofar as it sought the correction of the name of the
child from "Midael" to "Michael." He argued that although the correction sought
concerned only a clerical and innocuous error, it could not be granted because the
petition was basically for adoption, not the correction of an entry in the civil registry
under Rule 108 of the Rules of Court. The RTC dismissed the opposition of the
Solicitor General on the ground that Rule 108 of the Rules of Court (Cancellation
or Correction of Entries in the Civil Registry) applies only to the correction of
entries concerning the civil status of persons. The Court of Appeals affirmed in toto
the decision of the RTC.
ISSUE:
(1) Whether or not a petition for adoption may be denied on the ground that the
publication thereof contains the misspelled name of the person to be adopted.
(2) Whether or not Rule 108 of the Rules of Court applies to a correction of name
prayed for in a petition for adoption.
HELD:
(1)
The answer is in the affirmative. The present case involves an obvious clerical error
in the name of the child sought to be adopted. In this case the correction involves
merely the substitution of the letters "ch" for the letter "d," so that what appears as
"Midael" as given name would read "Michael." Even the Solicitor General admits
that the error is a plainly clerical one. The purpose of the publication requirement
is to give notice so that those who have any objection to the adoption can make their
objection known. That purpose has been served by publication of notice in this case.
(2)
The answer is in the affirmative. Rule 108 of the Rules of Court applies to this case
and because its provision was not complied with, the decision of the trial court,
insofar as it ordered the correction of the name of the minor, is void and without
force or effect. This case falls under letter "(o)," of Rule 108, referring to "changes
of name."
Now 3 of this Rule further provides that the civil registrar and all persons who
have or claim any interest which would be affected thereby shall be made parties to
the proceeding.
Page | 164

Compilation of Digested Cases for Remedial Law Review II


by: K. M. T. BUELA

The local civil registrar is thus an indispensable party, without whom no final
determination of the case can be had. As he was not impleaded in this case much
less given notice of the proceeding, the decision of the trial court, insofar as it
granted the prayer for the correction of entry, is void. The absence of an
indispensable party in a case renders ineffectual all the proceedings subsequent to
the filing of the complaint including the judgment.
NB: SC ruled that granting of correction of name is invalid for failure to comply
with the provisions of Rule 108 but failed to discuss whether correction of name
may be prayed for in a petition for adoption.

EUGENIO R. REYES, et al. vs. LIBRADA F. MAURICIO (deceased) and


LEONIDA F. MAURICIO
G.R. No. 175080, November 24, 2010
FACTS:
Respondents Librada F. Mauricio (Librada filed before the DARAB of Malolos,
Bulacan a complaint for annulment of contract denominated as Kasunduan and
between Librada and Petitioner Eugenio as parties.
Librada died during the pendency of the case and was substituted by her alleged
daughter Leonida F. Mauricio. Leonidas legal standing as a party was assailed by
Eugenio. Eugenio submitted that the complaint was rendered moot with the death
of Librada, Godofredos sole compulsory heir. Eugenio contended that Leonida is
a mere ward of Godofredo and Librada, thus, not a legal heir.
ISSUE: whether or not legal standing of person who substituted the original
complainant may be assailed of by questioning the validity formers adoption by
the latter in an action for annulment of contract
HELD:
The answer is in the negative. It is settled law that filiation cannot be collaterally
attacked. The legitimacy of the child cannot be contested by way of defense or as a
collateral issue in another action for a different purpose.
The same rule is applied to adoption such that it cannot also be made subject to a
collateral attack. In Reyes v. Sotero, Supreme Court reiterated that adoption cannot
be assailed collaterally in a proceeding for the settlement of a decedents estate.
Furthermore, in Austria v. Reyes, Supreme Court declared that the legality of the
adoption by the testatrix can be assailed only in a separate action brought for that
purpose and cannot be subject to collateral attack.
Page | 165

Compilation of Digested Cases for Remedial Law Review II


by: K. M. T. BUELA

IN THE MATTER OF THE ADOPTION OF STEPHANIE NATHY


ASTORGA GARCIA
G.R. No. 148311, March 31, 2005
FACTS:
Honorato B. Catindig, herein petitioner, filed a petition to adopt his minor
illegitimate child Stephanie Nathy Astorga Garcia. He alleged therein, among
others, that Stephanie was born on June 26, 1994; that her mother is Gemma
Astorga Garcia; that Stephanie has been using her mothers middle name and
surname; and that he is now a widower and qualified to be her adopting parent. He
prayed that Stephanies middle name Astorga be changed to Garcia, her mothers
surname, and that her surname Garcia be changed to Catindig, his surname.
The trial court granted the petition for adoption and declared that the minor shall be
known as STEPHANIE NATHY CATINDIG. Petitioner filed a motion for
clarification and/or reconsideration praying that Stephanie should be allowed to use
the surname of her natural mother (GARCIA) as her middle name.
The trial court denied petitioners motion for reconsideration holding that there is
no law or jurisprudence allowing an adopted child to use the surname of his
biological mother as his middle name.
ISSUE: Whether or not an illegitimate child, upon adoption by her natural father,
use the surname of her natural mother as her middle name.
HELD:
There is no law regulating the use of a middle name. The middle name or the
mothers surname is only considered in Article 375(1) of the Civil Code, in case
there is identity of names and surnames between ascendants and descendants, in
which case, the middle name or the mothers surname shall be added.
Notably, the law is likewise silent as to what middle name an adoptee may use. In
the case of an adopted child, the law provides that the adopted shall bear the
surname of the adopters. What it only expressly allows, as a matter of right and
obligation, is for the adoptee to bear the surname of the adopter, upon issuance of
the decree of adoption.
One of the effects of adoption is that the adopted is deemed to be a legitimate child
of the adopter for all intents and purposes pursuant to Article 189 of the Family
Code and Section 17 Article V of RA 8552.

Page | 166

Compilation of Digested Cases for Remedial Law Review II


by: K. M. T. BUELA
Being a legitimate child by virtue of her adoption, it follows that Stephanie is
entitled to all the rights provided by law to a legitimate child without discrimination
of any kind, including the right to bear the surname of her father and her mother, as
discussed above. This is consistent with the intention of the members of the Civil
Code and Family Law Committees as earlier discussed. In fact, it is a Filipino
custom that the initial or surname of the mother should immediately precede the
surname of the father.

F. Habeas Corpuz (Rule 102)


ERLINDA K. ILUSORIO vs. ERLINDA I. BILDNER and SYLVIA K.
ILUSORIO, JOHN DOE and JANE DOE
G.R. No. 139789, May 12, 2000
FACTS:
Erlinda Kalaw and Potenciano Ilusorio contracted matrimony and lived together for
a period of thirty (30) years. In 1972, they separated from bed and board for
undisclosed reasons.
On February 25, 1998, Erlinda filed with the Regional Trial Court, Antipolo City a
petition10 for guardianship over the person and property of Potenciano Ilusorio due
to the latters advanced age, frail health, poor eyesight and impaired judgment.
On May 31, 1998, after attending a corporate meeting in Baguio City, Potenciano
Ilusorio did not return to Antipolo City and instead lived at Cleveland
Condominium, Makati. On March 11, 1999, Erlinda filed with the Court of Appeals
a petition for habeas corpus to have the custody of lawyer Potenciano Ilusorio. She
alleged that respondents refused petitioners demands to see and visit her husband
and prohibited Potenciano from returning to Antipolo City.
ISSUE:
Whether or not a wife secure a writ of habeas corpus to compel her husband to live
with her in conjugal bliss
HELD:
The answer is no. Marital rights including coverture and living in conjugal dwelling
may not be enforced by the extra-ordinary writ of habeas corpus.
A writ of habeas corpus extends to all cases of illegal confinement or detention, or
by which the rightful custody of a person is withheld from the one entitled thereto.
It is available where a person continues to be unlawfully denied of one or more of
his constitutional freedoms, where there is denial of due process, where the
Page | 167

Compilation of Digested Cases for Remedial Law Review II


by: K. M. T. BUELA
restraints are not merely involuntary but are unnecessary, and where a deprivation
of freedom originally valid has later become arbitrary. It is devised as a speedy and
effectual remedy to relieve persons from unlawful restraint, as the best and only
sufficient defense of personal freedom
To justify the grant of the petition, the restraint of liberty must be an illegal
and involuntary deprivation of freedom of action. The illegal restraint of
liberty must be actual and effective, not merely nominal or moral.
The evidence shows that there was no actual and effective detention or deprivation
of lawyer Potenciano Ilusorios liberty that would justify the issuance of the writ.
The fact that lawyer Potenciano Ilusorio is about 86 years of age, or under
medication does not necessarily render him mentally incapacitated. Soundness of
mind does not hinge on age or medical condition but on the capacity of the
individual to discern his actions.

ATTY. EDWARD SERAPIO vs. SANDIGANBAYAN, et al.


G.R. No. 148468, January 28, 2003
FACTS:
Herein petitioner was charged with plunder together with former President Estrada
and Jinggoy Estrada before the Sandiganbayan. Before the arraignment, petitioner
filed a series of motions and/or petitions before the Sandiganbayan, among which
are as follows: (1) petition for bail; (2) motion to quash amended information; and
(3) petition for habeas corpus on the ground that the delay in proceeding with the
bail hearing was caused by the prosecution, thus, prosecution is deemed to have
waive its right to question the propriety of grant of bail.
Petitioner also assailed the decision of the Sandiganbayan in allowing the joint
hearing of petitioners petition for bail with that of President Estrada and Jinggoys
petition for bail.
ISSUE:
(1) Whether or not petitioner should first be arraigned before hearings of his
petition for bail may be conducted;
(2) Whether petitioner may file a motion to quash the amended Information during
the pendency of his petition for bail;
(3) Whether or not petition for bail of two defendants may be heard jointly;
(4) Whether or not petition for issuance of writ of habeas corpus may be availed of
on the ground of delay in the hearing of a petition for bail not as a matter of
right.

Page | 168

Compilation of Digested Cases for Remedial Law Review II


by: K. M. T. BUELA
HELD:
(1)
The arraignment of an accused is not a prerequisite to the conduct of hearings on
his petition for bail. A person is allowed to petition for bail as soon as he is deprived
of his liberty by virtue of his arrest or voluntary surrender. An accused need not
wait for his arraignment before filing a petition for bail.
To condition the grant of bail to an accused on his arraignment would be to place
him in a position where he has to choose between (1) filing a motion to quash and
thus delay his release on bail because until his motion to quash can be resolved, his
arraignment cannot be held, and (2) foregoing the filing of a motion to quash so that
he can be arraigned at once and thereafter be released on bail. This would undermine
his constitutional right not to be put on trial except upon a valid complaint or
Information sufficient to charge him with a crime and his right to bail.
(2)
These two reliefs have objectives which are not necessarily antithetical to each
other. There is no inconsistency exists between an application of an accused for bail
and his filing of a motion to quash. Bail is the security given for the release of a
person in the custody of the law, furnished by him or a bondsman, to guarantee his
appearance before any court as required under the conditions set forth under the
Rules of Court. Its purpose is to obtain the provisional liberty of a person charged
with an offense until his conviction while at the same time securing his appearance
at the trial. As stated earlier, a person may apply for bail from the moment that he
is deprived of his liberty by virtue of his arrest or voluntary surrender.
On the other hand, a motion to quash an Information is the mode by which an
accused assails the validity of a criminal complaint or Information filed against him
for insufficiency on its face in point of law, or for defects which are apparent in the
face of the Information. An accused may file a motion to quash the Information, as
a general rule, before arraignment.
(3)
There is no provision in the Revised Rules of Criminal Procedure or the Rules of
Procedure of the Sandiganbayan governing the hearings of two or more petitions
for bail filed by different accused or that a petition for bail of an accused be heard
simultaneously with the trial of the case against the other accused. The matter is
addressed to the sound discretion of the trial court.
However, in the cases at bar, the joinder of the hearings of the petition for bail of
petitioner with the trial of the case against former President Joseph E. Estrada is an
entirely different matter as it will prejudice the petitioner. Because although all
defendants were charged with plunder, the alleged specific violation as to each one
Page | 169

Compilation of Digested Cases for Remedial Law Review II


by: K. M. T. BUELA
of them is different. The petitioner is merely charged with conspired with the other
co-accused named in sub-paragraph (a) by "receiving or collecting, directly or
indirectly, on several instances, money x x x from illegal gambling, x x x in
consideration of toleration or protection of illegal gambling
Thus, with respect to petitioner, all that the prosecution needs to adduce to prove
that the evidence against him for the charge of plunder is strong are those related to
the alleged receipt or collection of money from illegal gambling as described in subparagraph (a) of the amended Information. With the joinder of the hearing of
petitioner's petition for bail and the trial of the former President, the latter will have
the right to cross-examine intensively and extensively the witnesses for the
prosecution in opposition to the petition for bail of petitioner.
The joinder of the hearing of petitioner's bail petition with the trial of former
President Joseph E. Estrada will be prejudicial to petitioner as it will unduly delay
the determination of the issue of the right of petitioner to obtain provisional liberty
and seek relief from this Court if his petition is denied by the respondent court.
(4)
The answer is in the negative. As a general rule, the writ of habeas corpus will not
issue where the person alleged to be restrained of his liberty in custody of an officer
under a process issued by the court which jurisdiction to do so. In exceptional
circumstances, habeas corpus may be granted by the courts even when the person
concerned is detained pursuant to a valid arrest or his voluntary surrender, for this
writ of liberty is recognized as "the fundamental instrument for safeguarding
individual freedom against arbitrary and lawless state action" due to "its ability to
cut through barriers of form and procedural mazes." Thus, in previous cases, we
issued the writ where the deprivation of liberty, while initially valid under the law,
had later become invalid, and even though the persons praying for its issuance were
not completely deprived of their liberty.
The Court finds no basis for the issuance of a writ of habeas corpus in favor of
petitioner. The general rule that habeas corpus does not lie where the person alleged
to be restrained of his liberty is in the custody of an officer under process issued by
a court which had jurisdiction to issue the same applies, because petitioner is under
detention pursuant to the order of arrest issued by the Sandiganbayan on April 25,
2001 after the filing by the Ombudsman of the amended information for plunder
against petitioner and his co-accused. Petitioner had in fact voluntarily surrendered
himself to the authorities on April 25, 2001 upon learning that a warrant for his
arrest had been issued.
A petition for habeas corpus is not the appropriate remedy for asserting one's
right to bail. It cannot be availed of where accused is entitled to bail not as a
Page | 170

Compilation of Digested Cases for Remedial Law Review II


by: K. M. T. BUELA
matter of right but on the discretion of the court and the latter has not abused
such discretion in refusing to grant bail, or has not even exercised said
discretion. The proper recourse is to file an application for bail with the court
where the criminal case is pending and to allow hearings thereon to proceed.
The ruling in Moncupa vs. Enrile that habeas corpus will lie where the deprivation
of liberty which was initially valid has become arbitrary in view of subsequent
developments finds no application in the present case because the hearing on
petitioner's application for bail has yet to commence. The delay in the hearing of
petitioner's petition for bail cannot be pinned solely on the Sandiganbayan or on the
prosecution for that matter. Petitioner himself is partly to be blamed with the series
of pleadings filed before the Sandiganbayan which totaled to 8 and 32 is those filed
by other defendants will be considered.

PANFILO LACSON, et al. vs. SECRETARY HERNANDO PEREZ, et al.


G.R. No. 147780, May 10, 2001
FACTS:
On May 1, 2001, President Macapagal-Arroyo issued Proclamation No. 38
declaring that there was a state of rebellion in the National Capital Region. She
likewise issued General Order No. 1 directing the Armed Forces of the Philippines
and the Philippine National Police to suppress the rebellion in the National Capital
Region. Warrantless arrests of several alleged leaders and promoters of the
"rebellion" were thereafter effected.
Aggrieved by the warrantless arrests, and the declaration of a "state of rebellion,"
which allegedly gave a semblance of legality to the arrests, a petition for
prohibition, injunction, mandamus, and habeas corpus (with an urgent application
for the issuance of temporary restraining order and/or writ of preliminary
injunction) filed by Panfilio M. Lacson, Michael Ray B. Aquino, and Cezar O.
Mancao. They alleged that they are under imminent danger of being arrested.
ISSUE: Whether or not petition for habeas corpus may be availed of against an
imminent danger from warrantless arrest.
HELD:
petitioners' contention in G. R. No. 147780 (Lacson Petition), 147781 (DefensorSantiago Petition), and 147799 (Lumbao Petition) that they are under imminent
danger of being arrested without warrant do not justify their resort to the
extraordinary remedies of mandamus and prohibition, since an individual subjected
to warrantless arrest is not without adequate remedies in the ordinary course of law.
Such an individual may ask for a preliminary investigation under Rule 112 of the
Page | 171

Compilation of Digested Cases for Remedial Law Review II


by: K. M. T. BUELA
Rules of Court, where he may adduce evidence in his defense, or he may submit
himself to inquest proceedings to determine whether or not he should remain under
custody and correspondingly be charged in court. Further, a person subject of a
warrantless arrest must be delivered to the proper judicial authorities within the
periods provided in Article 125 of the Revised Penal Code, otherwise the arresting
officer could be held liable for delay in the delivery of detained persons. Should the
detention be without legal ground, the person arrested can charge the arresting
officer with arbitrary detention. All this is without prejudice to his filing an action
for damages against the arresting officer under Article 32 of the Civil Code. Verily,
petitioners have a surfeit of other remedies which they can avail themselves of,
thereby making the prayer for prohibition and mandamus improper at this time
The application for the issuance of a writ of habeas corpus is not proper since its
purpose is to relieve petitioners from unlawful restraint a matter which remains
speculative up to this very day.

ANISAH IMPAL SANGCA vs. THE CITY PROSECUTOR OF CEBU


CITY
G.R. No. 175864, June 8, 2007
FACTS:
PDEA charged Lovely Impal Adam with violation of RA 9165. The inquest
prosecutor recommended the dismissal of the case but was disapproved by the City
Prosecutor. Consequently, an information charging Adam with violation of Section
5, Article 2 of R.A. No. 9165 was filed before RTC.
On petition for review before the Department of Justice, Secretary Raul M.
Gonzalez found no probable cause to hold Adam liable for the offense charged
The Justice Secretary directed the City Prosecutor of Cebu City to withdraw the
information. PDEA filed a motion for reconsideration but was denied by the Justice
Secretary on December 8, 2006.
On January 4, 2007, petitioner Anisah Impal Sangca filed the instant petition
praying for the issuance of a writ of habeas corpus and the release of Lovely Impal
Adam.
Finding that Adam could not be held liable for the crime charged, Judge Ingles
issued an Order on January 26, 2007 granting the Motion to Withdraw Information
and ordering the release of the accused, unless otherwise held for another valid
ground.

Page | 172

Compilation of Digested Cases for Remedial Law Review II


by: K. M. T. BUELA
ISSUE: Whether or not petition for habeas corpus may be availed of in case of
warrantless arrests where there is pending motion to withdraw information before
the trial court.
HELD:
Yes. A writ of habeas corpus extends to all cases of illegal confinement or detention
in which any person is deprived of his liberty, or in which the rightful custody of
any person is withheld from the person entitled to it. Its essential object and purpose
is to inquire into all manner of involuntary restraint and to relieve a person from it
if such restraint is illegal. The singular function of a petition for habeas corpus is to
protect and secure the basic freedom of physical liberty.

ANITA MANGILA vs. JUDGE HERIBERTO M. PANGILINAN, et al


G.R. No. 160739, July 17, 2013
FACTS:
petitioner Anita Mangila and four others were charged with seven criminal
complaints of syndicated estafa and violations of Republic Act No. 8042 (Migrant
Workers and Overseas Filipino Act of 1995) before the Municipal Trial Court
On the following day, June 17, 2003, Judge Heriberto M. Pangilinan, Presiding
Judge of the MTCC, conducted a preliminary investigation on the complaints. After
examining Miguel Aaron Palayon, one of the complainants, Judge Pangilinan
issued a warrant for the arrest of Mangila and her cohorts without bail. On the next
day, the entire records of the cases, including the warrant of arrest, were transmitted
to the City Prosecutor of Puerto Princesa City for further proceedings and
appropriate action in accordance with the prevailing rules. As a consequence,
Mangila was arrested on June 18, 2003 and detained at the headquarters on Taft
Avenue, Manila of the National Bureau of Investigation (NBI).
Claiming that Judge Pangilinan did not have the authority to conduct the
preliminary investigation; that the preliminary investigation he conducted was not
yet completed when he issued the warrant of arrest; and that the issuance of the
warrant of arrest was without sufficient justification or without a prior finding of
probable cause, Mangila filed in the CA a petition for habeas corpus to obtain her
release from detention. Her petition averred that the remedy of habeas corpus was
available to her because she could no longer file a motion to quash or a motion to
recall the warrant of arrest considering that Judge Pangilinan had already forwarded
the entire records of the case to the City Prosecutor who had no authority to lift or
recall the warrant.

Page | 173

Compilation of Digested Cases for Remedial Law Review II


by: K. M. T. BUELA
ISSUE: Whether or not a petition for habeas corpus may be availed of on the ground
that an accused in a criminal case when remedies such as motion to quash or motion
to recall warrant of arrest is no longer available.
HELD:
The answer is in the negative. Habeas corpus is not in the nature of a writ of error;
nor intended as substitute for the trial courts function. It cannot take the place of
appeal, certiorari or writ of error. The writ cannot be used to investigate and
consider questions of error that might be raised relating to procedure or on the
merits. The inquiry in a habeas corpus proceeding is addressed to the question of
whether the proceedings and the assailed order are, for any reason, null and void.
The writ is not ordinarily granted where the law provides for other remedies in the
regular course, and in the absence of exceptional circumstances. Moreover, habeas
corpus should not be granted in advance of trial. The orderly course of trial must be
pursued and the usual remedies exhausted before resorting to the writ where
exceptional circumstances are extant.
Hence, it cannot be issued where what is being questioned is the propriety of the
issuance of the warrant of arrest and other remedies to inquire into such issue is no
longer available. The function of habeas corpus, where the party who has appealed
to its aid is in custody under process, does not extend beyond an inquiry into the
jurisdiction of the court by which it was issued and the validity of the process upon
its face. It is not a writ of error.
There is no question that when the criminal complaints were lodged against Mangila
and her cohorts on June 16, 2003, Judge Pangilinan, as the Presiding Judge of the
MTCC, was empowered to conduct preliminary investigations involving "all crimes
cognizable by the proper court in their respective territorial jurisdictions." His
authority was expressly provided in Section 2, Rule 112 of the Revised Rules of
Criminal Procedure.
It is relevant to point out at this juncture that the authority of the MTC and MTCC
judges to conduct preliminary investigations was removed only effective on
October 3, 2005 pursuant to A.M. No. 05-8-26-SC.
With Mangilas arrest and ensuing detention being by virtue of the order lawfully
issued by Judge Pangilinan, the writ of habeas corpus was not an appropriate
remedy to relieve her from the restraint on her liberty. This is because the restraint,
being lawful and pursuant to a court process, could not be inquired into through
habeas corpus.

Page | 174

Compilation of Digested Cases for Remedial Law Review II


by: K. M. T. BUELA
G. Change of Name v. Correction/Cancellation of Entries (Rule 103 v. Rule 108)
MA. LOURDES BARRIENTOS ELEOSIDA vs. LOCAL CIVIL
REGISTRAR OF QUEZON CITY, and CARLOS VILLENA BORBON
G.R. No. 130277, May 9, 2002
FACTS:
Petitioner Ma. Lourdes Eleosida filed a petition seeking to correct the following
entries in the birth certificate of her son, Charles Christian: first, the surname
"Borbon" should be changed to "Eleosida;" second, the date of the parents' wedding
should be left blank; and third, the informant's name should be "Ma. Lourdes B.
Eleosida," instead of "Ma. Lourdes E. Borbon." In support of her petition, petitioner
alleged that she gave birth to her son out of wedlock on May 24, 1992; that she and
the boy's father, Carlos Borbon, were never married; and that the child is therefore
illegitimate and should follow the mother's surname. The petition impleaded the
Local Registrar of Quezon City and Carlos Villena Borbon as respondents.
The trial court motu proprio dismissed the petition for lack of merit. It ruled that,
only CLERICAL ERRORS OF A HARMLESS AND INNOCUOUS NATURE
may be the subject of a judicial order (contemplated under Article 412 of the New
Civil Code), authorizing changes or corrections. Thus, Petitioner filed the instant
petition for review.
ISSUE: whether corrections of entries in the certificate of live birth pursuant to
Article 412 of the Civil Code, in relation to Rule 108 of the Rules of Court may be
allowed even if the errors to be corrected are substantial and not merely clerical
errors of a harmless and innocuous nature.
HELD:
The answer is in the affirmative. Rule 108 of the Revised Rules of Court provides
the procedure for cancellation or correction of entries in the civil registry. The
proceedings under said rule may either be summary or adversary in nature. If the
correction sought to be made in the civil register is clerical, then the procedure to
be adopted is summary. If the rectification affects the civil status, citizenship or
nationality of a party, it is deemed substantial, and the procedure to be adopted is
adversary.
In was held in Republic vs. Valencia that even substantial errors in a civil registry
may be corrected and the true facts established under Rule 108 provided the parties
aggrieved by the error avail themselves of the appropriate adversary proceeding.
An appropriate adversary suit or proceeding is one where the trial court has
conducted proceedings where all relevant facts have been fully and properly
developed, where opposing counsel have been given opportunity to demolish the

Page | 175

Compilation of Digested Cases for Remedial Law Review II


by: K. M. T. BUELA
opposite party's case, and where the evidence has been thoroughly weighed and
considered.
The Court further laid down the procedural requirements to make the proceedings
under Rule 108 adversary, thus:
SEC. 3. Parties.When cancellation or correction of an entry in the civil register is
sought, the civil registrar and all persons who have or claim any interest which
would be affected thereby shall be made parties to the proceeding.
SEC. 4. Notice and publication.Upon the filing of the petition, the court shall, by
an order, fix the time and place for the hearing of the same, and cause reasonable
notice thereof to be given to the persons named in the petition. The court shall also
cause the order to be published once in a week for three (3) consecutive weeks in a
newspaper of general circulation in the province.
SEC. 5. Opposition.The civil registrar and any person having or claiming any
interest under the entry whose cancellation or correction is sought may, within
fifteen (15) days from notice, file his opposition thereto.

REPUBLIC OF THE PHILIPPINES vs. CARLITO I. KHO, et al.


G.R. No. 170340, June 29, 2007
FACTS:
Carlito and his siblings filed a verified petition for correction of entries in the civil
registry. The corrections to be made are as follows: (1) with respect to his and his
siblings birth certificate, the deletion of the word married opposite the phrase date
of marriage of parents as his parent were not legally married; (2) with respect to
his birth certificate alone, to change the citizenship of his mother from Chinese
to Filipino and to delete his second name; and (3) with respect to his marriage
certificate, to change the date of marriage from April 27, 1989 to January 21, 2000,
the date appearing in their marriage certificate.
As required, the petition was published for three consecutive weeks4 in Mindanao
Daily Patrol-CARAGA, a newspaper of general circulation, after which it was set
for hearing on August 9, 2001.
The trial court granted the petitions. Trial Courts decision was affirmed by the CA.
Herein petitioner assailed the decision of the Trial Court and the CA. Petitioner
contends that since the changes sought by respondents were substantial in nature,
they could only be granted through an adversarial proceeding in which

Page | 176

Compilation of Digested Cases for Remedial Law Review II


by: K. M. T. BUELA
indispensable parties, such as Marivel and respondents parents, should have been
notified or impleaded.
ISSUE: Whether or not a petition for correction of substantial entries in the birth
and marriage certificates for which corresponding publication was made is
invalidated by failure to implead indispensable parties, i.e. the mother and the wife
respectively.
HELD:
The answer is in the negative. The defect was cured by compliance with Section 4,
Rule 108, which requires notice by publication. The purpose precisely of Section 4,
Rule 108 is to bind the whole world to the subsequent judgment on the petition. The
sweep of the decision would cover even parties who should have been impleaded
under Section 3, Rule 108, but were inadvertently left out.
Verily, a petition for correction is an action in rem, an action against a thing and not
against a person. The decision on the petition binds not only the parties thereto but
the whole world. An in rem proceeding is validated essentially through publication.
Publication is notice to the whole world that the proceeding has for its object to bar
indefinitely all who might be minded to make an objection of any sort against the
right sought to be established. It is the publication of such notice that brings in the
whole world as a party in the case and vests the court with jurisdiction to hear and
decide it.
Parenthetically, it seems highly improbable that Marivel or Carlitos mother were
unaware of the proceedings. First, notices were sent to the residence of Carlito
which He shared with Marivel and their children. As for Carlitos mother, she was
presented as witness during the proceedings.

IN RE: PETITION FOR CHANGE OF NAME AND/OR


CORRECTION/CANCELLATION OF ENTRY IN CIVIL REGISTRY OF
JULIAN LIN CARULASAN WANG
G.R. No. 159966, March 30, 2005
FACTS:
The parents of Julian Lin Carulasan Wang plan to stay in Singapore. Since in
Singapore middle names or the maiden surname of the mother are not carried in a
persons name, they anticipate that Julian Lin Carulasan Wang will be discriminated
against because of his current registered name which carries a middle name. Thus,
Petitioner Julian Lin Carulasan Wang, a minor, represented by his mother Anna
Lisa Wang, filed a petition for change of name and/or correction/cancellation of
entry in the Civil Registry of Julian Lin Carulasan Wang. Petitioner sought to drop

Page | 177

Compilation of Digested Cases for Remedial Law Review II


by: K. M. T. BUELA
his middle name and have his registered name changed from Julian Lin Carulasan
Wang to Julian Lin Wang.
The trial court found that the reason given for the change of name sought in the
petitionthat is, that petitioner Julian may be discriminated against when studies
in Singapore because of his middle namedid not fall within the grounds
recognized by law. The trial court ruled that the change sought is merely for the
convenience of the child. Thus, trial court denied the petition.
ISSUE: Whether or not middle name may be dropped on the ground of
convenience.
HELD:
The answer is in the negative. Firstly, the law does not allow one to drop the middle
name from his registered name.
Middle names serve to identify the maternal lineage or filiation of a person as well
as further distinguish him from others who may have the same given name and
surname as he has. The Family Code gives legitimate children the right to bear the
surnames of the father and the mother, while illegitimate children shall use the
surname of their mother, unless their father recognizes their filiation, in which case
they may bear the fathers surname.
Secondly, petitioners reason of convenience for the change of his name could not
warrant favorable action on his petition. Before a person can be authorized to
change his name given him either in his certificate of birth or civil registry, he
must show proper or reasonable cause, or any compelling reason which may
justify such change. Otherwise, the request should be denied.
Among the grounds for change of name which have been held valid are: (a) when
the name is ridiculous, dishonorable or extremely difficult to write or pronounce;
(b) when the change results as a legal consequence, as in legitimation; (c) when the
change will avoid confusion; (d) when one has continuously used and been known
since childhood by a Filipino name, and was unaware of alien parentage; (e) a
sincere desire to adopt a Filipino name to erase signs of former alienage, all in good
faith and without prejudicing anybody; and (f) when the surname causes
embarrassment and there is no showing that the desired change of name was for a
fraudulent purpose or that the change of name would prejudice public interest.
In the case at bar, the only reason advanced by petitioner for the dropping his middle
name is convenience. However, how such change of name would make his
integration into Singaporean society easier and convenient is not clearly
established. That the continued use of his middle name would cause confusion and
Page | 178

Compilation of Digested Cases for Remedial Law Review II


by: K. M. T. BUELA
difficulty does not constitute proper and reasonable cause to drop it from his
registered complete name.

MA. CRISTINA TORRES BRAZA, et al. vs. THE CITY CIVIL


REGISTRAR OF HIMAMAYLAN CITY, NEGROS OCCIDENTAL, et al.
G.R. No. 181174, December 4, 2009
FACTS:
Petitioner Ma. Cristina and Pablo Sicad Braza, Jr. (Pablo) were married with three
(3) children. When Pablo died, Lucille Titular began introducing her co-respondent
minor Patrick Alvin Titular Braza Patrick as her and Pablo's son. Ma. Cristina
thereupon made inquiries in the course of which she obtained Patrick's birth
certificate which states that Patrick was the legitimated son of Pablo by reason of
his marriage to Lucille Titular subsequent to the birth of Patrick.
Contending that Patrick could not have been legitimated by the supposed marriage
between Lucille and Pablo, said marriage being bigamous on account of the valid
and subsisting marriage between Ma. Cristina and Pablo, petitioners prayed for (1)
the correction of the entries in Patrick's birth record with respect to his legitimation,
the name of the father and his acknowledgment, and the use of the last name
"Braza"; 2) a directive to Leon, Cecilia and Lucille, all surnamed Titular, as
guardians of the minor Patrick, to submit Parick to DNA testing to determine his
paternity and filiation; and 3) the declaration of nullity of the legitimation of Patrick
as stated in his birth certificate and, for this purpose, the declaration of the marriage
of Lucille and Pablo as bigamous.
ISSUE: Whether or not a petition for correction of birth entries may be made
praying the declaration of illegitimacy of filiation and of nullity of marriage of the
parents of the person whose birth certificate is sought to be corrected.
HELD:
The answer is in the negative. In a special proceeding for correction of entry under
Rule 108 (Cancellation or Correction of Entries in the Original Registry), the trial
court has no jurisdiction to nullify marriages and rule on legitimacy and filiation.
Rule 108 of the Rules of Court vis a vis Article 412 of the Civil Code charts the
procedure by which an entry in the civil registry may be cancelled or corrected. The
proceeding contemplated therein may generally be used only to correct clerical,
spelling, typographical and other innocuous errors in the civil registry. A clerical
error is one which is visible to the eyes or obvious to the understanding; an error
made by a clerk or a transcriber; a mistake in copying or writing, or a harmless
change such as a correction of name that is clearly misspelled or of a misstatement
Page | 179

Compilation of Digested Cases for Remedial Law Review II


by: K. M. T. BUELA
of the occupation of the parent. Substantial or contentious alterations may be
allowed only in adversarial proceedings, in which all interested parties are
impleaded and due process is properly observed.
The allegations of the petition filed before the trial court clearly show that
petitioners seek to nullify the marriage between Pablo and Lucille on the ground
that it is bigamous and impugn Patricks filiation in connection with which they ask
the court to order Patrick to be subjected to a DNA test. These causes of action are
governed not by Rule 108 but by A.M. No. 02-11-10-SC which took effect on
March 15, 2003, and Art. 171 of the Family Code, respectively, hence, the petition
should be filed in a Family Court as expressly provided in said Code. It is well to
emphasize that, doctrinally, validity of marriages as well as legitimacy and
filiation can be questioned only in a direct action seasonably filed by the proper
party, and not through collateral attack such as the petition filed before the
court a quo.

ROMMEL JACINTO DANTES SILVERIO vs. REPUBLIC OF THE


PHILIPPINES
G.R. No. 174689, October 22, 2007
FACTS:
Rommel Jacinto Dantes Silverio underwent sex change from male to female by way
of surgery. Thereafter, he filed a petition for the change of his first name and sex in
his birth certificate.
ISSUE: Whether or not a person may successfully petition for a change of name
and sex appearing in the birth certificate to reflect the result of a sex reassignment
surgery.
HELD:
A Persons First Name Cannot Be Changed On the Ground of Sex Reassignment
RA 9048 now governs the change of first name. RA 9048 likewise provides the
grounds for which change of first name may be allowed: (1) The petitioner finds
the first name or nickname to be ridiculous, tainted with dishonor or extremely
difficult to write or pronounce; (2) The new first name or nickname has been
habitually and continuously used by the petitioner and he has been publicly known
by that first name or nickname in the community; or (3) The change will avoid
confusion.
Petitioners basis in praying for the change of his first name was his sex
reassignment. He intended to make his first name compatible with the sex he
Page | 180

Compilation of Digested Cases for Remedial Law Review II


by: K. M. T. BUELA
thought he transformed himself into through surgery. However, a change of name
does not alter ones legal capacity or civil status. RA 9048 does not sanction a
change of first name on the ground of sex reassignment. Rather than avoiding
confusion, changing petitioners first name for his declared purpose may only create
grave complications in the civil registry and the public interest.
Before a person can legally change his given name, he must present proper or
reasonable cause or any compelling reason justifying such change.19 In addition,
he must show that he will be prejudiced by the use of his true and official name.20
In this case, he failed to show, or even allege, any prejudice that he might suffer as
a result of using his true and official name.
No Law Allows The Change of Entry In The Birth Certificate As To Sex On the
Ground of Sex Reassignment
Under RA 9048, a correction in the civil registry involving the change of sex is not
a mere clerical or typographical error. It is a substantial change for which the
applicable procedure is Rule 108 of the Rules of Court.
The entries envisaged in Article 412 of the Civil Code and correctable under Rule
108 of the Rules of Court are those provided in Articles 4072 and 4083 of the Civil
Code.
The acts, events or factual errors contemplated under Article 407 of the Civil Code
include even those that occur after birth. However, no reasonable interpretation of
the provision can justify the conclusion that it covers the correction on the ground
of sex reassignment.
To correct simply means "to make or set aright; to remove the faults or error from"
while to change means "to replace something with something else of the same kind
or with something that serves as a substitute." The birth certificate of petitioner
contained no error. All entries therein, including those corresponding to his first
name and sex, were all correct. No correction is necessary.

ART. 407. Acts, events and judicial decrees concerning the civil status of persons shall be recorded in the civil
register.
3

ART. 408. The following shall be entered in the civil register: (1) Births; (2) marriages; (3) deaths; (4) legal
separations; (5) annulments of marriage; (6) judgments declaring marriages void from the beginning; (7)
legitimations; (8) adoptions; (9) acknowledgments of natural children; (10) naturalization; (11) loss, or (12) recovery
of citizenship; (13) civil interdiction; (14) judicial determination of filiation; (15) voluntary emancipation of a minor;
and (16) changes of name.
Page | 181

Compilation of Digested Cases for Remedial Law Review II


by: K. M. T. BUELA
REPUBLIC OF THE PHILIPPINES vs. JENNIFER B. CAGANDAHAN
G.R. No. 166676, September 12, 2008
FACTS:
Jennifer Cagandahan filed a Petition for Correction of Entries in Birth Certificate
In her petition, she alleged that she was born on January 13, 1981 and was registered
as a female in the Certificate of Live Birth but while growing up, she developed
secondary male characteristics and was diagnosed to have Congenital Adrenal
Hyperplasia (CAH) which is a condition where persons thus afflicted possess both
male and female characteristics. She further alleged that she was diagnosed to have
clitoral hyperthropy in her early years and at age six, underwent an ultrasound
where it was discovered that she has small ovaries. At age thirteen, tests revealed
that her ovarian structures had minimized, she has stopped growing and she has no
breast or menstrual development. She then alleged that for all interests and
appearances as well as in mind and emotion, she has become a male person. Thus,
she prayed that her birth certificate be corrected such that her gender be changed
from female to male and her first name be changed from Jennifer to Jeff.
ISSUE:
(1) Whether or not correction of entry under rule 108 does not allows change of
"sex" or "gender" in the birth certificate on the ground of the petitioners
inborn medical condition.
(2) Whether or not change of name may be allowed without complying with Rule
103 when such change is merely incidental to the substantial correction as to
the gender following Rule 108.
HELD:
(1)
Under Rep. Act No. 9048, a correction in the civil registry involving the change of
sex is not a mere clerical or typographical error. It is a substantial change for which
the applicable procedure is Rule 108 of the Rules of Court.
(2)
As a rule the change of name is a matter of judicial discretion under Rule 103.
However, the consequence that respondents change of name merely recognizes his
preferred gender, we find merit in respondents change of name. Such a change will
conform with the change of the entry in his birth certificate from female to male.
The trial courts grant of respondents change of name from Jennifer to Jeff implies
a change of a feminine name to a masculine name. Thus, the change of name to
conform to the proper sex without filing a separate petition under Rule 103 is
proper.

Page | 182

Compilation of Digested Cases for Remedial Law Review II


by: K. M. T. BUELA
REPUBLIC OF THE PHILIPPINES vs. DR. NORMA S. LUGSANAY UY
G.R. No. 198010, August 12, 2013
FACTS:
respondent filed a Petition for Correction of Entry in her Certificate of Live Birth.
Her petition seeks to change her name from Anita Sy to Norma S. Lugsanay
following her mothers surname as her parents where never married. She also
contended that she is a Filipino citizen and not Chinese, and all her siblings bear the
surname Lugsanay and are all Filipinos.
In this case, it was only the Local Civil Registrar of Gingoog City who was
impleaded as respondent in the petition below. This, notwithstanding, the RTC
granted her petition and allowed the correction sought by respondent, which
decision was affirmed in toto by the CA. The CA held that respondents failure to
implead other indispensable parties was cured upon the publication of the Order
setting the case for hearing
ISSUE: Whether or not failure to implead indispensable parties aside from the local
civil registrar in petition for correction of entries in the birth certificate is cured by
the publication of the order setting the case for hearing.
HELD:
The answer is in the negative. While there may be cases where the Court held
that the failure to implead and notify the affected or interested parties may be
cured by the publication of the notice of hearing, earnest efforts were made by
petitioners in bringing to court all possible interested parties. Such failure was
likewise excused where the interested parties themselves initiated the corrections
proceedings; when there is no actual or presumptive awareness of the existence of
the interested parties; or when a party is inadvertently left out.
It is clear from the foregoing discussion that when a petition for cancellation or
correction of an entry in the civil register involves substantial and controversial
alterations, including those on citizenship, legitimacy of paternity or filiation, or
legitimacy of marriage, a strict compliance with the requirements of Rule 108 of
the Rules of Court is mandated. If the entries in the civil register could be corrected
or changed through mere summary proceedings and not through appropriate action
wherein all parties who may be affected by the entries are notified or represented,
the door to fraud or other mischief would be set open, the consequence of which
might be detrimental and far reaching.

Page | 183

Compilation of Digested Cases for Remedial Law Review II


by: K. M. T. BUELA
MINORU FUJIKI vs. MARIA PAZ GALELA MARINAY, et al.
G.R. No. 196049, June 26, 2013
FACTS:
Marinay contracted two marriages, first with Petitioner Fujiki and second with
Maekara. The first marriage ended without being legally annulled due to Fujikis
parents who does not favor the marriage. The second marriage ended due to an
alleged physical abuse committed by Maekara against Marinay.
Fujiki and Marinay met again in Japan and reestablished their relationship. In 2010,
Fujiki helped Marinay obtain a judgment from a family court in Japan which
declared the marriage between Marinay and Maekara void on the ground of bigamy.
On 14 January 2011, Fujiki filed a petition in the RTC entitled: "Judicial
Recognition of Foreign Judgment (or Decree of Absolute Nullity of Marriage)."
Fujiki prayed that (1) the Japanese Family Court judgment be recognized; (2) that
the bigamous marriage between Marinay and Maekara be declared void ab initio
under Articles 35(4) and 41 of the Family Code of the Philippines; and (3) for the
RTC to direct the Local Civil Registrar of Quezon City to annotate the Japanese
Family Court judgment on the Certificate of Marriage between Marinay and
Maekara and to endorse such annotation to the Office of the Administrator and Civil
Registrar General in the National Statistics Office (NSO).
The RTC motu proprio dismissed of the petition on the following grounds: (1) that
a special proceeding for correction of entry under Rule 108 (Cancellation or
Correction of Entries in the Original Registry) may not be done to recognize a
foreign judgment which is effect collaterally attacks the validity of or to nullify
marriages; and (2) that under A.M. No. 02-11-10-SC, a petition for declaration of
absolute nullity of void marriage may be filed solely by the husband or the wife.
ISSUE:
(1) Whether the Regional Trial Court can recognize the foreign judgment in a
proceeding for cancellation or correction of entries in the Civil Registry under
Rule 108 of the Rules of Court.
(2) Whether a husband or wife of a prior marriage can file a petition to recognize a
foreign judgment nullifying the subsequent marriage between his or her spouse
and a foreign citizen on the ground of bigamy.
HELD:
(1)
The answer is in the affirmative. A recognition of a foreign judgment only requires
proof of fact of the judgment, it may be made in a special proceeding for
cancellation or correction of entries in the civil registry under Rule 108 of the Rules
of Court. Rule 1, Section 3 of the Rules of Court provides that "a special proceeding
Page | 184

Compilation of Digested Cases for Remedial Law Review II


by: K. M. T. BUELA
is a remedy by which a party seeks to establish a status, a right, or a particular fact."
Rule 108 creates a remedy to rectify facts of a persons life which are recorded by
the State pursuant to the Civil Register Law or Act No. 3753. These are facts of
public consequence such as birth, death or marriage, which the State has an interest
in recording. In Corpuz v. Sto. Tomas this Court declared that "the recognition of
the foreign divorce decree may be made in a Rule 108 proceeding itself, as the
object of special proceedings (such as that in Rule 108 of the Rules of Court) is
precisely to establish the status or right of a party or a particular fact."
While it was repeatedly held that a petition for correction or cancellation of an entry
in the civil registry cannot substitute for a direct action to invalidate a marriage
under the Family Code, A.M. No. 02-11-10-SC and other related laws, this does not
apply in a petition for correction or cancellation of a civil registry entry based on
the recognition of a foreign judgment annulling a marriage where one of the parties
is a citizen of the foreign country. There is neither circumvention of the substantive
and procedural safeguards of marriage under Philippine law, nor of the jurisdiction
of Family Courts under R.A. No. 8369. A recognition of a foreign judgment is
not an action to nullify a marriage. It is an action for Philippine courts to
recognize the effectivity of a foreign judgment, which presupposes a case which
was already tried and decided under foreign law.
The procedure in A.M. No. 02-11-10-SC does not apply in a petition to recognize a
foreign judgment annulling a bigamous marriage where one of the parties is a
citizen of the foreign country. Neither can R.A. No. 8369 define the jurisdiction of
the foreign court.
Article 26 of the Family Code confers jurisdiction on Philippine courts to extend
the effect of a foreign divorce decree to a Filipino spouse without undergoing trial
to determine the validity of the dissolution of the marriage. The second paragraph
of Article 26 of the Family Code provides that "where a marriage between a Filipino
citizen and a foreigner is validly celebrated and a divorce is thereafter validly
obtained abroad by the alien spouse capacitating him or her to remarry, the Filipino
spouse shall have capacity to remarry under Philippine law." The second paragraph
of Article 26 of the Family Code only authorizes Philippine courts to adopt the
effects of a foreign divorce decree precisely because the Philippines does not allow
divorce. Philippine courts cannot try the case on the merits because it is tantamount
to trying a case for divorce.
(2)
The answer in the affirmative. Rule 108, Section 1 of the Rules of Court states: Any
person interested in any act, event, order or decree concerning the civil status of
persons which has been recorded in the civil register, may file a verified petition for

Page | 185

Compilation of Digested Cases for Remedial Law Review II


by: K. M. T. BUELA
the cancellation or correction of any entry relating thereto, with the Regional Trial
Court of the province where the corresponding civil registry is located.
Fujiki has the personality to file a petition to recognize the Japanese Family Court
judgment nullifying the marriage between Marinay and Maekara on the ground of
bigamy because the judgment concerns his civil status as married to Marinay. For
the same reason he has the personality to file a petition under Rule 108 to cancel
the entry of marriage between Marinay and Maekara in the civil registry on the basis
of the decree of the Japanese Family Court.
There is no doubt that the prior spouse has a personal and material interest in
maintaining the integrity of the marriage he contracted and the property relations
arising from it. There is also no doubt that he is interested in the cancellation of an
entry of a bigamous marriage in the civil registry, which compromises the public
record of his marriage.

REPUBLIC OF THE PHILIPPINES vs. MERLINDA L. OLAYBAR


G.R. No. 189538, February 10, 2014
FACTS:
Respondent requested from the National Statistics Office (NSO) a Certificate of No
Marriage (CENOMAR) as one of the requirements for her marriage with her
boyfriend of five years. Upon receipt thereof, she discovered that she was already
married to a certain Ye Son Sune, a Korean National, on June 24, 2002, at the Office
of the Municipal Trial Court in Cities (MTCC), Palace of Justice. She denied having
contracted said marriage and claimed that she did not know the alleged husband;
she did not appear before the solemnizing officer; and, that the signature appearing
in the marriage certificate is not hers. She, thus, filed a Petition for Cancellation of
Entries in the Marriage Contract, especially the entries in the wife portion thereof.5
Respondent impleaded the Local Civil Registrar of Cebu City, as well as her alleged
husband, as parties to the case.
During trial, respondent testified on her behalf and explained that she could not
have appeared before Judge Mamerto Califlores, the supposed solemnizing officer,
at the time the marriage was allegedly celebrated, because she was then in Makati
working as a medical distributor in Hansao Pharma. Respondent also presented as
witness a certain Eufrocina Natinga, an employee of MTCC, Branch 1, who
confirmed that the marriage of Ye Son Sune was indeed celebrated in their office,
but claimed that the alleged wife who appeared was definitely not respondent.
Lastly, a document examiner testified that the signature appearing in the marriage
contract was forged.

Page | 186

Compilation of Digested Cases for Remedial Law Review II


by: K. M. T. BUELA
Contrary to petitioners stand, the RTC held that it had jurisdiction to take
cognizance of cases for correction of entries even on substantial errors under Rule
108 of the Rules of Court being the appropriate adversary proceeding required.
Considering that respondents identity was used by an unknown person to contract
marriage with a Korean national, it would not be feasible for respondent to institute
an action for declaration of nullity of marriage since it is not one of the void
marriages under Articles 35 and 36 of the Family Code. Petitioner now comes
before the Court in this Petition for Review on Certiorari under Rule 45.
ISSUE: Whether or not a petition for correction of entry in the civil registrar may
be availed of to correct the marriage status of a person who claimed that she had
never been married.
HELD:
The answer is in the affirmative. While we maintain that Rule 108 cannot be availed
of to determine the validity of marriage, we cannot nullify the proceedings before
the trial court where all the parties had been given the opportunity to contest the
allegations of respondent; the procedures were followed, and all the evidence of the
parties had already been admitted and examined. Respondent indeed sought, not the
nullification of marriage as there was no marriage to speak of, but the correction of
the record of such marriage to reflect the truth as set forth by the evidence.
Otherwise stated, in allowing the correction of the subject certificate of marriage by
cancelling the wife portion thereof, the trial court did not, in any way, declare the
marriage void as there was no marriage to speak of.
Aside from the certificate of marriage, no such evidence was presented to show the
existence of marriage. Rather, respondent showed by overwhelming evidence that
no marriage was entered into and that she was not even aware of such existence.
The testimonial and documentary evidence clearly established that the only
"evidence" of marriage which is the marriage certificate was a forgery.

H. Prerogative Writs
DANIEL MASANGKAY TAPUZ et al. vs. HONORABLE JUDGE ELMO
DEL ROSARIO, et al.
G.R. No. 182484, June 17, 2008
FACTS:
Private respondents spouses Sanson claiming to be owners of 1+ hectare parcel of
land located at Boracay filed complaint for forcible entry on the ground of force,
intimidation and threats and damages with a prayer for the issuance of a writ of
preliminary mandatory injunction against the petitioners. The MCTC rendered a
Page | 187

Compilation of Digested Cases for Remedial Law Review II


by: K. M. T. BUELA
decision in the private respondents' favor. On appeal with RTC, RTC upheld the
decision of MCTC and subsequently issued a writ of permanent mandatory
injunction and order of demolition. Assailing the RTC decision, petitioners filed a
petition for review with the CA. CA denied the petition but petitioners filed an MR.
Pending resolution of the MR, petitioners filed herein petition for certiorari and for
the issuance of the writs of amparo and habeas data
Contrary to the factual findings of the MCTC, the factual allegations of the petition
for the issuance of the writ of amparo provides that the private respondents availed
of the help of armed men and intrude into the property alleged to be owned by
petitioners by firing shotguns and burning their houses.
Petition for a writ of habeas data is prayed for so that the PNP may release the report
on the burning of the homes of the petitioners and the acts of violence allegedly
employed against them by the private respondents.
ISSUE:
(1) Whether or not, a petition for issuance of writ of amparo may be availed of
against an alleged violence or threats committed against a persons life, property
and security arising from property dispute.
(2) Whether or not writ of habeas data may be availed of for purposes of mandating
the authorities to release an information on alleged arson committed in relation
to a property dispute where such information was never sought in the main
action over possession of the property in dispute.
HELD:
(1)
The answer is in the negative. Writ of Amparo is not a writ to protect concerns that
are purely property or commercial. The writ of amparo was originally conceived as
a response to the extraordinary rise in the number of killings and enforced
disappearances, and to the perceived lack of available and effective remedies to
address these extraordinary concerns. It is intended to address violations of or
threats to the rights to life, liberty or security, as an extraordinary and independent
remedy beyond those available under the prevailing Rules, or as a remedy
supplemental to these Rules.
The writ shall issue if the Court is preliminarily satisfied with the prima facie
existence of the ultimate facts determinable from the supporting affidavits that
detail the circumstances of how and to what extent a threat to or violation of the
rights to life, liberty and security of the aggrieved party was or is being committed.
Rather than acts of terrorism that pose a continuing threat to the persons of the
petitioners, the violent incidents alleged appear to us to be purely property-related
Page | 188

Compilation of Digested Cases for Remedial Law Review II


by: K. M. T. BUELA
and focused on the disputed land. Thus, if the petitioners wish to seek redress and
hold the alleged perpetrators criminally accountable, the remedy may lie more in
the realm of ordinary criminal prosecution rather than on the use of the
extraordinary remedy of the writ of amparo.
Where, as in this case, there is an ongoing civil process dealing directly with the
possessory dispute and the reported acts of violence and harassment, we see no
point in separately and directly intervening through a writ of amparo in the absence
of any clear prima facie showing that the right to life, liberty or security - the
personal concern that the writ is intended to protect - is immediately in danger or
threatened, or that the danger or threat is continuing. We see no legal bar, however,
to an application for the issuance of the writ, in a proper case, by motion in a
pending case on appeal or on certiorari, applying by analogy the provisions on the
co-existence of the writ with a separately filed criminal case.
(2)
The necessity or justification for the issuance of the writ, based on the insufficiency
of previous efforts made to secure information, has not also been shown. In sum,
the prayer for the issuance of a writ of habeas data is nothing more than the "fishing
expedition" that this Court - in the course of drafting the Rule on habeas data - had
in mind in defining what the purpose of a writ of habeas data is not. In these lights,
the outright denial of the petition for the issuance of the writ of habeas data is fully
in order.

ARMANDO Q. CANLAS, et al. vs. NAPICO HOMEOWNERS ASSN., I


XIII, INC., et al.
G.R. No. 182795, June 5, 2008
FACTS:
Petitioners are settlers in a certain parcel of land situated in Barangay Manggahan,
Pasig City. Their dwellings/houses have either been demolished as of the time of
filing of the petition, or is about to be demolished pursuant to a judgment of the
Supreme Court.
Petitioners alleging that they were deprived of their liberty, freedom and/or rights
to shelter filed the instant petition for the issuance of writ of amparo. They alleged
that the titles which is the basis of a final judgment ordering the demolition of their
houses is spurious and is issued by fraudulently by unprincipled land officials.
ISSUE: Whether or not a demolition of dwelling pursuant to a final and executory
judgment is a ground for the issuance of writ of amparo on the ground that
petitioners had been deprived of their liberty and/or right to shelter.
Page | 189

Compilation of Digested Cases for Remedial Law Review II


by: K. M. T. BUELA

HELD:
The answer is in the negative. The threatened demolition of a dwelling by virtue of
a final judgment affirmed with finality by the Supreme Court, is not included among
the enumeration of rights as stated in Section 1 of the Rule on the Writ of Amparo
for which the remedy of a writ of amparo is made available. Their claim to their
dwelling, assuming they still have any despite the final and executory judgment
adverse to them, does not constitute right to life, liberty and security. There is,
therefore, no legal basis for the issuance of the writ of amparo.

P/SUPT. FELIXBERTO CASTILLO et al. vs. DR. AMANDA T. CRUZ,


NIXON T. CRUZ, and FERDINAND T. CRUZ
G.R. No. 182165, November 25, 2009
FACTS:
Provincial Government of Bulacan is the winning party in an unlawful detainer case
against Respondent Spouses Cruz. Notwithstanding that the judgment in the said
case had become final and executory, the latter as able to obtain an injunction from
the RTC. RTC further ordered the remand of the case to MTC. Upon remand, MTC
again decided in favor of the Provincial Government of Bulacan and issued a writ
of demolition. Despite execution of the writ of execution, Respondent obtained a
TRO from the RTC.
Invoking the TRO, respondents refused to vacate the property and clashed with
Police Superintendent Felixberto Castillo et al., who were deployed by the City
Mayor to "protect, secure and maintain the possession of the property," entered the
property. Consequently, respondents were arrested.
Respondents later filed a "Respectful Motion-Petition for Writ of Amparo and
Habeas Data and averred that despite the TRO petitioners unlawfully entered the
property with the use of heavy equipment, tore down the barbed wire fences and
tents, and arrested them when they resisted petitioners entry
RTC granted the petition. Hence, the present petition for review on certiorari,
pursuant to Section 1910 of The Rule on the Writ of Amparo (A.M. No. 07-9-12SC),11 which is essentially reproduced in the Rule on the Writ of Habeas Data
(A.M. No. 08-1-16-SC).12
ISSUE: Whether or not a writ of amparo may be issued against those who allegedly
entered the petitioners property unlawfully in violation of a previously issued TRO.
HELD:
Page | 190

Compilation of Digested Cases for Remedial Law Review II


by: K. M. T. BUELA
The answer is in the negative. In the case of Tapuz v. Del Rosario, it was held that
a wirt of amparo is not a writ to protect concerns that are purely property or
commercial.
To thus be covered by the privilege of the writs, respondents must meet the
threshold requirement that their right tolife, liberty and security is violated or
threatened with an unlawful act or omission. Evidently, the present controversy
arose out of a property dispute between the Provincial Government and
respondents. Absent any considerable nexus between the acts complained of and its
effect on respondents right to life, liberty and security, the Court will not delve on
the propriety of petitioners entry into the property.
It bears emphasis that respondents petition did not show any actual violation,
imminent or continuing threat to their life, liberty and security. Bare allegations that
petitioners "in unison, conspiracy and in contempt of court, there and then willfully,
forcibly and feloniously with the use of force and intimidation entered and forcibly,
physically manhandled the petitioners (respondents) and arrested the herein
petitioners (respondents)" will not suffice to prove entitlement to the remedy of the
writ of amparo. No undue confinement or detention was present. In fact,
respondents were even able to post bail for the offenses a day after their arrest.

GEN. AVELINO I. RAZON vs. MARY JEAN B. TAGITIS


G.R. No. 182498, February 16, 2010
FACTS:
On December 3, 2009, the Supreme Court rendered confirming the enforced
disappearance of Engineer Morced N. Tagitis (Tagitis) and granting the Writ of
Amparo. The decision was based, among other considerations, on the finding that
Col. Julasirim Ahadin Kasim (Col. Kasim) informed the respondent Mary Jean
Tagitis (respondent) and her friends that her husband had been under surveillance
since January 2007 because an informant notified the authorities, through a letter,
that Tagitis was a liaison for the JI;5 that he was "in good hands" and under custodial
investigation for complicity with the JI after he was seen talking to one Omar Patik
and a certain "Santos" of Bulacan, a "Balik Islam" charged with terrorism (Kasim
evidence).
The Supreme Court considered Col. Kasims information, together with the
consistent denials by government authorities of any complicity in the disappearance
of Tagitis, the dismissive approach of the police authorities to the report of the
disappearance, as well as the haphazard investigations conducted that did not
translate into any meaningful results, to be indicative of government complicity in
the disappearance of Tagitis.
Page | 191

Compilation of Digested Cases for Remedial Law Review II


by: K. M. T. BUELA

Herein petitioners filed a motion for reconsideration on the ground that there was
no sufficient evidence to conclude that Col. Kasims disclosure unequivocally
points to some government complicity in the disappearance of Tagitis. Specifically,
the petitioners contend that Supreme Court erred in unduly relying on the raw
information given to Col. Kasim by a personal intelligence "asset" without any other
evidence to support it.
ISSUE: Whether or not the grant of Writ of Amparo on the basis of a hearsay
evidence is valid.
HELD:
Yes. although the Kasim evidence was patently hearsay (and was thus incompetent
and inadmissible under our rules of evidence), the unique evidentiary difficulties
posed by enforced disappearance cases compel us to adopt standards that were
appropriate and responsive to the evidentiary difficulties faced. We noted that while
we must follow the substantial evidence rule, we must also observe flexibility in
considering the evidence that we shall take into account. Thus, we introduced a new
evidentiary standard for Writ of Amparo cases in this wise:
The fair and proper rule, to our mind, is to consider all the pieces of evidence
adduced in their totality, and to consider any evidence otherwise inadmissible
under our usual rules to be admissible if it is consistent with the admissible
evidence adduced. In other words, we reduce our rules to the most basic test
of reason i.e., to the relevance of the evidence to the issue at hand and its
consistency with all the other pieces of adduced evidence, Thus, even hearsay
evidence can be admitted if it satisfies this minimum test.

IN THE MATTER OF THE PETITION FOR THE WRIT OF AMPARO


AND THE WRIT OF HABEAS DATA IN FAVOR OF MELISSA C.
ROXAS, MELISSA C. ROXAS vs. GLORIA MACAPAGAL-ARROYO et
al.
G.R. No. 189155, September 7, 2010
FACTS:
Petitioner, an American citizen of Filipino descent, filed a Petition for the Writs of
Amparo and Habeas Data before the Supreme Court. She alleged that while doing
a volunteer work in La Paz, Tarlacm she and her companions were abducted and
tortured for five day. Petitioner impleaded public officials occupying the uppermost
echelons of the military and police hierarchy as respondents, on the belief that it
was government agents who were behind her abduction and torture.

Page | 192

Compilation of Digested Cases for Remedial Law Review II


by: K. M. T. BUELA
In a Resolution dated 9 June 2009, this Court issued the desired writs and referred
the case to the Court of Appeals for hearing, reception of evidence and appropriate
action. CA granted the petition for issuance of writ of habeas data. However, the
Court of Appeals was not convinced that the military or any other person acting
under the acquiescence of the government, were responsible for the abduction and
torture of the petitioner.
Petitioner contested the CA decision and filed a petitioner for review on certiorari.
Petitioner argues that (1) the manner by which her abduction and torture was carried
out, as well as the sounds of construction, gun-fire and airplanes that she heard
while in detention, as these were detailed in her two affidavits and affirmed by her
in open court, are already sufficient evidence to prove government involvement;
and (2) petitioner invokes the doctrine of command responsibility to implicate the
high-ranking civilian and military authorities she impleaded as respondents in her
amparo petition.
ISSUE: Whether or not a government officials may be impleaded as respondent in
a writ of amparo on the ground of command responsibility.
HELD:
The answer is in the negative. The use by the petitioner of the doctrine of command
responsibility is legally inaccurate, if not incorrect. The doctrine of command
responsibility is a rule of substantive law that establishes liability and, by this
account, cannot be a proper legal basis to implead a party-respondent in an amparo
petition
According to Fr. Bernas, "command responsibility," in its simplest terms, means
the "responsibility of commanders for crimes committed by subordinate members
of the armed forces or other persons subject to their control in international wars or
domestic conflict."
Since the application of command responsibility presupposes an imputation of
individual liability, it is more aptly invoked in a full-blown criminal or
administrative case rather than in a summary amparo proceeding. The obvious
reason lies in the nature of the writ itself.
The writ of amparo is a protective remedy aimed at providing judicial relief
consisting of the appropriate remedial measures and directives that may be crafted
by the court, in order to address specific violations or threats of violation of the
constitutional rights to life, liberty or security. While the principal objective of its
proceedings is the initial determination of whether an enforced disappearance,
extralegal killing or threats thereof had transpiredthe writ does not, by so doing,

Page | 193

Compilation of Digested Cases for Remedial Law Review II


by: K. M. T. BUELA
fix liability for such disappearance, killing or threats, whether that may be criminal,
civil or administrative under the applicable substantive law.
The remedy provides rapid judicial relief as it partakes of a summary proceeding
that requires only substantial evidence to make the appropriate reliefs available to
the petitioner; it is not an action to determine criminal guilt requiring proof beyond
reasonable doubt, or liability for damages requiring preponderance of evidence, or
administrative responsibility requiring substantial evidence that will require full and
exhaustive proceedings.
It must be clarified, however, that the inapplicability of the doctrine of command
responsibility in an amparo proceeding does not, by any measure, preclude
impleading military or police commanders on the ground that the complained acts
in the petition were committed with their direct or indirect acquiescence. In which
case, commanders may be impleadednot actually on the basis of command
responsibilitybut rather on the ground of their responsibility, or at least
accountability.
Responsibility refers to the extent the actors have been established by substantial
evidence to have participated in whatever way, by action or omission, in an enforced
disappearance, as a measure of the remedies this Court shall craft, among them, the
directive to file the appropriate criminal and civil cases against the responsible
parties in the proper courts. Accountability, on the other hand, refers to the measure
of remedies that should be addressed to those who exhibited involvement in the
enforced disappearance without bringing the level of their complicity to the level of
responsibility defined above; or who are imputed with knowledge relating to the
enforced disappearance and who carry the burden of disclosure; or those who carry,
but have failed to discharge, the burden of extraordinary diligence in the
investigation of the enforced disappearance.

EDITA T. BURGOS vs. GEN. HERMOGENES ESPERON et al.


G.R. No. 178497, February 4, 2014
FACTS:
This case relates the proceedings on the enforced disappearance of Jonas Burgos.
A petition for the issuance of writ of habeas corpus and writ of amparo was filed
before the SC.
SC, pursuant to CHRs initial report, issued a writ of habeas corpus but hold in
abeyance the ruling on the merits of Amparo and referred the same to CA to allow
Lt. Baliaga, Jr. to comment on the CHR Report. CA denied petition for issuance of
writ of habeas corpus (The CA held that the issue in the petition for habeas corpus
Page | 194

Compilation of Digested Cases for Remedial Law Review II


by: K. M. T. BUELA
is not the illegal confinement or detention of Jonas, but his enforced disappearance)
and granted the issuance of writ of amparo. The CA found that the totality of the
evidence supports the petitioners allegation that the military was involved in the
enforced disappearance of Jonas. Thus, the CA held that Lt. Baliaga was responsible
and the AFP and the PNP were accountable for the enforced disappearance of Jonas.
CA directed PNP and AFT to conduct an exhaustive investigation of the enforced
disappearance of Jonas Burgos. CHR was also directed to continue with its own
independent investigation on the enforced disappearance of Jonas Burgos with the
same degree of diligence required under the Rule on the Writ of Amparo.
On April 1, 2013, the petitioner filed an Ex Parte Motion Ex Abundanti Cautela
asking the Court among others to: xxx (2) issue a writ of Amparo on the basis of
the newly discovered evidence (the sealed attachment to the motion); and (3) refer
the cases to the CA for further hearing on the newly discovered evidence.
The petitioner alleged that she received from a source (who requested to remain
anonymous) documentary evidence proving that an intelligence unit of the 7th
Infantry Division of the Philippine Army and 56th Infantry Battalion, operating
together, captured Jonas on April 28, 2007 at Ever Gotesco Mall, Commonwealth
Avenue, Quezon City.
ISSUE: Whether or not after the grant of petition for issuance of writ of amparo, a
new writ of amparo may be issued on the basis of newly discovered evidence
HELD:
No. The SC resolved to deny the motion for re-issuance of the writ of amparo and
to refer the case to the CA based on newly discovered evidence.
It should be emphasized that while the Rule on the Writ of Amparo accords the
Court a wide latitude in crafting remedies to address an enforced disappearance, it
cannot (without violating the nature of the writ of Amparo as a summary remedy
that provides rapid judicial relief) grant remedies that would complicate and prolong
rather than expedite the investigations already ongoing. Note that the CA has
already determined with finality that Jonas was a victim of enforced disappearance.
The ROLE of SC in a writ of Amparo proceeding is merely to determine whether
an enforced disappearance has taken place; to determine who is responsible or
accountable; and to define and impose the appropriate remedies to address the
disappearance.
As we held in Razon, Jr. v. Tagitis, the writ merely embodies the Courts directives
to police agencies to undertake specified courses of action to address the enforced
Page | 195

Compilation of Digested Cases for Remedial Law Review II


by: K. M. T. BUELA
disappearance of an individual. The Writ of Amparo serves both a preventive and a
curative role. It is curative as it facilitates the subsequent punishment of perpetrators
through the investigation and remedial action that it directs. The focus is on
procedural curative remedies rather than on the tracking of a specific criminal
or the resolution of administrative liabilities.
In this case, the beneficial purpose of the Writ of Amparo has been served with the
CAs final determination of the persons responsible and accountable for the
enforced disappearance of Jonas and the commencement of criminal action against
Lt. Baliaga. At this stage, criminal, investigation and prosecution proceedings are
already beyond the reach of the Writ of Amparo proceeding now before us.

MARICRIS D. DOLOT vs. HON. RAMON PAJE (DENR)


G.R. No. 199199, August 27, 2013
FACTS:
On September 15, 2011, petitioners filed a petition for continuing mandamus,
damages and attorneys fees with the RTC of Sorsogon. On September 16, 2011,
the case was summarily dismissed for lack of jurisdiction. RTC relied on SC
Administrative Order (A.O.) No. 7 defining the territorial areas of the Regional
Trial Courts in Regions 1 to 12, and Administrative Circular (Admin. Circular) No.
23-2008, designating the environmental courts "to try and decide violations of
environmental laws x x x committed within their respective territorial jurisdictions."
The petitioners filed a motion for reconsideration but it was denied. RTC further
ruled that: (1) there was no final court decree, order or decision yet that the public
officials allegedly failed to act on, which is a condition for the issuance of the writ
of continuing mandamus; (2) the case was prematurely filed as the petitioners
therein failed to exhaust their administrative remedies; and (3) they also failed to
attach judicial affidavits and furnish a copy of the complaint to the government or
appropriate agency, as required by the rules. Petitioner Dolot went straight to this
Court on pure questions of law.
ISSUE:
(1) Whether or not RTC may motu proprio dismiss a petition for continuing
mandamus on the ground of lack of jurisdiction following A.O. No. 7 defining
the territorial areas of the Regional Trial Courts in Regions 1 to 12.
(2) Whether or not a final judgment finding that the public officials failed to act on
the contemplated environmental issue is a condition precedent for the issuance
of the writ of continuing mandamus.
(3) Whether or not a petition for issuance of continuing mandamus requires the
attachment of judicial affidavits.
Page | 196

Compilation of Digested Cases for Remedial Law Review II


by: K. M. T. BUELA

HELD:
(1)
Such reasoning is plainly erroneous. The RTC cannot solely rely on SC A.O. No. 7
and Admin. Circular No. 23-2008 and confine itself within its four corners in
determining whether it had jurisdiction over the action filed by the petitioners.
These administrative order issued by the Court merely provide for the venue where
an action may be filed. The Court does not have the power to confer jurisdiction on
any court or tribunal as the allocation of jurisdiction is lodged solely in Congress.
It also cannot be delegated to another office or agency of the Government.
Venue relates only to the place of trial or the geographical location in which an
action or proceeding should be brought and does not equate to the jurisdiction of
the court. It is intended to accord convenience to the parties, as it relates to the place
of trial, and does not restrict their access to the courts. Consequently, the RTCs
motu proprio dismissal of Civil Case No. 2011-8338 on the ground of lack of
jurisdiction is patently incorrect. At most, the error committed by the petitioners in
filing the case with the RTC of Sorsogon was that of improper venue.
Similarly, it would serve the higher interest of justice if the Court orders the transfer
of Civil Case No. 2011 8338 to the RTC of Irosin for proper and speedy resolution,
with the RTC applying the Rules in its disposition of the case.
(2)
The RTCs mistaken notion on the need for a final judgment, decree or order is
apparently based on the definition of the writ of continuing mandamus under
Section 4(c), Rule 1 of the Rules, to wit: Continuing mandamus is a writ issued by
a court in an environmental case directing any agency or instrumentality of the
government or officer thereof to perform an act or series of acts decreed by final
judgment which shall remain effective until judgment is fully satisfied.
The final court decree, order or decision erroneously alluded to by the RTC actually
pertains to the judgment or decree that a court would eventually render in an
environmental case for continuing mandamus and which judgment or decree shall
subsequently become final.
Under the Rules, after the court has rendered a judgment in conformity with Rule
8, Section 7 and such judgment has become final, the issuing court still retains
jurisdiction over the case to ensure that the government agency concerned is
performing its tasks as mandated by law and to monitor the effective performance
of said tasks. It is only upon full satisfaction of the final judgment, order or decision
that a final return of the writ shall be made to the court and if the court finds that
Page | 197

Compilation of Digested Cases for Remedial Law Review II


by: K. M. T. BUELA
the judgment has been fully implemented, the satisfaction of judgment shall be
entered in the court docket. A writ of continuing mandamus is, in essence, a
command of continuing compliance with a final judgment as it "permits the court
to retain jurisdiction after judgment in order to ensure the successful
implementation of the reliefs mandated under the courts decision."
(3)
RTC erred in ruling that the petition is infirm for failure to attach judicial affidavits.
Rule 8 requires that the petition should be verified, contain supporting evidence and
must be accompanied by a sworn certification of non-forum shopping. There is
nothing in Rule 8 that compels the inclusion of judicial affidavits, albeit not
prohibited. It is only if the evidence of the petitioner would consist of testimony of
witnesses that it would be the time that judicial affidavits (affidavits of witnesses in
the question and answer form) must be attached to the petition/complaint.

Page | 198

Compilation of Digested Cases for Remedial Law Review II


by: K. M. T. BUELA
EVIDENCE
A. Preliminary Consideration
ONG CHIA vs. REPUBLIC OF THE PHILIPPINES and THE COURT OF
APPEALS
G.R. No. 127240, March 27, 2000
FACTS:
Petitioner was born in Amoy, China. In 1932, as a nine-year old boy, he arrived at
the port of Manila on board the vessel "Angking." Since then, he has stayed in the
Philippines where he found employment and eventually started his own business,
married a Filipina, with whom he had four children. On July 4, 1989, at the age of
66, he filed a verified petition to be admitted as a Filipino citizen under C.A. No.
473, otherwise known as the Revised Naturalization Law.
The trial court granted the petition and admitted petitioner to Philippine citizenship.
On appeal, CA reversed the trial court and denied petitioners application for
naturalization. It ruled that due to the importance of naturalization cases, the State
is not precluded from raising questions not presented in the lower court and brought
up for the first time on appeal.
Petitioners principal contention is that the appellate court erred in considering the
documents which had merely been annexed by the State to its appellants brief and,
on the basis of which, justified the reversal of the trial courts decision. Not having
been presented and formally offered as evidence during the trial, they are mere
"scraps of paper devoid of any evidentiary value
ISSUE: Whether or not the rules on evidence applies to a petition for naturalization.
HELD:
The answer is in the negative. Rule 1, Sec. 4 provides that: These rules shall not
apply to land registration, cadastral and election cases, naturalization and
insolvency proceedings, and other cases not herein provided for, except by analogy
or in a suppletory character and whenever practicable and convenient.
Prescinding from the above, the rule on formal offer of evidence (Rule 132, 34)
now being invoked by petitioner is clearly not applicable to the present case
involving a petition for naturalization. The only instance when said rules may be
applied by analogy or suppletorily in such cases is when it is "practicable and
convenient." That is not the case here, since reliance upon the documents presented
by the State for the first time on appeal, in fact, appears to be the more practical and
convenient course of action considering that decisions in naturalization proceedings
Page | 199

Compilation of Digested Cases for Remedial Law Review II


by: K. M. T. BUELA
are not covered by the rule on res judicata. Consequently, a final favorable
judgment does not preclude the State from later on moving for a revocation of the
grant of naturalization on the basis of the same documents.

CECILIA ZULUETA vs. COURT OF APPEALS and ALFREDO MARTIN


G.R. No. 107383, February 20, 1996
FACTS:
Petitioner Cecilia Zulueta, wife of private respondent Alfredo Martin, entered the
clinic of her husband, a doctor of medicine, and in the presence of her mother, a
driver and private respondents secretary, forcibly opened the drawers and cabinet
in her husbands clinic and took 157 documents consisting of private
correspondence between Dr. Martin and his alleged paramours, greetings cards,
cancelled checks, diaries, Dr. Martins passport, and photographs. The documents
and papers were seized for use in evidence in a case for legal separation and for
disqualification from the practice of medicine which petitioner had filed against her
husband.
Dr. Martin brought this action below for recovery of the documents and papers and
for damages against petitioner. After trial, RTC rendered judgment for private
respondent. The writ of preliminary injunction earlier issued was made final and
petitioner. Cecilia Zulueta and her attorneys and representatives were enjoined from
"using or submitting/admitting as evidence" the documents and papers in question.
On appeal, the Court of Appeals affirmed the decision of the Regional Trial Court.
Hence this petition.
ISSUE: whether or not documents and/or correspondence taken by one spouse
without the consent of the owner spouse may be used by former against the latter
as evidence in an action for disqualification to practice a profession.
HELD:
The documents and papers in question are inadmissible in evidence. The
constitutional injunction declaring "the privacy of communication and
correspondence to be inviolable" is no less applicable simply because it is the wife
(who thinks herself aggrieved by her husbands infidelity) who is the party against
whom the constitutional provision is to be enforced. The only exception to the
prohibition in the Constitution is if there is a "lawful order from a court or when
public safety or order requires otherwise, as prescribed by law." Any violation of
this provision renders the evidence obtained inadmissible "for any purpose in any
proceeding."

Page | 200

Compilation of Digested Cases for Remedial Law Review II


by: K. M. T. BUELA
The intimacies between husband and wife do not justify any one of them in breaking
the drawers and cabinets of the other and in ransacking them for any telltale
evidence of marital infidelity. A person, by contracting marriage, does not shed
his/her integrity or his right to privacy as an individual and the constitutional
protection is ever available to him or to her.
The law insures absolute freedom of communication between the spouses by
making it privileged. Neither husband nor wife may testify for or against the other
without the consent of the affected spouse while the marriage subsists. Neither may
be examined without the consent of the other as to any communication received in
confidence by one from the other during the marriage, save for specified exceptions.
But one thing is freedom of communication; quite another is a compulsion for each
one to share what one knows with the other. And this has nothing to do with the
duty of fidelity that each owes to the other.

PEOPLE OF THE PHILIPPINES vs. JOEL YATAR alias "KAWIT


G.R. NO. 150224, May 19, 2004
FACTS:
Appellant Yatar was charged and convicted of Rape with Homicide by the trial
court. The basis of the conviction rest on circumstantial evidence gathered from the
testimony of various witnesses, to: (1) the presence of the accused at the crime scene
within the timeframe of the approximate time of death of the victim; (2) at one point
prior to the commission accused was seen wearing a white shirt with collar; (3)
latter on he was seen wearing a dirty white shirt with collar; (4) when the body of
the victim was found, a dirty white shirt was seen beside her; (5) the dirty white
shirt with collar found at the crime scene was stained by blood; (6) when the blood
stain and accuseds blood was subjected to DNA testing, it was found that it
contained the same DNA; (7) that when semen found inside the victims body was
subjected to DNA testing, it was found to be identical to that of accuseds DNA.
In an attempt to exclude the DNA evidence, the appellant contends that the blood
sample taken from him as well as the DNA tests were conducted in violation of his
right to remain silent as well as his right against self-incrimination under Secs. 12
and 17 of Art. III of the Constitution.
ISSUE: Whether or not, taking of accuseds blood sample and subjecting the same
to DNA testing is inadmissible in evidence as it amounts to violation of his right
against self-incrimination.
HELD:

Page | 201

Compilation of Digested Cases for Remedial Law Review II


by: K. M. T. BUELA
Accuseds contention is untenable. The kernel of the right is not against all
compulsion, but against testimonial compulsion. The right against selfincrimination is simply against the legal process of extracting from the lips of the
accused an admission of guilt. It does not apply where the evidence sought to be
excluded is not an incrimination but as part of object evidence.
It was held in People v. Rondero that although accused-appellant insisted that hair
samples were forcibly taken from him and submitted to the National Bureau of
Investigation for forensic examination, the hair samples may be admitted in
evidence against him, for what is proscribed is the use of testimonial compulsion or
any evidence communicative in nature acquired from the accused under duress.
Hence, a person may be compelled to submit to fingerprinting, photographing,
paraffin, blood and DNA, as there is no testimonial compulsion involved. Under
People v. Gallarde, where immediately after the incident, the police authorities took
pictures of the accused without the presence of counsel, we ruled that there was no
violation of the right against self-incrimination. The accused may be compelled to
submit to a physical examination to determine his involvement in an offense of
which he is accused.
It must also be noted that appellant in this case submitted himself for blood
sampling which was conducted in open court on March 30, 2000, in the presence of
counsel.

NENA LAZALITA* TATING vs. FELICIDAD TATING MARCELLA, et


al.
G.R. NO. 155208, March 27, 2007
FACTS:
On 1969, Daniela sold her property to her granddaughter, herein petitioner Nena
Lazalita Tating. As a consequence, title thereto was transferred in the name of Nena.
She declared the property in her name for tax purposes and paid the real estate taxes
due thereon for the years 1972
1988
Daniela died on July 29, 1988. On 1989, Danielas heirs herein respondents found
a sworn statement executed by Danila stating that she had actually no intention of
selling the property; the true agreement between her and Nena was simply to
transfer title over the subject property in favor of the latter to enable her to obtain a
loan by mortgaging the subject property.
On September 6, 1989, Respondents filed a complaint with the RTC praying for the
nullification of the Deed of Absolute Sale executed by Daniela in her favor,
Page | 202

Compilation of Digested Cases for Remedial Law Review II


by: K. M. T. BUELA
cancellation of the TCT issued in the name of Nena, and issuance of a new title and
tax declaration in favor of the heirs of Daniela.
RTC rendered its judgment in favor of the plaintiffs. CA Affirmed its decision.
Petitioner asserts that the sole evidence which persuaded both the RTC and the CA
in holding that the subject deed was simulated was the Sworn Statement of Daniela
dated December 28, 1977. However, petitioner argues that said Sworn Statement
should have been rejected outright by the lower courts considering that Daniela has
long been dead when the document was offered in evidence, thereby denying
petitioner the right to cross-examine her.
ISSUE: Whether or not a sworn statement/affidavit of a deceased may be given
probative value for purposes of deciding a complaint.
HELD:
No. It is settled that affidavits are classified as hearsay evidence since they are not
generally prepared by the affiant but by another who uses his own language in
writing the affiant's statements, which may thus be either omitted or misunderstood
by the one writing them. Moreover, the adverse party is deprived of the opportunity
to cross-examine the affiant. For this reason, affidavits are generally rejected for
being hearsay, unless the affiants themselves are placed on the witness stand to
testify thereon. The Court finds that both the trial court and the CA committed error
in giving the sworn statement probative weight. Since Daniela is no longer available
to take the witness stand as she is already dead, the RTC and the CA should not
have given probative value on Daniela's sworn statement for purposes of proving
that the contract of sale between her and petitioner was simulated and that, as a
consequence, a trust relationship was created between them.

PEOPLE OF THE PHILIPPINES vs. RODRIGO SALAFRANCA Y


BELLO
G.R. No. 173476, February 22, 2012
FACTS:
Rodrigo Salafranca y Bello was charged and convicted of murder for the fatal
stabbing of Johnny Bolanon. On appeal, his conviction was affirmed by the CA.
The basis of the conviction is the testimony of the victims uncle Rodolfo B. Estao
that on their way to the hospital Bolanon told him that it was Salafranca who had
stabbed him.
ISSUE: Whether or not a statement made to another person by a victim of murder
before he died is admissible as evidence.

Page | 203

Compilation of Digested Cases for Remedial Law Review II


by: K. M. T. BUELA

HELD:
Yes. An ante-mortem declaration of a victim of murder, homicide, or parricide that
meets the conditions of admissibility under the Rules of Court and pertinent
jurisprudence is admissible either as a dying declaration or as a part of the res gestae,
or both.
A dying declaration, although generally inadmissible as evidence due to its hearsay
character, may nonetheless be admitted when the following requisites concur,
namely: (a) that the declaration must concern the cause and surrounding
circumstances of the declarants death; (b) that at the time the declaration is
made, the declarant is under a consciousness of an impending death - There is
ample authority for the view that the declarants belief in the imminence of his death
can be shown by the declarants own statements or from circumstantial evidence,
such as the nature of his wounds, statements made in his presence, or by the opinion
of his physician; (c) that the declarant is competent as a witness; and (d) that
the declaration is offered in a criminal case for homicide, murder, or parricide,
in which the declarant is a victim.
All the requisites were met herein. Bolanon communicated his ante-mortem
statement to Estao, identifying Salafranca as the person who had stabbed him. At
the time of his statement, Bolanon was conscious of his impending death, having
sustained a stab wound in the chest. Bolanon would have been competent to testify
on the subject of the declaration had he survived. Lastly, the dying declaration was
offered in this criminal prosecution for murder in which Bolanon was the victim.
Res gestae refers to the circumstances, facts, and declarations that grow out of the
main fact and serve to illustrate its character and are so spontaneous and
contemporaneous with the main fact as to exclude the idea of deliberation and
fabrication. A declaration or an utterance is deemed as part of the res gestae
and thus admissible in evidence as an exception to the hearsay rule when the
following requisites concur, to wit: (a) the principal act, the res gestae, is a
startling occurrence; (b) the statements are made before the declarant had
time to contrive or devise; and (c) the statements must concern the occurrence
in question and its immediately attending circumstances.
The requisites for admissibility of a declaration as part of the res gestae concur
herein. Surely, when he gave the identity of the assailant to Estao, Bolanon was
referring to a startling occurrence, i.e., his stabbing by Salafranca. Bolanon was then
on board the taxicab that would bring him to the hospital, and thus had no time to
contrive his identification of Salafranca as the assailant. His utterance about
Salafranca having stabbed him was made in spontaneity and only in reaction to the

Page | 204

Compilation of Digested Cases for Remedial Law Review II


by: K. M. T. BUELA
startling occurrence. The statement was relevant because it identified Salafranca as
the perpetrator.

SCC CHEMICALS CORPORATION vs. THE HONORABLE COURT OF


APPEALS, et al.
G.R. No. 128538, February 28, 2001
FACTS:
SCC Chemicals Corporation obtained a loan from State Investment House Inc.
(SIHI). Upon failure of SCC to pay, SIHI filed an action for a sum of money. During
Pre-Trial, SCC admitted the existence of the loan executed through its officers. SIHI
presented one witness to prove its claim. The cross-examination of said witness was
postponed several times due to one reason or another at the instance of either party.
The case was calendared several times for hearing but each time, SCC or its counsel
failed to appear despite notice. SCC was finally declared by the trial court to have
waived its right to cross-examine the witness of SIHI and the case was deemed
submitted for decision.
Trial court promulgated its decision in favor of SIHI. The appellate court affirmed
in toto the judgment. SCC elevated the case before the SC with the following
contentions: (1) that SIHI introduced documentary evidence through the testimony
of a witness whose competence was not established and whose personal knowledge
of the truthfulness of the facts testified to was not demonstrated in violation of
Sections 36, Rule 130; and (2) that due execution and authenticity of private
documents evidencing the loan was not proved during trial.
ISSUE:
(1) Whether or not a defendant who failed to conduct cross-examination due to its
own fault may questioned the admissibility of the evidence for violation of
hearsay rule.
(2) Whether or not the due execution of loan documents is necessary when the
existence of the loan had already been admitted during pre-trial.
HELD:
(1)
Petitioners reliance on Section 36, Rule 130 of the Rules of Court is misplaced. As
a rule, hearsay evidence is excluded and carries no probative value. However, the
rule does admit of an exception. Where a party failed to object to hearsay evidence,
then the same is admissible. The rationale for this exception is to be found in the
right of a litigant to cross-examine. It is settled that it is the opportunity to crossexamine which negates the claim that the matters testified to by a witness are
hearsay. However, the right to cross-examine may be waived. The repeated
Page | 205

Compilation of Digested Cases for Remedial Law Review II


by: K. M. T. BUELA
failure of a party to cross-examine the witness is an implied waiver of such right.
Petitioner was afforded several opportunities by the trial court to cross-examine the
other partys witness. Petitioner repeatedly failed to take advantage of these
opportunities. No error was thus committed by the respondent court when it
sustained the trial courts finding that petitioner had waived its right to crossexamine the opposing partys witness. It is now too late for petitioner to be raising
this matter of hearsay evidence.chanrob1es virtua1.
(2)
No. Respondent SIHI had no need to present the original of the documents as there
was already a judicial admission by petitioner at pre-trial of the execution of the
promissory note and receipt of the demand letter. It is now too late for petitioner to
be questioning their authenticity. Its admission of the existence of these documents
was sufficient to establish its obligation. Petitioner failed to submit any evidence to
the contrary or proof of payment or other forms of extinguishment of said
obligation.

B. What Need Not Be Proved


LANDBANK OF THE PHILIPPINES vs. SPOUSES VICENTE BANAL
and LEONIDAS ARENAS-BANAL
G.R. NO. 143276, July 20, 2004
FACTS:
Spouses Banal, respondents, are the registered owners of 19+ hectares of
agricultural land situated in Camarines Norte. A portion of the land was
compulsorily acquired by DAR pursuant to the Comprehensive Agrarian Reform
Law of 1988. Petitioner Landbank valued the expropriated portion at P173,918.55.
Unsatisfied with the Landbank valuation and the subsequent affirmance of such by
PARAD, Respondents filed before the RTC a petition for determination of just
compensation impleaded as respondents were the DAR and the Landbank.
After the pre-trial, the court issued an Order dispensing with the hearing and
directing the parties to submit their respective memoranda. Trial court computed
the just compensation for a total of P703,137.00, which is beyond respondents
valuation of P623,000.00. In concluding that the valuation of respondents property,
RTC merely took judicial notice of the average production figures in another case
pending before it and applied the same to instant case without conducting a hearing.

Page | 206

Compilation of Digested Cases for Remedial Law Review II


by: K. M. T. BUELA
ISSUE: Whether or not a court may take judicial notice of the records of one case
pending before it and apply the same to another case also pending with it without
conducting trial and without the knowledge or consent of the parties.
HELD:
The answer is in the negative. Well-settled is the rule that courts are not authorized
to take judicial notice of the contents of the records of other cases even when said
cases have been tried or are pending in the same court or before the same judge.
They may only do so in the absence of objection and with the knowledge of the
opposing party, which are not obtaining here.
Section 3, Rule 129 of the Revised Rules on Evidence is explicit on the necessity
of a hearing before a court takes judicial notice of a certain matter, to wit:
SEC. 3.Judicial notice, when hearing necessary. During the
trial, the court, on its own initiative, or on request of a party,
may announce its intention to take judicial notice of any
matter and allow the parties to be heard thereon.
After the trial, and before judgment or on appeal, the proper
court, on its own initiative or on request of a party, may take
judicial notice of any matter and allow the parties to be
heard thereon if such matter is decisive of a material issue
in the case.
The RTC failed to observe the above provisions.

THE PEOPLE OF THE PHILIPPINES, plaintiff-appellee, vs. JAILON


KULAIS et al.
G.R. Nos. 100901-08, July 16, 1998
FACTS:
Appellants were charge with five (5) counts of kidnapping for ransom and three (3)
counts of kidnapping before the RTC. Trial court convicted the seven accused
positively identified by the victims.
Appellant Kulais argues that he was denied due process when the trial court took
judicial notice of the testimony given in another case by one Lt. Melquiades
Feliciano, who was the team leader of the government troops that captured him and
his purported cohorts. Because he was allegedly deprived of his right to crossexamine a material witness in the person of Lieutenant Feliciano, he contends that
the latters testimony should not be used against him.
Page | 207

Compilation of Digested Cases for Remedial Law Review II


by: K. M. T. BUELA

ISSUE: Whether or not a court may take judicial notice of the testimony of one
witness in a case pending before it and use the same to another case also pending
with it.
HELD:
No. As a general rule, courts should not take judicial notice of the evidence
presented in other proceedings, even if these have been tried or are pending in the
same court, or have been heard and are actually pending before the same judge. This
is especially true in criminal cases, where the accused has the constitutional right to
confront and cross-examine the witnesses against him.
Having said that, we note, however, that even if the court a quo did take judicial
notice of the testimony of Lieutenant Feliciano, it did not use such testimony in
deciding the cases against the appellant. Hence, Appellant Kulais was not denied
due process. His conviction was based mainly on the positive identification made
by some of the kidnap victims. These witnesses were subjected to meticulous crossexaminations conducted by appellants counsel.

MENANDRO B. LAUREANO vs. COURT OF APPEALS AND


SINGAPORE AIRLINES LIMITED
G.R. No. 114776. February 2, 2000
FACTS:
Sometime in 1978, plaintiff is an expatriate employed by Respondent Singapore
Airlines Limited on a contractual basis which is stipulated to last for five (5) years.
However, due to recession, Respondent decided to terminate some of their pilots,
included in the termination is herein plaintiff.
Plaintiff filed the instant case for damages due to illegal termination of contract of
services before the court a quo. Defendant contends that the complaint is for illegal
dismissal together with a money claim arising out of and in the course of plaintiffs
employment "thus it is the Labor Arbiter and the NLRC who have the jurisdiction
pursuant to Article 217 of the Labor Code" and that, since plaintiff was employed
in Singapore, all other aspects of his employment contract and/or documents
executed in Singapore. Thus, defendant postulates that Singapore laws should apply
and courts thereat shall have jurisdiction.
ISSUE: Whether or not courts may take judicial notice of foreign law.
HELD:

Page | 208

Compilation of Digested Cases for Remedial Law Review II


by: K. M. T. BUELA
The answer is in the negative. The Philippine Courts do not take judicial notice of
the laws of Singapore. The defendant that claims the applicability of the Singapore
Laws to this case has the burden of proof. The defendant has failed to do so.
Therefore, the Philippine law should be applied.

CASAN MACODE MAQUILING vs. COMMISSION ON ELECTIONS, et


al.
G.R. NO. 195649, July 2, 2013
FACTS:
Arnado filed herein Motion for Reconsideration assailing the previous decision of
the Supreme Court as to his disqualification to run as mayor. Respondent cites
Section 349 of the Immigration and Naturalization Act of the United States as
having the effect of expatriation when he executed his Affidavit of Renunciation of
American Citizenship on April 3, 2009 and thus claims that he was divested of his
American citizenship.
ISSUE: Whether or not courts may consider applicability of foreign laws to a case
upon reference to it by one of the parties.
HELD:
The Court cannot take judicial notice of foreign laws, which must be presented as
public documents of a foreign country and must be "evidenced by an official
publication thereof." Mere reference to a foreign law in a pleading does not suffice
for it to be considered in deciding a case.

PEOPLE OF THE PHILIPPINES vs. KHADDAFY JANJALANI


G.R. No. 188314, January 10, 2011
FACTS:
Herein accused were charged of multiple murder in relation to the Valentines Day
bombing. The prosecution presented documents furnished by the Department of
Justice, confirming that shortly before the explosion, the spokesperson of the Abu
Sayyaf Group - Abu Solaiman - announced over radio station DZBB that the group
had a Valentine's Day "gift" for former President Gloria Macapagal-Arroyo. After
the bombing, he again went on radio and warned of more bomb attacks.
As stipulated during pretrial, accused Trinidad gave ABS-CBN News Network an
exclusive interview sometime after the incident, confessing his participation in the
Valentine's Day bombing incident. In another exclusive interview on the network,
accused Baharan likewise admitted his role in the bombing incident. Finally,
Page | 209

Compilation of Digested Cases for Remedial Law Review II


by: K. M. T. BUELA
accused Asali gave a television interview, confessing that he had supplied the
explosive devices for the 14 February 2005 bombing. The bus conductor identified
the accused Baharan and Trinidad, and confirmed that they were the two men who
had entered the RRCG bus on the evening of 14 February.
Accused were convicted largely due to the testimony of Asali who turned state
witness and the testimony of the conductor who identified the two accused to be
those who alighted from his bus shortly before the bomb exploded.
Accused contend that the testimony of Asali is inadmissible pursuant to Sec. 30,
Rule 130 of the Rules of Court.
ISSUE: Whether or not testimony of a co-conspirator made during a televised
interview and repeated during trial may be admissible as evidence against a coconspirator.
HELD:
The answer is in the affirmative. Section 30.
Admission by conspirator.
The act or declaration of a conspirator relating to the conspiracy and during its
existence, may be given in evidence against the co-conspirator after the conspiracy
is shown by evidence other than such act of declaration.
While Section 30, Rule 130 of the Rule of Court provides that statements made by
a conspirator against a co-conspirator are admissible only when made during the
existence of the conspiracy, if the declarant repeats the statement in court, his
extrajudicial confession becomes a judicial admission, making the testimony
admissible as to both conspirators.
Section 30, Rule 130 of the Rules of Court applies only to extrajudicial acts or
admissions and not to testimony at trial where the party adversely affected has
the opportunity to cross-examine the declarant.
Distinction must be made between extrajudicial and judicial confessions. An
extrajudicial confession may be given in evidence against the confessant but not
against his co-accused as they are deprived of the opportunity to cross-examine him.
A judicial confession is admissible against the declarant's co-accused since the latter
are afforded opportunity to cross-examine the former.

Page | 210

Compilation of Digested Cases for Remedial Law Review II


by: K. M. T. BUELA
REPUBLIC OF THE PHILIPPINES vs. SANDIGANBAYAN et al.
G.R. No. 152375, December 16, 2011
FACTS:
In 1993, SB ordered the consolidation cases related to the recovery of the ill-gotten
wealth of Marcos Family and cronies. At the trial of Civil Case No. 0009, the
petitioner filed a Motion stating that petitioner wishes to adopt in Civil Case No.
0009 testimonies and the documentary exhibits presented and identified by them in
another related case. This motion partly denied insofar as the adoption of
testimonies on oral deposition of Maurice V. Bane and Rolando Gapud for the
reason that said deponents are not available for cross-examination in this Court by
the respondents.
Petitioner then filed another motion asking SB to take judicial notice of the facts
established by the Bane deposition, together with the marked exhibits appended
thereto. This was again denied by the SB, to wit: Judicial notice is found under Rule
129. This provision refers to the Courts duty to consider admissions made by the
parties in the pleadings, or in the course of the trial or other proceedings in resolving
cases before it. The duty of the Court is mandatory and in those cases where it is
discretionary, the initiative is upon the Court. Such being the case, the Court finds
the Urgent Motion and/or Request for Judicial Notice as something which need not
be acted upon as the same is considered redundant.
Petitioners 3rd motion was again denied by SB. Hence, petitioner filed the instant
motion alleging grave abuse of discretion on the part of SBs refusal to take judicial
notice of or to admit the Bane deposition as part of its evidence. Petitioner asserts
that the case where the Bane deposition was originally taken, introduced and
admitted in evidence is but a "child" of the "parent" case, Civil Case No. 0009;
under this relationship, evidence offered and admitted in any of the "children" cases
should be considered as evidence in the "parent" case.
ISSUE: Whether or not courts in trying consolidated cases may take judicial notice
of testimony and evidence presented in one of the cases consolidated.
HELD:
In adjudicating a case on trial, generally, courts are not authorized to take judicial
notice of the contents of the records of other cases, even when such cases have been
tried or are pending in the same court, and notwithstanding that both cases may have
been tried or are actually pending before the same judge. This rule though admits
of exceptions.
As a matter of convenience to all the parties, a court may properly treat all or any
part of the original record of a case filed in its archives as read into the record of a
Page | 211

Compilation of Digested Cases for Remedial Law Review II


by: K. M. T. BUELA
case pending before it, when, with the knowledge of, and absent an objection from,
the adverse party, reference is made to it for that purpose, by name and number or
in some other manner by which it is sufficiently designated; or when the original
record of the former case or any part of it, is actually withdrawn from the archives
at the court's direction, at the request or with the consent of the parties, and admitted
as a part of the record of the case then pending.
Courts must also take judicial notice of the records of another case or cases, where
sufficient basis exists in the records of the case before it, warranting the dismissal
of the latter case.
The petitioner itself admits that the present case has generated a lot of cases, which,
in all likelihood, involve issues of varying complexity. If we follow the logic of the
petitioners argument, we would be espousing judicial confusion by
indiscriminately allowing the admission of evidence in one case, which was
presumably found competent and relevant in another case, simply based on the
supposed lineage of the cases.

C. Rules of Admissibility
1. Object/Real Evidence
JUNIE MALILLIN Y. LOPEZ vs. PEOPLE OF THE PHILIPPINES
G.R. No. 172953, April 30, 2008
FACTS:
On the strength of a warrant of search and seizure, a team of five police officers
raided the residence of petitioner. The search allegedly yielded two (2) plastic
sachets of shabu and five (5) empty plastic sachets containing residual morsels
of the said substance. Accordingly, petitioner was charged with violation illegal
possession of dangerous drugs under RA 9165.
Accused was convicted. He appealed from the said decision and averred that
there was irregularity on the conduct of the search.
During the trial, the following persons were presented as witnesses: (1) two
members of the raiding team, i.e., Esternon and Bulanon; and (2) the forensic
chemist who examined the seized items. The member of the raiding team who
conducted the recording and marking of the seized items were not presented.
Furthermore the item seized was immediately sent to the laboratory without
presenting the same to the judge who issued the warrant. Furthermore, the
forensic chemist admitted that all seven sachets were delivered to the laboratory

Page | 212

Compilation of Digested Cases for Remedial Law Review II


by: K. M. T. BUELA
by Esternon in the afternoon of the same day that the warrant was executed. The
items seized were not presented to the judge who issued the warrant.
ISSUE: Whether or not testimony of the raiding team and the forensic chemist
is sufficient to establish the identity of the seized items by virtue of a search
warrant.
HELD:
No. Prosecutions for illegal possession of prohibited drugs necessitates that the
elemental act of possession of a prohibited substance be established with moral
certainty, together with the fact that the same is not authorized by law. The
dangerous drug itself constitutes the very corpus delicti of the offense and
the fact of its existence is vital to a judgment of conviction. Essential therefore
in these cases is that the identity of the prohibited drug be established beyond
doubt. Section 21 of the Implementing Rules and Regulations of R.A. No. 9165
mandates that the officer acquiring initial custody of drugs under a search
warrant must conduct the photographing and the physical inventory of the item
at the place where the warrant has been served. The chain of custody rule
requires that there be testimony about every link in the chain, from the
moment the object seized was picked up to the time it is offered in evidence,
in such a way that every person who touched it would describe how and
from whom it was received, where it was and what happened to it while in
the witness possession, the condition in which it was received and the
condition in which it was delivered to the next link in the chain.
Esternon deviated he brought the seized items immediately to the police station
for the alleged purpose of making a "true inventory" thereof, but there appears
to be no reason why a true inventory could not be made in petitioner's house.
Likewise, Esternon's failure to deliver the seized items to the court demonstrates
a departure from the directive in the search warrant that the items seized be
immediately delivered to the trial court with a true and verified inventory of the
same, as required by Rule 126, Section 1246 of the Rules of Court. People v.
Go characterized this requirement as mandatory in order to preclude the
substitution of or tampering with said items by interested parties. Thus, as a
reasonable safeguard, People vs. Del Castillo declared that the approval by the
court which issued the search warrant is necessary before police officers can
retain the property seized and without it, they would have no authority to retain
possession thereof and more so to deliver the same to another agency. Mere
tolerance by the trial court of a contrary practice does not make the practice
right because it is violative of the mandatory requirements of the law and it
thereby defeats the very purpose for the enactment.

Page | 213

Compilation of Digested Cases for Remedial Law Review II


by: K. M. T. BUELA

PEOPLE OF THE PHILIPPINES vs. FELIMON PAGADUAN y


TAMAYO
G.R. No. 179029, August 12, 2010
FACTS:
Captain de Vera, SPO1 Balido and PO3 Almarez conducted a buy-bust
operation which resulted to the apprehension of the herein accused. Thereafter,
the buy-bust team brought the appellant to the Diadi Police Station for
investigation. At the police station, Captain de Vera prepared a request for
laboratory examination. The appellant was transferred to the Diadi Municipal
Jail where he was detained. Two days later, or on December 29, 2003, PO3
Almarez transmitted the letter-request, for laboratory examination, and the
seized plastic sachet to the PNP Crime Laboratory, where they were received
by PO2 Fernando Dulnuan. Police Senior Inspector (PSI) Alfredo Quintero, the
Forensic Chemist of the PNP Crime Laboratory, conducted an examination on
the specimen submitted, and found it to be positive for the presence of shabu.
The accused was charged with violation of RA 9165 and was convicted.
Accused appealed the decision of the RTC and claimed that: (1) his guilt was
not prove beyond reasonable doubt due to failure of the police did not conduct
a prior surveillance on him before conducting the buy-bust operation; and (2) a
period of two days had elapsed from the time the shabu was confiscated to the
time it was forwarded to the crime laboratory for examination.
ISSUE: Whether or not chain of custody was sufficiently established when the
two day period which lapsed between the day of apprehension and day of turnover to the laboratory was not explained.
HELD:
No. The chain of custody rule requires that there be testimony about every
link in the chain, from the moment the object seized was picked up to the
time it is offered in evidence, in such a way that every person who touched
it would describe how and from whom it was received, where it was and
what happened to it while in the witness possession, the condition in which
it was received and the condition in which it was delivered to the next link
in the chain.
The strict compliance to the chain of custody rule may not always be possible
under field conditions. For this reason, the implementing rules provides that
"non-compliance with these requirements under justifiable grounds, as
long as the integrity and the evidentiary value of the seized items are

Page | 214

Compilation of Digested Cases for Remedial Law Review II


by: K. M. T. BUELA
properly preserved by the apprehending officer/team, shall not render void
and invalid such seizures of and custody over said items."
Thus, noncompliance thereof is not necessarily fatal to the prosecutions case;
police procedures in the handling of confiscated evidence may still have some
lapses, as in the present case. These lapses, however, must be recognized and
explained in terms of their justifiable grounds, and the integrity and evidentiary
value of the evidence seized must be shown to have been preserved.
In the present case, the prosecution did not bother to offer any explanation to
justify the failure of the police to conduct the required physical inventory,
photographing of the seized drugs and the details of the turn over during the two
day period.
The records show that the seized specimen was forwarded by PO3 Almarez to
the PNP Crime Laboratory on December 29, 2003, where it was received by
PO2 Dulnuan, and later examined by PSI Quintero. However, the person from
whom PO3 Almarez received the seized illegal drug for transfer to the crime
laboratory was not identified. As earlier discussed, the identity of the duty desk
officer who received the shabu, as well as the person who had temporary
custody of the seized items for two days, had not been established.

NARCISO SALAS vs. ANNABELLE MATUSALEM


G.R. No. 180284, September 11, 2013
FACTS:
Annabelle Matusalem (respondent) filed a complaint for Support/Damages
against Narciso Salas and claimed that petitioner is the father of her son
Christian Paulo Salas. In his answer, Petitioner denied paternity of the child
Christian Paulo.
At the trial, respondent and her witness Grace Murillo testified. Murillo
corroborated respondents testimony as to the payment by petitioner of
apartment rental, his weekly visits to respondent and financial support to her,
his presence during and after delivery of respondents baby, respondents
attempted suicide through sleeping pills overdose and hospitalization for which
she paid the bill, her complaint before the police authorities and meeting with
petitioners wife at the headquarters. Respondent also presented the following
evidence: (1) certificate of live birth of Christian Paulo which she allegedly
filled up with entries supposedly dictated by the Petitioner; and (2) baptismal
certificate.

Page | 215

Compilation of Digested Cases for Remedial Law Review II


by: K. M. T. BUELA
On April 5, 1999, the trial court rendered its decision in favor of respondent.
CA affirmed the decision of the trial court. Hence, Petitioner filed a petition for
review on certiorari on the ground that CA erred in holding that the filiation of
Christian Paulo was duly established pursuant to article 175 in relation to article
172 of the Family Code and existing jurisprudence and therefore entitled to
support from the petitioner
ISSUE: Whether or not illegitimate filiation may be proved by mere testimonial
evidence that the alleged father provided financial support and unsigned birth
certificate and love letters.
HELD:
The answer is in the negative. Under Article 175 of the Family Code of the
Philippines, illegitimate filiation may be established in the same way and on the
same evidence as legitimate children.
Article 172 of the Family Code of the Philippines states:
The filiation of legitimate children is established by any of the
following:
(1) The record of birth appearing in the civil register or a final
judgment; or
(2) An admission of legitimate filiation in a public document or
a private handwritten instrument and signed by the parent
concerned.
In the absence of the foregoing evidence, the legitimate filiation
shall be proved by:
(1) The open and continuous possession of the status of a
legitimate child; or
(2) Any other means allowed by the Rules of Court and special
laws.
A certificate of live birth purportedly identifying the putative father is not
competent evidence of paternity when there is no showing that the putative
father had a hand in the preparation of the certificate. Thus, if the father did not
sign in the birth certificate, the placing of his name by the mother, doctor,
registrar, or other person is incompetent evidence of paternity. Neither can such
birth certificate be taken as a recognition in a public instrument and it has no
probative value to establish filiation to the alleged father.
Furthermore, while baptismal certificates may be considered public documents,
they can only serve as evidence of the administration of the sacraments on the

Page | 216

Compilation of Digested Cases for Remedial Law Review II


by: K. M. T. BUELA
dates so specified. They are not necessarily competent evidence of the veracity
of entries therein with respect to the childs paternity.
The rest of respondents documentary evidence consists of handwritten notes
and letters, hospital bill and photographs taken of petitioner and respondent
inside their rented apartment unit are not sufficient to establish Christian Paulos
filiation to petitioner as they were not signed by petitioner and contained no
statement of admission by petitioner that he is the father of said child. Thus,
even if these notes were authentic, they do not qualify under Article 172 (2) vis- vis Article 175 of the Family Code which admits as competent evidence of
illegitimate filiation an admission of filiation in a private handwritten
instrument signed by the parent concerned.

PEOPLE OF THE PHILIPPINES vs. RUPER POSING Y ALAYON


G.R. No. 196973, July 31, 2013
FACTS:
SPO1 Angeles together with PO1 Jesus Cortez, PO1 Ralph Nicart, and their
informant conducted a buy-bust operation which resulted to the apprehension
of herein accused. Consequently, the accused was charged with violation of both
Sections 5 and 11, Article II, of R.A. 9165. The trial court found Posing
GUILTY. The CA affirmed the ruling of the trial court.
On appeal, the accused-appellant, contended that the police officers failed to
comply with the proper procedure in the handling and custody of the seized
drugs, as provided under Section 21 of R. A. No. 9165, which ultimately
affected the chain of custody of the confiscated drugs.
The accused-appellant, argued that the following instances would constitute a
break in the chain of custody of the seized plastic sachets of shabu: (1) SPO1
Angeles failed to identify the duty officer to whom he turned over the alleged
confiscated shabu; (2) SPO1 Angeles was not able to recall who brought the
drug specimens to the crime laboratory; (3) SPO1 Angeles failed to mark the
confiscated sachets at the crime scene immediately after the accused-appellant
was arrested; and (4) the police officers failed to prepare an inventory report of
the confiscated drugs, no photographs of the same were taken in the presence of
the accused-appellant and that of a representative from the media or the
Department of Justice or any elected public official.
The People, through the Office of the Solicitor General, countered that although
the requirements under Section 21 of R. A. No. 9165 has been held to be
mandatory, non-compliance with the same, does not necessarily warrant an
Page | 217

Compilation of Digested Cases for Remedial Law Review II


by: K. M. T. BUELA
acquittal. In addition, it was averred that the police officers are entitled to the
presumption of regularity in the performance of official duties. Finally, the
accused-appellant did not interpose any evidence in support of his defense aside
from his bare denial.
ISSUE: Whether or not the chain of custody rule provided under Section 21 of
RA 9165 must be followed strictly.
HELD:
The answer is in the negative. In cases involving violations of Dangerous Drugs
Act, credence should be given to the narration of the incident by the prosecution
witnesses especially when they are police officers who are presumed to have
performed their duties in a regular manner, unless there is evidence to the
contrary. In this regard, the defense failed to show any ill motive or odious intent
on the part of the police operatives to impute such a serious crime that would
put in jeopardy the life and liberty of an innocent person, such as in the case of
appellant. Incidentally, if these were simply trumped-up charges against him, it
remains a question why no administrative charges were brought against the
police operatives.
In Malillin v. People, we laid down the chain of custody requirements that must
be met in proving that the seized drugs are the same ones presented in court: (1)
testimony about every link in the chain, from the moment the item was picked
up to the time it is offered into evidence; and (2) witnesses should describe the
precautions taken to ensure that there had been no change in the condition of
the item and no opportunity for someone not in the chain to have possession of
the item.
In this case, the prosecution was able to prove, through the testimonies of its
witnesses that the integrity of the seized item was preserved every step of the
process. After the sale of shabu and another sachet was discovered in the person
of accused-appellant, SPO1 Angeles, who was the poseur-buyer in the buy-bust
operation, marked the drug specimens, and then turned over the same to the
desk officer, who in turn handed it to PO1 Sales. The latter then prepared a
Request for Laboratory Examination, and on the same day, the specimens were
delivered by PO1 Nicart to the PNP Crime Laboratory for quantitative and
qualitative examination, conducted by Engr. Jabonillo.
Time and again, jurisprudence is consistent in stating that less than strict
compliance with the procedural aspect of the chain of custody rule does not
necessarily render the seized drug items inadmissible.

Page | 218

Compilation of Digested Cases for Remedial Law Review II


by: K. M. T. BUELA
PEOPLE OF THE PHILIPPINES vs. ASIR GANI y ALIH and
NORMINA GANI y GALOS4
G.R. No. 198318, November 27, 2013
FACTS:
Accused-appellants were charged with illegal sale of dangerous drugs in
violation of RA 9165. They were apprehended through a buy-bust operation
conducted by the joint team of NBI and PDEA. During the search incidental to
accused-appellants arrest, the buy-bust team seized from accused-appellants
possession two other sachets of shabu, the marked money, accused-appellant
Asirs .45 caliber pistol, and the motorcycle. However, the inventory report, did
not include the two other sachets of shabu seized from accused-appellants
possession. Thereafter, the buy-bust team brought accused-appellants to the
NBI Headquarters in Manila. Accused were charged and convicted by the trial
court.
During trial, SI Saul testified that in addition to the two plastic sachets of shabu
sold to him by accused-appellants, there were two more sachets of shabu
recovered from accused-appellants possession by the buy-bust team during the
body search conducted incidental to accused-appellants lawful arrest.
Accused-appellants further point out that the prosecutions evidence conflicted
as to the number of sachets of shabu seized from them. It was stipulated during
the pre-trial that there were four plastic sachets of shabu but prosecution witness
SI Saul testified that as poseur-buyer, he bought and received only two sachets
of shabu from accused-appellants. No details were provided about the seizure
of the other two sachets of shabu.
ISSUE: Whether or not there was substantial compliance to chain of custody
when not seized items where included in the inventory.
HELD:
The answer is in the affirmative. Jurisprudence has decreed that, in dangerous
drugs cases, the failure of the police officers to make a physical inventory and
to photograph the sachets of shabu, as well as to mark the sachets at the place
of arrest, do not render the seized drugs inadmissible in evidence or
automatically impair the integrity of the chain of custody of the said drugs. What
is of utmost importance is the preservation of the integrity and the evidentiary
value of the seized items, as these would be utilized in the determination of the
guilt or innocence of the accused.

Penned by Leonardo-De Castro, J.


Page | 219

Compilation of Digested Cases for Remedial Law Review II


by: K. M. T. BUELA
In this case, prosecution witness, SI Saul, was able to explain why there were
a total of four sachets of shabu presented during trial, when SI Saul only
bought two sachets during the buy- bust operation.

2. Documentary Evidence
Best Evidence Rule
CITIBANK, N.A. MASTERCARD vs. EFREN S. TEODORO
G.R. No. 150905. September 23, 2003
FACTS:
Respondent made various purchases through his credit card. Accordingly, he
was billed by petitioner for those purchases, for which he tendered various
payments. However, as of January 20, 1995, petitioner claims that respondent
was not able to pay his bills despite demand. This prompted petitioner to file a
Complaint for collection. During the trial, petitioner presented several sales
invoices or charge slips. Because all these copies appeared to bear the signatures
of respondent, the trial court deemed them sufficient proof of his purchases with
the use of the credit card. Accordingly, the MTC ordered him to pay petitioner
the amount of P24,388.36 plus interest and penalty fee.
On appeal, RTC affirmed the MTC Decision in toto. However, CA reversed the
decision of the trial court and ruled that photocopies of the sales invoices or
charge slips as proof of the obligation are insufficient to prove any liability on
respondents part.
ISSUE: Whether or not mere photocopies as proof of obligations is admissible
as evidence where unavailability of the original thereof was not explained or
proved during trial.
HELD:
The answer is in the negative. The original copies of the sales invoices are the
best evidence to prove the alleged obligation following Section 3, Rule 130.
Photocopies thereof are mere secondary evidence. As such, they are
inadmissible because petitioner, as the offeror, failed to prove any of the
exceptions provided under Section 35 of Rule 130 of the Rules of Court, as well

Section 3. Original document must be produced; exceptions. When the subject of inquiry is the contents of a
document, no evidence shall be admissible other than the original document itself, except in the following cases:
(a) When the original has been lost or destroyed, or cannot be produced in court, without bad faith on the part of
the offeror;
(b) When the original is in the custody or under the control of the party against whom the evidence is offered, and
the latter fails to produce it after reasonable notice;
5

Page | 220

Compilation of Digested Cases for Remedial Law Review II


by: K. M. T. BUELA
as the conditions of their admissibility. Because of the inadmissibility of the
photocopies in the absence of the originals, respondents obligation was not
established.
Before a party is allowed to adduce secondary evidence to prove the
contents of the original sales invoices, the offeror must prove the following:
(1) the existence or due execution of the original; (2) the loss and
destruction of the original or the reason for its nonproduction in court in
accordance with Section 56, Rule 130; and (3) on the part of the offeror, the
absence of bad faith to which the unavailability of the original can be
attributed. The correct order of proof is as follows: existence, execution,
loss, and contents.
In the present case, the existence of the original sales invoices was established
by the photocopies and the testimony of Hernandez. Petitioner, however, failed
to prove that the originals had been lost or could not be produced in court after
reasonable diligence and good faith in searching for them.

WILGEN LOON et al. vs. POWER MASTER, INC.et al.


G.R. No. 189404, December 11, 2013
FACTS:
Respondents employed and assigned the petitioners as janitors and leadsmen in
various PLDT offices in Metro Manila area. Subsequently, the petitioners filed
a complaint for money claims and illegal dismissal. Labor Arbiter (LA) partially
ruled in favor of the petitioners. Both parties appealed the LAs ruling with the
NLRC.
6 months after filing their notice of appeal, Respondents filed an unverified
supplemental appeal. They attached photocopied and computerized copies of
list of employees with automated teller machine (ATM) cards to the
supplemental appeal. This list also showed the amounts allegedly deposited in
the employees ATM cards. On the other hand, petitioners filed an Urgent
Manifestation and Motion where they asked for the deletion of the supplemental
appeal from the records because it allegedly suffered from infirmities. First, the

(c) When the original consists of numerous accounts or other documents which cannot be examined in court
without great loss of time and the fact sought to be established from them is only the general result of the whole;
and
(d) When the original is a public record in the custody of a public officer or is recorded in a public office.
SEC. 5. When original document is unavailable. When the original document has been lost or destroyed, or
cannot be produced in court, the offeror, upon proof of its execution or existence and the cause of its unavailability
without bad faith on his part, may prove its contents by a copy, or by a recital of its contents in some authentic
document, or by the testimony of witnesses in the order stated
6

Page | 221

Compilation of Digested Cases for Remedial Law Review II


by: K. M. T. BUELA
supplemental appeal was not verified. Second, it was belatedly filed six months
from the filing of the respondents notice of appeal with memorandum on
appeal. The petitioners pointed out that they only agreed to the respondents
filing of a responsive pleading until December 18, 2002. Third the attached
documentary evidence on the supplemental appeal bore the petitioners forged
signatures.
NLRC giving weight to the photocopy of computerized payroll records ruled in
favor of respondent. It maintained that the absence of the petitioners signatures
in the payrolls was not an indispensable factor for their authenticity. The CA
affirmed the NLRCs ruling.
ISSUE: Whether or not mere photocopies as documentary evidence filed 6
months from notice of appeal are admissible in evidence where there is an
allegation of forgery by the adverse party.
HELD:
The answer is in the negative.
While strict adherence to the technical rules of procedure is not required in labor
cases, the liberality of procedural rules is qualified by two requirements: (1) a
party should adequately explain any delay in the submission of evidence; and
(2) a party should sufficiently prove the allegations sought to be proven.
Respondents, in this case, failed to sufficiently prove the allegations sought to
be proven. Why the respondents photocopied and computerized copies of
documentary evidence were not presented at the earliest opportunity is a serious
question that lends credence to the petitioners claim that the respondents
fabricated the evidence for purposes of appeal.
While courts generally admit in evidence and give probative value to
photocopied documents in administrative proceedings, allegations of
forgery and fabrication should prompt the adverse party to present the
original documents for inspection. It was incumbent upon the respondents to
present the originals, especially in this case where the petitioners had submitted
their specimen signatures. Instead, the respondents effectively deprived the
petitioners of the opportunity to examine and controvert the alleged spurious
evidence by not adducing the originals. Failure to present the originals raises
the presumption that evidence willfully suppressed would be adverse if
produced.

Page | 222

Compilation of Digested Cases for Remedial Law Review II


by: K. M. T. BUELA
THERESITA DIMAGUILA et al. vs. JOSE and SONIA A. MONTEIRO
G.R. No. 201011, January 27, 2014
FACTS:
Respondent spouses filed their Complaint for Partition and Damages against the
petitioners. Later on the complaint was amended to an action for recovery.
Respondent alleged: (1) that the disputed property was originally owned by
Buenaseda; (2) had long been partitioned between her two sons, Perfecto and
Vitaliano Dimaguila; and (3) that owned 1/3 of the portion of Perfectos share
by virtue of a deed of sale executed between them and one of Perfectos heir,
Pedro.
While in their original answer, petitioner alleged that the subject property had
already been extra-judicially partitioned between the heirs of Vitaliano and
Perfecto, they subsequently changed their position when the complaint was
amended. They now claimed that the property was not partitioned with specific
metes and bounds. What is cleared among the heirs is their respective shares
thereof.
RTC ruled in favor of Spouses Monteiro. CA affirmed the ruling of the RTC.
The CA found that Spouses Monteiro had established their case by a
preponderance of evidence thru their presentation of the Deed of Extrajudicial
Partition, the certified true copy of cadastral map and the municipal assessor's
records. Hence, this petition for review on certiorari where petitioners argued
that the cadastral map, which serves as the basis of the alleged partition, is
inadmissible in violation of the best evidence rule and hearsay rule.
ISSUE: Whether or not a certified true copy of cadastral map is inadmissible
in evidence on the ground that it violates the best evidence rule and hearsay
rule.
HELD:
The answer is in the negative. Anent violation of Best Evidence Rule, Section
3(d) of Rule 130 of the Rules of Court provides that when the subject of
inquiry is the contents of a document, no evidence shall be admissible other
than the original document itself, except when the original is a public
record in the custody of a public officer or is recorded in a public office.
Section 7 of the same Rule provides that when the original of a document is in
the custody of a public officer or is recorded in a public office, its contents may
be proved by a certified copy issued by the public officer in custody thereof.
Section 24 of Rule 132 provides that the record of public documents may be
evidenced by a copy attested by the officer having the legal custody or the
record. Certified true copies of the cadastral map of Liliw and the corresponding
Page | 223

Compilation of Digested Cases for Remedial Law Review II


by: K. M. T. BUELA
list of claimants of the area covered by the map were presented by two public
officers.
As to the Hearsay Rule, Section 44 of Rule 130 of the Rules of Court similarly
provides that entries in official records are an exception to the rule. The
rule provides that entries in official records made in the performance of the duty
of a public officer of the Philippines, or by a person in the performance of a duty
specially enjoined by law, are prima facie evidence of the facts therein stated.
The necessity of this rule consists in the inconvenience and difficulty of
requiring the official's attendance as a witness to testify to the innumerable
transactions in the course of his duty. The document's trustworthiness consists
in the presumption of regularity of performance of official duty.
Cadastral maps are the output of cadastral surveys. The DENR is the department
tasked to execute, supervise and manage the conduct of cadastral surveys. It is,
therefore, clear that the cadastral map and the corresponding list of claimants
qualify as entries in official records as they were prepared by the DENR, as
mandated by law. As such, they are exceptions to the hearsay rule and are
primafacie evidence of the facts stated therein.

Parole Evidence
RAFAEL S. ORTAEZ vs. THE COURT OF APPEALS, et al.
G.R. No. 107372. January 23, 1997
FACTS:
Private respondents sold to petitioner two (2) parcels of registered land for a
consideration of P35,000.00 and P20,000.00, respectively as evidence by two
(2) deed of sale.
Private respondents received the payments for the above-mentioned lots, but
failed to deliver the titles to petitioner. When the latter demanded from the
former the delivery of said titles, Private respondents, refused on the ground that
the title of the first lot is in the possession of another person, and petitioners
acquisition of the title of the other lot is subject to certain conditions.
Offshoot, petitioner sued private respondents for specific performance before
the RTC. In their answer with counterclaim private respondents merely alleged
the existence of the following oral conditions which were never reflected in the
deeds of sale

Page | 224

Compilation of Digested Cases for Remedial Law Review II


by: K. M. T. BUELA
ISSUE: Whether or not parol evidence relating to certain stipulated condition
made orally may be admitted in evidence in addition to those expressly provided
in a contract.
HELD:
The answer is in the negative. Section 9 of Rule 130 of the Rules of Court, when
the terms of an agreement were reduced to writing, as in this case, it is deemed
to contain all the terms agreed upon and no evidence of such terms can be
admitted other than the contents thereof.
Considering that the written deeds of sale were the only repository of the truth,
whatever is not found in said instruments must have been waived and
abandoned by the parties. Examining the deeds of sale, we cannot even make
an inference that the sale was subject to any condition. As a contract, it is the
law between the parties. Oral testimony on the alleged conditions, coming from
a party who has an interest in the outcome of the case, depending exclusively
on human memory, is not as reliable as written or documentary evidence.
Spoken words could be notoriously unreliable unlike a written contract which
speaks of a uniform language.
Parol evidence herein sought to be introduced would vary, contradict or defeat
the operation of a valid instrument, hence, contrary to the rule that the parol
evidence rule forbids any addition to the terms of a written instrument by
testimony purporting to show that, at or before the signing of the
document, other or different terms were orally agreed upon by the parties.
Although parol evidence is admissible to explain the meaning of a contract, "it
cannot serve the purpose of incorporating into the contract additional
contemporaneous conditions which are not mentioned at all in the writing unless
there has been fraud or mistake." No such fraud or mistake exists in this case.

LAPULAPU FOUNDATION, INC. and ELIAS Q. TAN vs. COURT OF


APPEALS and ALLIED BANKING CORP.
G.R. No. 126006, January 29, 2004
FACTS:
Respondent bank filed instant collection suit against Petitioner Foundation and
previous president Tan for failure to pay four matured loan as evidenced by
promissory notes signed by Tan in his capacity as president of the said
Foundation.

Page | 225

Compilation of Digested Cases for Remedial Law Review II


by: K. M. T. BUELA
In disclaiming any liability for the loans, the petitioner Foundation maintains
that said loans were contracted by petitioner Tan in his personal capacity. On
the other hand, while admitting that the loans were his personal obligation,
petitioner Tan avers that the same is not yet due as he had an unwritten
agreement with the respondent Bank that these loans would be renewed on a
year-to-year basis and paid from the proceeds of his shares of stock in the
Lapulapu Industries Corp.
Trial Court ruled petitioners are liable to the bank solidarily. On appeal, the CA
affirmed the judgment of the court a quo. CA likewise rejected petitioner Tans
assertion that there was an unwritten agreement between him and the respondent
Bank that he would pay the loans from the proceeds of his shares of stocks in
the Lapulapu Industries Corp.
ISSUE: Whether or not an alleged unwritten agreement between the creditor
and the debtor which is not reflected on the promissory note (PN) evidencing
the loan is admissible in evidence in addition to the terms of the PN
HELD:
The answer is in the negative. The parol evidence rule likewise constrains this
Court to reject petitioner Tans claim regarding the purported unwritten
agreement between him and the respondent Bank on the payment of the
obligation. Section 9, Rule 130 of the of the Revised Rules of Court provides
that when the terms of an agreement have been reduced to writing, it is to be
considered as containing all the terms agreed upon and there can be, between
the parties and their successors-in-interest, no evidence of such terms other than
the contents of the written agreement.
In this case, the promissory notes are the law between the petitioners and the
respondent Bank. Nowhere was it stated therein that they would be renewed on
a year-to-year basis or rolled-over annually until paid from the proceeds of
petitioner Tans shares in the Lapulapu Industries Corp. Accordingly, this
purported unwritten agreement could not be made to vary or contradict the terms
and conditions in the promissory notes.
Evidence of a prior or contemporaneous verbal agreement is generally not
admissible to vary, contradict or defeat the operation of a valid contract. While
parol evidence is admissible to explain the meaning of written contracts, it
cannot serve the purpose of incorporating into the contract additional
contemporaneous conditions which are not mentioned at all in writing, unless
there has been fraud or mistake. No such allegation had been made by the
petitioners in this case.

Page | 226

Compilation of Digested Cases for Remedial Law Review II


by: K. M. T. BUELA

MODESTO LEOVERAS vs. CASIMERO VALDEZ


G.R. No. 169985, June 15, 2011
FACTS:
Respondent and the petitioner executed an Agreement, allotting their portions
of the subject property, to wit: Petitioner Modesto Leoveras 3,020 square
meters and Respondent Casimero Valdez 7,544.27 square meters.
In 1996, the respondent learned that the petitioner had already obtained in his
name two TCTs: one - covering an area of 3,020 square meters; and two covering an area of 1,004 square meters. Thus, respondent filed a complaint for
Annulment of Title, Reconveyance and Damages against the petitioner, seeking
the reconveyance of the 1,004-square meter portion on the ground that the
petitioner is entitled only to the 3,020 square meters identified in the parties'
Agreement.
In his defense, the petitioner claimed: (1) that the parties has agreed that the
extent of their ownership would be based on their actual possession; (2) that he
actually possessed and subsequently acquired has a total area of 4,024 square
meters, which he subdivided into two portions and caused to be covered by the
two TCTs in question; and (3) that respondent participated in executing an
Affidavit of confirmation of subdivision, which corrected the mistake in the
previously executed Agreement and confirmed the petitioner's ownership over
the disputed property.
The RTC dismissed the complaint. CA reversed the RTC. CA noted the
discrepancy between the respondent's signatures as appearing in the Affidavit,
on one hand, and the documents on record, on the other.
ISSUE: Whether or not an allegation of ownership that is contrary to those
expressly stated in an agreement may be used as evidence
HELD:
The answer is in the negative. The petitioner does not dispute the due execution
and the authenticity the Agreement entered into between him and the
respondent. However, he claims that since the Agreement does not reflect the
true intention of the parties, the Affidavit was subsequently executed in order to
reflect the parties' true intention. Factual findings of the CA holding that such
affidavit is spurious due to discrepancy of respondents signature therein leads
us to rely only on the agreement as the basis for the claim of ownership of both
parties.

Page | 227

Compilation of Digested Cases for Remedial Law Review II


by: K. M. T. BUELA
The petitioner's argument calls to fore the application of the parol evidence rule,
i.e., when the terms of an agreement are reduced to writing, the written
agreement is deemed to contain all the terms agreed upon and no evidence of
these terms can be admitted other than what is contained in the written
agreement. Whatever is not found in the writing is understood to have been
waived and abandoned.
To avoid the operation of the parol evidence rule, the Rules of Court allows a
party to present evidence modifying, explaining or adding to the terms of the
written agreement if he puts in issue in his pleading, as in this case, the failure
of the written agreement to express the true intent and agreement of the parties.
The failure of the written agreement to express the true intention of the parties
is either by reason of mistake, fraud, inequitable conduct or accident, which
nevertheless did not prevent a meeting of the minds of the parties.

Electronic Evidence
HEIRS OF LOURDES SAEZ SABANPAN vs. ALBERTO C.
COMORPOSA et al.
G.R. No. 152807, August 12, 2003
FACTS:
Petitioners filed an action for unlawful detainer against respondents and alleged
that the disputed property was owned by Marcos Saez, predecessor of
petitioners; that Marcos son Adolfo, for humanitarian reasons, allowed
respondents to occupy a portion of Marcos Saez land without paying any rental.
On 7 May 1998, a formal demand was made upon the respondents to vacate the
premises but the latter refused to vacate the same and claimed that they were
the legitimate claimants and the actual and lawful possessors of the premises.
MTC rendered judgment in favor of petitioners. On appeal, RTC reversed the
said decision. Affirming the RTC, the CA upheld the right of respondents as
claimants and possessors. The CA lend credence to the Certification issued by
the DENRs community environment and natural resources (CENR) officer was
proof that when the cadastral survey was conducted, the land was still alienable
and was not yet allocated to any person. Therefore, respondents after
sufficiently proving their actual, physical, open, notorious, exclusive,
continuous and uninterrupted possession thereof since 1960 have better right to
possess alienable and disposable land of the public domain.
Hence, this Petition, petitioners avers that CA gravely abuse its discretion in
giving weight to the CENR Officers Certification, which only bears the
facsimile of the alleged signature of a certain Jose F. Tagorda.
Page | 228

Compilation of Digested Cases for Remedial Law Review II


by: K. M. T. BUELA

ISSUE: Whether or not a certification issued by a public officer bearing a


facsimile signature is inadmissible in evidence.
HELD:
The rule stated in Garvida v. Sales Jr that Pleadings filed via fax machines
are not considered originals and are at best exact copies. As such, they are not
admissible in evidence, as there is no way of determining whether they are
genuine or authentic is not applicable to the instant case.
The Certification, on the other hand, is being contested for bearing a facsimile
of the signature of CENR Officer Jose F. Tagorda. The facsimile referred to is
not the same as that which is alluded to in Garvida. The one mentioned here
refers to a facsimile signature, which is defined as a signature produced by
mechanical means but recognized as valid in banking, financial, and business
transactions.
Note that the CENR officer has not disclaimed the Certification. In fact, the
DENR regional director has acknowledged and used it as reference in his Order
dated April 2, 1998. If the Certification were a sham as petitioner claims, then
the regional director would not have used it as reference in his Order. Instead,
he would have either verified it or directed the CENR officer to take the
appropriate action, as the latter was under the formers direct control and
supervision.

ELLERY MARCH G. TORRES vs. PHILIPPINE AMUSEMENT and


GAMING CORPORATION (PAGCOR)
G.R. No. 193531, December 14, 2011
FACTS:
Petitioner was a Slot Machine Operations Supervisor (SMOS) of respondent
Philippine Amusement and Gaming Corporation (PAGCOR) who was
terminated due to his alleged participation in padding of Credit Meter Readings
(CMR) or slot machines at Casino Filipino-Hyatt.
Petitioner filed with the CSC a Complaint against PAGCOR for illegal
dismissal. CSC held that petitioner's appeal had already prescribed. The CSC
did not give credit to petitioner's claim that he sent a facsimile transmission of
his letter reconsideration within the period prescribed by the Uniform Rules on
Administrative Cases in the Civil Service. It found that a verification of one of
the telephone numbers where petitioner allegedly sent his letter reconsideration
disclosed that such number did not belong to the PAGCOR's Office of the Board
Page | 229

Compilation of Digested Cases for Remedial Law Review II


by: K. M. T. BUELA
of Directors; and that petitioner should have mentioned about the alleged
facsimile transmission at the first instance when he filed his complaint and not
only when respondent PAGCOR raised the issue of prescription in its Comment.
Petitioner contends that he filed his letter reconsideration of his dismissal on
August 13, 2007, which was within the 15-day period for filing the same; and
that he did so by means of a facsimile transmission sent to the PAGCOR's Office
of the Board of Directors. He claims that the sending of documents thru
electronic data message, which includes facsimile, is sanctioned under Republic
Act No. 8792, the Electronic Commerce Act of 2000. Petitioner further
contends that since his letter reconsideration was not acted upon by PAGCOR,
he then filed his complaint before the CSC.
ISSUE: Whether or not a letter reconsideration filed through facsimile is
allowed
HELD:
Even assuming arguendo that petitioner indeed submitted a letter
reconsideration which he claims was sent through a facsimile transmission, such
letter reconsideration did not toll the period to appeal. The mode used by
petitioner in filing his reconsideration is not sanctioned by the Uniform Rules
on Administrative Cases in the Civil Service. As we stated earlier, the motion
for reconsideration may be filed only in two ways, either by mail or personal
delivery.
In Garvida v. Sales, Jr., it was held inadmissible in evidence the filing of
pleadings through fax machines and ruled that: x x x A facsimile is not a
genuine and authentic pleading. It is, at best, an exact copy preserving all the
marks of an original. Without the original, there is no way of determining on its
face whether the facsimile pleading is genuine and authentic and was originally
signed by the party and his counsel. It may, in fact, be a sham pleading. x x x
Moreover, a facsimile transmission is not considered as an electronic evidence
under the Electronic Commerce Act. In MCC Industrial Sales Corporation v.
Ssangyong Corporation, it was held that a facsimile transmission cannot be
considered as electronic evidence. It is not the functional equivalent of an
original under the Best Evidence Rule and is not admissible as electronic
evidence.

Page | 230

Compilation of Digested Cases for Remedial Law Review II


by: K. M. T. BUELA
RUSTAN ANG y PASCUA vs. THE HONORABLE COURT OF
APPEALS and IRISH SAGUD
G.R. No. 182835, April 20, 2010
FACTS:
Rustan Ang was charged of violation of the Anti-Violence Against Women and
Their Children Act for purposeful sending Short Messaging Service (SMS)
using his mobile phone, a pornographic picture to one Irish Sagud, who was his
former girlfriend, whereby the face of the latter was attached to a completely
naked body of another woman making it to appear that it was said Irish Sagud
who is depicted in the said obscene and pornographic picture thereby causing
substantial emotional anguish, psychological distress and humiliation to the said
Irish Sagud.
Rustan claims that the obscene picture sent to Irish through a text message
constitutes an electronic document. Thus, it should be authenticated by means
of an electronic signature, as provided under Section 1, Rule 5 of the Rules on
Electronic Evidence (A.M. 01-7-01-SC).
ISSUE: Whether or not text messages to be admissible as evidence in a criminal
case must be authenticated following the Electronic Evidence Rule.
HELD:
The answer is in the negative. Electronic Evidence Rule do not apply to the
present criminal action. The said Rules applies only to civil actions, quasijudicial proceedings, and administrative proceedings. (A.M. No. 01-7-01-SC,
Rule 1, Section 2.)

3. Testimonial Evidence
a. Qualifications: one who can perceive and perceiving can make known
his perception
i. Ability to observe/perceive
ii. Ability to recall/remember
iii. Ability to relate/communicate

Page | 231

Compilation of Digested Cases for Remedial Law Review II


by: K. M. T. BUELA
b. Disqualifications
LUISA NAVARRO MARCOS* vs. THE HEIRS OF THE LATE DR.
ANDRES NAVARRO, JR
G.R. No. 198240, July 03, 2013
FACTS:
Petitioner discovered that respondents are claiming exclusive ownership of the
subject lot. Respondents based their claim on the Affidavit of Transfer of Real
Property where Andres, Sr. (common ascendant of both petitioner and
respondent) donated the subject lot to Andres, Jr. Believing that the affidavit is
a forgery, the sisters, requested a handwriting examination of the affidavit. The
PNP handwriting expert PO2 Mary Grace Alvarez found that Andres, Sr.s
signature on the affidavit and the submitted standard signatures of Andres, Sr.
were not written by one and the same person.
Thus, the sisters sued the respondents for annulment of the deed of donation.
After the pre-trial, respondents moved to disqualify PO2 Alvarez as a witness.
They argued that the RTC did not authorize the handwriting examination of the
affidavit. RTC granted respondents motion and disqualified PO2 Alvarez as a
witness. Petitioners elevated the case before the CA by way of petition for
certiorari. CA denied the petition
ISSUE: Whether or not experts testimony who is not authorized by the trial
court is disqualified from being a witness.
HELD:
No. A witness must only possess all the qualifications and none of the
disqualifications provided in the Rules of Court. Section 20, Rule 130 of the
Rules on Evidence provides the qualifications of a witness, i.e., all persons who
can perceive, and perceiving, can make known their perception to others, may
be witnesses.
The disqualifications are as follows: (1) Section 19, Rule 130 disqualifies those
who are mentally incapacitated and children whose tender age or immaturity
renders them incapable of being witnesses; (2) Section 20 of the same rule
provides for disqualification based on conflicts of interest or on relationship; (3)
Section 21 provides for disqualification based on privileged communications;
and (4) Section 15 of Rule 132 may not be a rule on disqualification of
witnesses but it states the grounds when a witness may be impeached by the
party against whom he was called. The specific enumeration of disqualified
witnesses excludes the operation of causes of disability other than those
mentioned in the Rules.

Page | 232

Compilation of Digested Cases for Remedial Law Review II


by: K. M. T. BUELA
As a handwriting expert of the PNP, PO2 Alvarez can surely perceive and make
known her perception to others. We have no doubt that she is qualified as a
witness. She cannot be disqualified as a witness since she possesses none of the
disqualifications specified under the Rules. Respondents motion to disqualify
her should have been denied by the RTC for it was not based on any of these
grounds for disqualification.
The RTC rather confused the qualification of the witness with the credibility
and weight of her testimony. Moreover, Section 49, Rule 130 of the Rules of
Evidence is clear that the opinion of an expert witness may be received in
evidence

PEOPLE OF THE PHILIPPINES vs. SALVADOR GOLIMLIM @


"BADONG"
G.R. No. 145225, April 2, 2004
FACTS:
Accused was charged and convicted of the crime of rape committed against
Evelyn G. Canchela (Evelyn), a mental retardate who is the niece of the
accuseds wife. The trial court in convicting the accused gave credence to the
testimony of Evelyn
ISSUE: Whether or not a mental retardate is not qualified to become a witness
due to her mental state.
HELD:
The answer is in the negative. Sections 20 of Rule 130 of the Revised Rules of
Court provides that all persons who can perceive, and perceiving, can make
known their perception to others, may be witnesses. On the other hand, Section
21 of the same rule provides that the following persons cannot be witnesses: (a)
Those whose mental condition, at the time of their production for examination,
is such that they are incapable of intelligently making known their perception
to others; (b) Children whose mental maturity is such as to render them
incapable of perceiving the facts respecting which they are examined and of
relating them truthfully.
That Evelyn is a mental retardate does not disqualify her as a witness nor render
her testimony bereft of truth. A mental retardate or a feebleminded person is
not, per se, disqualified from being a witness, her mental condition not being a
vitiation of her credibility. It is now universally accepted that intellectual
weakness, no matter what form it assumes, is not a valid objection to the

Page | 233

Compilation of Digested Cases for Remedial Law Review II


by: K. M. T. BUELA
competency of a witness so long as the latter can still give a fairly intelligent
and reasonable narrative of the matter testified to.
It cannot then be gainsaid that a mental retardate can be a witness, depending
on his or her ability to relate what he or she knows. If his or her testimony is
coherent, the same is admissible in court.

MAXIMO ALVAREZ vs. SUSAN RAMIREZ


G.R. NO. 143439, October 14, 2005
FACTS:
Accused was charged with arson for setting fire in his sister-in-laws house
while the latters family is inside including the formers estranged wife . During
trial, his wife Esperanza was presented as witness to prove that the accused
Maximo Alvarez committed all the elements of the crime being charged.
Petitioner filed a motion to disqualify Esperanza from testifying against him
pursuant to Rule 130 of the Revised Rules of Court on marital disqualification.
Trial court granted the motion and disqualified Esperanze. The prosecution filed
a motion for reconsideration but was denied in the other assailed Order dated
October 19, 1999. This prompted respondent Susan Ramirez to file with the CA
a Petition for Certiorari. On May 31, 2000, the Appellate Court rendered a
Decision nullifying and setting aside the assailed Orders issued by the trial
court.
ISSUE: Whether or not a wife is disqualified to testify against her husband to
prove a crime committed purposely committed to injure the former.
HELD:
No. Section 22, Rule 130 of the Revised Rules of Court provides that during
their marriage, neither the husband nor the wife may testify for or against the
other without the consent of the affected spouse, except in a civil case by one
against the other, or in a criminal case for a crime committed by one against the
other or the latter's direct descendants or ascendants. The reasons given for the
rule are: (1) There is identity of interests between husband and wife; (2) If one
were to testify for or against the other, there is consequent danger of perjury;
(3) The policy of the law is to guard the security and confidences of private life,
even at the risk of an occasional failure of justice, and to prevent domestic
disunion and unhappiness; and (4) Where there is want of domestic tranquility
there is danger of punishing one spouse through the hostile testimony of the
other.

Page | 234

Compilation of Digested Cases for Remedial Law Review II


by: K. M. T. BUELA
But like all other general rules, the marital disqualification rule has its own
exceptions, both in civil actions between the spouses and in criminal cases for
offenses committed by one against the other. For instance, where the marital
and domestic relations are so strained that there is no more harmony to be
preserved nor peace and tranquility which may be disturbed, the reason
based upon such harmony and tranquility fails. In such a case, identity of
interests disappears and the consequent danger of perjury based on that identity
is non-existent. Likewise, in such a situation, the security and confidences of
private life, which the law aims at protecting, will be nothing but ideals, which
through their absence, merely leave a void in the unhappy home.
The act of private respondent in setting fire to the house of his sister-in-law
Susan Ramirez, knowing fully well that his wife was there, and in fact with the
alleged intent of injuring the latter, is an act totally alien to the harmony and
confidences of marital relation which the disqualification primarily seeks to
protect.

PEOPLE OF THE PHILIPPINES vs. HON. MARIANO


CASTAEDA, JR. and BENJAMIN F. MANALOTO
G.R. No. L-46306, February 27, 1979

C.

FACTS:
Benjamin Manaloto sold the conjugal lot without his wifes consent by forging
the latters signature in a contract of sale. Hence, Benjamin was charged with
falsification of public document by his wife.
At the trial, the prosecution called the complainant-wife to the witness stand but
the defense moved to disqualify her as a witness, invoking the Marital
Disqualification Rule under Sec. 20, Rule 130. The prosecution opposed said
motion to disqualify on the ground that the case falls under the exception to the
rule, contending that it is a "criminal case for a crime committed by one against
the other."
ISSUE: Whether or not a wife may testify against her husband in a criminal
case for falsification of public document done by forging the signature of the
wife a contract of sale of conjugal property.
HELD:
The case is an exception to the marital disqualification rule, as a criminal case
for a crime committed by the accused-husband against the witness wife.

Page | 235

Compilation of Digested Cases for Remedial Law Review II


by: K. M. T. BUELA
The act complained of as constituting the crime of Falsification of Public
Document is the forgery by the accused of his wifes signature in a deed of sale,
thereby making it appear therein that said wife consented to the sale of a house
and lot belonging to their conjugal partnership when in fact and in truth she did
not. It must be noted that had the sale of the said house and lot, and the signing
of the wifes name by her husband in the deed of sale, been made with the
consent of the wife, no crime could have been charged against said husband.
Clearly, therefore, it is the husbands breach of his wifes confidence which
gave rise to the offense charged. And it is this same breach of trust which
prompted the wife to make the necessary complaint with the Office of the
Provincial Fiscal which, accordingly, filed the aforesaid criminal case with the
Court of First Instance of Pampanga. To rule, therefore, that such criminal case
is not one for a crime committed by one spouse against the other is to advance
a conclusion which completely disregards the factual antecedents of the instant
case.

ENRIQUE RAZON vs. INTERMEDIATE APPELLATE COURT and


VICENTE B. CHUIDIAN
G.R. No. 74306, March 16, 1992
FACTS:
Respondent, as administrator of the estate Juan Chuidian, filed an action for
specific performance against petitioner praying that the later be compelled to
deliver the stock certificate representing the share holdings of Juan Chuidian in
E. Razon, Inc. (ERI).
During trial, petitioner testified that: (1) all the shares of stock in the name of
stockholders of record of the corporation were fully paid for by defendant,
Razon; (2) said shares are subject to the agreement between defendants and
incorporators; (3) petitioner distributed shares of stock previously placed in the
names of the withdrawing nominal incorporators to some friends including Juan
T. Chuidian; (4) the shares of stock were actually owned and remained in the
possession of Razon; and (5) Thus, the stock certificate under the name of the
late Chuidian actually belongs to the petitioner with the understanding that he
shall remain in possession thereof until such time as he was paid therefor by the
other nominal incorporators/stockholders.
Trial court ruled that the real owner of the stocks is the petitioner. On appeal,
the CA reversed the decision of the trial court. CA ruled that petitioner is
disqualified from being a witness under the dead mans statute (Section 20 (a)
Rule 130). Petitioner, assailing CAs decision, contends that: (1) dead mans
statute is inapplicable in this case; (2) respondent did not object to his oral
Page | 236

Compilation of Digested Cases for Remedial Law Review II


by: K. M. T. BUELA
testimony; and (3) the petitioner was subjected to a rigid cross examination
regarding such testimony.
ISSUE:
(1) Whether or not dead mans statute disqualifies a defendant from testifying
against the claims of an administrator in relation to a transaction entered into
by the deceased during his lifetime.
(2) Whether or not failure to object to a testimony on the ground of dead mans
statute rule constitutes as a waiver to object to the admissibility of such
testimony.
HELD:
(1)
The reason for the rule is that if persons having a claim against the estate of the
deceased or his properties were allowed to testify as to the supposed statements
made by him (deceased person), many would be tempted to falsely impute
statements to deceased persons as the latter can no longer deny or refute them,
thus unjustly subjecting their properties or rights to false or unscrupulous claims
or demands. The purpose of the law is to guard against the temptation to give
false testimony in regard to the transaction in question on the part of the
surviving party.
The rule, however, delimits the prohibition it contemplates in that it is
applicable to a case against the administrator or its representative of an
estate upon a claim against the estate of the deceased person.
In the instant case, the testimony excluded by the appellate court is that of
petitioner as defendant in an action commenced by the administrator of the
estate of the late Juan Chuidian to recover shares of stock in E. Razon, Inc.
allegedly owned by the late Juan T. Chuidian. It is clear, therefore, that the
testimony of the petitioner is not within the prohibition of the rule. The case was
not filed against the administrator of the estate, nor was it filed upon claims
against the estate.
(2)
Granting that the petitioners testimony is within the prohibition of Section 20
(a), Rule 130 of the Rules of Court, the private respondent is deemed to have
waived the rule. It is also settled that the court cannot disregard evidence which
would ordinarily be incompetent under the rules but has been rendered
admissible by the failure of a party to object thereto.

Page | 237

Compilation of Digested Cases for Remedial Law Review II


by: K. M. T. BUELA
LILIBETH SUNGA-CHAN and CECILIA SUNGA vs. LAMBERTO T.
CHUA
G.R. No. 143340, August 15, 2001
FACTS:
Respondent filed a complaint against petitioners who are the daughter and wife
of the deceased Jacinto L. Sunga, for "Winding Up of Partnership Affairs,
Accounting, Appraisal and Recovery of Shares and Damages with Writ of
Preliminary Attachment. Petitioners filed their Answer with Compulsory
Counterclaims, contending that respondent does not have a cause of action
against them, and that the trial court has no jurisdiction over the nature of the
action, the SEC being the agency that has original and exclusive jurisdiction
over the case. As counterclaim, petitioner sought attorneys fees and expenses
of litigation.
During trial Respondent testified that: (1) in 1977, he verbally entered into a
partnership with Jacinto in the distribution of LPG in Manila; (2) for business
convenience, respondent and Jacinto allegedly agreed to register the business
name of their partnership under the name of Jacinto as a sole proprietorship; (3)
upon Jacintos death, petitioners took over the operations and control of the
partnerhsip without respondents consent; (4) despite respondents repeated
demands upon petitioners for accounting, inventory, appraisal, winding up and
restitution of his net shares in the partnership, petitioners failed to comply; and
(5) on March 31, 1991, petitioner disbursed out of the partnership funds the
amount of P200,000.00 representing partial payment of the formers share in
the partnership, with a promise that the former would make the complete
inventory and winding up of the properties of the business establishment.
The trial court, giving weight to Respondents testimony which was
corroborated by another witness who work with the deceased during his
lifetime, ruled in favor of the Respondent. On appeal, the CA affirmed the trial
courts decision. Petitioners assails the finding of the trial court and the CA and
argued that these courts were proscribed from hearing the testimonies of
respondent and his witness, Josephine, to prove the alleged partnership three
years after Jacintos death. To support this argument, petitioners invoke the
"Dead Mans Statute" or "Survivorship Rule" under Section 23, Rule 130 of the
Rules of Court.
ISSUE: Whether or not dead mans statute applies to a complainant/witness
who is also a defendant due to counterclaim of the original defendant.
HELD:

Page | 238

Compilation of Digested Cases for Remedial Law Review II


by: K. M. T. BUELA
The answer is in the negative. The "Dead Mans Statute" provides that if one
party to the alleged transaction is precluded from testifying by death, insanity,
or other mental disabilities, the surviving party is not entitled to the undue
advantage of giving his own uncontradicted and unexplained account of the
transaction. But before this rule can be successfully invoked to bar the
introduction of testimonial evidence, it is necessary that: (1) the witness is a
party or assignor of a party to a case or persons in whose behalf a case is
prosecuted; (2) the action is against an executor or administrator or other
representative of a deceased person or a person of unsound mind; (3) the
subject-matter of the action is a claim or demand against the estate of such
deceased person or against person of unsound mind; (4) the testimony refers to
any matter of fact which occurred before the death of such deceased person or
before such person became of unsound mind.
Two reasons forestall the application of the "Dead Mans Statute" to this case.
First, petitioners filed a compulsory counterclaim against respondent in their
answer before the trial court, and with the filing of their counterclaim,
petitioners themselves effectively removed this case from the ambit of the
"Dead Mans Statute." Well entrenched is the rule that when it is the
executor or administrator or representatives of the estate that sets up the
counterclaim, the plaintiff, herein respondent, may testify to occurrences
before the death of the deceased to defeat the counterclaim. Moreover, as
defendant in the counterclaim, respondent is not disqualified from testifying as
to matters of fact occurring before the death of the deceased, said action not
having been brought against but by the estate or representatives of the deceased.
Second, the testimony of Josephine is not covered by the "Dead Mans Statute"
for the simple reason that she is not "a party or assignor of a party to a case or
persons in whose behalf a case is prosecuted." Records show that respondent
offered the testimony of Josephine to establish the existence of the partnership
between respondent and Jacinto. Petitioners insistence that Josephine is the
alter ego of respondent does not make her an assignor because the term
"assignor" of a party means "assignor of a cause of action which has arisen, and
not the assignor of a right assigned before any cause of action has arisen."
Plainly then, Josephine is merely a witness of respondent, the latter being the
party plaintiff.

Page | 239

Compilation of Digested Cases for Remedial Law Review II


by: K. M. T. BUELA
TERESITA P. BORDALBA vs. COURT OF APPEALS, HEIRS OF
NICANOR JAYME
G.R. No. 112443, January 25, 2002
FACTS:
Lot 1242 was originally owned by the late spouses Carmeno Jayme and
Margarita Espina de Jayme. In 1947, the property was extraj-judicially
partitioned in the following manner: 1/3 to their grandchild Nicanor Jayme; 1/3
to their daughter Elena Jayme Vda. de Perez; and 1/3 to an unidentified party.
Petitioner, daughter of Elena, filed an application for issuance of a Free Patent
over the same lot 1242. When the application was granted and corresponding
OCT was issued, petitioner subdivided the property into 6 lots and disposed the
two parcels thereof. Upon learning of the issuance of the Free Patent and OCT,
as well as the conveyances made by petitioner, respondents filed with RTC the
instant complaint for annulment and cancellation of the Free Patent and OCT
against petitioner and purchasers.
Petitioner averred that Lot No. 1242 was acquired by her through purchase from
her mother, who was in possession of the lot in the concept of an owner since
1947. However, on cross-examination, petitioner admitted that the existence of
the above-mentioned Deed of Extrajudicial Partition. She, however, identified
one of the signatures in the said Deed to be the signature of her mother.
The trial court, giving weight on the testimony of witnesses as to the existence
of the extra-judicial partition and finding that fraud was employed by petitioner
in obtaining Free Patent and OCT, declared said patent and title void and
ordered its cancellation. CA affirmed with modification the decision of the trial
court. Thus, petitioner filed the instant petition, assailing the decision of the CA.
Petitioner contends that the testimonies given by the witnesses for private
respondents which touched on matters occurring prior to the death of her mother
should not have been admitted by the trial court, as the same violated the dead
mans statute.
ISSUE: Whether Dead Mans Statute applies to disqualify the testimony of all
witnesses attesting to the existence of an agreement entered into by the deceased
during her lifetime.
HELD:
Dead Mans Statute finds no application in the present case. The dead mans
statute does not operate to close the mouth of a witness as to any matter of fact
coming to his knowledge in any other way than through personal dealings with
the deceased person, or communication made by the deceased to the witness.
Page | 240

Compilation of Digested Cases for Remedial Law Review II


by: K. M. T. BUELA
Since the claim of private respondents and the testimony of their witnesses in
the present case is based, inter alia, on the 1947 Deed of Extra-judicial Partition
and other documents, and not on dealings and communications with the
deceased, the questioned testimonies were properly admitted by the trial court.

JOSIELENE LARA CHAN vs. JOHNNY T. CHAN


G.R. No. 179786, July 24, 2013
FACTS:
Josielene Lara Chan filed a petition for the declaration of nullity of her marriage
to respondent Johnny Chan. During the pre-trial conference, Josielene premarked the Philhealth Claim Form1 that Johnny attached to his answer as proof
that he was forcibly confined at the rehabilitation unit of a hospital. The form
carried a physicians handwritten note that Johnny suffered from
methamphetamine and alcohol abuse. Following up on this point, on August
22, 2006 or before trial, Josielene filed with the RTC a request for the issuance
of a subpoena duces tecum addressed to Medical City, covering Johnnys
medical records when he was there confined. The request was accompanied by
a motion to be allowed to submit in evidence the records sought by subpoena
duces tecum.
Johnny opposed the motion, arguing that the medical records were covered by
physician-patient privilege. RTC sustained the opposition and denied
Josielenes motion. Josielene of course claims that the hospital records subject
of this case are not privileged since it is the testimonial evidence of the
physician that may be regarded as privileged. Section 24(c) of Rule 130 states
that the physician cannot in a civil case, without the consent of the patient, be
examined regarding their professional conversation. The privilege, says
Josielene, does not cover the hospital records, but only the examination of the
physician at the trial.
ISSUE:
(1) Whether or not a motion for issuance of subpoena duces tecum may be done
prior to the trial.
(2) Whether or not hospital records may not be the subject of a subpoena duces
tecum before trial on the ground of privilege communication.
HELD:
(1)
Yes. It is of course possible to treat Josielenes motion for the issuance of a
subpoena duces tecum covering the hospital records as a motion for production
of documents, a discovery procedure available to a litigant prior to trial. Section
Page | 241

Compilation of Digested Cases for Remedial Law Review II


by: K. M. T. BUELA
1, Rule 27 of the Rules of Civil Procedure. But the right to compel the
production of documents has a limitation: the documents to be disclosed are
not privileged.
Considering that hospital records may not be a subject for motion for production
of documents, it can only be offered during trial. Hence, Josielenes request for
subpoena duces tecum is premature. She will have to wait for trial to begin
before making a request for the issuance of a subpoena duces tecum covering
Johnnys hospital records. It is when those records are produced for examination
at the trial, that Johnny may opt to object, not just to their admission in evidence,
but more so to their disclosure.
(2)
The answer is in the affirmative. The physician-patient privileged
communication rule essentially means that a physician who gets information
while professionally attending a patient cannot in a civil case be examined
without the patients consent as to any facts which would blacken the latters
reputation. This rule is intended to encourage the patient to open up to the
physician, relate to him the history of his ailment, and give him access to his
body, enabling the physician to make a correct diagnosis of that ailment and
provide the appropriate cure. Any fear that a physician could be compelled in
the future to come to court and narrate all that had transpired between him and
the patient might prompt the latter to clam up, thus putting his own health at
great risk.
Disclosing hospital records would be the equivalent of compelling the physician
to testify on privileged matters he gained while dealing with the patient, without
the latters prior consent.
To allow, however, the disclosure during discovery procedure of the hospital
recordsthe results of tests that the physician ordered, the diagnosis of the
patients illness, and the advice or treatment he gave him would be to allow
access to evidence that is inadmissible without the patients consent. Physician
memorializes all these information in the patients records.

JUDGE UBALDINO A. LACUROM v. ATTY. ELLIS F. JACOBA and


ATTY. OLIVIA VELASCO-JACOBA
A.C. NO. 5921, March 10, 2006
FACTS:
Judge Lacurom filed the present complaint against respondents before the
Integrated Bar of the Philippines. The antecedent facts are as follows: (1)
Page | 242

Compilation of Digested Cases for Remedial Law Review II


by: K. M. T. BUELA
respondents law firm acts as the counsel in the unlawful detainer case appealed
to the sala of Judge Lacurom; (2) Judge Lacurom reversed the decision of the
MTC and ruled against the counsels client; (3) the losing party filed an MR
signed by Velasco-Jacoba; (4) the MR contains scathing remarks against the
Judge; (5) Judge Lacurom ordered Velasco-Jacoba to appear before his sala and
explain why she should not be held in contempt of court for the contents of her
motion; (6) Velasco-Jacoba explained that she is not the author of the motion as
she merely signed the same as it was the practice between her and her husband,
her co-counsel in the law firm; (7) Judge Lacurom found Velasco-Jacoba guilty
of contempt; (8) Velasco-Jacoba filed a petition for certiorari assailing the
decision of Judge Lacurom; (9) Judge Lacurom issued another order directing
Jacoba (husband of Velasco-Jacoba) to explain why he should not be held in
contempt; (10) Jacoba, in his answer, denied that he prepared the motion; (11)
as to against Velasco-Jacoba's statements implicating him, Jacoba invoked the
marital privilege rule in evidence; (12) Judge Lacurom later rendered a decision
finding Jacoba guilty of contempt of court.
ISSUE: Whether or not marital privilege rule may be invoked against a
statement made by one spouse against the other without actually denying the
contents of the statement.
HELD:
The answer is in the negative. Jacobas Answer with Second Motion for
Inhibition did not contain a denial of his wife's account. Instead, Jacoba
impliedly admitted authorship of the motion.
The marital privilege rule, being a rule of evidence, may be waived by failure
of the claimant to object timely to its presentation or by any conduct that may
be construed as implied consent. This waiver applies to Jacoba who impliedly
admitted authorship of the 30 July 2001 motion.

CLARITA J. SAMALA vs. ATTY. LUCIANO D. VALENCIA


A.C. No. 5439, January 22, 2007
FACTS:
Samala filed a complaint for disbarment against Atty. Luciano D. Valencia for
serving on two separate occasions as counsel for contending parties. Records
show that Valencia acted as counsel for Valdez in three separate cases. In of the
cases Valencia represented Valdez and Alba against Bayuga and Bustamante.
However, on a subsequent estafa case between Valdez and Alba, Valencia
represented Valdez against Alba his previous client.

Page | 243

Compilation of Digested Cases for Remedial Law Review II


by: K. M. T. BUELA
In his defense Valencia, respondent, avers that he already severed his
representation for Alba when the latter charged respondent with estafa.
ISSUE: Whether or not the termination of lawyer-client relationship allows a
counsel to act as counsel against a previous client.
HELD:
The answer is in the negative. The termination of the relation of attorney and
client provides no justification for a lawyer to represent an interest adverse to
or in conflict with that of the former client. The reason for the rule is that the
client's confidence once reposed cannot be divested by the expiration of the
professional employment. Consequently, a lawyer should not, even after the
severance of the relation with his client, do anything which will injuriously
affect his former client in any matter in which he previously represented him
nor should he disclose or use any of the client's confidences acquired in the
previous relation.
The stern rule against representation of conflicting interests is founded on
principles of public policy and good taste. It springs from the attorney's duty
to represent his client with undivided fidelity and to maintain inviolate the
client's confidence as well as from the injunction forbidding the
examination of an attorney as to any of the privileged communications of
his client.

COMMISSIONER JOSE T. ALMONTE et al. vs. HONORABLE


CONRADO M. VASQUEZ and CONCERNED CITIZENS
G.R. No. 95367, May 23, 1995
FACTS:
Pursuant to his investigation of an anonymous letter alleging that funds
representing savings from unfilled positions in the EIIB had been illegally
disbursed, Ombudsman issued a subpoena duces tecum requiring petitioners as
chief accountant and record custodian of the Economic Intelligence and
Investigation Bureau (EIIB) to produce "all documents relating to Personal
Services Funds for the year 1988" and all evidence such as vouchers from
enforcing his orders.
Petitioners do not question the power of the Ombudsman to issue a subpoena
duces tecum nor the relevancy or materially of the documents required to be
produced, to the pending investigation in the Ombudsman's office. Petitioners
claimed that they cannot be ordered to produce documents relating to personal
services and salary vouchers of EIIB employees on the plea that such documents
Page | 244

Compilation of Digested Cases for Remedial Law Review II


by: K. M. T. BUELA
are classified. Disclosure of the documents in question is resisted on the ground
that "knowledge of EIIB's documents relative to its Personal Services Funds and
its plantilla . . . will necessarily lead to knowledge of its operations, movements,
targets, strategies, and tactics and the whole of its being" and this could "destroy
the EIIB."
ISSUE: Whether or not privilege communication may be invoked against a
subpoena duces tecum enjoining the production of records relating to personal
services funds on the ground that they are state secrets.
HELD:
For information to be accorded confidentiality on the ground that such are state
secrets, the necessity of according such treatment must be shown.
Governmental privilege against disclosure is recognized with respect to state
secrets bearing on military, diplomatic and similar matters. This privilege is
based upon public interest of such paramount importance as in and of itself
transcending the individual interests of a private citizen, even though, as a
consequence thereof, the plaintiff cannot enforce his legal rights.
Where there is a strong showing of necessity, the claim of privilege should not
be lightly accepted, but even most compelling necessity cannot overcome the
claim of privilege if the court is ultimately satisfied that military secrets are at
stake. A fortiori, where necessity is dubious, a formal claim of privilege, made
under the circumstances of this case, will haw to prevail.
Where the claim of confidentiality does not rest on the need to protect military,
diplomatic or other national security secrets but on a general public interest in
the confidentiality of his conversations, courts have declined to find in the
Constitution an absolute privilege of the President against a subpoena
considered essential to the enforcement of criminal laws.
In the case at bar, there is no claim that military or diplomatic secrets will be
disclosed by the production of records pertaining to the personnel of the EIIB.
Indeed, EIIB's function is the gathering and evaluation of intelligence reports
and information regarding "illegal activities affecting the national economy,
such as, but not limited to, economic sabotage, smuggling, tax evasion, dollar
salting." Consequently, while in cases which involve state secrets it may be
sufficient to determine from the circumstances of the case that there is
reasonable danger that compulsion of the evidence will expose military
matters without compelling production, no similar excuse can be made for
a privilege resting on other considerations unless it falls under statutorily-

Page | 245

Compilation of Digested Cases for Remedial Law Review II


by: K. M. T. BUELA
created ones such as the Government's privilege to withhold the identity of
persons who furnish information of violations of laws.

c. Testimonial Privilege

d. Admissions
OSCAR CONSTANTINO et al. vs. HEIRS OF CONSTANTINO
G.R. No. 181508, October 02, 2013
FACTS:
In this case, there are two (2) deed of extrajudicial settlement involving estate
properties of Pedro Constantino, Sr., i.e., one in 1968 involving the 192 sqm
and another in 1992 involving the 240 sqm. The separate Deeds came into being
out of an identical intention of the signatories in both to exclude their co-heirs
of their rightful share in the entire estate of Pedro Sr.
Respondent, who are grandchildren of Pedro Sr. from Pedro Jr., filed a
complaint seeking to annul the 1992 extrajudicial settlement involving the
240sqm lot on the ground that they, who are also heirs of Pedro Sr., were
excluded thereto. On the other hand, Petitioners alleged that the respondents
have no cause of action against them considering that the respondents already
have their lawful share over the estate of Pedro Sr. by virtue of the 1968 Deed
of Extrajudicial Settlement with Waiver. During the pre-trial, respondents
admitted that they executed the 1968 Deed to partition the 192 sqm which is the
share of their predecessor Pedro Jr., in Pedro Sr.s Estate.
RTC rendered a Decision finding both plaintiffs and defendants in pari delicto.
On appeal, CA rule in favor of respondent and declared that the 1968 Deed
covering the 192 sq m lot which actually belongs to Pedro Jr., hence, not part
of the estate of Pedro Sr. Hence, heirs of Pedro Jr. (herein respondent), did not
adjudicate the 192 sqm lot unto themselves to the exclusion of all the other heirs
of Pedro Sr. Petitioners now assails the erroneous disregard by the CA of
stipulations and admissions during the pre-trial conference
ISSUE: Whether or not admissions made during pre-trial are binding upon the
parties.
HELD:
The answer is in the affirmative. Judicial admissions are legally binding on the
party making the admissions. Pre-trial admission in civil cases is one of the
instances of judicial admissions explicitly provided for under Section 7, Rule
Page | 246

Compilation of Digested Cases for Remedial Law Review II


by: K. M. T. BUELA
18 of the Rules of Court, which mandates that the contents of the pre-trial order
shall control the subsequent course of the action, thereby, defining and limiting
the issues to be tried. A party who judicially admits a fact cannot later challenge
the fact as judicial admissions are a waiver of proof; production of evidence is
dispensed with.
However, the general rule regarding conclusiveness of judicial admission upon
the party making it and the dispensation of proof admits of two exceptions: 1)
when it is shown that the admission was made through palpable mistake, and 2)
when it is shown that no such admission was in fact made. The latter exception
allows one to contradict an admission by denying that he made such an
admission. However, respondents failed to refute the earlier
admission/stipulation before and during the trial.

CONRADO C. DOLDOL vs. PEOPLE OF THE PHILIPPINES and


THE HONORABLE COURT OF APPEALS
G.R. NO. 164481, September 20, 2005
FACTS:
Provincial Auditor conducted an audit of the cash and cash account of Conrado
C. Doldol, the Municipal Treasurer of Urbiztondo, Pangasinan. The State
Auditors discovered that Doldol had a shortage of P801,933.26. The State
Auditors submitted their Report to the Provincial Auditor on their examinations
showing his shortages. On the same day, Doldol wrote the Provincial Treasurer
requesting that a re-audit be conducted on his cash and cash account, taking
exception to the findings of the State Auditors.
Instead of pursuing his request for a re-audit, Doldol opted to refund the missing
funds. On September 15, 1995, he remitted P200,000.00 to the Acting
Municipal Treasurer for which he was issued Official Receipt No. 436756.
Doldol promised to pay the balance of his shortage, as follows: P200,000.00 on
October 31, 1995, and P884,139.66 on or before November 30, 1995. However,
he reneged on his promise.
Two informations for malversation of public funds were then filed against
Doldol in the Regional Trial Court (RTC) of San Carlos City. Doldol was
convicted.
ISSUE: Whether or not, person convicted of malversation may assail his
conviction when he had already partially paid the alleged shortage.
HELD:
Page | 247

Compilation of Digested Cases for Remedial Law Review II


by: K. M. T. BUELA
The said payment, particularly when taken in conjunction with appellant's
commitment to gradually pay the remainder of the missing funds, is a clear offer
of compromise which must be treated as an implied admission of appellant's
guilt that he embezzled or converted the missing funds to his personal use.

e. Confessions
JOSUE R. LADIANA, Petitioner, v. PEOPLE OF THE PHILIPPINES
G.R. No. 144293, December 4, 2002
FACTS:
Accused, a member of the Integrated National Police (now PNP), was charged
with murder before the Sandiganbayan (SB) for the death of Fancisco San Juan.
During the trial, Cortez, the prosecutor who conducted the preliminary
investigation, testified that the accused executed before him a counter-affidavit
admitting the commission of the crime. Before Cortez was presented as witness,
Defense counsel made an admission as to the authorship, authenticity, and
voluntariness of the execution of the counter-affidavit of accused Ladiana,
which was subscribed and sworn to before Cortez. However, Accused Ladiana
allegedly did so in self-defense.
The same counter-affidavit became the basis of SB in convicting the accused.
The court a quo held that his Counter-Affidavit, in which he had admitted to
having fired the fatal shots that caused the victims death, may be used as
evidence against him.
On appeal with the SC, petitioner argued that the counter-affidavit cannot be
considered an extrajudicial confession as the same was executed during
custodial investigation with the assistance of a counsel.
ISSUE: Whether or not the admission of the commission of an offense while
invoking self-defense in a Counter-affidavit executed during preliminary
investigation without the assistance of a counsel may be admitted as an
extrajudicial confession against him.
HELD:
The answer is in the negative. It is only an admission. Sections 26 and 33 of
Rule 130 of the Revised Rules on Evidence distinguish one from the other as
follows:
"SEC. 26. Admissions of a party. The act, declaration or omission of a party
as to a relevant fact may be given in evidence against him.
Page | 248

Compilation of Digested Cases for Remedial Law Review II


by: K. M. T. BUELA

"SEC. 33. Confession. The declaration of an accused acknowledging his guilt


of the offense charged, or of any offense necessarily included therein, may be
given in evidence against him."
In a confession, there is an acknowledgment of guilt; in an admission, there is
merely a statement of fact not directly involving an acknowledgment of guilt or
of the criminal intent to commit the offense with which one is charged. 26 Thus,
in the case at bar, a statement by the accused admitting the commission of the
act charged against him but denying that it was done with criminal intent is an
admission, not a confession.
Petitioner admits shooting the victim which eventually led to the latters
death but denies having done it with any criminal intent. In fact, he claims
he did it in self-defense. Nevertheless, whether categorized as a confession or
as an admission, it is admissible in evidence against him as the voluntariness of
the execution thereof was admitted by the defense.

THE PEOPLE OF THE PHILIPPINES vs. FELICIANO ULIT y


TAMPOY
G.R. NOS. 131799-801, February 23, 2004
FACTS:
Upon the sworn complaint of the victim Lucelle Serrano, four Information were
filed against her uncle, the appellant for qualified rape.
The records also show that the appellant executed a Sinumpaang Salaysay while
detained at the barangay hall where he confessed to having raped the victim in
February 1997 and March 2, 1997. However, the trial court did not ask the
appellant whether he was assisted by counsel when he was brought to the Office
of the Public Prosecutor for inquest investigation. Neither did the court a quo
inquire about the circumstances and the appellants reasons for refusing to
execute the said waiver.
The records show that when the prosecution offered the appellants Sinumpaang
Salaysay in evidence to prove that he confessed to having raped the victim in
February 1997 and March 2, 1997, the appellant objected thereto on the ground
that he was not assisted by counsel and that he was coerced into signing the
same.

Page | 249

Compilation of Digested Cases for Remedial Law Review II


by: K. M. T. BUELA
The trial court convicted the appellant of rape on the basis of Lucelles sworn
statement, the testimony of her mother, the appellants statement executed in the
Barangay Chairmans Office, and the testimony of Dr. Armie Soreta-Umil.
ISSUE: Whether or not an admission made before a Barangay Chairman
without the assistance of a counsel may be used against the accuse.
HELD:
Although the appellant was not assisted by counsel at the time he gave his
statement to the barangay chairman and when he signed the same, it is still
admissible in evidence against him because he was not under arrest nor under
custodial investigation when he gave his statement.
The exclusionary rule is premised on the presumption that the defendant is
thrust into an unfamiliar atmosphere and runs through menacing police
interrogation procedures where the potentiality for compulsion, physical and
psychological, is forcefully apparent. As intended by the 1971 Constitutional
Convention, this covers investigation conducted by police authorities which will
include investigations conducted by the municipal police, the PC and the NBI
and such other police agencies in our government. The barangay chairman is
not deemed a law enforcement officer for purposes of applying Section 12(1)
and (3) of Article III of the Constitution. Under these circumstances, it cannot
be successfully claimed that the appellants statement before the barangay
chairman is inadmissible.

PEOPLE OF THE PHILIPPINES vs. BENJAMIN SAYABOC y


SEGUBA et al.
G.R. No. 147201, January 15, 2004
FACTS:
Herein accused were charged with murder. He was arrested pursuant to the
testimony of two (2) eyewitnesses. During his custodial investigation the
accused made an extrajudicial confession after the following was recited to him:
I would like to inform you Mr. Sayaboc that questions will be asked to you
regarding an incident last December 2, 1994 at the Rooftop, Brgy. Quezon,
Solano, Nueva Vizcaya, in connection with the shooting of Joseph Galam,
owner of the said Disco House as a result of his death. Before questions will be
asked of you I would like to inform you about your ri[g]hts under the new
Constitution of the Philippines, as follows: That you have the right to remain
silent or refuse to answer the questions which you think will incriminate you;

Page | 250

Compilation of Digested Cases for Remedial Law Review II


by: K. M. T. BUELA
That you have the right to seek the services of a counsel of your own choice or
if not, this office will provide you a lawyer if you wish.
The appellants argue that the extrajudicial confession of Sayaboc may not be
admitted in evidence against him because Atty. Cornejo, the PAO lawyer who
was his counsel during the custodial investigation, was not a competent,
independent, vigilant, and effective counsel. He was ineffective because he
remained silent during the entire proceedings.
ISSUE:
(1) Whether or not an extrajudicial confession made during custodial
investigation wherein the rights of the accused were merely recited to him
may be admissible in evidence against such accused.
(2) Whether a counsel who remain silent at the time the extrajudicial confession
was made qualifies as an independent counsel.
HELD:
(1)
The answer is in the negative. The showing of a spontaneous, free, and
unconstrained giving up of a right is missing.
The right to be informed requires "the transmission of meaningful information
rather than just the ceremonial and perfunctory recitation of an abstract
constitutional principle." It should allow the suspect to consider the effects and
consequences of any waiver he might make of these rights. More so when the
suspect is one like Sayaboc, who has an educational attainment of Grade IV,
was a stranger in Nueva Vizcaya, and had already been under the control of the
police officers for two days previous to the investigation, albeit for another
offense.
(2)
A counsel who remains silent all through-out the investigation shows that there
is lacking of a faithful attempt at each stage of the investigation to make
Sayaboc aware of the consequences of his actions.
The right to a competent and independent counsel means that the counsel should
satisfy himself, during the conduct of the investigation, that the suspect
understands the import and consequences of answering the questions
propounded. Counsel should be able, throughout the investigation, to explain
the nature of the questions by conferring with his client and halting the
investigation should the need arise. The duty of a lawyer includes ensuring that
the suspect under custodial investigation is aware that the right of an accused to
remain silent may be invoked at any time.
Page | 251

Compilation of Digested Cases for Remedial Law Review II


by: K. M. T. BUELA

CARLOS L. TANENGGEE vs. PEOPLE OF THE PHILIPPINES


G.R. No. 179448, June 26, 2013
FACTS:
Petitioner was charged with estafa through falsification of commercial
documents. The prosecution alleged that: (1) the petitioner as branch manager
caused the preparation of promissory notes (PN) and cashiers check in the
name of one of their valued client; (2) that by forging the signature of such
client, petitioner was able to obtain the proceeds of the loan evidenced by the
PN.
After the discovery of the irregular loans, an internal audit was conducted and
an administrative investigation was held in the Head Office of Metrobank,
during which appellant signed a written statement in the form of questions and
answers admitting the commission of the allegations in the Information. Trial
court convicted the accused. On appeal, the CA affirmed his conviction.
Elevating the case before the SC, petitioner avers that the written statement
should not be admitted as evidence against him as it was taken in violation of
his rights under Section 12, Article III of the Constitution, particularly of his
right to remain silent, right to counsel, and right to be informed of the first two
rights. Hence, the same should not have been admitted in evidence against him.
ISSUE: Whether or not a statement of an accused-employee made during
administrative investigation conducted by his employer may not be admitted an
evidence against the former on the ground that it was made without the
assistance of a counsel.
HELD:
The answer is in the negative. The constitutional proscription against the
admissibility of admission or confession of guilt obtained in violation of Section
12, Article III of the Constitution is applicable only in custodial interrogation.
Custodial interrogation means any questioning initiated by law enforcement
authorities after a person is taken into custody or otherwise deprived of his
freedom of action in any significant manner.
In the present case, while it is undisputed that petitioner gave an uncounselled
written statement regarding an anomaly discovered in the branch he managed,
the following are clear: (1) the questioning was not initiated by a law
enforcement authority but merely by an internal affairs manager of the bank;
and, (2) petitioner was neither arrested nor restrained of his liberty in any
significant manner during the questioning. Clearly, petitioner cannot be said to
Page | 252

Compilation of Digested Cases for Remedial Law Review II


by: K. M. T. BUELA
be under custodial investigation and to have been deprived of the constitutional
prerogative during the taking of his written statement.
No error can therefore be attributed to the courts below in admitting in evidence
and in giving due consideration to petitioners written statement as there is no
constitutional impediment to its admissibility.

f. Conduct and Character


PEOPLE OF THE PHILIPPINES vs. RAUL SANTOS y NARCISO
G.R. Nos. 100225-26, May 11, 1993
FACTS:
Santos was charged and convicted of murder and frustrated murder. On appeal,
accused makes the following assignment of errors: (1) his identification in the
police line-up by the two witnesses is inadmissible as he was not afforded his
right to counsel; (2) he questions the trial court for admitting a sworn statement
by one Ronaldo Guerrero, a witness in another criminal case accused was also
charged with the murder which had taken place in the very same site where
Bautista and Cupcupin were ambushed as such accused contends that the
affidavit of Ronaldo Guerrero was hearsay evidence, considering that the
prosecution did not present Ronaldo Guerrero as a witness during the trial.
ISSUE:
(1) Whether or not identification in the police line-up is not admissible on the
ground that the accused was not provided with a counsel.
(2) Whether the trial court may not admit a sworn statement of a witness taken
from another criminal case wherein the accused for both cases are one and
the same.
HELD:
(1)
There is "no real need to afford a suspect the service of counsel at police lineup. The customary practice is, of course, that it is the witness who is investigated
or interrogated in the course of a police line-up and who gives a statement to
the police, rather than the accused who is not questioned at all at that stage. In
the instant case, there is nothing in the record of this case which shows that in
the course of the line-up, the police investigators sought to extract any
admission or confession from appellant Santos.
(2)
Section 34, Rule 130 of the Rules of Court provides that Evidence that one did
or did not do a certain thing at one time is not admissible to prove that he did or
Page | 253

Compilation of Digested Cases for Remedial Law Review II


by: K. M. T. BUELA
did not do the same or a similar thing at another time; but it may be received to
prove a specific intent or knowledge, identity, plan, system, scheme, habit,
custom or usage and the like."
Trial court did not commit reversible error in admitting the Guerrero affidavit
for the limited purpose for proving knowledge or plan or scheme, and more
specifically, that appellant knew that the particular corner of two (2) particular
streets in Malabon was a good place to ambush a vehicle and its passengers. As
in fact, both in the instant case, as well as the case where Guerreros testimony
was originally presented, the scene of the crime is one and the same.

PEOPLE OF THE PHILIPPINES vs. ALFREDO NARDO y ROSALES


G.R. No. 133888, March 1, 2001
FACTS:
Herein accused was charged of raping his eldest daughter. The prosecution
presented the victim as its main witness, while, the defense presented a number
of witnesses who testified to different occasion for which the victim was caught
lying.
After the trial, trial court, giving credence to the testimony of the victim,
convicted the accused. On appeal, the accused presented series of letters
allegedly written by the victim to the defense counsel asking said counsel to
help her father be acquitted.
ISSUE:
(1) Whether or not a minor witness credibility may be assailed by proving that
she lies on a number of occasion.
(2) Whether or not letters written by the witness after trial containing details
that is contrary to testimony made in open court constitutes recantation of
said testimony.
HELD:
(1)
The answer is in the negative. Rule 130, Section 34, of the Rules of Court
provides that: "Evidence that one did or did not do a certain thing at one time is
not admissible to prove that he did nor did not do the same or a similar thing at
another time; but it may be received to prove a specific intent or knowledge,
identity, plan, system, scheme, habit, custom or usage, and the like." While
lying may constitute a habit, we believe that the falsehoods committed by
Lorielyn, assuming them for the moment to be true, are petty and

Page | 254

Compilation of Digested Cases for Remedial Law Review II


by: K. M. T. BUELA
inconsequential. They are not as serious as charging ones own father of the
sordid crime of rape, with all of its serious repercussions.
Furthermore, as a rule, findings by the trial court on the credibility of witnesses
are not to be disturbed, for the trial court is in a better position to pass upon the
same. Lastly, jurisprudence dictates that testimonies of child-victims are given
full weight and credit, since when a woman, more so if she is a minor, says that
she has been raped, she says in effect all that is necessary to show that rape was
committed. Youth and immaturity are generally badges of truth and sincerity.
(2)
An affidavit of recantation, being usually taken ex parte, would be considered
inferior to the testimony given in open court. It would be a dangerous rule to
reject the testimony taken before a court of justice simply because the witness
who gave it later on changed his/her mind for one reason or another. Such a rule
would make a solemn trial a mockery, and place the proceedings at the mercy
of unscrupulous witnesses.
Recantations are frowned upon by the courts because they can easily be
obtained from witnesses through intimidation or for monetary consideration.
A retraction does not necessarily negate an earlier declaration. Especially,
recantations made after the conviction of the accused deserve only scant
consideration.

REPUBLIC OF THE PHILIPPINES vs. HEIRS OF FELIPE ALEJAGA


SR.
G.R. No. 146030, December 3, 2002
FACTS:
Respondent Felipe Alejaga, Sr. filed Free Patent Application. In relation to the
said application, Recio, Land Inspector, submitted a report of his investigation
to the Bureau of Lands. Less than 3 months after the application, free patent was
issued.
Sometime in April 1979, the heirs of Ignacio Arrobang requested for an
investigation for irregularities in the issuance of the title of a foreshore land in
favor of respondent. Isagani Cartagena, Supervising Special Investigator,
submitted his Report. In that report, Recio supposedly admitted that he had not
actually conducted an investigation and ocular inspection of the parcel of land.
Thereafter, government instituted an action for Annulment/Cancellation of
Patent and Title and Reversion against respondent.

Page | 255

Compilation of Digested Cases for Remedial Law Review II


by: K. M. T. BUELA
Trial court ruled in favor of the petitioner. In reversing the RTC, CA brushed
aside as hearsay Isagani Cartagenas testimony that Land Inspector Efren L.
Recio had not conducted an investigation on the free patent application of Felipe
Alejaga Sr.
ISSUE: Whether or not testimony based on a report which relates an admission
of a third person who was not presented as witness is inadmissible in evidence
for being a hearsay.
HELD:
The answer is in the negative. A witness may testify as to the state of mind of
another person the latters knowledge, belief, or good or bad faith and the
formers statements may then be regarded as independently relevant without
violating the hearsay rule. Recios alleged admission may be considered as
"independently relevant."
Thus, because Cartagena took the witness stand and opened himself to crossexamination, the Investigation Report he had submitted to the director of the
Bureau of Lands constitutes part of his testimony. Those portions of the report
that consisted of his personal knowledge, perceptions and conclusions are not
hearsay. On the other hand, the part referring to the statement made by Recio
may be considered as independently relevant.
The doctrine on independently relevant statements holds that conversations
communicated to a witness by a third person may be admitted as proof that,
regardless of their truth or falsity, they were actually made. Evidence as to the
making of such statements is not secondary but primary, for in itself it (a)
constitutes a fact in issue or (b) is circumstantially relevant to the existence of
such fact.
Since Cartagenas testimony was based on the report of the investigation he had
conducted, his testimony was not hearsay and was, hence, properly admitted by
the trial court.

g. Hearsay Evidence Rule


ANNA LERIMA PATULA, vs. PEOPLE OF THE PHILIPPINES
G.R. No. 164457, April 11, 2012
FACTS:
Petitioner, a sales representative at Footluckers Chain of Stores, was charged
with estafa for failure to account for the proceeds of the sales and deliver the
collection to the said company.
Page | 256

Compilation of Digested Cases for Remedial Law Review II


by: K. M. T. BUELA

During the trial, prosecution, in order to prove that collectibles lawfully


belonging to the company where misappropriated by the accused, submitted the
following documentary evidence: (a) the receipts allegedly issued by petitioner
to each of her customers upon their payment, (b) the ledgers listing the accounts
pertaining to each customer with the corresponding notations of the receipt
numbers for each of the payments, and (c) the confirmation sheets accomplished
by Guivencan herself. The ledgers and receipts were marked and formally
offered as Exhibits B to YY, and their derivatives, inclusive. Prosecution also
presented Guivencan to testify on the entries in the documentary evidence.
Petitioners counsel interposed a continuing objection on the ground that the
figures entered in Exhibits B to YY and their derivatives, inclusive, were
hearsay because the persons who had made the entries were not themselves
presented in court.
ISSUE: Whether or not testimony of a witness pertaining to entries in a
document made by another person constitutes hearsay and may not be admitted
as evidence.
HELD:
Section 36 of Rule 130, Rules of Court, a rule that states that a witness can
testify only to those facts that she knows of her personal knowledge; that is,
which are derived from her own perception, except as otherwise provided in the
Rules of Court. The personal knowledge of a witness is a substantive
prerequisite for accepting testimonial evidence that establishes the truth of a
disputed fact. A witness bereft of personal knowledge of the disputed fact
cannot be called upon for that purpose because her testimony derives its value
not from the credit accorded to her as a witness presently testifying but from the
veracity and competency of the extrajudicial source of her information.
The reason for the exclusion of hearsay evidence is that the person from whom
the witness derived the information on the facts in dispute is not in court and
under oath to be examined and cross-examined.
Moreover, the theory of the hearsay rule is that when a human utterance is
offered as evidence of the truth of the fact asserted, the credit of the assert or
becomes the basis of inference, and, therefore, the assertion can be received as
evidence only when made on the witness stand, subject to the test of crossexamination. However, if an extrajudicial utterance is offered, not as an
assertion to prove the matter asserted but without reference to the truth of the
matter asserted, the hearsay rule does not apply. For example, in a slander case,
if a prosecution witness testifies that he heard the accused say that the
complainant was a thief, this testimony is admissible not to prove that the
Page | 257

Compilation of Digested Cases for Remedial Law Review II


by: K. M. T. BUELA
complainant was really a thief, but merely to show that the accused uttered those
words. This kind of utterance is hearsay in character but is not legal hearsay.
The distinction is, therefore, between (a) the fact that the statement was made,
to which the hearsay rule does not apply, and (b) the truth of the facts asserted
in the statement, to which the hearsay rule applies.
Hence, as Guivencans testimony intends to prove an asserted fact, i.e.,
misappropriation on the part of the accused through documentary evidence of
which the witness has no personal knowledge, the same is inadmissible for
being a hearsay evidence.

Dying Declaration
PEOPLE OF THE PHILIPPINES vs. CESARIO MONTAEZ
G.R. No. 148257, March 17, 2004
FACTS:
Accused was charged of murdering Perlito Ollanes. Prosecution presented
Perlitos brother, Edmund, as witness. Edmund testified that on the day his
brother died the latter upon inquiry as to who shot him answered the Accuseds
name three times.
ISSUE: Whether or not accused may be convicted on account of a witness
testimony recounting the words of a deceased person few moments before the
latter died.
HELD:
Yes. Perlitos statement that it was the appellant who shot him was a dying
declaration. The statement is highly reliable, having been made in extremity
when the declarant is at the point of death and when any hope of survival is
gone, when every motive to falsehood is silenced, and when the mind is induced
by the most powerful considerations to speak the truth. Even if the declarant did
not make a statement that he was at the brink of death, the degree and
seriousness of the words and the fact that death superseded shortly afterwards
may be considered as substantial evidence that the declaration was made by the
victim with full realization that he was in a dying condition.

Page | 258

Compilation of Digested Cases for Remedial Law Review II


by: K. M. T. BUELA
Declaration against Interest
PEOPLE OF THE PHILIPPINES, vs. THEODORE BERNAL et al.
G.R. No. 113685, June 19, 1997
FACTS:
Accused was charged with Kidnapping Openda, Jr. During trial, the
prosecution, in order to prove that accused has a motive in perpetrating the
alleged crime, presented Enriquez, a common friend of both the accused and the
victim, as witness. Enriquez testified that Openda, Jr. confided to him that the
latter is having an affair with accuseds wife.
The trial court, giving credence to Enriquezs testimony as well as testimony of
other witnesses attesting to the circumstances prior to the alleged abduction,
convicted the accused. Accused assailing the decision of the trial court and for
admitting the testimony of Enriquez.
ISSUE: Whether or not testimony made by a witness as to a statement made a
deceased person that is against the interest of the latter may be admissible in
evidence as against a third person.
HELD:
Openda, Jr.s revelation to Enriquez regarding his illicit relationship with
Bernals wife is admissible in evidence, pursuant to Section 38, Rule 130 of the
Revised Rules on Evidence, viz.:
"Sec. 38. Declaration against interest. The declaration made by a person
deceased, or unable to testify, against the interest of the declarant, if the fact
asserted in the declaration was at the time it was made so far contrary to
declarants own interest, that a reasonable man in his position would not have
made the declaration unless he believed it to be true, may be received in
evidence against himself or his successors-in-interest and against third persons."
A statement may be admissible when it complies with the following requisites,
to wit:" (1) that the declarant is dead or unable to testify; (2) that it relates to a
fact against the interest of the declarant; (3) that at the time he made said
declaration the declarant was aware that the same was contrary to his aforesaid
interest; and (4) that the declarant had no motive to falsify and believed such
declaration to be true.
Openda, Jr., having been missing since his abduction, cannot be called upon to
testify. His confession to Enriquez, definitely a declaration against his own
interest, since his affair with Naty Bernal was a crime, is admissible in evidence

Page | 259

Compilation of Digested Cases for Remedial Law Review II


by: K. M. T. BUELA
because no sane person will be presumed to tell a falsehood to his own
detriment.

Declaration about Pedigree


CORAZON DEZOLLER TISON and RENE R. DEZOLLER vs.
COURT OF APPEALS and TEODORA DOMINGO
G.R. No. 121027, July 31, 1997
FACTS:
Martin Guerrero, the surviving spouse and only heir of Teodora Dezoller
Guerrero (TDG), sold the property originally owned by the latter to herein
respondent Teodora Domingo. Thereafter, Transfer Certificate of Title No.
374012 was issued in the latters name. When Martin Guerrero died, herein
petitioners, alleging to be TDGs niece and nephew, filed an action for
reconveyance claiming that they are entitled to inherit one-half of the property
in question by right of representation from TDG. During the trial, Corazon, one
of the petitioners, testified that she is the niece of TDG and submitted
documentary evidence such as pictures, baptismal certificate etc. to prove the
alleged filiation.
Private respondent filed a Demurrer to Plaintiffs Evidence on the ground that
petitioners failed to prove their legitimate filiation with the deceased Teodora
Guerrero. The trial court issued an order granting the demurrer to evidence. In
upholding the dismissal, respondent Court of Appeals declared that the
documentary evidence presented by herein petitioners, such as the baptismal
certificates, family picture, and joint affidavits are all inadmissible and
insufficient to prove and establish filiation
ISSUES:
(1) Whether or not a trial court may dismissed an action for reconveyance on
the ground of complainants failure to prove his alleged filiation on which
the cause of action is anchored.
(2) Whether or not testimony as to filiation to a deceased person is inadmissible
for being a hearsay evidence.
HELD:
(1)
The answer is in the negative. The court a quo and respondent appellate court
have regrettably overlooked the universally recognized presumption on
legitimacy. The presumption of legitimacy in the Family Code actually fixes a
civil status for the child born in wedlock, and that civil status cannot be attacked
collaterally. The legitimacy of the child can be impugned only in a direct action
Page | 260

Compilation of Digested Cases for Remedial Law Review II


by: K. M. T. BUELA
brought for that purpose, by the proper parties, and within the period limited by
law. The burden of proof rests not on herein petitioners who have the benefit of
the presumption in their favor, but on private respondent who is disputing the
same.
(2)
The primary proof to be considered in ascertaining the relationship between the
parties concerned is the testimony of Corazon Dezoller Tison to the effect that
Teodora Dezoller Guerrero in her lifetime, or sometime in 1946, categorically
declared that the former is Teodoras niece. Such a statement is considered a
declaration about pedigree which is admissible, as an exception to the hearsay
rule, under Section 39, Rule 130 of the Rules of Court, subject to the following
conditions: (1) that the declarant is dead or unable to testify; (2) that the
declarant be related to the person whose pedigree is the subject of inquiry; (3)
that such relationship be shown by evidence other than the declaration; and (4)
that the declaration was made ante litem motam, that is, not only before the
commencement of the suit involving the subject matter of the declaration, but
before any controversy has arisen thereon.
There is no dispute with respect to the first, second and fourth elements. What
remains for analysis is the third element, that is, whether or not the other
documents offered in evidence sufficiently corroborate the declaration made by
Teodora Dezoller Guerrero in her lifetime regarding the pedigree of petitioner
Corazon Dezoller Tison or, if at all, it is necessary to present evidence other
than such declaration.
Distinction must be made as to when the relationship of the declarant may be
proved by the very declaration itself, or by other declarations of said declarant,
and when it must be supported by evidence aliunde. The general rule, therefore,
is that where the party claiming seeks recovery against a relative common to
both claimant and declarant, but not from the declarant himself or the
declarants estate, the relationship of the declarant to the common relative may
not be proved by the declaration itself. There must be some independent proof
of this fact. As an exception, the requirement that there be other proof than the
declarations of the declarant as to the relationship, does not apply where it is
sought to reach the estate of the declarant himself and not merely to establish a
right through his declarations to the property of some other member of the
family.
We are sufficiently convinced, and so hold, that the present case is one instance
where the general requirement on evidence aliunde may be relaxed. Petitioners
are claiming a right to part of the estate of the declarant herself.

Page | 261

Compilation of Digested Cases for Remedial Law Review II


by: K. M. T. BUELA

Family Reputation
FRANCISCO L. JISON vs. COURT OF APPEALS and MONINA
JISON
G.R. No. 124853, February 24, 1998
FACTS:
Monina Jison filed a petition for recognition as an illegitimate child of petitioner
Francisco Jison. In her complaint, she alleged that: (1) at the end of 1945 or the
start of 1946, however, FRANCISCO impregnated Esperanza F. Amolar (who
was then employed as the nanny of FRANCISCO's daughter, Lourdes); (2)
MONINA was born on 6 August 1946, in Dingle, Iloilo; (3) since childhood,
she had enjoyed the continuous, implied recognition as an illegitimate child of
FRANCISCO by his acts and that of his family; and (4) that FRANCISCO gave
her support and spent for her education, such that she obtained a Master's
degree, became a certified public accountant (CPA) and eventually, a Central
Bank examiner.
At trial on the merits, MONINA presented as documentary evidence letters
written by Franciscos relatives as proof of her recognition as illegitimate
daughter of the latter.
The trial court dismissed the complaint. On appeal, CA reversed the ruling of
the trial court and held that Monina was able to establish her filiation as
FRANCISCO's illegitimate daughter not just preponderant but overwhelming
evidence on record. Francisco elevated the case before the SC and assailed the
admissibility of the letters of his relatives.
ISSUE: Whether or not letter of the relatives of a putative father is admissible
in evidence as part of the family reputation.
HELD:
The answer is in the negative.
Under Rule 130, Section 39, the contents of these documents may not be
admitted, there being no showing that the declarants-authors were dead or
unable to testify, neither was the relationship between the declarants and
MONINA shown by evidence other than the documents in question.
Neither may it be admitted under under Rule 130, Section 40.
Rule 130, Section 40, provides:

Page | 262

Compilation of Digested Cases for Remedial Law Review II


by: K. M. T. BUELA
Sec. 40. Family reputation or tradition regarding pedigree. The reputation
or tradition existing in a family previous to the controversy, in respect to the
pedigree of any one of its members, may be received in evidence if the witness
testifying thereon be also a member of the family, either by consanguinity or
affinity. Entries in family bibles or other family books or charts, engravings on
rings, family portraits and the like may be received as evidence of pedigree.
(emphasis supplied)
It is evident that this provision may be divided into two (2) parts: the portion
containing the first underscored clause which pertains to testimonial evidence,
under which the documents in question may not be admitted as the authors
thereof did not take the witness stand; and the section containing the second
underscored phrase. What must then be ascertained is whether letter presented
in this case as private documents, fall within the scope of the clause "and the
like" as qualified by the preceding phrase "entries in family bibles or other
family books or charts, engravings on rights and family portraits,"
We hold that the scope of the enumeration contained in the second portion of
this provision, in light of the rule of ejusdem generis, is limited to objects which
are commonly known as "family possessions," or those articles which represent,
in effect, a family's joint statement of its belief as to the pedigree of a person.
These have been described as objects "openly exhibited and well known to the
family," or those "which, if preserved in a family, may be regarded as giving a
family tradition." Plainly then, letters, as private documents not constituting
"family possessions" as discussed above, may not be admitted on the basis of
Rule 130, Section 40.
Neither may these exhibits be admitted on the basis of Rule 130, Section 41
regarding common reputation, it having been observed that: the weight of
authority appears to be in favor of the theory that it is the general repute, the
common reputation in the family, and not the common reputation in community,
that is a material element of evidence going to establish pedigree. Thus, matters
of pedigree may be proved by reputation in the family, and not by reputation in
the neighborhood or vicinity, except where the pedigree in question is marriage
which may be proved by common reputation in the community.

Page | 263

Compilation of Digested Cases for Remedial Law Review II


by: K. M. T. BUELA
Res gestae
PEOPLE OF THE PHILIPPINES v. FRANK LOBRIGAS et al.
G.R. No. 147649, December 17, 2002
FACTS:
Accused together with others were charged with murder for killing Felix
Taylaran. During the trial, prosecution presented two (2) witnesses, namely:
Castor Guden, owner of the farm where the victim work, and Rosa Solarte,
daughter of the victim.
Castor testified that: (1) on February 19, 1996, Felix Taylaran, upon returning
to Castor Guden from Mantes (one of the accused) store, was with bruises on
his face and injuries all over his body; (2) he told Castor that he was mauled by
accused-appellant Frank Lobrigas, Accused Marlito Lobrigas and Teodorico
Mante at the store; and (3) the next day, Felix Taylaran died. On the other hand,
Rosa Taylaran Solarte, testified that a day after the mauling, her father came to
her house and told her that he was beaten up by Frank Lobrigas, Marlito
Lobrigas and Teodorico Mante. He told her that he was in pain and felt weak.
Accused-appellant had a different version of the events. He denied the
accusation and testified that Felix had too much to drink, he became rowdy and
drew his knife. This was snatched from him by Mario Granderos and turned
over to Mante, who was a barangay councilman.
Accused-appellant insists that the statements made by the victim to Castor
Guden and Rosa Solarte cannot be considered dying declarations for they were
made not under the consciousness of an impending death. Neither can they be
deemed part of the res gestae because the victim was drunk and mad at Teodoro
Mante for taking away his knife.
ISSUE: Whether or not testimony of Castor and Rosa are admissible in
evidence as res gestae.
HELD:
A declaration is deemed part of the res gestae and admissible in evidence as an
exception to the hearsay rule when the following requisites concur: (1) the
principal act, the res gestae, is a startling occurrence; (2) the statements were
made before the declarant had time to contrive or devise; and (3) the statements
must concern the occurrence in question and its immediately attending
circumstances. 6 All these requisites concur in the case at bar. The principal act,
the mauling of the victim, was a startling occurrence. The declarations were
made shortly after the mauling incident while the victim was still under the
exciting influence of the startling occurrence, without any prior opportunity to
Page | 264

Compilation of Digested Cases for Remedial Law Review II


by: K. M. T. BUELA
contrive a story implicating Accused-Appellant. The declaration concerns the
circumstances surrounding the mauling of Felix Taylaran.
However, the declaration made by the victim to his daughter does not satisfy
the second requirement of spontaneity because they were made a day after the
incident and the exciting influence of the startling occurrence was no longer
present. Nevertheless, we hold that Rosa Solartes testimony on what her father
told her constitutes independent relevant statements distinct from hearsay, and
are thus admissible not as to the veracity thereof, but as proof of the fact that
they had been uttered.
Under the doctrine of independently relevant statements, only the fact that
such statements were made is relevant, and the truth or falsity thereof is
immaterial. The hearsay rule does not apply, hence, the statements are
admissible as evidence. Evidence as to the making of such statement is not
secondary but primary, for the statement itself may constitute a fact in issue or
be circumstantially relevant as to the existence of such a fact.

PEOPLE OF THE PHILIPPINES vs. GILBERTO VILLARICO et al.


G.R. No. 158362, April 04, 2011
FACTS:
Accused were charged of murder for the death of Haide Cagatan. During the
trial, prosecution presented the following witnesses: (1) Remedios, sister-in-law
of the victim, who testified that she saw accused pointing their gun at the victim;
(2) Lolita Cagatan, mother of the victim, who testified that she was at the sala
when she heard gunshots followed by seeing the victim wounded and asking for
help stating that he was shot by Berting (accused); (3) Francisco, father of the
victim; who testified that he also heard gunshots and saw accused aiming their
guns upward and were about to leave.
RTC convicted the four accused of homicide aggravated by dwelling. The RTC
accorded faith to the positive identification of the accused by the Prosecution's
witnesses. On intermediate review, the CA modified the RTC's decision and
convicted the accused with murder. The accused contend that the Prosecution
witnesses did not actually see who had shot Haide and that Lolitas testimony
is a hearsay.
ISSUE: Whether or not testimony relating the last statement of the victim
immediately after the shooting incident is admissible in evidence.
HELD:
Page | 265

Compilation of Digested Cases for Remedial Law Review II


by: K. M. T. BUELA
The answer is in the affirmative. The statement was admissible against the
accused as an exception to the hearsay rule under Section 42, Rule 130 of the
Rules of Court, which provides: Statements made by a person while a startling
occurrence is taking place or immediately prior or subsequent thereto with
respect to the circumstances thereof, may be given in evidence as part of the res
gestae. So, also, statements accompanying an equivocal act material to the issue,
and giving it a legal significance, may be received as part of the res gestae.
The term res gestae refers to "those circumstances which are the undesigned
incidents of a particular litigated act and which are admissible when illustrative
of such act." In a general way, res gestaeincludes the circumstances, facts, and
declarations that grow out of the main fact and serve to illustrate its character
and which are so spontaneous and contemporaneous with the main fact as to
exclude the idea of deliberation and fabrication. The rule on res gestae
encompasses the exclamations and statements made by either the participants,
victims, or spectators to a crime immediately before, during, or immediately
after the commission of the crime when the circumstances are such that the
statements were made as a spontaneous reaction or utterance inspired by the
excitement of the occasion and there was no opportunity for the declarant to
deliberate and to fabricate a false statement.
The test of admissibility of evidence as a part of the res gestae is whether the
act, declaration, or exclamation is so intimately interwoven or connected with
the principal fact or event that it characterizes as to be regarded a part of the
principal fact or event itself, and also whether it clearly negatives any
premeditation or purpose to manufacture testimony. A declaration or an
utterance is thus deemed as part of the res gestae that is admissible in evidence
as an exception to the hearsay rule when the following requisites concur: (a) the
principal act, the res gestae, is a startling occurrence; (b) the statements were
made before the declarant had time to contrive or devise; and (c) the statements
must concern the occurrence in question and its immediately attending
circumstances.
We find that the requisites concurred herein. Firstly, the principal act - the
shooting of Haide - was a startling occurrence. Secondly, his statement to his
mother about being shot by the group of Berting was made before Haide had
time to contrive or to devise considering that it was uttered immediately after
the shooting. And, thirdly, the statement directly concerned the startling
occurrence itself and its attending circumstance (that is, the identities of the
assailants). Verily, the statement was reliable as part of theres gestae for being
uttered in spontaneity and only in reaction to the startling occurrence.

Page | 266

Compilation of Digested Cases for Remedial Law Review II


by: K. M. T. BUELA
Entries in the course of business
PHILIPPINE AIRLINES, INC. vs. JAIME J. RAMOS et al.
G.R. No. 92740, March 23, 1992
FACTS:
Respondents filed an action for damages against petitioner alleging the
following: (1) they are passengers of PAL Flight No. 264 on September 24,
1985; (2) they check-in at least one (1) hour before the published departure time;
(3) no one was at the check-in counter until 30 minutes before departure; (4)
upon checking-in, they were informed that their tickets were cancelled and the
seats awarded to chance passengers; (5) they have to take the bus instead; and
(6) they suffered damages due to the cancellation.
Petitioner disclaims any liability, claiming that the non-accommodation of
Respondent on said flight was due to their having check-in late for their flight.
During the trial, defendant presented the check-in counter clerk at their Naga
Branch on the date of respondents scheduled flight. The clerk testified that: (1)
the respondents were late and that he noted the time of check-in on their tickets;
and (2) there were other passengers who came late before the respondents. In
relation to the testimony, two documentary evidence were offered, namely: (1)
the ticket bearing the notation late 4:02 of the clerk; and (2) the passenger
manifest showing the other names of other passengers who were also late.
Respondent objected to the documentary evidence submitted and argued that
those are self-serving.
ISSUE: Whether or not the entries made on a ticket by employees of a party in
the course of their business may not be given weight on the ground that the same
is self-serving.
HELD:
The answer is in the negative. The plane tickets of the private respondents with
the notation "late 4:02" stamped on the flight coupon by the check-in clerk
immediately upon the check-in of private respondents and the passenger
Manifest of Flight PR 264 which showed the non-accommodation of Capati and
Go and the private respondents are entries made in the regular course of business
which the, private respondents failed to overcome with substantial and
convincing evidence other than their testimonies.
Consequently, they carry more weight and credence. A writing or document
made contemporaneously with a transaction in which are evidenced facts
pertinent to an issue, when admitted as proof of those facts, is ordinarily

Page | 267

Compilation of Digested Cases for Remedial Law Review II


by: K. M. T. BUELA
regarded as more reliable proof and of greater probative force than the oral
testimony of a witness as to such facts based upon memory and recollection
Spoken words could be notoriously unreliable as against a written document
that speaks a uniform language
Private respondents only objection to these documents is that they are selfserving cannot be sustained. The hearsay rule will not apply in this case as
statements, acts or conduct accompanying or so nearly connected with the main
transaction as to form a part of it, and which illustrate, elucidate, qualify or
characterize the act, are admissible as part of the res gestae.

Entries in Official Records


RUDY LAO vs. STANDARD INSURANCE CO., INC.
G.R. No. 140023, August 14, 2003
FACTS:
Petitioner Lao, owner of a Fuso truck insured by respondent Standard Insurance
Co., Inc., filed a claim with the latter. However, the claim was denied by the
insurance company on the ground that the driver of the insured truck, Leonardo
Anit, as stated in the Police Blotter, did not possess a proper drivers license at
the time of the accident. The restriction in Leonardo Anits drivers license
provided that he can only drive four-wheeled vehicles weighing not more than
4,500 kgs. Since the insured truck he was driving weighed more than 4,500 kgs.,
he therefore violated the "authorized driver" clause of the insurance policy.
Thus, petitioner filed an action for breach of contract and damages. During trial,
Respondent offered as evidence the police blotter and presented the
investigating police officer who made the entries on the said blotter report. On
the other hand, petitioner offered in evidence the Motor Vehicle Accident
Report stating that the driver at the time of the accident is not Anit but Giddie
Boy. The said report was made three days after the accident or on April 27,
1985.
RTC dismissed the complaint and this was affirmed by CA on appeal. Petitioner
assails the admissibility and evidentiary weight given to the police blotter, as a
basis for the factual finding of the RTC and the CA. He contends that the same
entry was belied by the Motor Vehicle Accident Report and testimony of the
investigating policeman himself.
ISSUE: Whether or not admissibility of a police blotter may be assailed on the
ground that it contains entries that is contrary to another police report made by
the same investigating officer who made the blotter.
Page | 268

Compilation of Digested Cases for Remedial Law Review II


by: K. M. T. BUELA

HELD:
Entries in police records made by a police officer in the performance of the duty
especially enjoined by law are prima facie evidence of the fact therein stated,
and their probative value may be either substantiated or nullified by other
competent evidence. Although police blotters are of little probative value, they
are nevertheless admitted and considered in the absence of competent evidence
to refute the facts stated therein.
The police blotter was admitted under Rule 130, Section 44 of the Rules of
Court. Under the said rule, the following are the requisites for its admissibility:
(a) that the entry was made by a public officer, or by another person, specially
enjoined by law to do so; (b) that it was made by the public officer in the
performance of his duties, or by such other person in the performance of a duty
specially enjoined by law; and (c) that the public officer or other person had
sufficient knowledge of the facts by him stated, which must have been acquired
by him personally or through official information.
In this case the police blotter was identified and formally offered as evidence.
The person who made the entries was likewise presented in court; he identified
and certified as correct the entries he made on the blotter. The information was
supplied to the entrant by the investigating officer who did not protest about any
inaccuracy when the blotter was presented to him in comparison to the accident
report he made three (3) days after the accident. No explanation was likewise
given by the investigating officer for the alleged interchange of names.

MEYNARDO SABILI, PETITIONER, vs.


ELECTIONS AND FLORENCIO LIBREA
G.R. No. 193261, April 24, 2012

COMMISSION

ON

FACTS:
When petitioner filed his COC for mayor of Lipa City for the 2010 elections, he
stated therein that he had been a resident of the city for two (2) years and eight
(8) months. Private respondent Florencio Librea filed a Petition to Deny Due
Course and to Cancel Certificate of Candidacy and to Disqualify a Candidate
for Possessing Some Grounds for Disqualification. He alleged that petitioner
failed to comply with the one-year residency requirement under Section 39 of
the Local Government Code.
In order to prove his compliance with the residency requirement, petitioner
presented as evidence his barangay certificate. The COMELEC in disqualifying
the petitioner did not consider the Certification issued by Pinagtong-ulan
Page | 269

Compilation of Digested Cases for Remedial Law Review II


by: K. M. T. BUELA
Barangay Captain Dominador Honrade. COMELEC brushed it aside on the
ground that the said Certification was not sworn to before a notary public and,
hence, cannot be relied on. Subsequently, petitioner presented another,
substantially identical, Certification from the said Pinagtong-ulan Barangay
Captain, save for the fact that it had now been sworn to before a notary public.
ISSUE: Whether or not barangay certificate is inadmissible in evidence on the
ground that it is not notarized.
HELD:
The answer is in the negative. Rule 130, Section 44 of the Rules of Court
provides: Entries in official records made in the performance of his duty by a
public officer of the Philippines, or by a person in the performance of a duty
specially enjoined by law, are prima facie evidence of the facts therein stated.
Three (3) requisites must concur for entries in official records to be admissible
in evidence: (a) The entry was made by a public officer, or by another person
specially enjoined by law to do so; (b) It was made by the public officer in the
performance of his duties, or by such other person in the performance of a duty
specially enjoined by law; and (c) The public officer or other person had
sufficient knowledge of the facts stated by him, which facts must have been
acquired by him personally or through official information.
As to the first requisite, the Barangay Secretary is required by the Local
Government Code to keep an updated record of all inhabitants of the
barangay. Regarding the second requisite, it is the business of a punong
barangay to know who the residents are in his own barangay. Anent the third
requisite, the BarangayCaptains exercise of powers and duties concomitant to
his position requires him to be privy to these records kept by the Barangay
Secretary.

Commercial Lists
Spouses ANTONIO and LORNA QUISUMBING vs. MANILA
ELECTRIC COMPANY
G.R. No. 142943, April 3, 2002
FACTS:
Meralco team conducted their inspection at petitioners meter and found alleged
meter tampering, they immediately disconnected petitioners' electrical supply.
During the inspection the following persons were present, four (4) MERALCO
inspection personnel and the secretary of appellees.

Page | 270

Compilation of Digested Cases for Remedial Law Review II


by: K. M. T. BUELA
Plaintiffs-appellees filed a complaint for damages with prayer for the issuance
of a writ of preliminary mandatory injunction. On the other hand, MERALCO
filed a counterclaim with respect to the unpaid bills of herein plaintiff. During
the trial, MERALCO presented as witness its Senior Billing Computer Officer
to testify as to the unpaid bills of the plaintiff. The said testimony as
corroborated with the documentary evidence showing unpaid bills as well as the
laboratory test results proving the tampering.
Trial court ruled that immediate disconnection was illegal due to lack of due
process. On appeal, the CA reversed the trial courts decision and dismissed the
complaint. CA likewise upheld respondent's counterclaim for the billing
differential representing the value of petitioners' used but unregistered electrical
consumption, which had been established without being controverted. Petitioner
elevated the case before the SC by way of petition for review on certiorari. In
their memorandum, petitioner assailed ruling upholding the validity of the
disconnection and denying their claim for damages. The petitioner did not
questioned the computation of the differential billing both during the trial as
well as in their memorandum submitted before the SC. The only defense
presented by petitioner is that they cannot be held liable thereof because the bills
are already outstanding when they transferred to their residence.
ISSUE: Whether or not a party may be held liable for unpaid bills based on
the uncontroverted documentary and testimonial evidence.
HELD:
Yes. The mere presentation by petitioners of a Contract to Sell with Assumption
of Mortgage does not necessarily mean that they are no longer liable for the
billing differential. There was no sufficient evidence to show that they had not
been actually residing in the house before the date of the said document. Lorna
Quisumbing herself admitted that they did not have any contract for electrical
service in their own name. Hence, petitioners effectively assumed the bills of
the former occupants of the premises.
The evidence it presented, both documentary and testimonial, sufficiently
proved the amount of the differential.
Not only did respondent show how the meter examination had been conducted
by its experts, but it also established the amount of P193,332.96 that petitioners
owed respondent. The procedure through which this amount was arrived at was
testified to by Meralco's Senior Billing Computer Officer. His testimony was
corroborated by documentary evidence showing the account's billing history
and the corresponding computations. Neither do we doubt the documents of
inspections and examinations presented by respondent to prove that, indeed
Page | 271

Compilation of Digested Cases for Remedial Law Review II


by: K. M. T. BUELA
there had been meter tampering that resulted in unrecorded and unpaid electrical
consumption.

Learned Treaties

Testimony at a Former Proceeding


PEOPLE OF THE PHILIPPINES vs. LANIE ORTIZ-MIYAKE
G.R. Nos. 115338-39, September 16, 1997
FACTS:
Accused-appellant Lanie Ortiz-Miyake was charged with illegal recruitment in
large scale on a complaint initiated by Elenita Marasigan, Imelda Generillo and
Rosamar del Rosario.
In convicting appellant of illegal recruitment in large scale, the lower court
adopted a previous decision of Branch 78 of the Metropolitan Trial Court of
Paraaque as a basis for the judgment. Said previous decision was a conviction
for estafa involving the same circumstances in the instant case, wherein
complainants Generillo and Del Rosario charged appellant with two counts of
estafa. This decision was not appealed and had become final and executory.
On appeal, the OSG argued that the Makati court could not validly adopt the
facts embodied in the decision of the Paraaque court to show that illegal
recruitment was committed against Generillo and Del Rosario as well. Illegal
recruitment was allegedly proven to have been committed against only one
person, particularly, Elenita Marasigan. Appellant, therefore, may only be held
guilty of simple illegal recruitment and not of such offense in large scale.
ISSUE: Whether or not a trial court may adopt the findings of fact and decision
of another court involving the same parties and incidents.
HELD:
The answer is in the negative. Trial courts utilization of and reliance on the
previous decision of the Paraaque court must be rejected. Every conviction
must be based on the findings of fact made by a trial court according to its
appreciation of the evidence before it. A conviction may not be based merely
on the findings of fact of another court, especially where what is presented is
only its decision sans the transcript of the testimony of the witnesses who
testified therein and upon which the decision is based.

Page | 272

Compilation of Digested Cases for Remedial Law Review II


by: K. M. T. BUELA
A previous decision or judgment, while admissible in evidence may only prove
that an accused was previously convicted of a crime. 30 It may not be used to
prove that the accused is guilty of a crime charged in a subsequent case, in lieu
of the requisite evidence proving the commission of the crime, as said previous
decision is hearsay. To sanction its being used as a basis for conviction in a
subsequent case would constitute a violation of the right of the accused to
confront the witnesses against him.

HARRY L. GO, et al. vs. THE PEOPLE OF THE PHILIPPINES and


HIGHDONE COMPANY, LTD., et al.
G.R. NO. 185527, July 18, 2012
FACTS:
Petitioners were charged before the MTC for Other Deceits under Article 318
of the Revised Penal Code. The prosecution's complaining witness, Li Luen
Ping, a frail old businessman from Laos, Cambodia.
The private prosecutor filed with the MeTC a Motion to Take Oral Deposition
of Li Luen Ping, alleging that he was being treated for lung infection.
Notwithstanding petitioners' Opposition, the MeTC granted the motion.
Petitioners sought its reconsideration which the MeTC denied, prompting
petitioners to file a Petition for Certiorari before the RTC.
RTC granted the petition and declared the MeTC Orders null and void.11The
RTC held that Section 17, Rule 23 on the taking of depositions of witnesses in
civil cases cannot apply suppletorily to the case since there is a specific
provision in the Rules of Court with respect to the taking of depositions of
prosecution witnesses in criminal cases, which is primarily intended to
safeguard the constitutional rights of the accused to meet the witness against
him face to face.
The prosecution elevated the case to the CA. CA reversed the ruling of the RTC.
ISSUE: Whether or not testimony of a witness is a criminal case may be taken
by way of oral deposition.
HELD:
The answer is in the negative. The procedure for testimonial examination of an
unavailable prosecution witness is covered under Section 15, Rule 119. The
examination of witnesses must be done orally before a judge in open court. This
is true especially in criminal cases where the Constitution secures to the accused
his right to a public trial and to meet the witnessess against him face to face.
Page | 273

Compilation of Digested Cases for Remedial Law Review II


by: K. M. T. BUELA
The requirement is the "safest and most satisfactory method of investigating
facts" as it enables the judge to test the witness' credibility through his manner
and deportment while testifying. It is not without exceptions, however, as the
Rules of Court recognizes the conditional examination of witnesses and the use
of their depositions as testimonial evidence in lieu of direct court testimony.
For purposes of taking the deposition in criminal cases, more particularly of a
prosecution witness who would forseeably be unavailable for trial, the
testimonial examination should be made before the court, or at least before the
judge, where the case is pending as required by the clear mandate of Section 15,
Rule 119 of the Revised Rules of Criminal Procedure.

Child Witness
PEOPLE OF THE PHILIPPINES vs. EDWIN IBAEZ Y ALBANTE
and ALFREDO NULLA Y IBAEZ
G.R. No. 197813, September 25, 2013
FACTS:
Appellants were all charged in an Information for Murder of Wilfredo Atendido
y Dohenog.
The prosecution presented the victims wife, Rowena and minor daughter,
Rachel (10 years old) as witnesses. Rachel testified that she saw the appellants
killed her father. The defense, on the other hand, presented Aniceta as witness
whose testimony discredit that of Rachel. Aniceta testified testified that she and
Rachel were out on that day selling doormats and only returned at 6:00 p.m.
Thus, Rachel could not have witnessed the murder of Wilfredo.
The trial court convicted the accused. The conviction was affirmed by the CA.
Appellant, on appeal with the SC, tried to further discredit Rachels testimony
by arguing that Rachel was a mere child who had studied only until the first
grade of elementary school and could barely read, and did not know how to tell
time.
ISSUE: Whether or not a child witness may be disqualified on the ground that
she does not know how to read and tell time.
HELD:
The answer is in the negative.e With exceptions provided in the Rules of Court,
all persons who can perceive, and perceiving, can make known their perception
to others, may be witnesses. That is even buttressed by the Rule on Examination
of a Child Witness which specifies that every child is presumed qualified to be
a witness. To rebut this presumption, the burden of proof lies on the party
Page | 274

Compilation of Digested Cases for Remedial Law Review II


by: K. M. T. BUELA
challenging the child's competence. Only when substantial doubt exists
regarding the ability of the child to perceive, remember, communicate,
distinguish truth from falsehood, or appreciate the duty to tell the truth in court
will the court, motu proprio or on motion of a party, conduct a competency
examination of a child. Thus, petitioners flimsy objections on Rachels lack of
education and inability to read and tell time carry no weight and cannot
overcome the clear and convincing testimony of Rachel as to who killed her
father.

h. Opinion Rule
Expert Witness
PEOPLE OF THE PHILIPPINES vs. PO2 ALBERT ABRIOL et al.
G.R. No. 123137, October 17, 2001
FACTS:
Herein accused was charged and convicted of murder by the trial court. On
appeal, appellants argue that the prosecutions circumstantial evidence against
them is weak, ambiguous, and inconclusive. Specifically, appellants contend
that the testimony of P/Inspector Lemuel Caser, the prosecutions ballistics
expert, clearly shows that: (1) He is ignorant about such ballistics instruments
such as the micrometer, goniometer, and pressure barrel. (2) He is not
conversant with "the required references concerning ballistics," particularly
books on the subject by foreign authorities. (3) He could not "scientifically
determine the caliber of a bullet." Since P/Inspector Caser lacked adequate
training and expertise in ballistics, they claim that his opinion that the test
bullets and cartridges matched the slugs and cartridges recovered from the scene
of the crime was not reliable. Appellants also assail Casers failure to take the
necessary photographs to support his findings.
ISSUE: Whether or not testimony of an expert witness which was given
credence of the trial court may be assailed on appeal on the ground of
incompetence.
HELD:
An expert witness is "one who belongs to the profession or calling to which the
subject matter of the inquiry relates and who possesses special knowledge on
questions on which he proposes to express an opinion." There is no definite
standard of determining the degree of skill or knowledge that a witness must
possess in order to testify as an expert. It is sufficient that the following factors
be present: (1) training and education; (2) particular, first-hand familiarity with
the facts of the case; and (3) presentation of authorities or standards upon which
Page | 275

Compilation of Digested Cases for Remedial Law Review II


by: K. M. T. BUELA
his opinion is based. The question of whether a witness is properly qualified
to give an expert opinion on ballistics rests with the discretion of the trial
court.
In the instant case, P/Inspector Caser qualifies as a ballistics expert. He is a
licensed criminologist, trained at the Ballistics Command and Laboratory
Center in Fort Bonifacio, in the PNP Crime Laboratory in Camp Crame, and in
the National Bureau of Investigation. He had previously testified as an expert
witness in at least twenty-seven (27) murder and homicide cases all over the
country. An expert witness need not present comparative microphotographs of
test bullets and cartridges to support his findings. Examination under a
comparison microscope showing that the test bullet and the evidence bullet both
came from the same gun is sufficient. Moreover, the ballistician conclusively
found similar characteristic markings in the evidence, test cartridges and slugs.

LAURA and ERIBERTO BAUTISTA vs. HON. COURT OF APPEALS


and FERNANDO MORELOS
G.R. NO. 158015, August 11, 2004
FACTS:
During his lifetime, Cesar Morelos sold and conveyed a parcel of land in favor
of his niece, petitioner Laura Morelos Bautista, as evidenced by a "Deed of
Absolute Sale. Respondent Fernando Morelos, claiming to be the illegitimate
child of Cesar Morelos, instituted a complaint for the declaration of nullity of
sale and title with damages.
At the trial, Respondent presented testimonies of expert witnesses who claimed
that the signature of Cesar Morelos on the Deed of Absolute Sale and the
fingerprint appearing on his Residence Certificate were not his. Petitioners
countered that the Deed of Absolute Sale was valid.
On the other hand, petitioner presented Carmelita Marcelino who testified that
she saw Cesar Morelos and petitioner Laura Bautista sign the Deed in question.
The trial court dismissed the complaint. On appeal, the CA relying on the
testimony of the expert witness reversed the trial courts decision.
ISSUE: Whether or not the testimonies of expert witnesses are conclusive to be
a strong basis to nullify a duly executed and notarized deed of absolute sale as
against the testimony of one who witness the signing of the deed.
HELD:
Page | 276

Compilation of Digested Cases for Remedial Law Review II


by: K. M. T. BUELA
The answer is in the negative. It is well-settled that a duly notarized contract
enjoys the prima facie presumption of authenticity and due execution as well as
the full faith and credence attached to a public instrument. To overturn this legal
presumption, evidence must be clear, convincing and more than merely
preponderant to establish that there was forgery that gave rise to a spurious
contract.
A finding of forgery does not depend entirely on the testimony of handwriting
experts. Although such testimony may be useful, the judge still exercises
independent judgment on the issue of authenticity of the signatures under
scrutiny; he cannot rely on the mere testimony of the handwriting expert.
In the case at bar, the presumption of validity and regularity prevails over
allegations of forgery and fraud. As against direct evidence consisting of the
testimony of a witness who was physically present at the signing of the contract
and who had personal knowledge thereof, the testimony of an expert witness
constitutes indirect or circumstantial evidence at best. Carmelita Marcelino, the
witness to the Deed of Absolute Sale, confirmed the genuineness, authenticity
and due execution thereof. Having been physically present to see the decedent
Cesar Morelos and petitioner Laura Bautista affix their signatures on the
document, the weight of evidence preponderates in favor of petitioners.

BOBBY ABEL AVELINO Y BULAWAN vs. PEOPLE OF THE


PHILIPPINES
G.R. No. 181444, July 17, 2013
FACTS:
Herein accused was charged with murder. During the trial, the defense presented
as one of its witness Scene of the Crime Operative (SOCO) PSI Lito D.
Cabamongan to testify on the possible position of the gunman based on the
wounds sustained by the victim. With Cabamongans testimony, the defense
intends to assail the testimony of the eye-witnesses, Manalangsang and Caada,
as to the position of the gunman. Cabamongans testimony was offered as an
ordinary witness.
On appeal, the CA sustained the conviction of the accused and disregarded the
testimony of soco Cabamongan. Aggrieved, petitioner now seeks to reverse his
conviction, arguing that the CA erred in relying on the testimonies of the
prosecution witnesses Manalangsang and Caada and disregarding the
inconsistencies between the statements of Manalangsang and the findings of the
medico-legal and SOCO PSI Cabamongan as to the position of the gunman.

Page | 277

Compilation of Digested Cases for Remedial Law Review II


by: K. M. T. BUELA
ISSUE: Whether or not testimony of an expert presented as an ordinary witness
may be admitted as evidence and be considered against testimony of eyewitnesses.
HELD:
No. Expert evidence is admissible only if: (a) the matter to be testified to is one
that requires expertise, and (b) the witness has been qualified as an expert. In
this case, counsel for the petitioner failed to make the necessary qualification
upon presenting Cabamongan during trial.

Ordinary Witness
PEOPLE OF THE PHILIPPINES vs. EMILIANO DURANAN
G.R. Nos. 134074-75, January 16, 2001
FACTS:
Herein accused was charged with two (2) counts of rape. Complainant Nympha
Lozada, who was 25 years old at the time of the incidents in question, is
considered to be retarded and finished up to the sixth grade only. During the
trial, the prosecution presented three witnesses, namely, complainant Nympha
Lozada y de Lara, complainants mother Virginia de Lara Lozada, and the
attending medico-legal officer at Camp Crame, Dr. Rosalina O. Cosidon.
Accused was convicted by the trial court.
On appeal, Accused-appellant contends that he cannot be convicted of rape
since the victims mental age was not proven. He argues that an essential
element for the prosecution for rape of a mental retardate is a psychiatric
evaluation of the complainants mental age to determine if her mental age is
under twelve. He further claims that only in cases where the retardation is
apparent due to the presence of physical deformities symptomatic of mental
retardation can the mental evaluation be waived. In this case, only the mother
of the complainant testified as to the latters metal age and fitness.
ISSUE: Whether or not an ordinary witness testimony on the mental fitness of
the complainant who is a mental retardate may be admissible in evidence.
HELD:
The answer is in the affirmative. Rule 130, Section 50 of the Revised Rules on
Evidence provides: the opinion of a witness for which proper basis is given may
be received in evidence regarding (a) the identity of a person about whom he
has adequate knowledge; (b) a handwriting with which he has sufficient
familiarity; and (c) the mental sanity of a person with whom he is sufficiently
acquainted.
Page | 278

Compilation of Digested Cases for Remedial Law Review II


by: K. M. T. BUELA

A non-expert witness may give his opinion as to the sanity or insanity of


another, when based upon conversations or dealings which he has had with such
person, or upon his appearance, or upon any fact bearing upon his mental
condition, with the witness own knowledge and observation, he having first
testified to such conversations, dealings, appearance or other observed facts, as
the basis for his opinion. In the case at bar, Virginia Lozada testified on the
mental condition of her daughter

i. Character Evidence
PEOPLE OF THE PHILIPPINES vs. RAFAEL DIOPITA y GUZMAN
G.R. No. 130601, December 4, 2000
FACTS:
Accused was charged with Robbery with Rape. During the trial, the accused
was positively identified by the victim as her assailant. The trial court formally
rejected his defense of alibi and convicted him of the crime charged;
consequently, accused-appellant is now before us on appeal. Accused-appellant
in his brief, maintains that it was impossible for him to have committed the
crime charged since he is a person of good moral character, holding as he does
the position of "Ministerial Servant" in the congregation of Jehovahs
Witnesses, and that he is a godly man, a righteous person, a responsible family
man and a good Christian who preaches the word of God.
ISSUE: Whether or not the accused may be acquitted on the ground of his god
moral character.
HELD:
An accused is not entitled to an acquittal simply because of his previous good
moral character and exemplary conduct. The affirmance or reversal of his
conviction must be resolved on the basic issue of whether the prosecution had
discharged its duty of proving his guilt beyond any peradventure of doubt. Since
the evidence of the crime in the instant case is more than sufficient to convict,
the evidence of good moral character of accused-appellant is unavailing.The
fact that accused-appellant is endowed with such "sterling" qualities hardly
justifies the conclusion that he is innocent of the crime charged. Similarly, his
having attained the position of "Ministerial Servant" in his faith is no guarantee
against any sexual perversion and plunderous proclivity on his part. Indeed,
religiosity is not always an emblem of good conduct, and it is not the unreligious
alone who succumbs to the impulse to rob and rape.

Page | 279

Compilation of Digested Cases for Remedial Law Review II


by: K. M. T. BUELA
D. Burden of Proof and Presumptions
1. Burden of Proof v. Burden of Evidence
FAR EAST BANK & TRUST COMPANY (FEBTC) vs. ROBERT MAR
CHANTE
G.R. No. 170598, October 09, 2013
FACTS:
Instant complaint was filed by petitioner against Chante to recover from Chan
the principal sum of P770,488.30 representing the unpaid balance of the amount
fraudulently withdrawn from Chans ATM. FEBTC alleged that between 8:52
p.m. of May 4, 1992 and 4:06 a.m. of May 5, 1992, Chan had withdrawn funds
totaling P967,000.00 from the PNB-MEGALINK ATM facility at the Manila
Pavilion Hotel in Manila; that the withdrawals were done in a series of 242
transactions with the use of the same machine, at P4,000.00/withdrawal; and
that the transactions were processed and recorded by the respective computer
systems of PNB and MEGALINK despite the following circumstances, namely:
(a) the offline status of the branch of account (FEBTC Ongpin Branch); (b)
Chans account balance being only P198,511.70 at the time; (c) the maximum
withdrawal limit of the ATM facility being P50,000.00/day; and (d) his
withdrawal transactions not being reflected in his account, and no debits or
deductions from his current account with the FEBTC Ongpin Branch being
recorded. FEBTC asserted further that defendant took advantage of a system
bug which allowed the excessive withdrawals.
Chan denied liability and instead insisted that he had been actually home at the
time of the withdrawals. He alluded to a possible inside job as the cause of
the supposed withdrawals, citing a newspaper report to the effect that an
employee of FEBTCs had admitted having debited accounts of its depositors
by using his knowledge of computers as well as information available to him.
Chan claimed that it would be physically impossible for any human being like
him to stand long hours in front of the ATM facility just to withdraw funds.
ISSUE: Whether or not a civil action may be decided in favor of the plaintiff
where the defendant relies on bare and uncorroborated denial of the formers
allegation.
HELD:
The answer is in the negative. The party who alleges an affirmative fact has the
burden of proving it because mere allegation of the fact is not evidence of it.
Verily, the party who asserts, not he who denies, must prove.
In civil cases, the burden of proof is on the party who would be defeated if no
evidence is given on either side. This is because our system frees the trier of
Page | 280

Compilation of Digested Cases for Remedial Law Review II


by: K. M. T. BUELA
facts from the responsibility of investigating and presenting the facts and
arguments, placing that responsibility entirely upon the respective parties. The
burden of proof, which may either be on the plaintiff or the defendant, is on the
plaintiff if the defendant denies the factual allegations of the complaint in the
manner required by the Rules of Court; or on the defendant if he admits
expressly or impliedly the essential allegations but raises an affirmative defense
or defenses, that, if proved, would exculpate him from liability.
Burden of proof is a term that refers to two separate and quite different concepts,
namely: (a) the risk of non-persuasion, or the burden of persuasion, or simply
persuasion burden; and (b) the duty of producing evidence, or the burden of
going forward with the evidence, or simply the production burden or the burden
of evidence.
In its first concept, it is the duty to establish the truth of a given proposition or
issue by such a quantum of evidence as the law demands in the case at which
the issue arises. In its other concept, it is the duty of producing evidence at the
beginning or at any subsequent stage of trial in order to make or meet a prima
facie case. Generally speaking, burden of proof in its second concept passes
from party to party as the case progresses, while in its first concept it rests
throughout upon the party asserting the affirmative of the issue.
Being the plaintiff, FEBTC must rely on the strength of its own evidence instead
of upon the weakness of Chans evidence. Its burden of proof thus required it to
preponderantly demonstrate that his ATM card had been used to make the
withdrawals, and that he had used the ATM card and PIN by himself or by
another person to make the fraudulent withdrawals. Otherwise, it could not
recover from him any funds supposedly improperly withdrawn from the ATM
account.

2. Presumptions
a. Conclusive
IBAAN RURAL BANK INC. vs. THE COURT OF APPEALS and MR.
and MRS. RAMON TARNATE
G.R. No. 123817, December 17, 1999
FACTS:
Respondent spouses entered into a Deed of Absolute Sale with Assumption of
Mortgage of the lots in question from its original owner Spouses Reyes. As,
Private respondents failed to pay the loan and the bank extra-judicially
foreclosed on the mortgaged lots. At the public auction, the bank was the sole
Page | 281

Compilation of Digested Cases for Remedial Law Review II


by: K. M. T. BUELA
bidder. Consequently, a certificate of sale was issued. The said certificate stated
that redemption period expires two (2) years from the registration of the sale.
Certificate of sale was registered on October 16, 1979.
Within the two (2) year period, private respondents offered to redeem the
foreclosed lots and tendered the redemption amount. However, petitioner Bank
refused and argued that the right to redeem had prescribed, as more than one
year had elapsed from the registration of the Certificate of Sale. Private
respondents filed a complaint to compel the bank to allow their redemption of
the foreclosed lots. They argued that they were entitled to redeem the foreclosed
lots because they offered to redeem and tendered the redemption price before
October 16, 1981, the deadline of the 2-year.
ISSUE: Whether or not failure to previously contest the redemption period
stated on the certificate of sale precludes the bank from asserting it as a defense
to oppose the exercise of right of redemption.
HELD:
By its silence and inaction, petitioner misled private respondents to believe that
they had two years within which to redeem the mortgage. After the lapse of two
years, petitioner is estopped from asserting that the period for redemption was
only one year and that the period had already lapsed. Estoppel in pais arises
when one, by his acts, representations or admissions, or by his own silence when
he ought to speak out, intentionally or through culpable negligence, induces
another to believe certain facts to exist and such other rightfully relies and acts
on such belief, so that he will be prejudiced if the former is permitted to deny
the existence of such facts.
When petitioner received a copy of the Certificate of Sale registered in the
Office of the Register of Deeds of Lipa City, it had actual and constructive
knowledge of the certificate and its contents. For two years, it did not object to
the two-year redemption period provided in the certificate. Thus, it could be said
that petitioner consented to the two-year redemption period specially since it
had time to object and did not. When circumstances imply a duty to speak on
the part of the person for whom an obligation is proposed, his silence can be
construed as consent.

Page | 282

Compilation of Digested Cases for Remedial Law Review II


by: K. M. T. BUELA
SPOUSES REYNALDO ALCARAZ and ESMERALDA ALCARAZ
PEDRO M. TANGGA-AN et al.
G.R. No. 128568, April 9, 2003
FACTS:
Petitioners leased a building from Respondents predecessor (Virginia). At the
time of the perfection of the contract, the petitioner spouses, as lessees, were
aware that the NHA, and not Virginia, the lessor, owned the land on which the
rented house stood yet they signed the same, obliged themselves to comply with
the terms thereof for five years and performed their obligations as lessees for
two years.
After two years from the effectivity of the lease contract, Respondents filed a
complaint for unlawful detainer, with damages against petitioner for failure to
pay rent. On the other hand, the petitioner spouses alleged that they paid the rent
to the new owners (Virgilio and Angelita) of the lot where the building stood
and not to respondents since the latter supposedly no longer had the legal right
to collect rentals. Petitioner claimed that the lease contract ceased to be effective
because Virgilios assumption of ownership of the land stripped the respondents
of ownership of the building.
MTC rendered a decision in favor of plaintiffs. MTC ruled that petitioner failed
to show that the subject house belonged to Virgilio. On the other hand, the
respondents proved that the property in question is registered in their name. On
appeal, the RTC affirmed the decision of the MTC based on the petitioners
failure to present any documentary evidence modifying or amending the
contract of lease to justify the transfer of payment of the monthly rental to
Virgilio Tanga-an who claims only as the registered owner of the lot on which
the leased house is located.
ISSUE: Whether or not a defendant is a suit, to which the cause of action arises
from a contract may assailed the operation of such contract by disputing a
previously affirmed fact.
HELD:
No. Petitioner were aware that the lot in question was not owned by the lessors
at the time the lease contract was entered into. After recognizing the validity of
the lease contract for two years, the petitioner spouses are barred from alleging
the automatic cancellation of the contract on the ground that the respondents
lost ownership of the house after Virgilio acquired title over the lot. Section 2,
Rule 131 of the Rules of Court provides as a conclusive presumption that:

Page | 283

Compilation of Digested Cases for Remedial Law Review II


by: K. M. T. BUELA
Sec. 2. Conclusive presumptions. The following are instances
of conclusive presumptions:
(a) Whenever a party has, by his own declaration, act, or
omission, intentionally and deliberately led another to believe a
particular thing true, and to act upon such belief, he cannot, in
any litigation arising out of such declaration, act or omission,
be permitted to falsify it;
x
x
x
b. Disputable
HOSPICIO D. ROSAROSO et al. vs. LUCILA LABORTE SORIA et al.
G.R. No. 194846, June 28, 2013
FACTS:
Petitioners filed a complaint for nullity of the SPA authorizing Respondent to
sell the subject disputed lots to Meridian, as well as the deed of sale entered into
pursuant to the said SPA. Petitioner alleged that the said lot had already sold to
them by their predecessor Luis Rosaroso.
During trial the second wife of Luis, Lourdes testified deed of sale in favor of
petitioners, was obtained through fraud, deceit and trickery. She explained that
they signed the prepared deed out of pity because petitioners told them that it
was necessary for a loan application.
RTC ruled in favor of petitioners. On appeal, the CA reversed and set aside the
RTC decision. The CA ruled that the first deed of sale in favor of petitioners
was void because they failed to prove that they indeed tendered a consideration
for the four (4) parcels of land. It relied on the testimony of Lourdes that
petitioners did not pay her husband. The price or consideration for the sale was
simulated to make it appear that payment had been tendered when in fact no
payment was made at all.
ISSUE: Whether or not the validity of a contract may be assailed on a mere
testimony that it has no consideration.
HELD:
The answer is in the negative. Under Section 3, Rule 131 of the Rules of Court,
the following are disputable presumptions: (1) private transactions have been
fair and regular; (2) the ordinary course of business has been followed; and (3)
there was sufficient consideration for a contract. These presumptions operate
against an adversary who has not introduced proof to rebut them. They create
the necessity of presenting evidence to rebut the prima facie case they created,
Page | 284

Compilation of Digested Cases for Remedial Law Review II


by: K. M. T. BUELA
and which, if no proof to the contrary is presented and offered, will prevail. The
burden of proof remains where it is but, by the presumption, the one who has
that burden is relieved for the time being from introducing evidence in support
of the averment, because the presumption stands in the place of evidence unless
rebutted.
In this case, the respondents failed to trounce the said presumption. Aside from
their bare allegation that the sale was made without a consideration, they failed
to supply clear and convincing evidence to back up this claim. It is elementary
in procedural law that bare allegations, unsubstantiated by evidence, are not
equivalent to proof under the Rules of Court.
The CA decision ran counter to this established rule regarding disputable
presumption. It relied heavily on the account of Lourdes who testified that the
children of Luis approached him and convinced him to sign the deed of sale,
explaining that it was necessary for a loan application, but they did not pay the
purchase price for the subject properties. This testimony, however, is selfserving and would not amount to a clear and convincing evidence required by
law to dispute the said presumption. As such, the presumption that there was
sufficient consideration will not be disturbed.

HEIRS OF CIPRIANO TRAZONA vs. HEIRS OF DIONISIO


CAADA
G.R. No. 175874, December 11, 2013
FACTS:
Cipriano Trazona owned Lot No. 5053H covered by Tax Declaration No.
07764. Sometime in 1997, when the heirs of Cipriano, herein petitioners, tried
to secure a copy of Tax Declaration No. 07764, they were informed that Tax
Declaration No. 07764 had been cancelled and, in lieu thereof, Tax Declaration
No. 23959 was issued on 24 June 1996 in the name of Dionisio, the owner of
the property adjacent to Ciprianos lot. Apparently, respondents had caused the
issuance of Tax Declaration No. 23959 by submitting a Deed of Absolute Sale
supposedly executed by Cipriano in favor of Dionisio. The deed of sale covers
a portion of Ciprianos property which was encroached upon by Dioniso during
the formers lifetime, but the new tax declaration issued covers the whole
property of Cipriano.
Consequently, petitioners filed a Complaint against respondents for quieting of
title, annulment of deed of sale, cancellation of Tax Declaration No. 23959.
Petitioners alleged therein that the Deed of Absolute Sale dated 27 June 1956

Page | 285

Compilation of Digested Cases for Remedial Law Review II


by: K. M. T. BUELA
was a forgery. During the trial, petitioners presented an expert witness testifying
to the forgery of Ciprianos signature on the assailed deed.
RTC ruled in favor of petitioners. Based on RTC Judges observation Ciprianos
signature on the assailed deed bares a difference as compared to other
documents bearing his signature. On appeal, CA ruled that petitioners had failed
to prove by requisite evidence their allegation that the assailed deed was a
forgery. The deed, being a notarized document, enjoyed the presumption of
authenticity and due execution. Also, the fact that it was an ancient document
that remained unaltered after so many years, bodes well for its authenticity.
ISSUE: Whether or not the presumption of regularity of a notarized ancient
document may be assailed by the testimony of an expert witness and
independent observation of the trial court as to the forgery of signature therein.
HELD:
The answer is in the affirmative. It is true that notarized documents are accorded
evidentiary weight as regards their due execution. Nevertheless, while notarized
documents enjoy the presumption of regularity, this presumption is disputable.
They can be contradicted by evidence that is clear, convincing, and more than
merely preponderant.
In this case, clear and convincing evidence that is enough to overturn the
presumption of regularity of the assailed deed was presented. First, the
document examiner determined that the signature of Cipriano in the assailed
deed had been forged. No issue has been raised about his expertise. Second, the
RTC did not just rely on expert testimony in ruling that the signature was forged.
It likewise supported its finding that the signature was forged through
independent observation.
Lastly, when the record management analyst from the Bureau of Archives
presented the assailed deed, the paper was noted to be white, while its supposed
contemporaries in the bunch from where it was taken had turned yellow with
age. Further, when the analyst was asked the question of when the assailed deed
was received by the Bureau of Archives, she answered that it was forwarded to
them only on 28 September 1987 by RTC Region 7, Notarial Division.

Page | 286

Compilation of Digested Cases for Remedial Law Review II


by: K. M. T. BUELA
3. Suppression of Testimony
PEOPLE OF THE PHILIPPINES vs. ROBERTO PADRIGONE
G.R. No. 137664, May 9, 2002
FACTS:
Accused Roberto Padrigone, a.k.a. Roberto San Miguel, Jocel Ibanita, Michael
San Antonio and Abel Triumpante entered the dwelling of the Contridas sisters
at 3:00 a.m. of January 3, 1995, and at knifepoint successively raped Rowena
Contridas, a 16 year old lass. The victim became insane after the incident and
was not able to testify in Court. Nimfa Contridas, her fourteen year old sister,
who was also present that time narrated the incident when her elder sisters
innocence was forcibly violated. The trial court, disregarding the Accuseds
defense of denial and alibi, convicted the accused.
On appeal, herein Accused-appellant claims that the prosecution suppressed
evidence by not presenting Rowena, the victim, when the latter should have had
her sane moments. As a consequence, the trial court deprived appellant of the
opportunity to cross-examine her when she allegedly declared before the Chief
of Police of Buhi that it was only appellant who raped her which declaration
became the basis for the latters conviction.
ISSUE: Whether or not failure to present as witness the victim of a crime who
became insane by reason of such offense amounts to suppression of evidence.
HELD:
The answer is in the negative. The non-presentation of Rowena on the witness
stand cannot be considered as suppression of evidence. Under Rule 131, Section
3(e) of the Rules of Court, the rule that "evidence willfully suppressed would
be adverse if produced" does not apply if (a) the evidence is at the disposal of
both parties; (b) the suppression was not willful; (c) it is merely corroborative
or cumulative; and (d) the suppression is an exercise of a privilege.
Plainly, there was no suppression of evidence in this case. First, the defense had
the opportunity to subpoena Rowena even if the prosecution did not present her
as a witness. Instead, the defense failed to call her to the witness stand. Second,
Rowena was certified to be suffering from "Acute Psychotic Depressive
Condition" and thus "cannot stand judicial proceedings yet." The nonpresentation, therefore, of Rowena was not willful. Third, in any case, while
Rowena was the victim, Nimfa was also present and in fact witnessed the
violation committed on her sister.

Page | 287

Compilation of Digested Cases for Remedial Law Review II


by: K. M. T. BUELA
METROPOLITAN BANK & TRUST COMPANY vs. COURT OF
APPEALS and G.T.P. DEVELOPMENT CORPORATION
G.R. No. 122899, June 8, 2000
FACTS:
Mr. Chia and Respondent GTP entered into a contract of sale with assumption
of mortgage wherein the latter assumes the formers indebtedness with
Metrobank. Respondent, pursuant to the balance declared by Metrobank prior
to the execution of the sale, paid Chias loan balance with Metrobank in the
amount of P116K+.
This notwithstanding, petitioner METROBANK refused to release the real
estate mortgage on the subject property despite repeated requests, thus
prompting respondent GTP to file an action for specific performance against
petitioner METROBANK and Mr. Chia. Metrobank refused to discharge of the
real estate mortgage on the claim that the subject property still secures "other
unliquidated past due loans as there exist a stipulation in subject Deeds of
Mortgage that mortgagors debts subsequently obtained would be covered by
the same security. Hence, the payment made by GPT does not extinguish the
mortgage. The trial court ruled in favor of the respondent. On appeal, CA
reversed the decision. Respondent filed a motion for reconsideration.
With this unfavorable turn of events, respondent GTP, filed a motion for
reconsideration with alternative prayer to require METROBANK to furnish
appellee (GTP) of the alleged unpaid balance of Mr. Chia." At the re-scheduled
date of oral arguments where METROBANK was supposed to bring before the
CA the current statement of the mortgage debt of Mr. Chia secured by the deeds
of mortgage sought to be released, METROBANKs counsel did not appear.
Consequently, CA reversed its previous decision.
ISSUE: Whether or not failure of the mortgagee to present the proof of loan
secured by the mortgage estopped him from further asserting the existence of
such liability
HELD:
The answer is in the affirmative. It is a well-settled rule that when the evidence
tends to prove a material fact which imposes a liability on a party, and he has it
in his power to produce evidence which from its very nature must overthrow the
case made against him if it is not founded on fact, and he refuses to produce
such evidence, the presumption arises that the evidence, if produced, would
operate to his prejudice, and support the case of his adversary.

Page | 288

Compilation of Digested Cases for Remedial Law Review II


by: K. M. T. BUELA
The scheduled oral arguments before the CA was supposed to be Metrobanks
golden opportunity to prove the existence the "other unliquidated past due
loans" which is the basis of its refusal to release the mortgage property. But
Metrobank failed to appear thereon.
No rule of law is better settled than that a party having it in his power to prove
a fact, if it exists, which, if proved, would benefit him, his failure to prove it
must be taken as conclusive that the fact does not exist."

4. Official Duty
FILOMENA G. DELOS SANTOS vs. COMMISSION ON AUDIT
G.R. No. 198457, August 13, 2013
FACTS:
Congressman Cuenco entered into a Memorandum of Agreement with the
Vicente Sotto Memorial Medical Center (VSMMC) appropriating to the
hospital the amount of P1,500,000.00 from his Priority Development Assistance
Fund (PDAF) to cover the medical assistance of indigent patients under the
Tony N' Tommy (TNT) Health Program. It was agreed that Cuenco shall
identify and recommend the patients who may availed of the program.
Several years after the enforcement of the MOA, allegations of forgery and
falsification of prescriptions and referrals for the availment of medicines under
the TNT Program surfaced. Consequently, an audit thereof was conducted and
rampant violations of bidding and audit procedure were revealed. Thereafter,
Special Audit Team Supervisor, Boado disallowed the amount of P3,386,697.10
for the payment of drugs and medicines for anti-rabies with falsified
prescription and documents, and holding petitioners, together with other
VSMMC officials, solidarily liable therefor.
By way of defense, petitioners nonetheless argue that VSMMC was merely a
passive entity in the disbursement of funds under the TNT Program and, thus,
invoke good faith in the performance of their respective duties, capitalizing on
the failure of the assailed Decisions of the CoA to show that their lapses in the
implementation of the TNT Program were attended by malice or bad faith.
ISSUE: Whether or not assertion of good faith in the performance of a public
function prevails over factual findings revealing violations of rules and
regulations in the performance of such function.
HELD:
Page | 289

Compilation of Digested Cases for Remedial Law Review II


by: K. M. T. BUELA
Jurisprudence holds that, absent any showing of bad faith and malice, there is a
presumption of regularity in the performance of official duties. However, this
presumption must fail in the presence of an explicit rule that was violated.
Petitioners failed to make a case justifying their non-observance of existing
auditing rules and regulations, and of their duties under the MOA. Evidently,
petitioners neglect to properly monitor the disbursement of Cuenco's PDAF
facilitated the validation and eventual payment of 133 falsified prescriptions and
fictitious claims for anti-rabies vaccines supplied by both the VSMMC and Dell
Pharmacy, despite the patent irregularities borne out by the referral slips and
prescriptions related thereto. Had there been an internal control system installed
by petitioners, the irregularities would have been exposed, and the hospital
would have been prevented from processing falsified claims and unlawfully
disbursing funds from the said PDAF.

PEOPLE OF THE PHILIPPINES vs. HADJI SOCOR CADIDIA


G.R. No. 191263, October 16, 2013
FACTS:
Accused was charged with violation of RA 9165. The prosecution presented
Trayvilla, a Non-Uniformed Personnel of the PNP, who testified that on 31 July
2002 at around 6:30 in the morning, while performing her duty as a female
frisker assigned at the NAIA Terminal I, she frisked the accused Cadidia upon
her entry at the departure area and she noticed something unusual and thick in
the area of Cadidias buttocks. Upon inquiry, Cadidia answered that it was only
her sanitary napkin which caused the unusual thickness. Not convinced with
Cadidias explanation, Trayvilla and her female co-employee Bagsican brought
the accused to the comfort room inside the domestic airport to check. When she
and Bagsican asked Cadidia to remove her underwear, they discovered that
inside were two sachets of shabu. The two sachets of shabu were turned over to
their supervisor SPO3 Musalli I. Appang (SPO3 Appang). Trayvilla recalled
that Cadidia denied that the two sachets of shabu were hers and said that she
was only asked by an unidentified person to bring the same.
During trial, accused interposed the defense of frame-up. Both the trial court
and the CA, on appeal, conviced the accused. The accused also assails the
application of presumption of regularity in the performance of duties of the
witnesses. She claimed that the self- serving testimonies of Trayvilla and
Bagsican failed to overcome her presumption of innocence guaranteed by the
Constitution.
ISSUE: Whether or not the presumption of regularity in the performance of
duties of a public officer may be assailed by bare allegations of frame-up.
Page | 290

Compilation of Digested Cases for Remedial Law Review II


by: K. M. T. BUELA

HELD:
The answer is in the affirmative. In cases involving violations of the Dangerous
Drugs Act, credence is given to prosecution witnesses who are police officers
for they are presumed to have performed their duties in a regular manner, unless
there is evidence to the contrary suggesting ill-motive on the part of the police
officers.
In this case, the prosecution witnesses were unable to show ill-motive for the
police to impute the crime against Cadidia. Trayvilla was doing her regular duty
as an airport frisker when she handled the accused who entered the x-ray
machine of the departure area. There was no pre-determined notice to
particularly search the accused especially in her private area. The unusual
thickness of the buttocks of the accused upon frisking prompted Trayvilla to
notify her supervisor SPO3 Appang of the incident. The subsequent search of
the accused would only show that the two female friskers were just doing their
usual task when they found the illegal drugs inside accuseds underwear. This
is bolstered by the fact that the accused on the one hand and the two friskers on
the other were unfamiliar to each other. Neither could they harbour any ill-will
against each other. The allegation of frame-up and denial of the accused cannot
prevail over the positive testimonies of three prosecution witnesses who
corroborated on circumstances surrounding the apprehension.

5. Cohabitation
PEOPLE OF THE PHILIPPINES vs. JESUS EDUALINO
G.R. No. 119072, April 11, 1997
FACTS:
Accused Jesus Edualino was charged with consummated rape of a pregnant
woman.
Prosecution presented the following witnesses: (1) Rowena Nantiza
victim/complainant; Aileen Yayen eyewitness; and Dr. Rogelio Divinagracia
medico-legal. On the other hand, accused-appellant relies on alternative
defenses of alibi and consent on the part of complainant. While accusedappellant's defense before the trial court alleges that he had left the scene of the
incident together with defense witness Calixto Flora, he alternatively raises
before this Court the contention that the elements of the crime of rape have not
been established.

Page | 291

Compilation of Digested Cases for Remedial Law Review II


by: K. M. T. BUELA
Accused-appellant raises the issue of the character of complainant Rowena
Nantiza. It is argued that a responsible and decent married woman, who was
then three (3) months pregnant, would not be out at two (2) o'clock in the
morning getting drunk much less would a decent Filipina ask a man to
accompany her to drink beer. It is contended that complainant merely concocted
the charge of rape to save her marriage since her husband had found out that she
was using drugs and drinking alcohol and even made a spectacle of herself when
she tried to seduce accused-appellant on 11 May 1994 while she was under the
influence of drug and alcohol.
ISSUE: Whether or not the accused may question his conviction by assailing
the character of the victim.
HELD:
It should be pointed out that the moral character of a rape victim is immaterial
in the prosecution and conviction of the accused. The Court has ruled that
prostitutes can be the victims of rape.
In the present case, even if accused-appellant's allegations that the victim was
drunk and under the influence of drugs and that she (the victim) cannot be
considered a decent and responsible married woman, were true, said
circumstances will not per se preclude a finding that she was raped.
The Court has repeatedly held that a medical examination of the victim is not a
prerequisite in prosecutions for rape. A person accused of rape can be convicted
solely on the testimony of the victim provided the testimony is credible, natural,
convincing and otherwise consistent with human nature and the course of
things.

6. Legitimacy

7. Survivorship

Page | 292

Compilation of Digested Cases for Remedial Law Review II


by: K. M. T. BUELA

E. Presentation of Evidence
1. Order of Presentation of Evidence
PEOPLE OF THE PHILIPPINES vs. LEONARDO FABRE y
VICENTE
G.R. No. 146697, July 23, 2002
FACTS:
Leonardo Fabre was adjudged guilty by the Regional Trial Court of raping his
own daughter Marilou Fabre.
At the trial, the prosecution presented the testimony of Marilou, that of Adela
Fabre, her mother and the wife of the accused, and that of Dr. Reinerio Jalalon,
the doctor who examined Marilou, along with the medico-legal certificate
issued by Dr. Jalalon, the sworn statement of Adela, and the criminal complaint
signed by both Marilou and Adela. The defense, during its turn in the
presentation of evidence, countered with the testimony of the accused himself.
On appeal, the defense argues, that the testimony of appellant should acquire
added strength for the failure of the prosecution to conduct cross-examination
on him and to present any rebuttal evidence.
ISSUE: Whether or not the trial court should give full weight as to the veracity
of a testimony which was not subjected to cross-examination by the adverse
party.
HELD:
The answer is in the negative. The cross-examination of a witness is a
prerogative of the party against whom the witness is called. The purpose of
cross-examination is to test the truth or accuracy of the statements of a witness
made on direct examination. The party against whom the witness testifies may
deem any further examination unnecessary and instead rely on any other
evidence theretofore adduced or thereafter to be adduced or on what would be
believed is the perception of the court thereon. Certainly, the trial court is not
bound to give full weight to the testimony of a witness on direct examination
merely because he is not cross-examined by the other party.

Page | 293

Compilation of Digested Cases for Remedial Law Review II


by: K. M. T. BUELA
2. Leading and Misleading Questions
PEOPLE OF THE PHILIPPINES vs. JESUS PEREZ y SEBUNGA
G.R. No. 142556, February 5, 2003
FACTS:
For automatic review is the Decision of the Regional Trial Court finding
appellant Jesus S. Perez guilty of raping Mayia P. Ponseca and imposing on
appellant the death penalty.
On appeal, Appellant contends that his identification in open court by Mayia
was highly irregular. Appellant points out that the prosecutor had already
identified him as the man wearing an orange t-shirt when the prosecutor asked
Mayia to identify her alleged rapist. Appellant stresses that when Mayia
identified him in open court, she referred to him as a man named "Johnny" and
did not give any description or any identifying mark. Moreover, appellant
claims he was alone in the cell when Mayia identified him after the police
arrested him. Appellant bewails that the identification was not done with the
usual police line-up.
ISSUE: Whether or not the testimony of the minor-victim should be expunged
on the ground that leading questions were asked during her testimony in court.
HELD:
The answer is in the negative. As a rule, leading questions are not allowed.
However, the rules provide for exceptions when the witness is a child of tender
years as it is usually difficult for such child to state facts without prompting or
suggestion. Leading questions are necessary to coax the truth out of their
reluctant lips. In the case at bar, the trial court was justified in allowing leading
questions to Mayia as she was evidently young and unlettered, making the recall
of events difficult, if not uncertain.
The trend in procedural law is to give wide latitude to the courts in exercising
control over the questioning of a child witness. The reasons are spelled out in
our Rule on Examination of a Child Witness, which took effect on December
15, 2000, namely, (1) to facilitate the ascertainment of the truth, (2) to ensure
that questions are stated in a form appropriate to the developmental level of the
child, (3) to protect children from harassment or undue embarrassment, and (4)
avoid waste of time. Leading questions in all stages of examination of a child
are allowed if the same will further the interests of justice."

Page | 294

Compilation of Digested Cases for Remedial Law Review II


by: K. M. T. BUELA
3. Impeachment
PEOPLE OF THE PHILIPPINES vs. JAIME CASTILLANO, SR. et al.
G.R. No. 139412, April 2, 2003
FACTS:
This is an appeal from the Decision1 of the Regional Trial Court convicting
appellants Ronald Castillano alias "Nono" and Jaime Castillano, Jr. of murder
of the Diosdado Volante.
On appeal, Appellant Jaime, Jr. avers that the prosecution failed to prove his
guilt beyond reasonable doubt of the crime charged. He asserts that the
testimony of Luz Volante, the widow of Diosdado, was inconsistent with her
testimony during the preliminary examination in the municipal trial court and
her sworn statement before the police investigators as well as the testimonies of
SPO1 Fornillos and SPO4 Jaime Favier, and the physical evidence on record.
On the other hand, the Office of the Solicitor General asserts that the credibility
of the testimony of Luz, the prosecutions principal witness, cannot be
impeached via her testimony during the preliminary examination before the
municipal trial court nor by her sworn statement given to the police investigators
for the reason that the transcripts and sworn statement were neither marked and
offered in evidence by the appellants nor admitted in evidence by the trial court.
Moreover, the appellants did not confront Luz with her testimony during the
preliminary examination and her sworn statement to the police investigators.
Luz was not, therefore, accorded a chance to explain the purported
inconsistencies, as mandated by Section 13, Rule 132 of the Revised Rules of
Evidence
ISSUE: Whether or not a testimony made in open court may be impeached by
asserting that the said testimony is inconsistent with those made by the witness
in the preliminary examination.
HELD:
The answer is in the negative. Before the credibility of a witness and the
truthfulness of his testimony can be impeached by evidence consisting of his
prior statements which are inconsistent with his present testimony, the crossexaminer must lay the predicate or the foundation for impeachment and thereby
prevent an injustice to the witness being cross-examined. The witness must be
given a chance to recollect and to explain the apparent inconsistency between
his two statements and state the circumstances under which they were made.51
This Court held in People v. Escosura52 that the statements of a witness prior
to her present testimony cannot serve as basis for impeaching her credibility
Page | 295

Compilation of Digested Cases for Remedial Law Review II


by: K. M. T. BUELA
unless her attention was directed to the inconsistencies or discrepancies and she
was given an opportunity to explain said inconsistencies. In a case where the
cross-examiner tries to impeach the credibility and truthfulness of a witness via
her testimony during a preliminary examination.
In this case, the appellants never confronted Luz with her testimony during the
preliminary examination and her sworn statement. She was not afforded any
chance to explain any discrepancies between her present testimony and her
testimony during the preliminary examination and her sworn statement. The
appellants did not even mark and offer in evidence the said transcript and sworn
statement for the specific purpose of impeaching her credibility and her present
testimony. Unless so marked and offered in evidence and accepted by the trial
court, said transcript and sworn statement cannot be considered by the court.
4. Reference to Memorandum
PEOPLE OF THE PHILIPPINES vs. ANTONIO PLASENCIA y
DESAMPARADO
G.R. No. 90198, November 7, 1995
FACTS:
Antonio Plasencia, Roberto Descartin and Joelito (Julito) Descartin were
accused of robbery with homicide. The Regional Trial Court did not give
credence to the defense of alibi. It convicted the three accused of murder
The instant appeal was interposed by the three convicted appellants. Appellants
attack the credibility of the prosecution's lone eyewitness. It is asserted that the
testimony of Francisca Espina should not be given worth since, while testifying,
she would at times be seen reading some notes written on her left palm.
ISSUE: Whether or not testimony of a witness, who during her testimony was
seen looking at some notes written on her palm, should not be given credence
HELD:
The answer is in the negative. The use of memory aids during an examination
of a witness is not altogether proscribed. Section 16, Rule 132, of the Rules of
Court states: A witness may be allowed to refresh his memory respecting a fact,
by anything written or recorded by himself or under his direction at the time
when the fact occurred, or immediately thereafter, or at any other time when
the fact was fresh in his memory and he knew that the same was correctly written
or recorded; but in such case the writing or record must be produced and may
be inspected by the adverse party, who may, if he chooses, cross-examine the
witness upon it and may read it in evidence. So, also, a witness may testify from
Page | 296

Compilation of Digested Cases for Remedial Law Review II


by: K. M. T. BUELA
such a writing or record, though he retain no recollection of the particular facts,
if he is able to swear that the writing or record correctly stated the transaction
when made; but such evidence must be received with caution. (Emphasis
supplied.)
Allowing a witness to refer to her notes rests on the sound discretion of the trial
court. In this case, the exercise of that discretion has not been abused; the
witness herself has explained that she merely wanted to be accurate on dates
and like details.

ROSELLA D. CANQUE vs. THE COURT OF APPEALS and SOCOR


CONSTRUCTION CORPORATION
G.R. No. 96202, April 13, 1999
FACTS:
Petitioner entered into two contracts with private respondent Socor
Construction Corporation. Due to Petitioners refusal to pay the amount billed
by the Respondent, the latter brought a suit to recover from the former the sum
of P299,717.75, plus interest at the rate of 3% a month.
To prove the unpaid bills of the petitioner, private respondent presented its Book
of Collectible Accounts and their bookkeeper, Dolores Aday to testify on the
entries of the said book. During the trial, Aday admitted that she had no personal
knowledge of the facts constituting the entry. She said she made the entries
based on the bills given to her. But she has no knowledge of the truth or falsity
of the facts stated in the bills. The deliveries of the materials stated in the bills
were supervised by "an engineer for such functions.
The trial court, giving due weight to the plaintiffs Book of Collectible
Accounts, as well as Adays testimony, ruled in favor of respondent. On appeal,
said decision was affirmed by the CA. Petitioner contends that Adays
testimony is considered a hearsay for lack of personal knowledge of the entries
made as the information entered was merely provided to her by the engineers of
the respondent.
It is nonetheless argued by private respondent that although the entries cannot
be considered an exception to the hearsay rule, they may be admitted under Rule
132, Section 10 of the Rules of Court. On the other hand, petitioner contends
that evidence which is inadmissible for the purpose for which it was offered
cannot be admitted for another purpose.
ISSUES:
Page | 297

Compilation of Digested Cases for Remedial Law Review II


by: K. M. T. BUELA
(1) Whether or not testimony of a witness as to entries made by her based on
bills provided to her is inadmissible on the ground of hearsay evidence rule.
(2) Whether or not the testimony of a witness as to entries made by her based
on information provided to her may be admissible as memorandum used to
refresh the memory of the witness
HELD:
(1)
The answer is in the affirmative. When the witness had no personal knowledge
of the facts entered by him, and the person who gave him the information is
individually known and may testify as to the facts stated in the entry which is
not part of a system of entries where scores of employees have intervened, such
entry is not admissible without the testimony of the informer.
(2)
The answer is in the negative. Assuming that the book of collectible accounts
presented by the respondent would qualify as a memorandum, the
memorandum used to refresh the memory of the witness does not constitute
evidence, and may not be admitted as such, for the simple reason that the
witness has just the same to testify on the basis of refreshed memory. In
other words, where the witness has testified independently of or after his
testimony has been refreshed by a memorandum of the events in dispute, such
memorandum is not admissible as corroborative evidence. It is self-evident that
a witness may not be corroborated by any written statement prepared wholly by
him. He cannot be more credible just because he supports his open-court
declaration with written statements of the same facts even if he did prepare them
during the occasion in dispute, unless the proper predicate of his failing memory
is priorly laid down. What is more, even where this requirement has been
satisfied, the express injunction of the rule itself is that such evidence must be
received with caution, if only because it is not very difficult to conceive and
fabricate evidence of this nature. This is doubly true when the witness stands to
gain materially or otherwise from the admission of such evidence

5. Classes of Documents
YASUO IWASAWA vs. FELISA CUSTODIO GANGAN AND THE
LOCAL CIVIL REGISTRAR OF PASAY CITY
G.R. No. 204169, September 11, 2013
FACTS:
Petitioner, a Japanese national married Private respondent in 2002. In July 2009,
petitioner noticed his wife become depressed. Suspecting that something might
Page | 298

Compilation of Digested Cases for Remedial Law Review II


by: K. M. T. BUELA
have happened in the Philippines, he confronted his wife about it. To his shock,
private respondent confessed to him that she received news that her previous
husband passed away. This prompted petitioner to file a petition for the
declaration of his marriage to private respondent as null and void on the ground
that their marriage is a bigamous one
During trial, aside from his testimony, petitioner also offered the following
pieces of documentary evidence issued by the NSO: (1) Certificate of Marriage
between petitioner and private respondent to prove the fact of marriage between
the parties on November 28, 2002; (2) Certificate of Marriage between private
respondent and Raymond Maglonzo Arambulo to prove the fact of marriage
between the parties on June 20, 1994
Certificate of Death of Raymond Maglonzo Arambulo; (3) Certification from
the NSO to the effect that there are two entries of marriage recorded by the
office pertaining to private respondent
RTC rendered the assailed decision. It ruled that there was insufficient evidence
to prove private respondents prior existing valid marriage to another man. It
held that while petitioner offered the certificate of marriage of private
respondent to Arambulo, it was only petitioner who testified about said
marriage.
The RTC ruled that petitioners testimony is unreliable because he has no
personal knowledge of private respondents prior marriage nor of Arambulos
death which makes him a complete stranger to the marriage certificate between
private respondent and Arambulo and the latters death certificate. It further
ruled that petitioners testimony about the NSO certification is likewise
unreliable since he is a stranger to the preparation of said document.
ISSUE: Whether or not the testimony of the NSO records custodian certifying
the authenticity and due execution of the public documents issued by said office
was necessary before they could be accorded evidentiary weight.
HELD:
The answer in the negative. There is no question that the documentary evidence
submitted by petitioner are all public documents. Art. 410 of the Civil Code
provides that the books making up the civil register and all documents relating
thereto shall be considered public documents and shall be prima facie evidence
of the facts therein contained.
As public documents, they are admissible in evidence even without further
proof of their due execution and genuineness. Thus, the RTC erred when it
disregarded said documents on the sole ground that the petitioner did not present
Page | 299

Compilation of Digested Cases for Remedial Law Review II


by: K. M. T. BUELA
the records custodian of the NSO who issued them to testify on their authenticity
and due execution since proof of authenticity and due execution was not
anymore necessary. Moreover, not only are said documents admissible, they
deserve to be given evidentiary weight because they constitute prima facie
evidence of the facts stated therein. And in the instant case, the facts stated
therein remain unrebutted since neither the private respondent nor the public
prosecutor presented evidence to the contrary.

ASIAN TERMINALS, INC. vs. PHILAM INSURANCE CO., INC.


G.R. No. 181163, July 24, 2013
FACTS:
Nichimen Corporation shipped to Universal Motors 219 packages containing
120 units of brand new Nissan Pickup Truck on board the vessel S/S "Calayan
Iris" from Japan to Manila. The shipment was insured with Philam against all
risks. When the package arrived and was unloaded by ATI, it was found that the
package marked as 03-245-42K/1 was in bad order. The shipment was
withdrawn by R.F. Revilla Customs Brokerage, Inc., the authorized broker of
Universal Motors, and delivered to the latters warehouse. Owing to the extent
of the damage to said cargoes, Universal Motors declared them a total loss.
Universal Motors filed a formal claim for damages against Westwind, ATI10
and R.F. Revilla Customs Brokerage, Inc. When Universal Motors demands
remained unheeded, it sought reparation from and was compensated by Philam.
Accordingly, Universal Motors issued a Subrogation Receipt in favor of Philam.
RTC rendered judgment in favor of Philam and ordered Westwind and ATI to
pay Philam. On appeal, the CA affirmed with modification the ruling of the
RTC. When the case was elevated to the SC, petitioners objected to the
admission of Marine Certificate and the Subrogation Receipt for being hearsay
as they were not authenticated by the persons who executed them.
ISSUE: Whether or not certificates and/or receipts offered as evidence required
authentication.
HELD:
The answer is in the affirmative. The nature of documents as either public or
private determines how the documents may be presented as evidence in court.
Public documents, as enumerated under Section 19, Rule 132 of the Rules of
Court, are self-authenticating and require no further authentication in order to
be presented as evidence in court.

Page | 300

Compilation of Digested Cases for Remedial Law Review II


by: K. M. T. BUELA
In contrast, a private document is any other writing, deed or instrument executed
by a private person without the intervention of a notary or other person legally
authorized by which some disposition or agreement is proved or set forth.
Lacking the official or sovereign character of a public document, or the
solemnities prescribed by law, a private document requires authentication in the
manner prescribed under Section 20, Rule 132 of the Rules:
SEC. 20. Proof of private document. Before any private document offered as
authentic is received in evidence, its due execution and authenticity must be
proved either:
(a) By anyone who saw the document executed or written; or
(b) By evidence of the genuineness of the signature or handwriting of the maker.
Any other private document need only be identified as that which it is claimed
to be.
The requirement of authentication of a private document is excused only in four
instances, specifically: (a) when the document is an ancient one within the
context of Section 21, Rule 132 of the Rules; (b) when the genuineness and
authenticity of the actionable document have not been specifically denied under
oath by the adverse party; (c) when the genuineness and authenticity of the
document have been admitted; or (d) when the document is not being offered as
genuine.
Indubitably, Marine Certificate No. 708-8006717-4 and the Subrogation
Receipt are private documents which Philam and the consignee, respectively,
issue in the pursuit of their business. Since none of the exceptions to the
requirement of authentication of a private document obtains in these cases, said
documents may not be admitted in evidence for Philam without being properly
authenticated.

6. Offer of Evidence
LOMISES ALUDOS, deceased, substituted by FLORA ALUDOS vs.
JOHNNY M. SUERTE
G.R. No. 165285, June 18, 2012
FACTS:
Lomises acquired from the Baguio City Government the right to occupy two
stalls in the Hangar Market in Baguio City. Lomises entered into an agreement
with respondent Johnny M. Suerte for the transfer of all improvements and
rights over the two market stalls. Before full payment could be made, however,
Page | 301

Compilation of Digested Cases for Remedial Law Review II


by: K. M. T. BUELA
Lomises backed out of the agreement and returned the P68,000.00.Thus, Johnny
filed a complaint against Lomises for specific performance with damages.
RTC nullified the agreement between Johnny and Lomises for failure to secure
the consent of the Baguio City Government to the agreement. The RTC found
that Lomises was a mere lessee of the market stalls, and the Baguio City
Government was the owner-lessor of the stalls. On appeal, CA agreed with the
RTC that the assignment of the leasehold rights was void for lack of consent of
the lessor, the Baguio City Government. The sale of the improvements,
however, was valid because these were Lomises private properties.
Lomises, however, objects to the CA ruling upholding the validity of the
agreement insofar as it involved the sale of improvements on the stalls. Lomises
alleges that the sale of the improvements should similarly be voided because it
was made without the consent of the Baguio City Government, the owner of the
improvements, pursuant to the May 1, 1985 lease contract.
ISSUE: Whether or not a document repeatedly mentioned in the pleadings of
the parties which was presented only on appeal may be considered for purposes
of adjudicating the case.
HELD:
The answer is in the negative. The CA has already rejected the evidentiary value
of the May 1, 1985 lease contract between the Baguio City Government and
Lomises, as it was not formally offered in evidence before the RTC; in fact, the
CA admonished Lomises lawyer, Atty. Lockey, for making it appear that it was
part of the records of the case. Under Section 34, Rule 132 of the Rules of Court,
the court shall consider no evidence which has not been formally offered. "The
offer of evidence is necessary because it is the duty of the court to rest its
findings of fact and its judgment only and strictly upon the evidence offered by
the parties. Unless and until admitted by the court in evidence for the purpose
or purposes for which such document is offered, the same is merely a scrap of
paper barren of probative weight." Although the contract was referred to in
Lomises answer to Johnnys complaint and marked as Exhibit "2" in his pretrial brief, a copy of it was never attached. In fact, a copy of the May 1, 1985
lease contract "surfaced" only after Lomises filed a motion for reconsideration
of the CA decision.

Page | 302

Compilation of Digested Cases for Remedial Law Review II


by: K. M. T. BUELA
WESTMONT INVESTMENT CORPORATION
FRANCIA, JR. et al.
G.R. No. 194128, December 7, 2011

vs.

AMOS

P.

FACTS:
Respondents filed a Complaint for Collection of Sum of Money and Damages
arising from their investments against petitioner Westmont Investment
Corporation (Wincorp) and respondent Pearlbank Securities Inc. the case was
set for the presentation of the defense evidence of Wincorp.
On March 7, 2003, three (3) days before the scheduled hearing, Wincorp filed
a written motion to postpone the hearing. The RTC denied Wincorps Motion
to Postpone and considered it to have waived its right to present evidence. The
Motion for Reconsideration of Wincorp was likewise denied.
On September 27, 2004, the RTC rendered a decision in favor of the Francias
and held Wincorp solely liable to them. The CA affirmed with modification the
ruling of the RTC. Wincorp filed an MR with the CA attaching to the said
motion evidentiary evidence which it was not able to present during trial.
ISSUE: Whether or not documents attached to a motion for reconsideration
made before the appellate court may be considered for purposes of adjudicating
the merits of the case.
HELD:
The answer is in the negative. Section 34, Rule 132 of the Rules on Evidence
states that: "The court shall consider no evidence which has not been formally
offered. The purpose for which the evidence is offered must be specified."
A formal offer is necessary because judges are mandated to rest their findings
of facts and their judgment only and strictly upon the evidence offered by the
parties at the trial. Its function is to enable the trial judge to know the purpose
or purposes for which the proponent is presenting the evidence. On the other
hand, this allows opposing parties to examine the evidence and object to its
admissibility. Moreover, it facilitates review as the appellate court will not be
required to review documents not previously scrutinized by the trial court.
Evidence not formally offered during the trial cannot be used for or against a
party litigant. Neither may it be taken into account on appeal.
The rule on formal offer of evidence is not a trivial matter. Failure to make a
formal offer within a considerable period of time shall be deemed a waiver to
submit it. Consequently, any evidence that has not been offered shall be
excluded and rejected.
Page | 303

Compilation of Digested Cases for Remedial Law Review II


by: K. M. T. BUELA

Prescinding therefrom, the very glaring conclusion is that all the documents
attached in the motion for reconsideration of the decision of the trial court and
all the documents attached in the defendant-appellants brief filed by defendantappellant Wincorp cannot be given any probative weight or credit for the sole
reason that the said documents were not formally offered as evidence in the trial
court because to consider them at this stage will deny the other parties the right
to rebut them.
7. Tender of Excluded Evidence

F. Weight and Sufficiency of Evidence


1. Hierarchy of Evidence
Proof Beyond Reasonable Doubt
PEOPLE OF THE PHILIPPINES vs. DELFIN CALISO
G.R. No. 183830, October 19, 2011
FACTS:
Caliso was arraigned and tried for rape with homicide, but the Regional Trial
Court found him guilty of murder for the killing of AAA, a mentally-retarded
16-year old girl, and sentenced him to death.
On appeal, CA affirmed Calisos conviction for murder based on the same
ratiocinations the RTC had rendered. The CA also relied on the identification
by Amegable of Caliso, despite his back being turned towards her during the
commission of the crime. The CA ruled that she made a positive identification
of Caliso as the perpetrator of the killing, observing that the incident happened
at noon when the sun had been at its brightest, coupled with the fact that
Amegables view had not been obstructed by any object at the time that AAAs
body had been submerged in the water
ISSUE: Whether or not the testimony of a witness attesting to identifying the
perpetrator of a crime by merely seeing his back constitutes positive
identification that warrants conviction.
HELD:
The answer is in the negative. No matter how honest Amegables testimony
might have been, her identification of Caliso by a sheer look at his back for a
few minutes could not be regarded as positive enough to generate that moral

Page | 304

Compilation of Digested Cases for Remedial Law Review II


by: K. M. T. BUELA
certainty about Caliso being the perpetrator of the killing, absent other reliable
circumstances showing him to be AAAs killer.
In every criminal prosecution, the identity of the offender, like the crime itself,
must be established by proof beyond reasonable doubt. Indeed, the first duty of
the Prosecution is not to prove the crime but to prove the identity of the criminal,
for even if the commission of the crime can be established, there can be no
conviction without proof of identity of the criminal beyond reasonable doubt.
There are two types of positive identification. A witness may identify a suspect
or accused in a criminal case as the perpetrator of the crime as an eyewitness to
the very act of the commission of the crime. This constitutes direct evidence.
There may, however, be instances where,although a witness may not have
actually seen the very act of commission of a crime, he may still be able to
positively identify a suspect or accused as the perpetrator of a crime as for
instance when the latter is the person or one of the persons last seen with the
victim immediately before and right after the commission of the crime. This is
the second type of positive identification, which forms part of circumstantial
evidence, which, when taken together with other pieces of evidence constituting
an unbroken chain, leads to only fair and reasonable conclusion, which is that
the accused is the author of the crime to the exclusion of all others. If the actual
eyewitnesses are the only ones allowed to possibly positively identify a suspect
or accused to the exclusion of others, then nobody can ever be convicted unless
there is an eyewitness, because it is basic and elementary that there can be no
conviction until and unless an accused is positively identified. Such a
proposition is absolutely absurd, because it is settled that direct evidence of the
commission of a crime is not the only matrix wherefrom a trial court may draw
its conclusion and finding of guilt. If resort to circumstantial evidence would
not be allowed to prove identity of the accused on the absence of direct
evidence, then felons would go free and the community would be denied proper
protection.
Amegables identification of Caliso as the perpetrator did not have unassailable
reliability, the only means by which it might be said to be positive and sufficient.
The test to determine the moral certainty of an identification is its
imperviousness to skepticism on account of its distinctiveness. To achieve such
distinctiveness, the identification evidence should encompass unique physical
features or characteristics, like the face, the voice, the dentures, the
distinguishing marks or tattoos on the body, fingerprints, DNA, or any other
physical facts that set the individual apart from the rest of humanity.

Page | 305

Compilation of Digested Cases for Remedial Law Review II


by: K. M. T. BUELA
PEOPLE OF THE PHILIPPINE vs. FELIMON PATENTES y
ZAMORA
G.R. No. 190178, February 12, 2014
FACTS:
Accused was charged with forcible abduction with rape. During the trial,
complainant testified that she was abducted by the accused and brought to the
latters residence where the former was repeatedly rape for 8 days. However, in
the course the complainants testimony, she testified that in between those 8
days, she was able to visit her grandmother.
Furthermore, when prosecution presented Dr. Cruz testified that he examined
AAA. In his report, he noted the following observations about AAA: (1)
contusion on the breast caused by a kiss mark; (2) hymen was intact and can
readily admit a normal-sized erect male penis without sustaining any injury; and
(3) vaginal canal was negative for spermatozoa. Dr. Cruz also added that he
cannot tell whether it was AAAs first sexual intercourse as the vagina was not
injured but had healed lacerations.
On the other hand, the defense presented Wilma Enriquez, a friend of the
complainant, who testified that after the dates wherein the latter was supposedly
rape, the latter visited her and discussed plans about marrying the accused. Trial
court convicted the accused. On appeal with CA, conviction of the accused was
confirmed.
ISSUE: Whether or not a person accused of rape may be convicted based solely
on the testimony of victim who positively identified him to be the perpetrator
notwithstanding blatant inconsistencies therein.
HELD:
In reviewing rape cases, the Court is guided by the following principles: (1) to
accuse a man of rape is easy, but to disprove the accusation is difficult, though
the accused may be innocent; (2) inasmuch as only two persons are usually
involved in the crime of rape, the testimony of the complainant should be
scrutinized with great caution; and (3) the evidence for the prosecution must
stand or fall on its own merit and should not be allowed to draw strength from
the weakness of the evidence for the defense. So long as the private
complainants testimony meets the test of credibility, the accused may be
convicted on the basis thereof. The time-honored test in determining the value
of the testimony of a witness is its compatibility with human knowledge,
observation and common experience of man. Thus, whatever is repugnant to the
standards of human knowledge, observation and experience becomes incredible
and must lie outside judicial cognizance.
Page | 306

Compilation of Digested Cases for Remedial Law Review II


by: K. M. T. BUELA

The following inconsistencies in complainants testimony is found to be


incompatible to human experience: (1) the admission that she during her
abduction she was brought to accuseds house where 8 family members of the
latter also resides; (2) that she was not able to ask for help from any of the family
member nor any of them was able to realize that accused was keeping her
against her will; (3) the discussion of wedding plans with her friend after her
ordeal; (4) admission that she was able to visit her grandmother within the
period of the alleged abduction when she was supposed to be kept inside
accuseds house against her will; and (5) lastly, that she was repeated mauled
by accused on their way to his to residence and within the duration of abduction
but no physical injuries was seen during the medical examination conducted
after the incident. A conviction in a criminal case must be supported by proof
beyond reasonable doubt, which means a moral certainty that the accused is
guilty; the burden of proof rests upon the prosecution. In the case at bar, the
prosecution has failed to discharge its burden of establishing with moral
certainty the truthfulness of the charge that appellant had carnal knowledge of
AAA against her will using threats, force or intimidation.

Clear and Convincing


SUPREME COURT vs. EDDIE V. DELGADO et al.
A.M. No. 2011-07-SC, October 4, 2011
FACTS:
On 2 June 2011, SC Associate Justice and Second Division Chairperson
Antonio T. Carpio caused the transmittal of two (2) sealed Agenda to the Office
of Clerk of Court Second Division (OCC-SD).
The said Agenda contain an itemized list of cases taken up by the Courts
Second Division during the sessions held on the concerned date and the
handwritten marginal notes of Justice Carpio noting the specific actions adopted
by the division on each case. Owing to the confidential nature of the contents of
an Agenda, the OCC-SD follows a very strict procedure in handling them.Thus,
only a few specified personnel within the OCC-SD are authorized to have access
to an Agenda e.g., only Ms. Puno is authorized to receive and open; only four
(4) persons are authorized to photocopy.
Herein Respondents were charged with grave misconduct for taking specific
pages in the said agenda without being authorized thereto. The complicity of
each respondent are as follows: Madeja and Florendo asked respondent Delgado
for a copy of several items included in the 30 May 2011 Agenda. Acceding to
the request, respondent Delgado removed pages 58, 59 and 70 from a copy of
Page | 307

Compilation of Digested Cases for Remedial Law Review II


by: K. M. T. BUELA
the Agenda entrusted to him for stitching and gave them to respondents Madeja
and Florendo.
During the initial and formal investigation, Delgado admitted that he removed
and took the said pages from the agenda and gave the same to Madeja and
Florendo. However, while respondents Madeja and Florendo admitted during
the initial investigation that they asked for and, in fact, obtained the missing
pages in the 30 May 2011 Agenda, they vehemently denied having been
involved in the taking of the missing Agenda pages during the formal
investigation of the OAS.
ISSUE: Whether or not mere denials made by respondent during formal
investigation warrants prevails against the contrary testimony of their corespondent implicating them in the alleged deed.
HELD:
The answer is in the negative. The basic principle in Evidence is that denials,
unless supported by clear and convincing evidence, cannot prevail over the
affirmative testimony of truthful witnesses.
It was never shown that respondent Delgado was motivated by any ill will in
implicating respondents Madeja and Florendo. As a witness, the credibility of
respondent Delgado remained unsullied.

GOVERNMENT OF HONG KONG SPECIAL ADMINISTRATIVE


REGION vs. HON. FELIXBERTO T. OLALIA, JR. and JUAN
ANTONIO MUOZ
G.R. No. 153675, April 19, 2007
FACTS:
petitioner Hong Kong Special Administrative Region filed with the RTC of
Manila a petition for the extradition of private respondent. For his part, private
respondent filed, in the same case,- a petition for bail which was opposed by
petitioner.
Initially, the petition for bail was denied by Judge Bernardo. On October 22,
2001, Judge Bernardo, Jr. inhibited himself from further hearing the case. It was
then raffled off to Branch presided by respondent judge. On October 30, 2001,
private respondent filed a motion for reconsideration of the Order denying his
application for bail. This was granted by respondent judge.
On December 21, 2001, petitioner filed an urgent motion to vacate the above
Order, but it was denied by respondent judge in his Order. Hence, the instant
Page | 308

Compilation of Digested Cases for Remedial Law Review II


by: K. M. T. BUELA
petition. Petitioner alleged that the trial court committed grave abuse of
discretion amounting to lack or excess of jurisdiction in admitting private
respondent to bail; that there is nothing in the Constitution or statutory law
providing that a potential extraditee has a right to bail, the right being limited
solely to criminal proceedings.
ISSUE:
(1) What should be the quantum of evidence needed to grant such bail to a
potential extraditee.
HELD:
Clear and convincing evidence. Extradition has thus been characterized as the
right of a foreign power, created by treaty, to demand the surrender of one
accused or convicted of a crime within its territorial jurisdiction, and the
correlative duty of the other state to surrender him to the demanding state. It is
not a criminal proceeding. Even if the potential extraditee is a criminal, an
extradition proceeding is not by its nature criminal, for it is not punishment for
a crime, even though such punishment may follow extradition. It is sui generis,
tracing its existence wholly to treaty obligations between different nations. It is
not a trial to determine the guilt or innocence of the potential extraditee. Nor is
it a full-blown civil action, but one that is merely administrative in character. Its
object is to prevent the escape of a person accused or convicted of a crime and
to secure his return to the state from which he fled, for the purpose of trial or
punishment.
But while extradition is not a criminal proceeding, it is characterized by the
following: (a) it entails a deprivation of liberty on the part of the potential
extraditee and (b) the means employed to attain the purpose of extradition is
also "the machinery of criminal law." This is shown by Section 6 of P.D. No.
1069 (The Philippine Extradition Law) which mandates the "immediate arrest
and temporary detention of the accused" if such "will best serve the interest of
justice." We further note that Section allows the requesting state "in case of
urgency" to ask for the "provisional arrest of the accused, pending receipt of the
request for extradition;" and that release from provisional arrest "shall not
prejudice re-arrest and extradition of the accused if a request for extradition is
received subsequently."
While administrative in character, the standard of substantial evidence used in
administrative cases cannot likewise apply given the object of extradition law
which is to prevent the prospective extraditee from fleeing our jurisdiction. In
his Separate Opinion in Purganan, then Associate Justice, now Chief Justice
Reynato S. Puno, proposed that a new standard which he termed "clear and
convincing evidence" should be used in granting bail in extradition cases.
Page | 309

Compilation of Digested Cases for Remedial Law Review II


by: K. M. T. BUELA
According to him, this standard should be lower than proof beyond reasonable
doubt but higher than preponderance of evidence. The potential extraditee must
prove by "clear and convincing evidence" that he is not a flight risk and will
abide with all the orders and processes of the extradition court.

PEOPLE OF THE PHILIPPINES vs. ALFONSO FONTANILLA y


OBALDO
G.R. No. 177743, January 25, 2012
FACTS:
Jose Olais was walking along the provincial road when Alfonso Fontanilla
suddenly struck him in the head with a piece of wood called bellang. Olais fell
facedown to the ground, but Fontanilla hit him again in the head with a piece of
stone. Fontanilla desisted from hitting Olais a third time only because Joel
Marquez and Tirso Abunan, the sons-in-law of Olais, shouted at him, causing
him to run away. Marquez and Abunan rushed their father-in-law to a medical
clinic, where Olais was pronounced dead on arrival. Consequently, Fontanilla
was charged with murder.
At the trial, Fontanilla claimed self-defense. Prosecution presented the
physician who conducted the autopsy on the cadaver of Olais. She attested that
her post-mortem examination showed that Olais had suffered a fracture on the
left temporal area of the skull, causing his death. She opined that a hard object
or a severe force had hit the skull of the victim more than once, considering that
the skull had been already fragmented and the fractures on the skull had been
radiating.
The RTC rejected Fontanillas plea of self-defense by observing that he had "no
necessity to employ a big stone, inflicting upon the victim a mortal wound
causing his death" due to the victim attacking him only with bare hands. It noted
that Fontanilla did not suffer any injury despite his claim that the victim had
mauled him; that Fontanilla did not receive any treatment, and no medical
certificate attested to any injury he might have suffered, having been
immediately released from the hospital.
ISSUE: Whether or not testimony of accused alleging self-defense may be
given credence amidst gravity of the injury sustained by the victim.
HELD:
The answer is in the negative. Fontanilla pleaded self-defense. In order for selfdefense to be appreciated, he had to prove by clear and convincing evidence the
following elements: (a) unlawful aggression on the part of the victim; (b)
Page | 310

Compilation of Digested Cases for Remedial Law Review II


by: K. M. T. BUELA
reasonable necessity of the means employed to prevent or repel it; and (c) lack
of sufficient provocation on the part of the person defending himself. Unlawful
aggression is the indispensable element of self-defense, for if no unlawful
aggression attributed to the victim is established, self-defense is unavailing, for
there is nothing to repel.
The plea of self-defense was thus belied, for the weapons used by Fontanilla
and the location and number of wounds he inflicted on Olais revealed his intent
to kill, not merely an effort to prevent or repel an attack from Olais. We consider
to be significant that the gravity of the wounds manifested the determined effort
of the accused to kill his victim, not just to defend himself.

Preponderance of Evidence
PHILIPPINE COMMERCIAL INTERNATIONAL BANK
ANTONIO B. BALMACEDA and ROLANDO N. RAMOS
G.R. No. 158143, September 21, 2011

vs.

FACTS:
PCIB filed an action for recovery of sum of money with damages against
Antonio Balmaceda. PCIB alleged that Balmaceda, by taking advantage of his
position as branch manager, fraudulently obtained and encashed 34 Managers
checks amounting to 11M+. PCIB also impleaded Ramos as one of the
recipients of a portion of the proceeds from Balmacedas alleged fraud. For
failure to file an answer, Balmaceda was declared in default. On the other hand,
Ramos filed an Answer denying any knowledge of Balmacedas scheme.
Ramos admitted receiving money from Balmaceda as payment for the fighting
cocks that he sold to Balmaceda, but maintained that he had no knowledge of
the source of Balmacedas money.
RTC issued a decision in favor of PCIB. On appeal, the CA dismissed the
complaint against Ramos, holding that no sufficient evidence existed to prove
that Ramos colluded with Balmaceda in the latters fraudulent manipulations.8
According to the CA, the mere fact that Balmaceda made Ramos the payee in
some of the Managers checks does not suffice to prove that Ramos was
complicit in Balmacedas fraudulent scheme. It was also observed that other
persons were also named as payees in the checks that Balmaceda acquired and
encashed, and PCIB only chose to go after Ramos. With PCIBs failure to prove
Ramos actual participation in Balmacedas fraud, no legal and factual basis
exists to hold him liable.

Page | 311

Compilation of Digested Cases for Remedial Law Review II


by: K. M. T. BUELA
PCIB insists that since Ramos defense anchored on mere denial of any
participation in Balmacedas wrongdoing is an intrinsically weak defense, it
was error for the CA to exonerate Ramos from any liability.
ISSUE: Whether or not appellate court may exonerate defendant on the basis
of his mere denial of the imputed wrong in a civil case.
HELD:
The answer is in the affirmative. While mere denial by the defendant is
intrinsically weak evidence, this does not mean that the complainant may just
rely on the weakness of the defense presented by the former. The rule is that the
party asserting a claim must still provide proof of his allegation. In civil cases,
the party carrying the burden of proof must establish his case by a
preponderance of evidence, or evidence which, to the court, is more worthy of
belief than the evidence offered in opposition.
On its face, all that PCIBs evidence proves is that Balmaceda used Ramos
name as a payee when he filled up the application forms for the Managers
checks. But, as the CA correctly observed, the mere fact that Balmaceda made
Ramos the payee on some of the Managers checks is not enough basis to
conclude that Ramos was complicit in Balmacedas fraud; a number of other
people were made payees on the other Managers checks yet PCIB never alleged
them to be liable, nor did the Bank adduce any other evidence pointing to
Ramos participation that would justify his separate treatment from the others.

DRA. LEILA A. DELA LLANA vs. REBECCA BIONG


G.R. No. 182356, December 04, 2013
FACTS:
Joel Primero, a truck driver employed by the Respondent, was involved in a
vehicular accident which resulted to minor injuries of Petitioner Dra. dela Llana.
The traffic investigation that Joel was recklessly imprudent in driving the truck.
More than one month after the incident, Petitioner was diagnose with whiplash
injury which she attributed to the accident that had happened.
Petitioner had to undergo physical therapy and eventually a spine surgery to
treat the injury. However, while the surgery was successful, petitioner can no
longer practice her profession. Consequently, petitioner sued the respondent for
damages for the medical expenses and loss of earnings
At the trial, Dra. dela Llana presented herself as an ordinary witness and Joel as
a hostile witness. Dra. dela Llana reiterated that she lost the mobility of her arm
Page | 312

Compilation of Digested Cases for Remedial Law Review II


by: K. M. T. BUELA
because of the vehicular accident. To prove her claim, she identified and
authenticated a medical certificate dated issued by Dr. Milla. The medical
certificate stated that Dra. dela Llana suffered from a whiplash injury.
The RTC ruled in favor of Dra. dela Llana. But on appeal, CA reversed the RTC
ruling. It held that Dra. dela Llana failed to establish a reasonable connection
between the vehicular accident and her whiplash injury by preponderance of
evidence.
ISSUE: Whether or not mere assertion that damages suffered was caused by a
certain event imputable against the defendant without explaining the causal link
between the injury and the imputed act warrants a ruling in favor of the
complainant.
HELD:
No. In civil cases, a party who alleges a fact has the burden of proving it. He
who alleges has the burden of proving his allegation by preponderance of
evidence or greater weight of credible evidence. The reason for this rule is that
bare allegations, unsubstantiated by evidence, are not equivalent to proof. In
short, mere allegations are not evidence.
In the present case, the burden of proving the proximate causation between
Joels negligence and Dra. dela Llanas whiplash injury rests on Dra. dela Llana.
She must establish by preponderance of evidence that Joels negligence, in its
natural and continuous sequence, unbroken by any efficient intervening cause,
produced her whiplash injury, and without which her whiplash injury would not
have occurred.
Under Article 2176 of the Civil Code, the elements necessary to establish a
quasi-delict case are: (1) damages to the plaintiff; (2) negligence, by act or
omission, of the defendant or by some person for whose acts the defendant must
respond, was guilty; and (3) the connection of cause and effect between such
negligence and the damages.
Notably, Dra. dela Llana anchors her claim mainly on three pieces of evidence:
(1) the pictures of her damaged car, (2) the medical certificate dated November
20, 2000, and (3) her testimonial evidence. However, none of these pieces of
evidence show the causal relation between the vehicular accident and the
whiplash injury. In other words, Dra. dela Llana, during trial, did not adduce the
factum probans or the evidentiary facts by which the factum probandum or the
ultimate fact can be established, as fully discussed below.

Page | 313

Compilation of Digested Cases for Remedial Law Review II


by: K. M. T. BUELA
ZACARIA A. CANDAO, et al. vs. PEOPLE OF THE PHILIPPINES
AND SANDIGANBAYAN
G.R. Nos. 186659-710, October 19, 2011
FACTS:
COA conducted an expanded audit on the financial transactions and operations
of ORG-ARMM for the period July 1992 to March 1993. The Special Audit
Office Report submitted by the audit team shown that illegal withdrawals were
made from the depository accounts of the agency through the issuance of checks
payable to the order of petitioners without the required disbursement vouchers.
Office of the Ombudsman-Mindanao, filed in the Sandiganbayan (SB) criminal
cases for malversation of public funds against petitioners. SB convicted herein
petitioners. The prosecutions lone witness was Heidi L. Mendoza. On crossexamination, witness Mendoza testified that due to security reasons, the audit
team failed to conduct entry and exit conference. SB convicted the petitioner
and held that by their act of co-signing the subject checks without the required
disbursement vouchers of the amounts covered by the 43 checks constitutes
illegal withdrawals.
Petitioners contend that SB committed a reversible error in not applying the
"equipoise rule" which if applied would have resulted in the acquittal of the
accused-petitioners.
ISSUE: Whether or not equipoise rule finds application in the instant case.
HELD:
There is therefore no merit in petitioners argument that the Sandiganbayan
erred in not applying the equipoise rule.
Under the equipoise rule, where the evidence on an issue of fact is in equipoise
or there is doubt on which side the evidence preponderates, the party having the
burden of proof loses. The equipoise rule finds application if the inculpatory
facts and circumstances are capable of two or more explanations, one of
which is consistent with the innocence of the accused and the other
consistent with his guilt, for then the evidence does not fulfill the test of
moral certainty, and does not suffice to produce a conviction. Such is not
the situation in this case because the prosecution was able to prove by adequate
evidence that one of the peitioners failed to account for funds under his custody
and control upon demand, specifically for the P21,045,570.64 illegally
withdrawn from the said funds.

Page | 314

Compilation of Digested Cases for Remedial Law Review II


by: K. M. T. BUELA
Petitioners are both accountable public officers within the meaning of Article
217 of the Revised Penal Code, as amended. No checks can be prepared and no
payment can be effected without their signatures on a disbursement voucher and
the corresponding check. In other words, their indispensable participation of
petitioners in the issuance of the subject checks to effect illegal withdrawals of
ARMM funds was therefore duly established by the prosecution and the
Sandiganbayan did not err in ruling that they acted in conspiracy with petitioner
Haron in embezzling and misappropriating such funds.

Substantial Evidence
OFFICE OF THE OMBUDSMAN vs. ANTONIO T. REYES7
G.R. No. 170512, October 5, 2011
FACTS:
Reyes and Pealoza were charged with grave misconduct pursuant to a
complaint-affidavit executed by Jaime Acero. Pealoza filed a counter-affidavit
implicating Reyes as the mastermind behind the practice of extorting money
from those who failed the drivers licensure exam. In addition to Pealozas
affidavit, he submitted two other affidavits of LTO employees who allegedly
had first-hand knowledge of the practice of Reyes of imposing and pocketing
additional fees. Reyes was not furnished with the copies of two other affidavits
submitted by Pealoza.
Ombudsman convicted Reyes of grave misconduct with penalty of dismissal.
Reyes elevated the case to the Court of Appeals via a Petition for Review. Court
of Appeals granted the petition of Reyes and reversed the judgment of the Office
of the Ombudsman-Mindanao. In assailing the judgment of the Court of
Appeals, petitioner avers that the findings of fact of the Office of the
Ombudsman are entitled to great weight and must be accorded full respect and
credit as long as they are supported by substantial evidence. Petitioner argues
that it is not the task of the appellate court to weigh once more the evidence
submitted before an administrative body and to substitute its own judgment for
that of the administrative agency with respect to the sufficiency of evidence.
ISSUE: Whether or not findings of substantial evidence of administrative or
quasi-judicial body is not subject to review by appellate courts.
HELD:
The answer is in the negative. In administrative and quasi-judicial proceedings,
only substantial evidence is necessary to establish the case for or against a party.
Substantial evidence is more than a mere scintilla of evidence. It is that amount
7

Penned by LEONARDO DE CASTRO, J.:


Page | 315

Compilation of Digested Cases for Remedial Law Review II


by: K. M. T. BUELA
of relevant evidence that a reasonable mind might accept as adequate to support
a conclusion, even if other minds, equally reasonable, might conceivably opine
otherwise.
Dadulo v. Court of Appeals reiterates that in reviewing administrative decisions,
it is beyond the province of this Court to weigh the conflicting evidence,
determine the credibility of witnesses, or otherwise substitute its judgment for
that of the administrative agency with respect to the sufficiency of evidence.
However, while it is not the function of the Court to analyze and weigh the
parties' evidence all over again, an exception thereto lies as when there is serious
ground to believe that a possible miscarriage of justice would thereby result.
After carefully perusing the records of this case, we find that the above-cited
exception, rather than the general rule, applies herein. Considering that
petitioner rendered its Decision on the basis of evidence that were not disclosed
to Reyes, Reyes right to due process was violated. A judgment in an
administrative case that imposes the extreme penalty of dismissal must not only
be based on substantial evidence but also rendered with due regard to the rights
of the parties to due process.

XAVIER C. RAMOS vs. BPI FAMILY SAVINGS BANK INC. and/or


ALFONSO L. SALCEDO, JR.
G.R. No. 203186, December 4, 2013
FACTS:
Ramos was employed by BPI Family as Vice-President for Dealer Network
Marketing/Auto Loans Division. His duties and responsibilities includes the
receipt and approval of applications for auto loans from auto dealers and
salesmen. During his tenure, a person pretending to be their valued client Acosta
secured another auto loan from BPI Family which had remained unpaid. After
investigation, BPI Family discovered that: (1) a person misrepresented herself
as Acosta and succeeded in obtaining the delivery of a Toyota Prado pursuant
to the Purchase Order (PO) and Authority to Deliver (ATD) issued by Ramos;
(2) Ramos released these documents without the prior approval of BPI Familys
credit committee.
Consequently, Ramos employment was severed and his last pay and benefits
were deducted with a portion of the losses incurred by BPI due to the Acosta
incident. Claiming that the deductions made by BPI Family were illegal, Ramos
filed a complaint for underpayment of retirement benefits against BPI. Labor
Arbiter (LA) dismissed Ramoss complaint. NLRC reversed the LA in a
Decision alleged negligence committed by Ramos was not substantially proven
Page | 316

Compilation of Digested Cases for Remedial Law Review II


by: K. M. T. BUELA
as he was not expected to personally examine all loan documents that pass
through his hands or to require the client to personally appear before him
because he has subordinates to do those details for him.
CA affirmed the finding of negligence on the part of Ramos, holding that Ramos
was remiss in his duty. But it also attributed negligence on the part of BPI
Family since it sanctioned the practice of issuing the PO and ATD prior to the
approval of the credit committee. Finding BPI Familys negligence to be
concurrent with Ramos, the CA found it improper to deduct the entire
P546,000.00 from Ramos s retirement benefits and, instead, equitably reduced
the same to the amount of P200,000.00.
ISSUE: Whether or not findings of fact by a labor tribunal may be assailed by
petition for certiorari.
HELD:
As a general rule, in certiorari proceedings under Rule 65 of the Rules of Court,
the appellate court does not assess and weigh the sufficiency of evidence upon
which the Labor Arbiter and the NLRC based their conclusion. The query in
this proceeding is limited to the determination of whether or not the NLRC acted
without or in excess of its jurisdiction or with grave abuse of discretion in
rendering its decision. However, as an exception, the appellate court may
examine and measure the factual findings of the NLRC if the same are not
supported by substantial evidence.
The Court has not hesitated to affirm the appellate courts reversals of the
decisions of labor tribunals if they are not supported by substantial evidence .
The requirement that the NLRCs findings should be supported by substantial
evidence is clearly expressed in Section 5, Rule 133 of the Rules of Court which
provides that "in cases filed before administrative or quasi- judicial bodies, a
fact may be deemed established if it is supported by substantial evidence, or that
amount of relevant evidence which a reasonable mind might accept as adequate
to justify a conclusion."
Applying the foregoing considerations, the Court finds the CA to have erred in
attributing grave abuse of discretion on the part of the NLRC in finding that the
deduction made from Ramoss retirement benefits was improper as BPI was not
able to substantially prove its imputation of negligence against Ramos. Wellsettled is the rule that the burden of proof rests upon the party who asserts the
affirmative of an issue.

Page | 317

Compilation of Digested Cases for Remedial Law Review II


by: K. M. T. BUELA
Prima Facie Evidence
JESSE U. LUCAS vs. JESUS S. LUCAS
G.R. No. 190710, June 6, 2011
FACTS:
Petitioner, Jesse U. Lucas, filed a Petition to Establish Illegitimate Filiation
specifically seeking a DNA testing order to abbreviate the proceedings. RTC
dismissed the. The court opined that petitioner must first establish these four
procedural aspects before he can present evidence of paternity and filiation.
Petitioner failed to establish a prima facie case considering that (a) his mother
did not personally declare that she had sexual relations with respondent, and
petitioners statement as to what his mother told him about his father was clearly
hearsay; (b) the certificate of live birth was not signed by respondent; and (c)
although petitioner used the surname of respondent, there was no allegation that
he was treated as the child of respondent by the latter or his family.
Petitioner filed a motion for reconsideration which was granted by the RTC. On
appeal by the respondent to CA, CA remarked that petitioner filed the petition
to establish illegitimate filiation, specifically seeking a DNA testing order to
abbreviate the proceedings. It noted that petitioner failed to show that the four
significant procedural aspects of a traditional paternity action had been met. The
CA further held that a DNA testing should not be allowed when the petitioner
has failed to establish a prima facie case.
ISSUE: Whether a prima facie showing is necessary before a court can issue a
DNA testing order for purposes of proving filiation.
HELD:
Although a paternity action is civil, not criminal, the constitutional prohibition
against unreasonable searches and seizures is still applicable, and a proper
showing of sufficient justification under the particular factual circumstances of
the case must be made before a court may order a compulsory blood test.
as a preliminary matter, before the court may issue an order for compulsory
blood testing, the moving party must show that there is a reasonable possibility
of paternity. As explained hereafter, in cases in which paternity is contested and
a party to the action refuses to voluntarily undergo a blood test, a show cause
hearing must be held in which the court can determine whether there is sufficient
evidence to establish a prima facie case which warrants issuance of a court order
for blood testing.
The same condition precedent should be applied in our jurisdiction to protect
the putative father from mere harassment suits. Thus, during the hearing on the

Page | 318

Compilation of Digested Cases for Remedial Law Review II


by: K. M. T. BUELA
motion for DNA testing, the petitioner must present prima facie evidence or
establish a reasonable possibility of paternity.
Notwithstanding these, it should be stressed that the issuance of a DNA testing
order remains discretionary upon the court. The court may, for example,
consider whether there is absolute necessity for the DNA testing. If there is
already preponderance of evidence to establish paternity and the DNA test result
would only be corroborative, the court may, in its discretion, disallow a DNA
testing.

Probable Cause
PHILIPPINE NATIONAL BANK vs. AMELIO TRIA and JOHN DOE
G.R. No. 193250, April 25, 2012
FACTS:
Respondent Tria was a former Branch Manager of PNB-MWSS. MWSS opened
a Current Account with the said PNB Branch and made an initial deposit of PhP
6,714,621.13. On April 22, 2004, PNB-MWSS received a letter-request from
MWSS instructing the deduction of PhP 5,200,000 from C/A and the issuance
of the corresponding managers check payable to a certain "Atty. Rodrigo A.
Reyes." The letter-request, supporting documents, and Managers Check
Application Form were then evaluated by the banks Sales and Service Officer
(SSO), Bagasani, who found the same to be in order. Thus, a managers check
was issued payable to Atty. Reyes.
On April 26, 2004, PNB-MWSS received cash delivery from PNBs Cash
Center in the amount of PhP 8,660,000. Onthe same day, respondent Tria
accompanied Atty. Reyes in presenting Managers Check No. 1165848 to
PNBs Circle Branch. SSO, Flandez of PNB-MWSS Sale and Service Head,
Veniegas and placement of Trias signature on the check above the handwritten
note "PAYEE IDENTIFIED AMELIO C. TRIA."
It was later on found that MWSS did not authorize the issuance of the said
managers check. PNB conducted its own investigation and, at its conclusion,
sought to hold Tria liable for qualified theft. During preliminary investigation,
the Assistant City Prosecutor (ACP) issued a Resolution finding no probable
cause against Tria. ACPs resolution was affirmed both by DOJ and CA.
ISSUE: Whether or not there is probable cause to prosecute Tria with qualified
theft.
HELD:
Page | 319

Compilation of Digested Cases for Remedial Law Review II


by: K. M. T. BUELA
Yea. While discretionary authority to determine probable cause in a preliminary
investigation to ascertain sufficient ground for the filing of an information rests
with the executive branch, such authority is far from absolute. It may be subject
to review when it has been clearly used with grave abuse of discretion. And
indeed, grave abuse of discretion attended the decision to drop the charges
against Tria as there was more than probable cause to proceed against him for
qualified theft.
It must be emphasized at the outset that what is necessary for the filing of a
criminal information is not proof beyond reasonable doubt that the person
accused is guilty of the acts imputed on him, but only that there is probable
cause to believe that he is guilty of the crime charged.
Probable cause, for purposes of filing a criminal information, are such facts as
are sufficient to engender a well-founded belief that a crime has been committed
and that the accused is probably guilty thereof. It is the existence of such facts
and circumstances as would excite the belief in a reasonable mind, acting on the
facts within the knowledge of the prosecutor, that the person charged was guilty
of the crime for which he is to be prosecuted. A finding of probable cause needs
only to rest on evidence showing that, more likely than not, a crime has been
committed and that it was committed by the accused.
The acts of Tria and the relevant circumstances that led to the encashment of
the check provide more than sufficient basis for the finding of probable cause
to file an information against him and John Doe/Atty. Reyes for qualified theft.
In fact, it is easy to infer from the factual milieu of the instant case the existence
of all the elements necessary for the prosecution of the crime of qualified theft.

RUBEN DEL CASTILLO vs. PEOPLE OF THE PHILIPPINES


G.R. No. 185128, January 30, 2012
FACTS:
Pursuant to a confidential information that petitioner was engaged in selling
shabu, police officers headed by SPO3 Bienvenido Masnayon, after conducting
surveillance and test-buy operation at the house of petitioner, secured a search
warrant from the RTC which was granted. During the implementation of the
warrant, petitioner escaped and illegal drugs were found inside a nipa hut in
front of his rented two-story residence.
Consequently, an Information was filed before the RTC against petitioner,
charging him with violation of Section 16, Article III of R.A. 6425. RTC
convicted the accused. The finding of conviction was affirmed by the CA. On
Page | 320

Compilation of Digested Cases for Remedial Law Review II


by: K. M. T. BUELA
appeal with the SC, petitioner assailed the validity of the issuance of the search
warrant on the ground that the police officer who applied for the same as no
personal knowledge of the basis for the issuance of such warrant.
ISSUE: Whether or not validity of the issuance of a search warrant may be
questioned on appeal on the ground that the person applying for the warrant
had no personal knowledge of the cause for the issuance thereof.
HELD:
The requisites for the issuance of a search warrant are: (1) probable cause is
present; (2) such probable cause must be determined personally by the judge;
(3) the judge must examine, in writing and under oath or affirmation, the
complainant and the witnesses he or she may produce; (4) the applicant and the
witnesses testify on the facts personally known to them; and (5) the warrant
specifically describes the place to be searched and the things to be seized.
A finding of probable cause needs only to rest on evidence showing that, more
likely than not, a crime has been committed and that it was committed by the
accused. Probable cause demands more than bare suspicion; it requires less than
evidence which would justify conviction.
The judge, in determining probable cause, is to consider the totality of the
circumstances made known to him and not by a fixed and rigid formula, and
must employ a flexible, totality of the circumstances standard. The existence
depends to a large degree upon the finding or opinion of the judge conducting
the examination. This Court, therefore, is in no position to disturb the factual
findings of the judge which led to the issuance of the search warrant. A
magistrate's determination of probable cause for the issuance of a search warrant
is paid great deference by a reviewing court, as long as there was substantial
basis for that determination. Substantial basis means that the questions of the
examining judge brought out such facts and circumstances as would lead a
reasonably discreet and prudent man to believe that an offense has been
committed, and the objects in connection with the offense sought to be seized
are in the place sought to be searched. A review of the records shows that in the
present case, a substantial basis exists.

Page | 321

Compilation of Digested Cases for Remedial Law Review II


by: K. M. T. BUELA
Iota of Evidence (Circumstantial)
PEOPLE OF THE PHILIPPINES vs. ALBERTO ANTICAMARA y
CABILLO et al.
G.R. No. 178771, June 8, 2011
FACTS:
Appellants are charged with the crimes of Murder and of Kidnapping/Serious
Illegal Detention in two separate Information.
During the trial, AAA, one of the victims, testified: (1) that on May 7, 2002,
while she and the victim Abad were sleeping inside the house of the Estrella
family, several persons entered to rob the place; (2) Inside the house, she saw
and recognized the appellants, and heard one of them uttering "somebody will
die; (3) appellants took her outside the house and pushed her into the Revo
(vehicle); (4) inside the Revo, she saw inside Abad Sulpacio who was
blindfolded and with his hands tied; (5) the last time that she saw Abad Sulpacio
was when he was dragged out from the vehicle; (6) during her captivity, AAA
was repeatedly rape by the appellants.
RTC found the appellants guilty of the crime of Kidnapping/Serious Illegal
Detention of AAA and Murder of Abad. Considering that the victim AAA was
raped during her detention, the maximum penalty of DEATH was imposed. CA
affirmed RTCs ruling. On appeal with SC, appellants assailed the conviction
of murder on the ground that there was no eye-witness presented that they
indeed kill Abad.
ISSUE: Whether or not an accused may be convicted even if there are no eyewitness on the commission of the offense.
HELD:
The answer is in the affirmative. The trial court found that although there was
no direct eyewitness in the killing of Abad, the prosecution adduced sufficient
circumstantial evidence to establish with moral certainty the identities and guilt
of the perpetrators of the crime.
Circumstantial evidence consists of proof of collateral facts and circumstances
from which the existence of the main fact may be inferred according to reason
and common experience . Circumstantial evidence is sufficient to sustain
conviction if: (a) there is more than one circumstance; (b) the facts from which
the inferences are derived are proven; (c) the combination of all circumstances
is such as to produce a conviction beyond reasonable doubt. A judgment of
conviction based on circumstantial evidence can be sustained when the
circumstances proved form an unbroken chain that results in a fair and
Page | 322

Compilation of Digested Cases for Remedial Law Review II


by: K. M. T. BUELA
reasonable conclusion pointing to the accused, to the exclusion of all others, as
the perpetrator.
In the case at bar, although no one directly saw the actual killing of Sulpacio,
the prosecution was able to paint a clear picture that the appellants took Sulpacio
away from the house of the Estrellas, tied and blindfolded him, and brought him
to another place where he was repeatedly shot and buried.

PEOPLE OF THE PHILIPPINES, vs. EFREN DEOCAMPO et al.


G.R. No. 185212, February 15, 2012
FACTS:
Maritess Alolod, Efren Deocampo, Edwin Deocampo, and Elmer Deocampo
were charged with double murder. RTC convicted them considering the
following circumstantial evidence: (1) Efren had always been banned from the
old couples house because they strongly disapproved his relationship with
Maritess; (2) The old couple were enjoying good health before of May 27, 1998;
(3) On May 28 they suddenly went missing; (4) On the night of May 27 the
security guard at Salaman Institute saw Efren and Edwin standing on the school
side of the fence next to the old couples house. The next day, the guard
discovered that the fence wire had been cut; (5) At about 2:00 a.m. of May 28 a
`neighbor heard the sound of a woman sobbing and what seemed like the
butchering of a pig; (6) At break of dawn, a witness saw Efren in the Alolod
kitchen; (7) From then on Efren and his brothers frequented the old couples
house, with Efren wearing the old mans watch; (8) Maritess lied about her
adoptive parents going to Cotabato City / Davao City for medical treatment
when people started looking for them; (9) A witness heard Efren instructing
Maritess to plant more camote on a pile of red soil beside the house; and (10)
The bodies of the old couple were found underneath those plants.
On appeal, Accused Efren Deocampo assailed their conviction based on
circumstantial evidence.
ISSUE: Whether or not the CA erred in affirming the RTCs finding that
accused Efren was responsible for the murder of the Alolod couple based on
circumstantial evidence.
HELD:
The rule of evidence that applies when no witness saw the commission of the
crime provides:

Page | 323

Compilation of Digested Cases for Remedial Law Review II


by: K. M. T. BUELA
SEC. 4. Circumstantial evidence, when sufficient. Circumstantial evidence is
sufficient for conviction if: (a) There is more than one circumstance; (b) The
facts from which the inferences are derived are proven; and (c) The
combination of all the circumstances is such as to produce a conviction beyond
reasonable doubt.
The circumstances must constitute an unbroken chain that inexorably leads to
one fair conclusion: the accused committed the crime to the exclusion of all
others.
In this case, the trial court was correct in holding that the accused was guilty
due to the following facts: (1) the presence of the accused in the victims house
and used of the latters things when he had always been banned therefrom
during the lifetime of the victim; (2) the cover-up story of the adopted daughter
that her parent went away for a medical check-up; (3) the planting of camote on
the ground where the victims where found buried; (4) testimony of a neighbor
hearing a scream the night before the victims went missing then seeing the
accused inside the house of the victims. All these when taken together leads
only to one conclusion, i.e., that the accused is guilty of the crime charged.

Page | 324

You might also like